Sie sind auf Seite 1von 622

i

INTRODUCTION TO FINANCIAL ACCOUNTING


THE A R E WAY

MICHAEL A. WILLIAMS

W and B Publications
ii

All rights reserved. No part of this book may be reproduced in any


form or by any means, without permission in writing from the
Publisher.

DEDICATED TO

This book is dedicated to all the individuals who encouraged me to


produce the 3rd. edition. Thank you.

NEVER

GIVE

UP
PREFACE iii

This introductory textbook in Financial Accounting was written for


several reasons:

(1) To provide a local textbook for Introduction To Financial


Accounting;

(2) To emphasize the practical discipline of Financial Accounting;

(3) To improve the pedagogical nature of teaching Financial


Accounting using the A R E method; and

(4) To provide students with a solid conceptual and practical


foundation for the study of Financial Accounting.

My teaching of Financial Accounting spans twenty-seven (27) years


(including 17 years at the University of the West Indies, Mona) and
over those years I have developed an awareness of the areas in
Financial Accounting which students find very difficult to
understand and master. I have exercised special care in those areas
to make them clear and understandable.

Financial Accounting is a cycle and the sequencing of chapters


emphasizes this. To maintain the students' attention and interest,
the text mixes an active voice with a clear and concise writing
style emphasizing distinctive captions, large print, frequent use
of examples, and adequate spacing of sentences.

The favourable comments from undergraduate and postgraduate


students who read the second edition have convinced me that my
innovative style of writing and years of practical experience were
instrumental in the overwhelming success of the second edition.
People who I do not know have telephoned me indicating that they
borrowed a copy of the 2nd. Edition and it was so good that they
would like to purchase a copy for themselves. This convinced me
that it was time for the 3rd. edition.

Although several computer financial accounting packages are


available such as, Peach Tree Accounting, which assists in
preparing the financial statements faster than a manual financial
accounting system, this does not mean that students no longer need
to learn Financial Accounting. Remember when it comes to computers
"garbage in garbage out." So a thorough understanding of financial
accounting principles and concepts is necessary whether you use a
computer accounting system or a manual accounting system.
iv

ACKNOWLEDGMENT
My thanks and gratitude is extended to Miguel Williams, Michelle
Williams, and Andrew Foster. The preparation of any financial
accounting textbook is a long and demanding project and they worked
long hours in preparing this edition. The team’s constructive
suggestions and innovative ideas were instrumental in making this
third edition complete, accurate and colourful.
BRIEF CHAPTER CONTENTS
CHAPTER 1 FINANCIAL ACCOUNTING – THE A R E WAY

CHAPTER 2 CONCEPTS AND STANDARDS

CHAPTER 3 BUSINESS ORGANIZATIONS

CHAPTER 4 RECORDING TRANSACTIONS IN BOOKS OF ORIGINAL


ENTRY

CHAPTER 5 POSTING TO THE GENERAL LEDGER AND


SUBSIDIARY LEDGERS

CHAPTER 6 ADJUSTMENTS

CHAPTER 7 PROPERTY, PLANT AND EQUIPMENT

CHAPTER 8 FINAL ACCOUNTS AND FINANCIAL STATEMENTS

CHAPTER 9 CASH FLOW STATEMENT

CHAPTER 10 INCOMPLETE RECORDS

CHAPTER 11 BANK RECONCILIATION

CHAPTER 12 PARTNERSHIP ACCOUNTS

CHAPTER 13 COMPANY ACCOUNTS

CHAPTER 14 DEPARTMENTAL ACCOUNTS

CHAPTER 15 ACCOUNTING FOR NON-PROFIT ORGANIZATIONS

CHAPTER 16 MANUFACTURING ACCOUNTS

CHAPTER 17 FINANCIAL STATEMENT ANALYSIS

CHAPTER 18 STOCK TAKING AND PAYROLL ACCOUNTING


CONTENTS
CHAPTER ONE PAGE
FINANCIAL ACCOUNTING - THE A R E WAY 1
DEFINITION OF ACCOUNTING 2
NATURE OF FINANCIAL ACCOUNTING 3
PURPOSE OF ACCOUNTING 3
CAREERS IN ACCOUNTING 5
COMPUTERS IN ACCOUNTING 6
FINANCIAL ACCOUNTING CYCLE 9
ASSETS AND LIABILITIES 12
RECEIPTS AND PAYMENTS 15
EXPENSE AND INCOME 15
FUNDAMENTAL PRINCIPLE OF DEBIT AND CREDIT 16
RULES FOR POSTING TRANSACTIONS 18
A R E METHOD 19
APPLICATION OF A R E METHOD 25
TIME OUT 29
REVIEW QUESTIONS 30
ANSWERS TO SELECTED REVIEW QUESTIONS 32

CHAPTER TWO
CONCEPTS AND STANDARDS 33
CONCEPTS, CONVENTIONS AND GAAP 34
ACCOUNTING STANDARDS 39
REVIEW QUESTIONS 40
ANSWERS TO SELECTED REVIEW QUESTIONS 42

CHAPTER THREE
BUSINESS ORGANIZATIONS 43
MAIN TYPES OF BUSINESS ORGANIZATIONS 44
SOLE PROPRIETORSHIP OR SOLE TRADER 45
PARTNERSHIP 46
COMPANY 48
DOCUMENTS USED IN BUSINESS 50
SAMPLES OF DOCUMENTS USED IN BUSINESS 52
REVIEW QUESTIONS 65
ANSWERS TO SELECTED REVIEW QUESTIONS 67
CHAPTER FOUR
RECORDING TRANSACTIONS IN BOOKS OF ORIGINAL ENTRY 69
INTRODUCTION 70
CASH DISCOUNT 70
TRADE DISCOUNT 70
RECORDING TRANSACTIONS 71
SALES JOURNAL 74
PURCHASES JOURNAL 75
RETURNS INWARD JOURNAL 76
RETURNS OUTWARD JOURNAL 76
CASH BOOK 77
PETTY CASH BOOK 84
GENERAL JOURNAL 90
REVIEW QUESTIONS 91
ANSWERS TO SELECTED REVIEW QUESTIONS 93

CHAPTER FIVE
POSTING TO THE GENERAL LEDGER AND SUBSIDIARY LEDGERS 97
TYPES OF ACCOUNTS 98
CHART OF ACCOUNTS 99
GENERAL LEDGER 104
SUBSIDIARY LEDGERS 104
POSTING FROM BOOKS OF ORIGINAL ENTRY 105
BALANCING ACCOUNTS 120
TRIAL BALANCE 121
ERRORS THAT DO NOT AFFECT THE BALANCING OF THE
TRIAL BALANCE 123
ERRORS THAT AFFECT THE BALANCING OF THE TRIAL
BALANCE 124
SUSPENSE ACCOUNT 125
CORRECTION OF ERRORS 126
CONTROL ACCOUNTS 133
REVIEW QUESTIONS 139
ANSWERS TO SELECTED REVIEW QUESTIONS 141
CHAPTER SIX
ADJUSTMENTS 143
INTRODUCTION 144
ACCRUED EXPENSES 144
PREPAYMENTS 148
BAD DEBTS 151
ALLOWANCE FOR BAD DEBTS 153
OTHER ALLOWANCES 154
ACCRUED INCOME 157
UNEARNED INCOME 159
MARK-UP AND MARGIN 162
OTHER ADJUSTMENTS 165
REVIEW QUESTIONS 169
ANSWERS TO SELECTED REVIEW QUESTIONS 171

CHAPTER SEVEN
PROPERTY, PLANT AND EQIPMENT 175
CAPITAL EXPENDITURE AND REVENUE EXPENDITURE 176
DEPRECIATION 178
DEPRECIATION POLICY 178
METHODS OF PROVIDING FOR DEPRECIATION 179
STRAIGHT LINE OR FIXED INSTALMENT METHOD 179
REDUCING BALANCE OR DIMINISHING BALANCE METHOD 188
SUM OF THE YEARS' DIGIT METHOD 190
REVALUATION METHOD 191
DEPLETION METHOD 192
DISPOSAL OF PROPERTY, PLANT AND EQUIPMENT 193
REVIEW QUESTIONS 195
ANSWERS TO SELECTED REVIEW QUESTIONS 196

CHAPTER EIGHT
FINAL ACCOUNTS AND FINANCIAL STATEMENTS 205
INTRODUCTION 206
ADJUSTED TRIAL BALANCE 206
CLOSING ENTRIES 206
ACCOUNTANT'S WORKSHEET 210
PREPARATION OF TRADING AND PROFIT AND LOSS ACCOUNT 216
PREPARATION OF BALANCE SHEET 221
ACCOUNTING EQUATION 227
OPENING ENTRIES 228
LIMITATIONS OF INCOME STATEMENT 229
LIMITATIONS OF BALANCE SHEET 229
REVIEW QUESTIONS 230
ANSWERS TO SELECTED REVIEW QUESTIONS 236
CHAPTER NINE
CASH FLOW STATEMENT 247
INTRODUCTION 248
DIRECT METHOD 249
INDIRECT METHOD 254
REVIEW QUESTIONS 259
ANSWERS TO SELECTED REVIEW QUESTIONS 263

CHAPTER TEN
INCOMPLETE RECORDS 267
INTRODUCTION 268
CALCULATION OF PROFIT OR LOSS 269
PREPARATION OF FINAL ACCOUNTS 270
REVIEW QUESTIONS 277
ANSWERS TO SELECTED REVIEW QUESTIONS 280

CHAPTER ELEVEN
BANK RECONCILIATIONS 285
INTRODUCTION 286
TYPES OF CHEQUES 290
RECONCILIATION 293
REVIEW QUESTIONS 308
ANSWERS TO SELECTED REVIEW QUESTIONS 311

CHAPTER TWELVE
PARTNERSHIP ACCOUNTS 315
INTRODUCTION 316
PROFIT AND LOSS APPROPRIATION ACCOUNT 316
CAPITAL ACCOUNT 317
CURRENT ACCOUNT 317
FINAL ACCOUNTS AND BALANCE SHEET 319
STATEMENT OF CHANGES IN PARTNERS’ EQUITY 324
REVIEW QUESTIONS 327
ANSWERS TO SELECTED REVIEW QUESTIONS 331
CHAPTER THIRTEEN
COMPANY ACCOUNTS 339
INTRODUCTION 340
FORMATION OF A COMPANY 340
REGISTERED COMPANY 341
PRIVATE COMPANY 342
PUBLIC COMPANY 342
BOARD OF DIRECTORS 343
GENERAL MEETING 344
AUTHORIZED SHARE CAPITAL AND ISSUED SHARE CAPITAL 345
TYPES OF SHARES 345
DIVIDENDS 347
CAPITAL RESERVE AND REVENUE RESERVE 348
DEBENTURES AND BONDS 349
PROFIT AND LOSS APPROPRIATION ACCOUNT 349
FINAL ACCOUNTS AND BALANCE SHEET 350
STATEMENT OF CHANGES IN EQUITY 352
BALANCE SHEET ARRANGEMENT 356
CASH FLOW STATEMENT 359
REVIEW QUESTIONS 364
ANSWERS TO SELECTED REVIEW QUESTIONS 370

CHAPTER FOURTEEN
DEPARTMENTAL ACCOUNTS 377
INTRODUCTION 378
INDIRECT COSTS AND DIRECT COSTS 379
ALLOCATION OF INDIRECT COSTS 379
DEPARTMENTAL TRADING AND PROFIT AND LOSS STATEMENT 380
ROOMS DEPARTMENT INCOME STATEMENT 382
FOOD DEPARTMENT INCOME STATEMENT 385
BEVERAGE DEPARTMENT INCOME STATEMENT 389
REVIEW QUESTIONS 393
ANSWERS TO SELECTED REVIEW QUESTIONS 399

CHAPTER FIFTEEN
ACCOUNTING FOR NON-PROFIT ORGANIZATIONS 407
INTRODUCTION 408
DONATIONS RECEIVED BY THE CLUB 409
LIFE MEMBERSHIP FUND 410
ACCOUNTING POLICY 411
FINAL ACCOUNTS AND BALANCE SHEET 412
STATUTORY BODY 417
REVIEW QUESTIONS 421
ANSWERS TO SELECTED REVIEW QUESTIONS 425
CHAPTER SIXTEEN
MANUFACTURING ACCOUNTS 431
INTRODUCTION 432
COST OF RAW MATERIALS CONSUMED 433
PRIME COST 433
MANUFACTURING COST OR COST OF PRODUCTION 434
MANUFACTURING COST PER UNIT 434
MANUFACTURING ACCOUNT 435
WORK IN PROGRESS VALUED AT PRIME COST 440
COST CENTER TRADING ACCOUNT 442
PROFIT CENTER TRADING ACCOUNT 442
UNREALIZED PROFIT 443
FINAL ACCOUNTS FOR A MANUFACTURING ENTERPRISE 445
JOB COSTING 449
REVIEW QUESTIONS 452
ANSWERS TO SELECTED REVIEW QUESTIONS 456

CHAPTER SEVENTEEN
FINANCIAL STATEMENT ANALYSIS 463
INTRODUCTION 464
TYPES OF FINANCIAL STATEMENT ANALYSIS 465
SOLVENCY ANALYSIS 465
SHORT TERM SOLVENCY (LIQUIDITY) ANALYSIS 466
LONG TERM SOLVENCY ANALYSIS 470
PROFITABILITY ANALYSIS 473
SHARE PERFORMANCE ANALYSIS 475
HORIZONTAL ANALYSIS 476
VERTICAL ANALYSIS 476
TREND ANALYSIS 477
COMPARATIVE ANALYSIS 477
LIQUIDITY RATIOS 478
ASSET MANAGEMENT RATIOS 479
DEBT MANAGEMENT RATIOS 480
PROFITABILITY RATIOS 481
SHARE PERFORMANCE RATIOS 482
CASH FLOW STATEMENT RATIOS 485
REVIEW QUESTIONS 489
ANSWERS TO SELECTED REVIEW QUESTIONS 497
CHAPTER EIGHTEEN
STOCKTAKING AND PAYROLL ACCOUNTING 501
INTRODUCTION 502
TYPES OF CLOSING INVENTORY 503
PERPETUAL INVENTORY SYSTEM 504
PHYSICAL INVENTORY COUNT 505
STOCK RECONCILIATION 512
PAYROLL ACCOUNTING ENTRIES 521
TIMESHEET 527
PAYROLL SUMMARY 530
REVIEW QUESTIONS 533
ANSWERS TO SELECTED REVIEW QUESTIONS 536

ADDITIONAL PROBLEMS 539

CROSSWORD PUZZLE 585

ANSWER TO CROSSWORD PUZZLE 588

GLOSSARY 589

INDEX 600
1

Ch1
CHAPTER ONE

FINANCIAL ACCOUNTING
THE A R E WAY

CHAPTER OBJECTIVES
After completing this chapter you should be able to:

 Define Accounting.

 State clearly the nature of Accounting.

 State clearly the purpose of Accounting.

Draw a flowchart depicting the financial accounting


cycle.

 Distinguish between assets and liabilities.

 Distinguish between receipts and payments.

 Distinguish between expense and income.

 Explain the A R E method.


2

DEFINITION OF ACCOUNTING
Accounting is a financial information system that involves the
analysis, classification, recording, summarising, interpretation,
and the communication of business transactions in monetary terms.
Accounting can also be defined as the science of recording,
analysing, interpreting, and communicating quantitative and non-
quantitative information. It should be noted that accounting is
mainly concerned with quantitative information expressed in
monetary terms.

Before the development of specialised areas in accounting the terms


principles of accounts and bookkeeping were widely used when
referring to accounts. Principles of accounts now refer to the
concepts and conventions that are used in preparing the accounts.
Bookkeeping now refers to the recording of business transactions in
books of original entry. In a small organisation, one accountant
performs all the financial accounting functions. This accountant
therefore performs the bookkeeping function and all other
accounting functions. In medium size and large organisations, the
financial accounting functions are usually performed by several
accountants and accounting clerks. The accounting clerks are
usually responsible for performing the bookkeeping function, that
is, the simple financial accounting tasks.

The evolution of accounting as a science and the development of


financial accounting as a specialised area, relegated “principles
of accounts” and “bookkeeping” to subsets of financial accounting.
The recording of transactions (bookkeeping) and the preparation of
accounts are based on certain concepts and conventions (principles
of accounts). But the recording process, the concepts and
conventions are only just a section of financial accounting.
Financial accounting includes other things, such as, the
communication of financial information in the form of financial
statements.
3

NATURE OF ACCOUNTING
Accounting provides a wide range of financial information.
Information is provided on the cost of a product or service, the
break-even point of sales, the income tax owed to the government,
the value of property, plant and equipment, the net profit or net
loss made by the business, and so on.

Owners, managers and supervisors need financial information to make


business decisions. Accounting provides decision makers of business
entities, government, and non-profit organisations with relevant
information to allocate their scarce resources. So accounting
assist managers in the efficient allocation of resources. The board
of directors of a company, for example, would need to know if the
company made a net profit or net loss, before they can make a
decision about declaring dividends to shareholders.

The financial data that are used by decision makers are usually
communicated to them in accounting jargon, for example, gross
profit, sales revenue, budget deficit and so on. Accounting is
therefore referred to as “The language of businesses.”

PURPOSE OF ACCOUNTING
Businesses need to know the profit or loss at the end of a period
and the assets and liabilities of the business at the end of that
same period. Accounting provides this information. The purpose of
accounting is:

1. To provide managers, supervisors and employees with financial


information to make business decisions and to evaluate the
performance of projects, departments, divisions, branches,
subsidiaries and the business as a whole.

2. To provide trade creditors with financial information about


the profitability and liquidity of the business, so that they
can make decisions about whether to extend credit, the amount
of credit to extend and to assess the risk involved in
extending credit.

3. To provide Government agencies with financial information so


that they can determine the tax liability of businesses and
individuals.
4

4. To provide owners with financial information for them to make


investment decisions and to evaluate the performance of
managers.

5. To provide trade unions with financial information about the


profitability and liquidity of the business, so that they can
negotiate effectively for better wage and fringe benefits for
employees.

6. To provide Government with financial information about revenue


collections, revenue expenditure and capital expenditure, so
that they can better allocate scarce resources.

7. To provide individuals with financial information about


earnings and expenditure, so that they can determine their tax
liability, the level of credit to obtain and, or the level of
personal savings to make.

8. To provide members of a non-profit organisation with financial


information, so that they can assess the level and adequacy of
the services being provided by the organisation.

9. To provide financial institutions with financial information


about the profitability, short term solvency (ability of the
business to pay its debts in the short term), and long term
solvency (ability of the business to pay its debts in the long
term) of the business, so that they can make decisions about
whether to approve a loan request and to assess the risk
involved in advancing the loan.

10. To provide financial analysts with financial information


about the business so that they can make projections about
the future earnings and value of the business in order to
provide sound financial advice.

11. To provide managers with financial information for them to


evaluate the performance of employees, set selling price,
assess the profitability and solvency of the business.
5

CAREERS IN ACCOUNTING
Accounting can be divided into specialised areas such as, financial
accounting, management accounting, internal auditing, external
auditing, and taxation. Which area of accounting are you interested
in?

Financial accounting involves the recording of business


transactions, the preparation of accounts, the preparation of
financial statements, such as, the income statement, the statement
of changes in equity, the balance sheet and the cash flow
statement. The individual in charge of the accounting department in
a medium size or large organisation is usually referred to as the
financial controller or comptroller (a professional accountant).
The qualification required for this job is usually a master’s
degree in accounting or ACCA (association of certified and
chartered accountants) designation, or CPA (certified public
accountant) designation. The ACCA is a British qualification, but
the examination can be done locally in your country, through any of
the accounting institutes, such as, ICAJ (Institute of Chartered
Accountants of Jamaica), ICATT (Institute of Chartered Accountants
of Trinidad and Tobago), ICAB (Institute of Chartered Accountants
of Barbados), and the BICA (Bahamas Institute of Chartered
Accountants). The CPA is a North American qualification and
currently the examination can only be done in North America. Some
states in the U.S.A. allow non-US citizens to register and sit the
examination. But a first degree in accounting and in some cases
(the current trend) a master’s degree in accounting is required
before you can register for the CPA examination. Since the
examination is in North America, you will need a visa. Accounting
clerks and accountants report to the financial controller or
comptroller. The qualification required for accounting clerks is
usually CXC or GCE accounting. The qualification required for
accountants is usually a Bachelor of Science Degree (B.SC) with a
major or specialization in accounting.

Management accounting or managerial accounting involves the


preparation of performance reports, cost reports, break-even
analysis, budgets and special analyses, such as, whether to
manufacture a component part internally or to purchase the
component part from an external supplier. This level of accounting
usually requires the individual to have at least a first degree
majoring in or specializing in accounting. You could become a
professional management accountant by sitting and passing the
6

Chartered Institute of Management Accountants examination.

Internal auditing involves the examination and evaluation of the


internal controls and financial reporting within the organisation
and making recommendations for their improvement. The auditing is
done by a department within the organisation, thus the term
internal auditing. The employees in the internal auditing
department usually have a first degree majoring in or specializing
in accounting. You could become a professional internal auditor by
sitting and passing the CIA (certified internal auditor)
examination. The head of the internal audit department is usually a
professional accountant with a CIA, ACCA or CPA designation.

External auditing involves the independent examination of the


financial records of an organisation for an accounting period and
an expression of an opinion on the financial statements of the
organisation for that same accounting period. The auditing is done
by a certified or chartered accountant that is not an employee,
supervisor, manager or director of the organisation, thus the term
external auditing.

Taxation involves the examination of the income and expenses of a


company or individual for an accounting period, and the application
of the relevant tax laws in estimating the company or individual’s
tax liability. The tax accountant is usually a professional
accountant with a CPA or ACCA designation.

Whichever, field of accounting you decide to go into I know that


you will find it interesting and rewarding. I always say
accountants should be conservative, consistent and ethical. Let
this be your guide as you embark on your accounting career.

COMPUTERS IN ACCOUNTING

The manual recording of transactions in books of original entry,


the posting from books of original entry to the general ledger and
subsidiary ledgers, the extraction of a trial balance, and the
preparation of financial statements can all be done electronically
with the aid of computer hardware and computer software. Hardware
includes the CPU (central processing unit), monitor, keyboard,
mouse, speakers, scanner, and printer.
7

The computer software includes the operating system (e.g. Windows)


and accounting software (e.g. QuickBooks). In addition to the
accounting software the business will need spreadsheet software
such as Microsoft Excel and word processing software such as
Microsoft Word in order to produce some accounting reports that
management may require from time to time. The software contains a
menu bar, which displays all menu options available to the user.
When you select an option from the menu bar a sub-menu with options
will be displayed.

Organizations with a large number of transactions should use an


electronic system, since the electronic system has a faster
processing speed than a manual system. Furthermore, processing
errors are more likely to occur in the manual system than in the
electronic system. The correction of errors when processing a large
number of transactions in the manual system can be very time
consuming. Organisations that require reports at regular intervals
should use an electronic system. The electronic system can produce
reports much faster than the manual system.

The use of computers will expedite the processing of business


transactions and will assist management in producing accurate
financial statements and reports on a timely basis. Multiple
reports, such as, weekly sales per salesperson, costs and profit
per region, and aged analysis of trade debtors can be produced by
entering data once. Some accounting software allows the user to
import and export data, thus preventing the need to enter data more
than once. For example, Microsoft Great Plains (an accounting
software) allows you to import or export data to Microsoft Excel (a
spreadsheet software).

The electronic system when compared to the manual system has


certain drawbacks. The electronic system will be subject to higher
maintenance costs, hardware malfunctions, software viruses and
power cuts. Furthermore, an electronic system requires an
investment in hardware that could become obsolete in a short period
of time, investment in software that may require upgrading, and
investment in training personnel to use the electronic system. If
the electronic system is connected to the Internet, hackers may
steal private and valuable information from your computer.
Furthermore, the hard drive of the computer may “crash” and the
accounting information would be lost. That is why it is necessary
to back-up this accounting information.
8

To back-up means to save information on a diskette(s), or CD


(compact disc) or DVD (digital video display). The hard drive may
“crash” because of normal wear and tear or it may “crash” because
of a virus.

A virus can spread rapidly via the Internet by e-mail (electronic


mail). E-mail is a fast and inexpensive way of communicating with
employees, customers, investors and prospective
customers/employees/investors. In order to receive and send e-mail
you need an e-mail address on the World Wide Web (WWW). Do you have
an e-mail address? Now Michael did not want an e-mail address, he
said it was too complicated. Michael lives in Jamaica and his
fiancée lives and works in the USA. It is April 30, his fiancée’s
birthday and Michael did not remember to buy and post a birthday
card. Claire was already at work and he did not know her work
number, but he remembered her saying that whenever she receives e-
mail at work her computer notifies her. He took the letter she had
sent him with her e-mail address and went to his friend’s house.
Within 20 minutes his friend assisted him in obtaining an e-mail
address and sending her an e-mail birthday card. Before leaving his
friend’s house Michael’s cell phone rang, it was Claire calling him
to tell him thanks for the e-mail. This was Michael’s reply to his
fiancée, “I wanted you to receive your birthday card on your
birthday, you are so special, and so I used the Internet.” If you
don’t have an e-mail address, go and get one. Ask someone who has
one to assist you. From Michael’s example you can see that e-mail
is fast, cost effective and convenient.

A fast, cost effective and convenient way of doing business is E-


Business (electronic business). With the aid of a website and the
right software it is possible for a manufacturer of Jerk seasoning
in Portland, Jamaica to sell this seasoning to anyone in the world
at anytime. A customer can “log on” to the Internet day or night
and purchase the seasoning. But the customer will want to know that
the website is “secure” and confidential information such as their
credit card number is safe from hackers. Purchasing encryption
software and informing your prospective customers about this
software could achieve this. Your E-Business should be integrated
with your accounting software, to ensure that proper accounting
records are kept. Your accounting software should have an E-
Business module.
9
FINANCIAL ACCOUNTING CYCLE
The financial accounting cycle is a series of sequential steps
leading to the financial statements. The cycle is repeated each
accounting period. The accounting period is the time period for
which the accounts are prepared. The time period could be one
month, six months, one year or some other time period.

FINANCIAL ACCOUNTING CYCLE FLOWCHART


FOR BUSINESSES INVOLVED IN THE BUYING AND SELLING OF GOODS

BUSINESS TRANSACTIONS

OPENING ENTRIES RECORD TRANSACTIONS IN


BOOKS OF ORIGINAL ENTRY

BALANCE SHEET

POST TO THE GENERAL


LEDGER AND SUBSIDARY
LEDGERS
STATEMENT OF CHANGES IN
EQUITY
UNADJUSTED TRIAL BALANCE

PROFIT AND LOSS ACCOUNT


ACCOUNTANT’S WORKSHEET

TRADING ACCOUNT
ADJUSTMENTS

CLOSING ENTRIES ADJUSTED TRIAL BALANCE


10

If the business enterprise is involved in manufacturing of goods


then add a rectangle with MANUFACTURING ACCOUNT between the CLOSING
ENTRIES and TRADING ACCOUNT rectangles to the flowchart on page 10.
The new flowchart would then appear as follows:

FINANCIAL ACCOUNTING CYCLE FLOWCHART


FOR BUSINESSES INVOLVED IN THE MANUFACTURING OF GOODS

BUSINESS TRANSACTIONS

OPENING ENTRIES RECORD TRANSACTIONS IN


BOOKS OF ORIGINAL ENTRY

BALANCE SHEET

POST TO THE GENERAL


LEDGER AND SUBSIDARY
STATEMENT OF CHANGES IN LEDGERS
EQUITY

UNADJUSTED TRIAL BALANCE


PROFIT AND LOSS ACCOUNT

ACCOUNTANT’S WORKSHEET
TRADING ACCOUNT

ADJUSTMENTS
MANUFACTURING ACCOUNT

ADJUSTED TRIAL BALANCE


CLOSING ENTRIES
11

If the business enterprise is strictly a service business and is


not involved in the manufacturing and/or buying and selling of
goods then delete the TRADING ACCOUNT rectangle from the flowchart
on page 10. The new flowchart would then appear as follows:

FINANCIAL ACCOUNTING CYCLE FLOWCHART


FOR BUSINESSES INVOLVED IN PROVIDING SERVICES

BUSINESS TRANSACTIONS

OPENING ENTRIES RECORD TRANSACTIONS IN


BOOKS OF ORIGINAL ENTRY

BALANCE SHEET POST TO THE GENERAL


LEDGER AND SUBSIDARY
LEDGERS

STATEMENT OF CHANGES IN UNADJUSTED TRIAL BALANCE


EQUITY

ACCOUNTANT’S WORKSHEET

PROFIT AND LOSS ACCOUNT

ADJUSTMENTS

CLOSING ENTRIES
ADJUSTED TRIAL BALANCE
12

ASSETS AND LIABILITIES

ASSETS

Assets are economic resources which are owned or controlled (assets


leased for a long term period) by a firm and are expected to
benefit future operations. Assets can be further subdivided into
non-current assets and current assets.

NON-CURRENT ASSETS

Non-current assets are economic resources owned for a period


exceeding 12 months. Non-current assets can be further subdivided
into tangible assets, intangible assets, long-term receivables, and
long-term investments. Property, plant and equipment (tangible
assets) are assets purchased by the firm to assist it in earning
its profit or to enhance its earning power, for example, plant,
machinery, land and buildings, furniture and fittings. Tangible
assets are assets purchased for use in the business and not for
resale in the short run. Tangible assets are of a permanent or
long-lived nature (useful life exceeding one year) and are written
off against profits by charging an annual amount, that is,
depreciation. The depreciation is calculated so as to eliminate the
depreciable cost (original cost less scrap value). The principle of
depreciation will be explained in detail in the chapter on
Property, Plant and Equipment. (Chapter 7)

Land, buildings, furniture, fixtures and fittings, machines,


equipment, and motor vehicles are tangible assets. Goodwill,
patents, and trademarks represent intangible assets. Goodwill
represents the excess paid for a business because of the good
reputation of the business or the good location of the business. A
patent represents the value to the business for the legal right to
make or use or sell a particular invention. A trademark represents
the value to the business for the legal right to use a symbol or
words to distinguish the goods manufactured by that business.

Long-term receivables are amounts to be collected by the business


after more than one year. An example of a long term receivable is a
car loan given to an employee by the business to be repaid over
five years. The amount to be repaid by the employee within the next
12 months after the end of the financial year will be classified as
13

a current asset (current portion of long-term receivable). The


amount to be repaid by the employee after more than one year from
the end of the financial year will be classified as a long-term
receivable.

Long-term investments are investments being held with no intention


of selling within the next 12 months from the end of the financial
year.

CURRENT ASSETS

Current assets are assets that continually change form within a 12


months period during the course of business, for example, stock,
debtors, prepayments, short-term investments, current portion of
long-term receivable, and cash.

LIABILITIES

Liabilities are debts payable by the firm. Liabilities are the


opposite of assets. Liabilities can be further subdivided into
current liabilities, non-current liabilities and owners' equity.

CURRENT LIABILITIES

Current liabilities are short-term debts payable by the firm within


a 12 months period, for example, trade creditors, corporation tax
payable, current portion of long term loan and accruals.

NON-CURRENT LIABILITIES

Non-current liabilities are long-term debts payable by the firm


after a 12 months period, for example, long-term loans, debentures
and deferred taxation. An example of a non-current liability is a
loan obtained by the business to be repaid over four years. The
amount to be paid by the business within the next 12 months after
the end of the financial year will be classified as a current
14

liability (current portion of long-term loan). The amount to be


paid by the business after more than one year from the end of the
financial year will be classified as a long-term loan and reflected
under non-current liabilities.

OWNERS' EQUITY

Owners' equity comprises the capital invested in the business by


the owners plus the reserves (capital reserves and revenue
reserves).

EXAMPLES OF ASSETS EXAMPLES OF LIABILITIES


NON-CURRENT ASSETS NON-CURRENT LIABILITIES
Equipment Long term loans
Furniture and fittings Debentures
Plant and machinery Deferred taxation
Land and buildings
Motor vehicle OWNERS' EQUITY
Long term investment Capital
Long term receivable Capital reserves
Goodwill Revenue reserves
Patent
Trademark CURRENT LIABILITIES
Creditors
CURRENT ASSETS Current portion of loan
Debtors Accruals
Current portion of receivable Corporation tax payable
Stock Income tax payable
Petty cash Sales tax payable
Cash
Prepayments
15

RECEIPTS AND PAYMENTS


In accounting the firm is considered as a separate entity.
Therefore, all transactions are recorded from the point of view of
the firm and not from the point of view of the owner or owners of
the business. Any amount paid to the business is considered as a
receipt from the business point of view. Any amount paid by the
business is considered as a payment. Payments are the opposite of
receipts. If the business operates a Chequing (Current) Account at
the bank, then payments are usually made from this account by means
of a cheque. Lodgements of cash and/or cheque to the Current
Account are considered as receipts (bank receipts) from the
business point of view.

EXAMPLES OF RECEIPTS EXAMPLES OF PAYMENTS


Owner's start-up capital Paid wages by cash
$10,000 cash Paid rent by cheque
Lodgements to Current Account Sent cheque to R. Black
B. Sun paid the business cash Withdrew cash for personal use
R. Moon paid the business by
cheque

EXPENSE AND INCOME


Expenses are costs incurred during the accounting period in order
to earn income for the organization. Income is revenue earned
during the accounting period. Income is the opposite of expense.

EXAMPLES OF EXPENSES EXAMPLES OF INCOME


Salaries Cash sales
Insurance Credit sales
Discount allowed Discount received
Electricity Interest earned
Bad debt Rental income
Travelling Ordinary dividend income
Depreciation Preference dividend income
16

FUNDAMENTAL PRINCIPLE OF DEBIT AND CREDIT


The double entry system is the cornerstone of Financial Accounting.
Every transaction is twofold, that is, every transaction involves a
debit (DR.) entry in one account and a corresponding credit (CR.)
entry in another account. In other words:

FOR EVERY DEBIT (DR.) THERE IS A CORRESPONDING CREDIT (CR.)

AND

FOR EVERY CREDIT (CR.) THERE IS A CORRESPONDING DEBIT (DR.)

An account is a record of financial transactions or financial


entries expressed in monetary terms, usually for a specific period
of time. The account is usually given a title, for example, Fax
Ltd. Account. Therefore, you would record financial transactions or
financial entries relating to Fax Ltd. in this account.

An example of an account (A/C) is given below:

DR. FAX LTD. ACCOUNT CR.






LEFT HAND DEBIT

RIGHT HAND CREDIT

Did you notice that the shape of the account resembles the letter
T? That is why an account is sometimes referred to as a T Account.

The debit is the left hand side of an account. A debit entry in an


Asset Account or an Expense Account will increase the amount in the
account. On the other hand, a debit entry in a Liability Account or
an Income Account will decrease the amount in the account.
17

The credit is the right hand side of an account. A credit entry in


a Liability Account or an Income Account will increase the amount
in the account. However, a credit entry in an Asset Account or an
Expense Account will decrease the amount in the account.

AN EXAMPLE OF DOUBLE ENTRY

If an advance of $1,000 cash is received from Fax Ltd. on January


6, 2004, then the Cash Account will be debited with $1,000 and Fax
Ltd. Account will be credited with $1,000.

DR. Cash A/C $1,000


CR. Fax Ltd. A/C $1,000

DR. CASH ACCOUNT CR.


2004 $ ║
Jan. 6 Fax Ltd. A/C 1,000 ║




DR. FAX LTD. ACCOUNT CR.


║ 2004 $
║ Jan. 6 Cash A/C 1,000




So for every debit there is a corresponding credit, and for every


credit there is a corresponding debit.
18

RULES FOR POSTING TRANSACTIONS


CASH ACCOUNT - DEBIT RECEIPTS AND CREDIT PAYMENTS

BANK ACCOUNT - DEBIT RECEIPTS AND CREDIT PAYMENTS

EXPENSE ACCOUNTS - NORMALLY HAVE DEBIT BALANCES

INCOME ACCOUNTS - NORMALLY HAVE CREDIT BALANCES

PROFIT & LOSS ACCOUNT - DEBIT EXPENSES AND CREDIT INCOME

ASSET ACCOUNTS - NORMALLY HAVE DEBIT BALANCES

LIABILITY ACCOUNTS - NORMALLY HAVE CREDIT BALANCES

NOTE:
An Allowance Account is a Liability Account, and therefore this
account has a credit balance.
19

A R E METHOD

The A R E method simplifies the basic fundamentals of Financial


Accounting into an acronym with three letters:

A
R
E

But before we expound on the acronym, let us summarize briefly what


we have learnt so far.

(1) The opposite of asset is liability.


(2) The opposite of receipt is payment.
(3) The opposite of expense is income.
(4) For every debit there is a corresponding credit.
(5) The left hand represents the debit side of an account.

Now read over the five (5) points in the above summary until you
can repeat all of them aloud without looking at this page. Do not
be afraid of repeating the five points aloud in front of a mirror,
it will only take a few minutes. If you feel like a fool standing
in front of a mirror and repeating five sentences, then please do
not worry, this is a natural feeling. If a contractor builds a
house for you, you have to ensure that the foundation is solid.
Otherwise the house will collapse. So let me build a solid
foundation for Financial Accounting for you.

DO NOT READ THE NEXT PAGE until you can repeat the five points
above without looking at the page. You have been warned.

YOU CAN DO IT. NEVER GIVE UP!


20

Do you remember the five points? Fill in the five points in pencil
below. Do not peep at the previous page.

1. ____________________________________________________

2. ____________________________________________________

3. ____________________________________________________

4. ____________________________________________________

5. ____________________________________________________

For each correct answer give yourself 20%. If your total score is
less than 100% then you should rub out the above answers and revise
the previous page before you fill in the five points again.

YOU CAN DO IT. NEVER GIVE UP!

Did you score 100%? CONGRATULATIONS. You deserve a drink, nothing


too strong now. After getting yourself a drink, proceed to the next
page.
21

Congratulations in obtaining your first 100% in Financial


Accounting. Take another sip of your drink. Now anybody that tells
you that Financial Accounting is difficult must be joking.
Financial Accounting is as easy as A R E.

A = ASSET
R = RECEIPT OF CASH
E = EXPENSE

Asset DR.
Receipt of cash DR.
Expense DR.

Left hand Right hand

DR. FAX LTD. A/C CR.





To increase an Asset Account, you must debit the account.


To increase a Cash Account when cash is received, you must debit
the account.
To increase a Bank Account when cash is lodged to the Bank Account,
you must debit the account.
To increase an Expense Account, you must debit the account.

PLEASE NOTE: In all cases above an account is debited.

Now you already know that the opposite of asset is liability,


therefore if you debit an Asset Account to increase it, you will
credit a Liability Account to increase it.

DR. Asset Account to increase it

CR. Liability Account to increase it


22

Now you already know that the opposite of receipt is payment,


therefore if you increase the Cash Account by debiting it when cash
is received, then you will credit the Cash Account when a payment
by cash is made.

DR. Cash Account with receipt of cash

CR. Cash Account with payment of cash

If you increase the Bank Account by debiting it when money is


lodged, then you will credit the Bank Account when a payment is
made by cheque.

DR. Bank Account with lodgement of money

CR. Bank Account with payment by cheque

Now you already know that the opposite of expense is income,


therefore if you debit an Expense Account to increase it, you will
credit an Income Account to increase it.

DR. Expense Account to increase it

CR. Income Account to increase it

Let us now look at the total picture the A R E way.


Left Hand = DR. Right Hand = CR.

First we start with:

_______________ ___________________

A_________________ ______ _______________ ______

R_________________ ______ _______________ ______

E_________________ ______ _______________ ______


23

Then we fill in the blanks step by step. The left hand first.

Left Hand

Asset DR.

Receipt of cash DR.

Expense DR.

The next step is the right hand let us fill in the blanks.

Right Hand
The opposite of asset is liability Liability CR.

The opposite of receipt is payment Payment CR.

The opposite of expense is income Income CR.

So we now have:

Left Hand Right Hand

Asset DR. Liability CR.


Receipt of cash DR. Payment CR.
Expense DR. Income CR.
24

Fill in the blanks in pencil.

_______________ ___________________

A_________________ ______ _______________ ______

R_________________ ______ _______________ ______

E_________________ ______ _______________ ______

Did you fill in the blanks correctly? If you did, then you are
ready to apply the A R E method. If you did not, then you need to
repeat the previous page and try again.
25

APPLICATION OF A R E METHOD
EXAMPLE NUMBER 1

Mike Goodlooking Ltd. purchased furniture for use in the office for
$60,000 by cheque from Cheapside Office Furniture Ltd. Which
account will be debited and which account will be credited in the
General Ledger?

First we identify the accounts that are affected.

Furniture Account - this is what was paid for


Bank Account - since the payment is by cheque

The next step is to identify which account is debited and which


account is credited.

The Furniture Account represents an A_____ Account (Asset


Account). So since the firm is increasing the amount of furniture
it has, we will debit the Furniture Account, and if we debit the
Furniture Account we know that we automatically credit the Bank
Account.

Furniture Account DR. $60,000

DR. FURNITURE A/C CR.


$ ║ $
Bank A/C 60,000 ║


Bank Account CR. $60,000

DR. BANK A/C CR.


$ ║ $
║ Furniture A/C 60,000



26

EXAMPLE NUMBER 2

Goods purchased on credit for resale from M. Pencil amounting to


$5,000. Which account will be debited and which account will be
credited in the General Ledger?

First we identify the accounts that are affected.

If a Subsidiary Ledger is maintained by the business then the


accounts affected in the General Ledger are:

Creditors Account - since the goods are not yet paid for
Purchases Account - since the goods are purchased for resale

The next step is to identify which account is debited and which


account is credited.

Since we know that the Creditors Account represents a L_____


Account (Liability Account), we will start with this account.
Since the firm is increasing its liability by purchasing goods on
credit we will credit the Creditors Account, and if we credit the
Creditors Account we know that we automatically debit the
Purchases Account.

Purchases Account DR. $5,000

DR. PURCHASES A/C CR.


$ ║ $
Creditors A/C 5,000 ║


Creditors Account CR. $5,000

DR. CREDITORS A/C CR.


$ ║ $
║ Purchases A/C 5,000


27

If the business did not maintain a Creditors Subsidiary Ledger,


then instead of crediting a Creditors Account (an account for all
creditors) an account for M. Pencil would be credited. So the
double entry would be:

Purchases Account DR. $5,000

DR. PURCHASES A/C CR.


$ ║ $
M. Pencil A/C 5,000 ║




M. Pencil Account CR. $5,000

DR. M. PENCIL A/C CR.


$ ║ $
║ Purchases A/C 5,000





28

EXAMPLE NUMBER 3

Paid N. Francis wages amounting to $1,000 by cash.

Fill in the blanks in pencil.

Accounts affected:

____________________________________

____________________________________

Accounts debited and credited:

_________________ DR. $_______ ______________ CR. $_______

Accounts affected:

Wages Account - this is what was paid for


Cash Account - since the payment is by cash

The Wages Account represents an E______ Account (Expense


Account). Since the firm is increasing expenses by paying wages
we will debit the Wages Account, and if we debit the Wages
Account we know that we automatically credit the Cash Account.

Wages Account DR. $1,000

DR. WAGES A/C CR.


$ ║ $
Cash A/C 1,000 ║

Cash Account CR. $1,000

DR. CASH A/C CR.


$ ║ $
║ Wages A/C 1,000

29

Congratulations, you have just completed your first lesson in


Financial Accounting. So if anyone asks you if you know Accounts,
you can say - IT IS AS EASY AS

A
R
E

Time Out
If the symbol * appears at the beginning of a review question then
the answer for this question is provided at the end of the chapter.
30

REVIEW QUESTIONS
(1) What is the financial accounting cycle?

(2) Prepare a flowchart of the financial accounting cycle.

(3) Distinguish between expense and income.

(4) Distinguish between assets and liabilities.

(5) Explain the fundamental principle of debit and credit.

(6) What are the rules for posting transactions?

(7) Define the term accounting.

(8) Distinguish between accounting and bookkeeping.

(9) Outline the nature of accounting.

(10) Explain the usefulness of accounting information to trade


unions and Governments.

(11) Distinguish between an internal auditor and an external


auditor.

(12) * What does the acronym A R E mean?


(13) * Received $2,000 cash from M. Williams, a debtor. Which
accounts are affected in the General Ledger if the
business maintains a Debtors Subsidiary Ledger?

(14) * Sold goods on credit to S. Braitwaite for $6,000. Which


account should be debited and which account should be
credited in the General Ledger:
a. If a Debtors Subsidiary Ledger (Accounts Receivable
Subsidiary Ledger) is maintained?
b. If the business does not maintain a Debtors Subsidiary
Ledger?
31

(15) * Fill in the blanks with the following words:

ASSET LIABILITY EXPENSE INCOME

(a) Travelling ___________________

(b) Depreciation ___________________

(c) Land ___________________

(d) Wages ___________________

(e) Sales ___________________

(f) Fittings ___________________

(g) Discount allowed ___________________

(h) Plant ___________________

(i) Bad debt ___________________

(j) Discount received ___________________

(16) * Fill in the blanks with debit or credit.


a. Increase in asset account ____________
b. Increase in expense account ____________
c. Increase in liability account ____________
d. Increase in income account ____________
e. Decrease in asset account ____________
f. Decrease in expense account ____________
g. Decrease in liability account ____________
h. Decrease in income account ____________
i. Increase in allowance account ____________
32

ANSWERS TO SELECTED REVIEW QUESTIONS


(12) Asset, Receipt of Cash, Expense.

(13) General Ledger Accounts affected:


Cash Account
Debtors Account (Accounts Receivable Account)
Account debited - Cash Account
Account credited - Debtors Account

(14) (a) General Ledger Accounts affected:


Account debited - Debtors Account
Account credited - Sales Account

Please NOTE that in the Debtors or Accounts Receivable


Subsidiary Ledger S. Brathwaite Account is affected.

(b) General Ledger Accounts affected:


Account debited - S. Brathwaite Account
Account credited - Sales Account

Please NOTE that the personal account of S. Brathwaite is


debited since no Debtors Subsidiary Ledger is maintained.

(15) (a) Expense


(b) Expense
(c) Asset
(d) Expense
(e) Income
(f) Asset
(g) Expense
(h) Asset
(i) Expense
(j) Income

(16) (a) Debit


(b) Debit
(c) Credit
(d) Credit
(e) Credit
(f) Credit
(g) Debit
(h) Debit
(i) Credit
33

ch2
CHAPTER TWO

CONCEPTS AND STANDARDS

CHAPTER OBJECTIVES
After completing this chapter you should be able to:

 Explain the various concepts.

 State the purpose of accounting standards.


34

CONCEPTS, CONVENTIONS AND GAAP


Over the years, academics and accountants have developed a number
of concepts, conventions, practices, and principles with the
primary objective of ensuring that there is comparability between
the accounts produced within the same firm, and those produced by
other firms. These concepts, conventions, practices and principles
are referred to as Generally Accepted Accounting Principles
(G.A.A.P). The G.A.A.P. provides the basis upon which the accounts
are prepared, so it is very important that you read this section.
This section might appear highly theoretical at first glance,
without any practical application in the preparation of the
accounts. But all the concepts are applied in recording, analyzing,
and communicating the financial information of the firm.

ENTITY CONCEPT

Under the entity concept, it is assumed that the firm is a separate


entity from its owner(s). So the financial accounts are prepared
from the point of view of the firm, and not from the point of view
of the owner(s). So in recording transactions you must record them
from the point of view of the firm. Therefore, the capital invested
by the owner will be reflected under the liability section of the
Balance Sheet, the owner being a long-term creditor of the firm.
Mike Goodlooking started business with $200,000. From the point of
view of the business it now owes Mike Goodlooking $200,000.

MONEY MEASUREMENT CONCEPT

Under the money measurement concept, it is assumed that all


expenses, income, assets and liabilities are capable of being
measured in terms of a single monetary unit. This monetary unit
could be the dollar ($), the pound (£), the yen (¥), or some other
currency. So a monetary value must be placed on a transaction
before it can be reflected in the accounts.
35

HISTORICAL COST CONCEPT

Under the historical cost concept, transactions are recorded after


they occur. The historical cost is the monetary value of a
financial transaction that occurred in the past and is recorded in
the accounting records after the transaction occurred. Therefore
the assets and liabilities in the Balance Sheet are reflected at
their historical value. A motor vehicle was purchased on June 25,
2004 for US$10,000. The accountant recorded the transaction after
the transaction occurred. The US$10,000 paid for the motor vehicle
in the past now represents the historical cost of the motor
vehicle. This concept is closely related to the next concept, the
going concern concept.

GOING CONCERN CONCEPT

Under the going concern concept, it is assumed that the firm


(entity) will continue operations in the foreseeable future and
there is no intention to sell any significant part of the firm or
to liquidate the entire firm. This explains why a motor vehicle
which was purchased some years ago for US$4,000 and appreciated in
value to US$4,500 is still reflected in the accounts at US$4,000
(the historical cost). Since it is not expected that the firm will
be liquidated in the near future there is no need to reflect the
realizable value (US$4,500) in the accounts.

STABLE MONETARY UNIT CONCEPT

Under the stable monetary unit concept, it is assumed that the


value of the monetary unit used is constant over time. The
assumption of the stability of the monetary unit overlooks the fact
that its purchasing power is constantly changing. Many years ago
one Jamaican dollar could buy you a 'decent lunch' and an icy mint.
Today because of inflation that same $1 cannot buy you one icy
mint.
36

TIME PERIOD CONCEPT

Under the time period concept, it is assumed that the financial


activities of a firm can be divided into time periods, for example,
a month, or 3 months, or 6 months, or a year. In the long run it
does not matter if you value stock at cost or realizable value, if
the method of depreciation used is straight line or reducing
balance method. In the short run all these things matter to profit
measurement. In the long run we are all dead anyway.

DOUBLE ENTRY CONCEPT OR DUALITY CONCEPT

Under the double entry concept, every transaction in the General


Ledger is twofold. In other words, for every debit there is a
corresponding credit, and for every credit there is a corresponding
debit.

MATCHING CONCEPT OR ACCRUAL CONCEPT

Under the accrual concept, costs (expenses) incurred in one


financial period should be matched against revenues produced in
that same financial period as a result of those costs. For example,
if travelling expenses were incurred in January 2004 it should be
charged against income for January 2004, irrespective of whether
the travelling expenses were paid for in January 2004.

This is a specific income matching concept example. If the interest


income earned for 2003 is $10,000 and only $9,000 is received in
2003, the interest income that is matched with expenses for 2003
will be $10,000.

PRUDENCE CONCEPT OR CONSERVATISM CONCEPT

Under the prudence or conservatism concept, income should not be


anticipated but recognized only when it is realized in the form of
cash or other asset that can be treated as cash. If goods amounting
37

to $15,000 are sold on credit to Anthony Limited, then this credit


sale will be recognized as income since Anthony Limited is now a
debtor (asset) to the firm.

All expenses and losses that have arisen or are likely to arise in
respect of the financial period should be provided for, in that
financial period. If for example, the firm is sued for negligence
in 2004 and the judge ruled in favour of the plaintiff in 2004 and
will state the amount the firm will pay in damages in the next
financial year (2005). Then the firm must estimate the amount of
damages that the judge might award the plaintiff and provide for
the cost of these estimated damages in the accounts in 2004.

CONSISTENCY CONCEPT

Under the consistency concept, the accounting treatment of like


items should be consistent within each accounting period and over
successive accounting periods. For example, the depreciation policy
of a firm states that depreciation on computer equipment should be
charged using the reducing balance method. Therefore all computer
equipment (like items), whether the computer equipment in the
managing director's office or the computer equipment used by the
janitor, will be depreciated under the reducing balance method of
depreciation, within each accounting period and over successive
accounting periods.

MATERIALITY CONCEPT

Under the materiality concept, the accounting treatment of an item


will depend on its significance. Materiality depends on the size of
the firm. A firm with an annual turnover (sales) of $60,000 (Firm
X) and a firm with an annual turnover of $100 million (Firm Y) may
treat the purchase of a calculator costing $500 differently. To
Firm X with only $60,000 annual turnover, $500 might be material
(substantial), and therefore the purchase of the calculator is
treated as capital expenditure (acquisition of a tangible non-
current asset). To Firm Y with $100 million annual turnover, $500
might be immaterial, and the purchase of the calculator will
therefore be treated as revenue expenditure (as an expense).
38

A company with a materiality level of $10,000 purchased 100


calculators at $500 each. In this case we would consider the total
cost of the calculators ($50,000) and not the individual cost of
each calculator in deciding whether the purchase is capital
expenditure or revenue expenditure. Since the $50,000 is greater
than the materiality level of $10,000 the purchase of the
calculators would be treated as a capital expenditure.

OBJECTIVITY CONCEPT

Under the objectivity concept, the accounts are prepared in such a


way as to limit the scope for subjective judgement and bias, so
that the accounts can be authenticated by independent investigators
(auditors) as giving a true and fair view of the financial position
of the firm.
39

ACCOUNTING STANDARDS
Accounting standards provide guidelines for accountants in the
preparation of financial statements. If all accountants abide by
the same guidelines within the same accounting period and over
successive accounting periods, then intra-company comparison
(comparison of the financial statements of the same firm over
successive accounting periods), and inter-company comparison
(comparison of the financial statements of different firms) will be
meaningful. Accounting standards ensure that decision-makers are
provided with credible, relevant and reliable accounting data in
order to make their decisions. Professional accounting bodies such
as the International Accounting Standards Board prepare accounting
standards.

Examples of accounting standards are given below.

SSAP 12 - Accounting for Depreciation (British Standard)


FRS 1 - Cash Flow Statements (British Standard)
SFAS 47 - Disclosure of Long Term Obligations (American Standard)
IAS 18 - Revenue (International Standard)
IFRS 3 – Business Combinations (International Standard)
IPSAS 1 – Preparation of Financial Statements (Public Sector)

Meaning of the above abbreviations:

SSAP - Statement of Standard Accounting Practice


FRS - Financial Reporting Standard
SFAS - Statement of Financial Accounting Standard
IAS - International Accounting Standard
FRS – International Financial Reporting Standard
IPSAS – International Public Sector Accounting Standard

Globalization has led to an increase in the trend towards


harmonization of international accounting standards. The accounting
body given the responsibility of harmonizing international
accounting standards is the International Accounting Standards
Board (IASB). Some countries are now replacing their local
accounting standards with international accounting standards.
40

REVIEW QUESTIONS
(1)* G.A.A.P. means G______________ A______________ A_____________
P___________________.

(2)* Under the entity concept it is assumed that the firm is a


separate _____________ from its owner(s).

(3)* Under the going concern concept it is assumed that the firm
will continue operations in the foreseeable ______________.

(4)* Under the historical cost concept transactions are recorded


after they _______________.

(5)* Under the money measurement concept it is assumed that all


expenses, income, assets and liabilities are capable of being
measured in terms of a single monetary ______________.

(6)* Under the stable monetary unit concept it is assumed that the
value of the monetary ________________ used is constant over
_______________.

(7)* Under the accrual concept costs should be matched against


_________________which are produced as a result of those
_________________.

(8)* Under the prudence concept income should not be anticipated


but recognized only when it is ________________ in the form of
__________ or other asset which can be treated as
___________.

(9)* Under the consistency concept the accounting treatment of like


_____________ should be ____________________ within each
accounting period and over _____________________ accounting
periods.

(10)* Under the materiality concept the accounting treatment of an


item will depend on its _______________________.
41

EXERCISE 1

(i) Write short notes on the following:


(a) Entity concept
(b) Historical cost concept
(c) Going concern concept
(d) Duality concept
(e) Matching concept
(f) Conservatism concept
(g) Consistency concept

(ii) What is the purpose of accounting standards?

EXERCISE 2

(i) Relate the historical cost concept to the going concern


concept.
(ii) Is the concept of materiality relevant in a developing
country?

College Students
EXERCISE 3 *

Sam Wilkins started business with $100,000 cash. Sam decided to


debit Cash Account with $100,000 and credit Revenue Account with
$100,000.

Which is the most important accounting concept in recording this


transaction? Which account would you debit and which account would
you credit to record this transaction?

EXERCISE 4 *

Mike Super owns a building on King Street, Kingston, Jamaica. On


January 1, 2003 Mike rented the building to Chen Fries Retailers,
for a rental of $30,000 per month. For the 9 months ended September
30, 2003 Mike received $240,000 from Chen Fries Retailers for rent.

Which is the most important accounting concept in recording this


transaction? How much should Mike Super record as rental income for
the 9 months ended September 30, 2003?
42

ANSWERS TO SELECTED REVIEW QUESTIONS


(1) Generally Accepted Accounting Principles.

(2) Entity

(3) Future

(4) Occur

(5) Unit

(6) Unit
Time

(7) Revenues
Costs

(8) Realized
Cash
Cash

(9) Items
Consistent
Successive

(10) Significance

Exercise 3

Entity concept

Debit Cash Account $100,000


Credit Capital Account $100,000

Exercise 4

Matching concept

$270,000 (9 X $30,000 = $270,000)


43

Ch3
CHAPTER THREE

BU SINESS ORGA NIZATIONS

CHAPTER OBJECTIVES
After completing this chapter you should be able to:

 Describe the main types of business organizations.

 Distinguish between the main types of business


organizations.

 Distinguish between a private company and a public


company.

 Identify the various documents used in business.


44

MAIN TYPES OF BUSINESS ORGANIZATIONS


The main types of business organizations are Sole Proprietorship,
General Partnership and Limited Liability Company. The distinctive
features of each type are given below.

SOLE GENERAL LIMITED LIABILITY


PROPRIETORSHIP PARTNERSHIP COMPANY

OWNERSHIP One person Two or more Usually two or more


persons persons

LIABILITY Unlimited Unlimited Limited

CAPITAL Small capital Medium capital Large capital pool


pool pool

NAME OF OWNER Sole trader Partners Shareholders

START-UP COSTS Low Low High

TOP MANAGEMENT Sole trader Partners Board of Directors

FORMATION None None Memorandum of


DOCUMENTS Association and
Articles of
Association or
Articles of
Incorporation
45

SOLE PROPRIETORSHIP OR SOLE TRADER


A sole proprietorship or sole trader type of business is the oldest
legal form of business. One person owns and operates the business,
for example, a shopkeeper. It is easy to form and requires no legal
paperwork, except for those businesses that require some form of
licence to operate, for example, a bar. Sole proprietorship varies
in size from small, medium to large employing several persons, even
though the majority is small in size.

Since the sole trader is his or her own boss, it is assumed that
the sole trader has a great deal of freedom. But individuals who
were former employees often complain that after becoming a sole
trader they only have freedom in making decisions since they do not
have to report to anyone, but their personal freedom is restricted
since they have to spend so much time running the business.
The sole trader type of business requires low start-up costs but
the owner usually finds it difficult to raise large sums of
capital. The owner of a sole trader type of business usually
provides the majority of the capital for the business. Even though
the sole trader usually contributes the majority of the capital,
the sole trader's liability is not limited to the capital that he
or she contributes but extends to the individual's personal assets,
for example, the house or car owned by the sole trader. The sole
trader is therefore said to have unlimited liability.

Legally speaking, if the sole trader dies the business is


dissolved. If a new owner operates the business after the death of
the sole trader, at the same location using the same business name,
legally speaking it is a new business with the new owner having
unlimited liability.

From a legal standpoint the business is not a separate legal entity


from the owner of the business, that is, the business and the owner
are one and the same. From an accounting standpoint, applying the
entity concept, the sole trader business is a separate economic
entity, and the accounting records are prepared from the point of
view of this economic entity and not from the point of view of the
owner of the economic entity.
46

PARTNERSHIP
A partnership is the least popular form of business. A partnership
is a business that is owned by two or more persons, for example, a
firm of accountants. The ratio in which partners share profits and
losses does not necessarily bear any relationship to the capital
that each partner contributes. For example, partner John
contributes $200,000 and partner Tom contributes $100,000, but they
can both share profits and losses equally.

Partners should ensure that a written agreement is prepared


(Articles of Co-partnership), indicating among other things -

(i) Profit and loss sharing ratio;

(ii) The rate of interest on capital, if any;

(iii) The rate of interest on drawings, if any;

(iv) The amount of salary to be paid to a partner, if any;

(v) The rate of interest to be paid on a partner's loan.

In the event of a dispute between the partners the written


agreement provides the basis for settling the dispute.

The partnership type of business, just like the sole trader type of
business, requires low start-up costs and finds it difficult to
raise large sums of capital. But a partnership business is usually
able to raise more capital than a sole proprietorship. With more
than one person operating the business more talent and capital are
available to the business. The partnership is easy to form and
requires no legal paperwork, however in the interest of all
concerned it is wise for the partners to enter into a written
agreement to reduce the number of conflicts between the partners.
It should also be noted that the individuals entering into some
partnership businesses require a licence to operate, for example,
accountants, lawyers, doctors and dentists. It is the professional
that requires the licence to operate and not the partnership
business.
47

In a general partnership all the partners have unlimited liability.


In a limited partnership at least one partner must have unlimited
liability. In a limited partnership the limited partner's liability
is limited to the capital that he or she contributes to the
partnership. The limited partner is not involved in the running of
the partnership business.

In a partnership each partner is an agent for the partnership


business even if a partner enters into a transaction on behalf of
the partnership without the knowledge of the other partner(s). Lack
of continuity exists in a partnership business because if a partner
dies, becomes bankrupt, is declared mentally insane or withdraws
from the partnership voluntarily legally the partnership is
dissolved. But the remaining partners can form a new partnership
and continue operating the business. It is difficult to transfer
ownership in a partnership since the partner wanting to retire and
transfer his or her interest to a family member, or sell his or her
interest in the partnership must receive the consent of the other
partner(s). When a transfer of ownership takes place the existing
partnership is dissolved and a new partnership is formed.

From a legal standpoint the partnership business is not a separate


legal entity from the owners of the partnership business, that is,
the business and the owners are one and the same. From an
accounting standpoint, applying the entity concept, the partnership
business is a separate economic entity, and the accounting records
are prepared from the point of view of this economic entity and not
from the point of view of the owners of the economic entity.
48

COMPANY
In some countries two documents are needed to form a company, the
Memorandum of Association and the Articles of Association. The
Memorandum of Association will include such things as the name of
the company, the authorized share capital and the objects of the
company. On the other hand, the Articles of Association will
include such things as the voting rights of shareholders, election
of directors and the number of days notice that is required for an
annual general meeting. In other words, the Memorandum of
Association governs the external affairs of the company while the
Articles of Association governs the internal affairs. In other
countries only one document is needed, the Articles of
Incorporation. This one document is in fact a combination of the
Memorandum of Association and the Articles of Association.

To form a company a Memorandum of Association and an Articles of


Association or Articles of Incorporation are submitted to the
Registrar of Companies along with the relevant declaration, the
appropriate fees and stamp duties. If the Registrar approves the
formation of the company then the Registrar issues a Certificate of
Incorporation. When the Certificate of Incorporation (birth
certificate) is issued, the company comes into existence. A company
is an artificial person with a separate existence from its owners,
and therefore the death, bankruptcy or insanity of a shareholder
does not affect the continued existence of the company. Forming a
company is more difficult than forming a sole proprietorship or
partnership because of the legal requirements. This legal
requirement makes the formation of a company very costly.

A Board of Directors is elected by shareholders at an annual


general meeting or an extra-ordinary general meeting to run the
company. Companies are usually able to raise large sums of capital.
The Companies Act (law governing the operation of companies)
requires companies to hold an annual general meeting to elect a
Board of Directors, approve the audited accounts, appoint auditors,
and pass resolutions. Companies are also required to file returns
such as the declaration of assets to the Registrar of Companies.
The extra-ordinary general meeting (emergency meeting) is usually
called to pass important and urgent resolutions, for example, to
increase the authorized share capital. If a shareholder is unable
to attend the annual or extra-ordinary general meeting to vote,
49

then this shareholder can appoint someone to vote on his or her


behalf. This is done by means of a proxy. A proxy is a legal
document authorizing someone to vote on behalf of a shareholder at
a special annual or extra-ordinary general meeting.

Since the company is a person it is subject to taxation. This


taxation is referred to as corporation tax. When the company pays
dividends to its shareholders out of its after tax profits, the
shareholders are usually required to pay tax on the dividends they
receive. That is why companies are said to be subject to double
taxation. Double taxation does not apply all the time. For example
in Jamaica in 2002 the tax on dividends for companies listed on the
Jamaica Stock Exchange was 0%.

The majority of companies formed are companies with limited


liability. So the shareholder's liability is limited to the capital
that he or she agrees to contribute and this liability does not
extend to the shareholder's personal assets. Limited liability
companies can be subdivided into private companies and public
companies. The distinctive features are given below:

PRIVATE COMPANY PUBLIC COMPANY

Shareholders can sell shares to anyone No Yes

Can be listed on stock exchange No Yes

Can issue shares to the general public No Yes

Can issue debentures to the general No Yes


public

Can advertise the issue of it's shares No Yes


or debentures to the general public
50

DOCUMENTS USED IN BUSINESS

DOCUMENT PURPOSE INFORMATION ON DOCUMENT

SALES INVOICE Sent to customer for Date, invoice number, name of


goods or services business firm, name of customer, and
sold on credit quantity, description and monetary
value of goods or services sold.

PURCHASES INVOICE Received from Date, invoice number, supplier's


suppliers for goods name, name of individual or firm
or services purchasing the goods or services,
purchased on credit and quantity, description and
monetary value of goods or services
purchased.

RECEIPT Given to customers Date, name of firm, receipt number,


to indicate the amount of money received in words
amount of money they and figures, and the name of the
pay to the firm person or firm from whom the money
was received.

CHEQUE To make payment from Date, cheque number, name of firm,


Bank Current A/C amount paid in figures and words,
name of bank on which cheque is
drawn, name of person or firm to
whom cheque is paid, and authorizing
signature(s).

DEBIT NOTE To increase the Date, name of firm, reason for the
amount owed by debit note, name of customer, and
customers monetary value of debit note.

CREDIT NOTE To decrease the Date, name of firm, reason for the
amount owed by credit note, name of customer, and
customers monetary value of credit note.

DELIVERY SLIP To inform customers Date, quantity and description of


of goods being goods delivered, name of customer,
delivered name of supplier, and signature of
person receiving the goods.

PURCHASE ORDER Used to authorize Date, quantity and description of


the purchase of goods or services needed, name of
goods or services firm, name of supplier, and the
signature of the person authorizing
the purchase of the goods or
services.
51

DOCUMENT PURPOSE INFORMATION ON DOCUMENT

STOCK REQUISITION Used to request goods Date, quantity and description


from stores or the of goods requested, name of
warehouse firm, the signature of the
person requesting the goods,
and the signature of the
person authorizing the issue
of the goods.

PROFORMA INVOICE To indicate to a Date, name of firm, quantity


prospective customer and description of goods,
the cost of goods if he monetary value of goods, and
or she decides to the name of the prospective
purchase the goods customer.

GOODS RECEIVAL NOTE To inform the Accounts Date, quantity and description
Payable Clerk of the of goods received, name of
quantity of goods firm, name of firm delivering
received goods, and the signature of
the person receiving the
goods.

CREDIT CARD To inform the credit Account number, date of


STATEMENT cardholder of the statement, due date for
transactions during the payment, transaction date,
period and the minimum description of transaction,
payment required transaction amount, credit
line limit, credit line
available, interest rate, name
and address of firm issuing
credit card, name and address
of credit cardholder.

CHEQUE REQUISITION To authorize cheque Date, cheque number, name of


VOUCHER or PAYMENT payment individual or firm to be paid
VOUCHER (payee), amount to be paid in
figures and words, purpose for
which the payment is to be
made, name of bank on which
the cheque should be drawn,
and approval signature(s).

PETTY CASH VOUCHER To authorize petty cash Date, voucher number, purpose,
expenditure signature of person receiving
the cash, and the signature of
the person approving the petty
cash expenditure.

NOTE: Name of firm refers to the firm issuing the document.


52

SAMPLES OF DOCUMENTS USED IN BUSINESS

INVOICE
J. GOLDDIGGER INVOICE NUMBER __________
10 FREETOWN AVENUE
KINGSTON 50 DATE __________________
JAMAICA, W.I.
CUSTOMER:
Telephone: 809-8888888
Fax: 809-9999999
E-mail: jgolddigger@golsom.com.jm ADDRESS:

ORDER NO: SALES PERSON: TERMS:

QUANTITY DESCRIPTION UNIT PRICE AMOUNT

MANY THANKS FOR SUB-TOTAL $


YOUR ORDER SALES TAX $
TOTAL $

When J. Golddigger sends this invoice to a customer, from J.


Golddigger's point of view this invoice is a Sales Invoice, but
from the point of view of the customer who receives this invoice it
is a Purchases Invoice. Therefore, the invoice can be either a
Sales Invoice or a Purchases Invoice depending on whether it is
being recorded in the supplier's accounting books or the customer's
accounting books.
53

The Sales Invoice is usually completed in triplicate.


ORIGINAL - sent to customer (from the customer's point of view
this represents the Purchases Invoice)
DUPLICATE - for J. Golddigger's invoice file
TRIPLICATE - for customer's file in J. Golddigger's office

RECEIPT
J. GOLDDIGGER
10 FREETOWN AVENUE
KINGSTON 50
JAMAICA, W.I.

RECEIPT NUMBER: ________________________

RECEIVED FROM: ________________________

ADDRESS: ________________________

________________________

FOR _______________________________________________________________________

_______________________________________________________________ DOLLARS

_______________________________________________________________ CENTS

┌───┐
└───┘ CASH $__________________
┌───┐
└───┘ CREDIT CARD
┌───┐
└───┘ CHEQUE

RECEIVED BY:

The Receipt is usually completed in duplicate.


ORIGINAL - sent to person or organization from whom the money was
received
DUPLICATE - retained in J. Golddigger's Receipt Book
54

CHEQUE

WILLIAMS BANK JAMAICA LIMITED


WILLIAMS PLAZA, KINGSTON, JAMAICA CHEQUE NO. ______
NO. ________

DATE ____________
DATE _______

PAYEE ______
PAY TO THE
ORDER OF ___________________________________ $___________

PURPOSE SUM OF ____________________________________ DOLLARS

_____________________
$__________
J. GOLDDIGGER _____________________

The cheque stub to the left, indicating for example the cheque
number and purpose, is retained in the Cheque Book. The two lines
at the bottom right hand corner of the cheque are for the
authorizing signatures. Depending on the instructions given to the
bank, some Chequing Accounts require only one authorizing
signature.
55

DEBIT NOTE
J. GOLDDIGGER DEBIT NOTE NUMBER__________
10 FREETOWN AVENUE
KINGSTON 50 DATE ______________________
JAMAICA, W.I.
CUSTOMER:
Telephone: 809-8888888
Fax: 809-9999999
E-mail: jgolddigger@golsom.com.jm ADDRESS:

REASON FOR DEBIT:

QUANTITY INVOICE NUMBER/DESCRIPTION UNIT PRICE AMOUNT

SUB-TOTAL $
SALES TAX $
TOTAL $

APPROVED BY: ______________________

The Debit Note is usually completed in triplicate.


ORIGINAL - sent to customer
DUPLICATE - for J. Golddigger's debit note file
TRIPLICATE - for customer's file in J. Golddigger's office
56

CREDIT NOTE
J. GOLDDIGGER CREDIT NOTE NUMBER_________
10 FREETOWN AVENUE
KINGSTON 50 DATE ______________________
JAMAICA, W.I.

CUSTOMER:
Telephone: 809-8888888
Fax: 809-9999999
E-mail: jgolddigger@golsom.com.jm ADDRESS:

REASON FOR CREDIT:

QUANTITY INVOICE NUMBER/DESCRIPTION UNIT PRICE AMOUNT

SUB-TOTAL $
APPROVED BY:________________ SALES TAX $
TOTAL $

The Credit Note is usually completed in triplicate.


ORIGINAL - sent to customer
DUPLICATE - for J. Golddigger's credit note file
TRIPLICATE - for customer's file in J. Golddigger's office
57

DELIVERY SLIP
J. GOLDDIGGER DELIVERY SLIP NUMBER______
10 FREETOWN AVENUE
KINGSTON 50 DATE _____________
JAMAICA, W.I.

CUSTOMER:
Telephone: 809-8888888
Fax: 809-9999999
E-mail: jgolddigger@golsom.com.jm ADDRESS:
PURCHASE ORDER NUMBER _______

SPECIAL DIRECTIONS FOR ADDRESS _______________________________

_______________________________

QUANTITY IDENTIFICATION NUMBER DESCRIPTION OF GOODS

RECEIVED BY:

DATE RECEIVED:

The Delivery Slip is usually completed in duplicate.


ORIGINAL - sent to customer
DUPLICATE - retained in J. Golddigger's Delivery Book
58

PURCHASE ORDER
J. GOLDDIGGER PURCHASE ORDER NUMBER _______
10 FREETOWN AVENUE
KINGSTON 50 DATE ___________
JAMAICA, W.I.
SUPPLIER:
Telephone: 809-8888888
Fax: 809-9999999
E-mail: jgolddigger@golsom.com.jm ADDRESS:

SPECIAL INSTRUCTIONS:

Date when goods are required: __________________________


Method of delivery: __________________________
Credit terms required: __________________________
Other: __________________________

QUANTITY IDENTIFICATION DESCRIPTION OF UNIT PRICE TOTAL VALUE


NUMBER GOODS/SERVICES

SUB-TOTAL $
SALES TAX $
TOTAL AMOUNT $

REQUESTED BY:
APPROVED BY:

The Purchase Order is usually completed in duplicate.


ORIGINAL - sent to supplier
DUPLICATE - retained by J. Golddigger
59

STOCK REQUISITION
TO: _______________________________

FROM: ____________________ DEPARTMENT STOCK REQUISITION NUMBER __________

DATE ______________________________

CHARGE TO: _______________________________

QUANTITY IDENTIFICATION DESCRIPTION UNIT PRICE TOTAL VALUE


NUMBER

TOTAL $

REQUESTED BY:

APPROVED BY:

The Stock Requisition is usually completed in duplicate (for


internal use only).
ORIGINAL - sent to warehouse or stores
DUPLICATE - retained in J.Golddigger's Stock Requisition Book
60

PROFORMA INVOICE
J. GOLDDIGGER PROFORMA INVOICE NUMBER _________
10 FREETOWN AVENUE
KINGSTON 50 TO: _____________________________
JAMAICA, W.I.
ADDRESS: ________________________

Telephone: 809-8888888 ________________________


Fax: 809-9999999
E-mail: jgolddigger@golsom.com.jm
DATE _______________________

QUANTITY IDENTIFICATION DESCRIPTION UNIT PRICE TOTAL VALUE


NUMBER

SUB-TOTAL $
SALES TAX $
DELIVERY CHARGES $
TOTAL $

This Proforma Invoice is valid for only 30 days.


All prices are subject to change after 30 days.
PLEASE quote the Proforma Invoice number in all communications regarding
this Proforma Invoice.

The Proforma Invoice is usually completed in duplicate.


ORIGINAL - sent to prospective customer
DUPLICATE - retained by J. Golddigger
61

GOODS RECEIVAL NOTE


J. GOLDDIGGER GOODS RECEIVAL NUMBER ___________
10 FREETOWN AVENUE
KINGSTON 50 DATE ____________________________
JAMAICA, W.I.

RECEIVED FROM: ___________________________________

QUANTITY IDENTIFICATION DESCRIPTION OF GOODS COMMENTS


NUMBER

RECEIVED BY:

The Goods Receival Note is usually completed in duplicate (for


internal use only).
ORIGINAL - sent to Accounts Payable Clerk
DUPLICATE - retained in J. Golddigger's Goods Receival Book
62

CREDIT CARD STATEMENT

ACCOUNT NUMBER NEW BALANCE MINIMUM PAYMENT DUE DATE


$ $
WILLIAMS BANK JAMAICA LIMITED J. GOLDDIGGER
WILLIAMS PLAZA, KINGSTON, JAMAICA 10 FREETOWN AVENUE
KINGSTON 50
JAMAICA, W. I.

TRANSACTION DATE DESCRIPTION AMOUNT

CREDIT LINE LIMIT $ OPENING BALANCE $


CREDIT LINE AVAILABLE $ CHARGES $
INTEREST RATE PAYMENTS $
STATEMENT DATE NEW BALANCE $

Firms sometimes obtain credit cards and give them to managers to


pay for business expenses. Credit cards can be domestic (usage
restricted to a particular country) or international (can be used
in any country).
63

CHEQUE REQUISITION VOUCHER

DATE ___________________________

CHEQUE NUMBER __________________

AMOUNT $_______________________

NAME OF BANK ___________________

PAYEE _________________________________

PURPOSE ____________________________________________________________

____________________________________________________________

____________________________________________________________

APPROVED BY: ___________________

The Cheque Requisition or Payment Voucher is for internal use only.


64

PETTY CASH VOUCHER

VOUCHER NUMBER ____________ DATE _________________

NAME OF ACCOUNT _________________________

PURPOSE ____________________________________________________________

____________________________________________________________

____________________________________________________________

RECEIVED BY: ___________________ APPROVED BY: __________________

The Petty Cash Voucher is for internal use only. It is prepared for
each payment and is numbered in consecutive order.
65

REVIEW QUESTIONS
(1)* A sole proprietorship business is owned by _________________
person.

(2)* The top management in a company is referred to as a


B________________ of D___________________.

(3)* The documents needed to form a company are the


______________________ of _____________________ and the
______________________ of _____________________.

(4)* A cheque is used to make payment from a Bank _______________


Account.

(5)* A purchase order is used to ____________________ the purchase


of goods or services.

(6)* A delivery slip is used to inform customers of goods being


________________________.

(7)* A sales invoice is sent to customers who purchase goods or


services on _________________ from the firm.

(8)* A ________________ ___________________ is received from


suppliers who sell the firm goods or services on credit.

(9)* List the advantages and disadvantages of a sole trader.

(10)* List the advantages and disadvantages of a partnership.


66

EXERCISE 1

What is the purpose of the following documents?


(a) Debit Note
(b) Credit Note
(c) Stock Requisition
(d) Proforma Invoice
(e) Purchase Order

EXERCISE 2

(a) Identify five (5) distinctive features of a private company


and five (5) distinctive features of a public company.

(b) Distinguish between a partnership and a sole


proprietorship.

College Students

Exercise 3

a. Ray Hope wants to leave his current job and start his own
business as a sole trader. Ray Hope wants to operate a bar.
Ray Hope believes that this business will be easy to form and
no legal paperwork is involved. Ray also believes that he
will have complete freedom as a sole trader. Do you agree
with Ray Hope? Explain.

b. What does the term unlimited liability means to a sole


trader?

Exercise 4*

Mary’s sole proprietorship business experienced a sales growth of


48% for 2002. She wants to convert her sole trader business into a
private company. Before making her decision Mary would like to know
the advantages and disadvantages of a private company. Identify
four (4) advantages and four (4) disadvantages for Mary.
67

ANSWERS TO SELECTED REVIEW QUESTIONS


(1) One

(2) Board, Directors

(3) Memorandum, Association


Articles, Association

(4) Current

(5) Authorize

(6) Delivered

(7) Credit

(8) Purchases Invoice

(9) Advantages
a. Easy to form
b. Freedom to make decisions
c. Low start-up costs
d. No legal paperwork

Disadvantages
a. Lack of personal freedom
b. Difficult to raise large sums of capital
c. Unlimited liability
d. Lack of continuity

(10) Advantages
a. Easy to form
b. Low start-up costs
c. More talent and capital
d. No legal paperwork

Disadvantages
a. Difficult to raise large sums of capital
b. Unlimited liability
c. Lack of continuity
d. Conflicts between partners
68

Exercise 4

Advantages
a. Continuity
b. Ability to raise large sums of capital
c. Limited liability of owners
d. Professional management

Disadvantages
a. Difficult to form
b. High star-up costs
c. Double taxation
d. Must abide by the requirements of the Companies Act
69

ch4
CHAPTER FOUR

RECORDING TRANSACTIONS IN BOOKS


OF ORIGINAL ENTRY

CHAPTER OBJECTIVES
After completing this chapter you should be able to:

 Distinguish between cash discount and trade discount.

 State the purpose of each book of original entry.

 Explain how transactions are recorded in books of original


entry.

 Record transactions in books of original entry.


70

INTRODUCTION
From the accounting cycle flowchart, it can be seen that financial
transactions are first recorded in the books of original entry and
then posted to the General Ledger. In this section we will
concentrate on recording transactions in the books of original
entry. But first let us get a clear understanding of cash discount
and trade discount.

CASH DISCOUNT
Cash discount is given for prompt payment. Two types of cash
discount exist - discount allowed and discount received. Discount
allowed is a cash discount given by the firm to customers for
prompt payment. Discount received is a cash discount obtained by
the firm from suppliers for prompt payment. The discounts allowed
and the discounts received are both reflected in the accounts.
Discount allowed is an expense and discount received is income.

The term 2 in 15 net 30 means that a 2% cash discount is allowed if


payment is made within 15 days, but the maximum credit period is 30
days.

TRADE DISCOUNT
Trade discount is given by the firm to customers who buy goods in
bulk and is given by suppliers to the firm for bulk purchases. The
trade discount is deducted from the list price (gross selling
price) and only the net price is reflected in the accounts. Trade
discount is deducted at the date of sale and does not depend on the
date of payment. So if goods with a list price of $260,000 are
purchased from Uptown Ltd., and a trade discount of $10,000 is
obtained, then only $250,000 would be reflected in the accounts,
that is GROSS SELLING PRICE - TRADE DISCOUNT
$260,000 - $10,000 = $250,000

The trade discount of $10,000 would not be reflected in the


accounts.

NOTE: Trade discount depends on the quantity purchased, while cash


discount depends on the time of payment.
71

RECORDING TRANSACTIONS

BOOKS OF ORIGINAL ENTRY TRANSACTIONS RECORDED


Sales Journal Credit sale of goods or services
Purchases Journal Credit purchases of goods or services
Returns Inward Journal Sales returns
Returns Outward Journal Purchases returns
Cash Book Cash/Bank Receipts and payments
Petty Cash Book Petty cash receipts and payments
General Journal Opening and closing entries,
accruals, prepayments, unearned
income, deferred income, correction
of errors, writing off depreciation
and bad debts, the purchase of
tangible non-current assets on
credit, and other adjustments.
72

J. Golddigger had planned to start business on March 1, 2003, but


was unable to obtain a suitable office for rent until January 1,
2004. J. Golddigger's year end date is January 31.

Enter the following transactions in the proper book of original


entry.
$
Jan. 2,2004 J. Golddigger started business with cash 50,000
Jan. 2,2004 Lodged cash to Bank Current Account 40,000
Jan. 5,2004 Paid rent by cheque (cheque #001) 10,000
Jan. 7,2004 Cash for petty cash imprest system 1,000
Jan. 7,2004 Bus fare for messenger (voucher #01) 50
Jan.10,2004 Purchased furniture on credit for the
Office from O. Thomas (invoice #95) 20,000
Jan.10,2004 Purchased goods on credit from W. Dandy
(invoice #28) 10,000
Jan.12,2004 Bus fare for messenger (voucher #02) 40
Jan.12,2004 Postage stamps (voucher #03) 20
Jan.13,2004 Sold goods on credit to B. Back
(invoice #001) 7,000
Jan.13,2004 Purchased goods on credit from R. Shandy
(invoice #500) 25,000
Quantity discount received from R. Shandy 1,000
Jan.13,2004 Purchased goods on credit from S. Pie
(invoice #851) 15,000
Jan.13,2004 Returned goods to S. Pie 1,000
Jan.14,2004 Sold goods on credit to L. Hand
(invoice #002) 14,000
Jan.14,2004 Sold goods on credit to N. Foot
(invoice #003) 16,000
Jan.18,2004 Sold goods on credit to E. Head
(invoice #004) 10,000
Jan.18,2004 Sugar, milk and coffee (voucher #04) 250
Jan.19,2004 E. Head returned goods 200
Jan.20,2004 B. Back returned goods 100
Jan.23,2004 Amount paid to W. Dandy by cheque
(cheque #002) 9,000
Discount received from W. Dandy 1,000
Jan.25,2004 B. Back paid the business cash 1,000
Jan.25,2004 Amount received from L. Hand and lodged
in Bank Current Account 13,500
Discount allowed to L. Hand 500
Jan.31,2004 Electricity unpaid for January 2004 3,000
Jan.31,2004 Paid salaries by cheque (cheque #003) 8,000
Jan.31,2004 J. Golddigger withdrew cash for personal use 1,000
73

The first step is to identify the book of original entry in which each
transaction will be recorded by using the following key.

KEY
SJ - Sales Journal
PJ - Purchases Journal
GJ - General Journal
ROJ - Returns Outward Journal
RIJ - Returns Inward Journal
CB - Cash Book
PCB - Petty Cash Book

The most important words in each transaction are underlined.


$
2/1/04 Golddigger started business with cash 50,000 CB
2/1/04 Lodged cash to Bank Current Account 40,000 CB
5/1/04 Paid rent by cheque (cheque #001) 10,000 CB
7/1/04 Cash for petty cash imprest system 1,000 CB/PCB
7/1/04 Bus fare for messenger (voucher #01) 50 PCB
10/1/04 Purchased furniture on credit for the
office from O. Thomas (invoice # 95) 20,000 GJ
10/1/04 Purchased goods on credit from W. Shandy
(invoice # 28) 10,000 PJ
12/1/04 Bus fare for messenger (voucher #02) 40 PCB
12/1/04 Postage stamps (voucher #03) 20 PCB
13/1/04 Sold goods on credit to B.Back
(invoice #001) 7,000 SJ
13/1/04 Purchased goods on credit from R.Shandy
(invoice #500) 25,000 PJ
Quantity discount received from R.Shandy 1,000 -
13/1/04 Purchased goods on credit from S.Pie
(invoice #851) 15,000 PJ
13/1/04 Returned goods to S.Pie 1,000 ROJ
14/1/04 Sold goods on credit to L. Hand
(invoice # 002) 14,000 SJ
14/1/04 Sold goods on credit to N. Foot 16,000 SJ
18/1/04 Sold goods on credit to E.Head
(invoice #004) 10,000 SJ
18/1/04 Sugar, milk and coffee (voucher #04) 250 PCB
19/1/04 E.Head returned goods 200 RIJ
20/1/04 B.Back returned goods 100 RIJ
23/1/04 Amount paid to W.Dandy by cheque
(cheque #002) 9,000 CB
Discount received from W.Dandy 1,000 CB
25/1/04 B.Back paid the business cash 1,000 CB
25/1/04 Amount received from L.Hand and lodged
in Bank Current Account 13,500 CB
Discount allowed to L.Hand 500 CB
31/1/04 Electricity unpaid for January 2004 3,000 GJ
31/1/04 Paid salaries by cheque (cheque #003) 8,000 CB
31/1/04 J.Golddigger withdrew cash for personal use 1,000 CB
74

SALES JOURNAL
All sales of goods or services on credit are recorded in the Sales
Journal (Sales Book) on a monthly basis. All transactions with SJ
will be recorded in the Sales Journal in date order and invoice
number (#) order. The sales invoices should be used by the business
in sequential order. The folio column is a reference column and
will be utilized when we are posting the transactions to the
General Ledger.

J. GOLDDIGGER SALES JOURNAL PAGE 1

DATE PARTICULARS FOLIO INVOICE # AMOUNT

2004 $
Jan. 13 B. Back 001 7,000
Jan. 14 L. Hand 002 14,000
Jan. 14 N. Foot 003 16,000
Jan. 18 E. Head 004 10,000
TOTAL CREDIT SALES FOR 47,000
JANUARY 2004
75

PURCHASES JOURNAL
All purchases of goods or services on credit are recorded in the
Purchases Journal (Purchases Book) on a monthly basis. All
transactions with PJ will be recorded in the Purchases Journal in
date order. Since the invoices will be received from different
suppliers, the invoice numbers will vary. Therefore, in recording
the invoices in the Purchases Journal only date order is important.

The gross purchase price of goods purchased from R. Shandy is


$25,000. A quantity discount (trade discount) of $1,000 was
received from R. Shandy, so only the net purchases figure of
$24,000 will be recorded in the Purchases Journal.

Net Purchases = Gross Purchase Price - Trade Discount


= $25,000 - $1,000
= $24,000

The trade discount of $1,000 will not be reflected in the


accounting books.

J. GOLDDIGGER PURCHASES JOURNAL PAGE 1

DATE PARTICULARS FOLIO INVOICE # AMOUNT

2004 $
Jan. 10 W. Dandy 28 10,000
Jan. 13 R. Shandy 500 24,000
Jan. 13 S. Pie 851 15,000
TOTAL CREDIT PURCHASES FOR 49,000
JANUARY 2004
76

RETURNS INWARD JOURNAL


All sales returns are recorded in the Returns Inward Journal on a
monthly basis. Sales returns (returns inward) represent the value
of goods returned by customers to the business. The goods may be
returned because they were not what the customers ordered or for
some other reason. All transactions with RIJ will be recorded in
the Returns Inward Journal in date order.

J. GOLDDIGGER RETURNS INWARD JOURNAL PAGE 1

DATE PARTICULARS FOLIO AMOUNT

2004 $
Jan. 19 E. Head 200
Jan. 20 B. Back 100
TOTAL RETURNS INWARD FOR JANUARY 2004 300

RETURNS OUTWARD JOURNAL


All purchases returns are recorded in the Returns Outward Journal
on a monthly basis. Purchases returns (returns outward) represent
the value of goods returned to suppliers by the business. The goods
may be returned because they are damaged or for some other reason.
All transactions with ROJ will be recorded in the Returns Outward
Journal in date order.

J. GOLDDIGGER RETURNS OUTWARD JOURNAL PAGE 1

DATE PARTICULARS FOLIO AMOUNT

2004 $
Jan. 13 S. Pie 1,000
TOTAL RETURNS OUTWARD FOR JANUARY 2004 1,000
77

CASH BOOK
CASH BOOK USED AS A BOOK OF ORIGINAL ENTRY AND ALSO AS A PART OF
THE GENERAL LEDGER

All cash and bank (Current Account) transactions will be recorded


in the Cash Book. The Cash Book can be utilized as a book of
original entry and also as part of the General Ledger. Therefore
the Cash Book will be serving two separate functions - as a book of
original entry, and as part of the General Ledger - at the same
time. All transactions with CB will be recorded in the Cash Book.
Receipts of cash or lodgements to the bank will be recorded on the
receipts side of the Cash Book. Payments by cash and payments by
cheque will be recorded on the payment side of the Cash Book. J.
Golddigger's Cash Book will be utilized as a book of original entry
and as part of the General Ledger. Since the Cash Book is also part
of the General Ledger, the receipt side of the Cash Book will
represent the debit (DR.) and the payment side of the Cash Book
will represent the credit (CR.). So in the Cash Book two accounts
exist, the Cash Account and the Bank Account. The discount allowed
and the discount received columns in the Cash Book do not represent
accounts.

The firm should use its cheque book in numeric order. Payments by
cheque should be recorded in the Cash Book in date order and
numeric order (cheque number order). The Cash Book can be ruled up
so that other details about receipts and payments are reflected.
For example, we could reflect the name of the person or firm from
which cash and cheques are received, and the name of the person or
firm to which cash and cheques are paid.

Since the business is receiving $50,000 cash from J. Golddigger the


Cash Account is debited.The lodgement of cash amounting to $40,000
to the Bank Account is twofold. The first part of the transaction
is the payment of $40,000 cash to the bank (credit Cash Account
$40,000). The second part of the transaction is the receipt of
$40,000 when the cash is lodged to the Bank Account (debit Bank
Account $40,000). Since the double entry for this transaction is
recorded in the same book (the Cash Book) it is referred to as a
contra entry. This is why a C is placed in the folio column for
this transaction. An example of J. Golddigger's Cash Book being
used as a book of original entry and also as a part of the General
Ledger is shown on the following two pages (pages 78 and 79).
78

J. GOLDDIGGER CASH BOOK

DR. RECEIPTS
DATE PARTICULARS FOLIO DISCOUNT CASH BANK
ALLOWED A/C A/C
2004 $ $
Jan. 2 Capital A/C-
J. Golddigger 50,000
Jan. 2 Cash A/C C 40,000
Jan.25 B. Back A/C 1,000
Jan.25 L. Hand A/C 500 13,500

_____ _______ _______


500 51,000 53,500
===== ======= =======
Feb. 1 Balance b/d 9,000 26,500

Please NOTE that the payment side of the Cash Book is on the
following page (page 79).

The total cash receipts for any month include any opening cash
balance. The cash balance at the end of the month is obtained by
subtracting the total cash payments from the total cash receipts.
The total cash payments can equal the total cash receipts, leaving
a nil balance. But the total cash payments can never be greater
than the total cash receipts, since you cannot pay cash that you do
not possess.

The bank balance at the end of the month is obtained by comparing


the total receipts with the total payments. Since the firm can
obtain an overdraft (can withdraw more money than is in the Current
Account) from the bank, the total payments can exceed the total
receipts. A bank overdraft is reflected as a current liability in
the Balance Sheet.
79

CR. PAYMENTS PAGE 1


DATE PARTICULARS FOLIO CHEQUE DISCOUNT CASH BANK
NUMBER RECEIVED A/C A/C
2004 $ $
Jan. 2 Bank A/C C 40,000
Jan. 5 Rent A/C 001 10,000
Jan. 7 Petty Cash A/C 1,000
Jan.23 W. Dandy A/C 002 1,000 9,000
Jan.31 Salaries A/C 003 8,000
Jan.31 Drawings A/C 1,000
Jan.31 Balance c/d 9,000 26,500
1,000 51,000 53,500
===== ====== ======
80

CASH BOOK USED AS A BOOK OF ORIGINAL ENTRY ONLY

If the Cash Book is used only as a book of original entry, then the
following will apply:

(1) The Cash Book will contain no debit or credit side.

(2) No balance, whether opening or closing, will be reflected in


the Cash Book.

(3) A Cash Account is opened in the General Ledger and not in the
Cash Book, since the Cash Book is used only as a book of
original entry and not as part of the General Ledger.

(4) A Bank Account is opened in the General Ledger and not in the
Cash Book, since the Cash Book is used only as a book of
original entry and not as part of the General Ledger.

(5) The total cash receipts in the Cash Book for the period (day,
week or month) are debited in the Cash Account in the General
Ledger and credited to the respective accounts in the General
Ledger.

DR. Cash A/C $51,000


CR. Capital A/C $50,000
CR. B. Back A/C $ 1,000

Please see receipt side of J. Golddigger Cash Book on page 84.

(6) The total cash payments in the Cash Book for the period (day,
week or month) are credited in the Cash Account in the General
Ledger and debited to the respective accounts in the General
Ledger.

CR. Cash A/C $42,000


DR. Bank A/C $40,000
DR. Petty Cash A/C $ 1,000
DR. Drawings A/C $ 1,000

Please see payment side of J. Golddigger Cash Book on page 85.


81

(7) The total bank receipts in the Cash Book for the period (day,
week or month) are debited in the Bank Account in the General
Ledger and credited to the respective accounts in the General
Ledger.
DR. Bank A/C $53,500
CR. Cash A/C $40,000
CR. L. Hand A/C $13,500

Please see receipt side of J. Golddigger Cash Book on page 82.

(8) The total bank payments in the Cash Book for the period (day,
week or month) are credited in the Bank Account in the General
Ledger and debited to the respective accounts in the General
Ledger.

CR. Bank A/C $27,000


DR. Rent A/C $10,000
DR. W. Dandy A/C $ 9,000
DR. Salaries A/C $ 8,000

Please see payment side of J. Golddigger Cash Book on page 83.

An example of J. Golddigger's Cash Book being used only as a book


of original entry is shown on the following two pages (pages 82 and
83).
82

J. GOLDDIGGER CASH BOOK

RECEIPTS -
DATE PARTICULARS FOLIO DISCOUNT CASH BANK
ALLOWED -
2004 $ $
Jan. 2 Capital A/C-
J. Golddigger 50,000
Jan. 2 Cash A/C C 40,000
Jan.25 B. Back A/C 1,000
Jan.25 L. Hand A/C 500 13,500

_____ _______ _______


500 51,000 53,500
===== ======= =======

Please NOTE that the payment side of the Cash Book is on the
following page (page 83).
83

PAYMENTS PAGE 1
DATE PARTICULARS FOLIO CHEQUE DISCOUNT CASH BANK
NUMBER RECEIVED
2004 $ $
Jan. 2 Bank A/C C 40,000
Jan. 5 Rent A/C 001 10,000
Jan. 7 Petty Cash A/C 1,000
Jan.23 W. Dandy A/C 002 1,000 9,000
Jan.31 Salaries A/C 003 8,000
Jan.31 Drawings A/C 1,000 -
1,000 42,000 27,000
===== ====== ======
84

PETTY CASH BOOK


All small item cash transactions, such as payments for bus fare and
postage, are paid through the petty cashier. The cash that the
petty cashier has available to pay for small item expenditure is
called petty cash or petty cash float. The petty cashier is
provided with a fixed sum (imprest) and is reimbursed at intervals
for the exact amount of his or her petty cash disbursements
(payments). The reimbursement to the petty cashier is usually done
by means of a cheque payment. The petty cashier then cashes the
cheque at the bank. A petty cash voucher is prepared for each
payment, and is numbered in consecutive order.

The amount of the imprest depends on the size of the business, the
nature of the business, the rate of inflation, and the frequency
with which the petty cash is utilized. A limit should be placed on
the amount that the petty cashier can pay for any individual
expenditure. Expenditure above this specified amount should not be
paid by the petty cashier. If amounts exceeding $400 should not be
paid by the petty cashier, then an expenditure of $460 for pens
must be paid by cheque.

When the imprest falls below a certain minimum level a


reimbursement is requested by the petty cashier. The imprest should
last for at least one week without being reimbursed. Frequent
reimbursement (reimbursements in excess of once per week) is an
indication that the amount of the imprest is too low (could be the
result of inflation) and, or the petty cash system is being abused
(unnecessary payments being made). An example of a petty cash
imprest system is given below.

Imprest amount $4,000


No single payment can exceed (10% of imprest) $ 400
Minimum level (twice the single payment maximum) $ 800

Therefore, if payments amounting to $3,400 are made by the petty


cashier, a reimbursement of $3,400 will be requested by the petty
cashier. The reimbursement is requested by the petty cashier
because the petty cash balance of $600 ($4,000 - $3,400) fell below
the minimum level of $800.
85

PETTY CASH BOOK USED AS A BOOK OF ORIGINAL ENTRY AND ALSO AS A PART
OF THE GENERAL LEDGER

The Petty Cash Book can be utilized as a book of original entry and
also as part of the General Ledger. Therefore the Petty Cash Book
will be serving two separate functions - as a book of original
entry, and as part of the General Ledger - at the same time.
J. Golddigger's Petty Cash Book will be utilized as a book of
original entry and as part of the General Ledger. All transactions
with PCB will be recorded in the Petty Cash Book. Receipts of
cash or cheque will be debited in the Petty Cash Book in date
order. Petty cash payments will be credited in the Petty Cash Book
in date order and voucher number order. Under particulars on the
payment side we could reflect the name of the person or the name of
the firm that received the payment, if we wish.

Each petty cash payment is recorded in both the amount column and
in the respective analysis column (for example, bus fare column).
The analysis columns do not represent accounts.

The total petty cash receipts for any month include the opening
petty cash balance. The petty cash balance at the end of the month
is obtained by subtracting the total for the amount column (on the
payment side) from the total receipts. The total for the amount
column (on the payment side) can never be greater than the total
receipts, since you cannot pay cash that you do not possess.

An example of J. Golddigger's Petty Cash Book being used as a book


of original entry and also as a part of the General Ledger is shown
on the following two pages (pages 86 and 87).
86

J. GOLDDIGGER PETTY CASH BOOK

DR. RECEIPTS
DATE PARTICULARS FOLIO CHEQUE PETTY
NUMBER CASH A/C
2004 $
Jan. 7 Cash A/C - 1,000

-
1,000
=====
Feb. 1 Balance b/d 640

Please NOTE that the payment side of the Petty Cash Book is on the
following page (page 87).
87

CR. PAYMENTS PAGE 1


DATE PARTICULARS VOUCHER PETTY BUS POSTAGE OFFICE
NUMBER CASH A/C FARE SUPPLIES
2004 $ $ $ $
Jan. 7 Messenger 01 50 50
Jan.12 Messenger 02 40 40
Jan.12 Post Office 03 20 20
Jan.18 Supermarket 04 250 250
360 90 20 250
=======================
Jan.31 Balance c/d 640
1,000
=====

PETTY CASH BOOK USED AS A BOOK OF ORIGINAL ENTRY ONLY

If the Petty Cash Book is used only as a book of original entry,


then the following will apply:
(1) The Petty Cash Book will contain no debit or credit side.

(2) No balance, whether opening or closing, will be reflected in


the Petty Cash Book.

(3) A Petty Cash Account is opened in the General Ledger and not in
the Petty Cash Book, since the Petty Cash Book is used only as
a book of original entry and not as part of the General Ledger.

(4) The double entry for the receipt side of the Petty Cash Book is
completed when we post the payment side of the Cash Book.

(5) The total petty cash payments in the Petty Cash Book for the
period (day, week or month) are credited in the Petty Cash
Account in the General Ledger and debited to the respective
accounts in the General Ledger.

CR. Petty Cash A/C $360


DR. Bus Fare A/C $ 90
DR. Postage A/C $ 20
DR. Office Supplies A/C $250

An example of J. Golddigger's Petty Cash Book being used only as a


book of original entry is shown on the following two pages (pages
88 and 89).
88

J. GOLDDIGGER PETTY CASH BOOK

RECEIPTS
DATE PARTICULARS FOLIO CHEQUE PETTY
NUMBER CASH
2004 $
Jan. 7 Cash A/C - 1,000

-
1,000
=====

Please NOTE that the payment side of the Petty Cash Book is on the
following page (page 89).
89

PAYMENTS PAGE 1
DATE PARTICULARS VOUCHER PETTY BUS POSTAGE OFFICE
NUMBER CASH FARE SUPPLIES
2004 $ $ $ $
Jan. 7 Messenger 01 50 50
Jan.12 Messenger 02 40 40
Jan.12 Post Office 03 20 20
Jan.18 Supermarket 04 250 250
360 90 20 250
====== ======================
90

GENERAL JOURNAL
All transactions with GJ will be journalized, that is, recorded in
the General Journal in date order and journal number order. The
double entry for each transaction is recorded in the General
Journal (DR. and CR.), but the General Journal is not an account.
Each journal entry is assigned a number in consecutive order. So
the first journal entry in January is assigned number 1 and the
second journal entry is assigned number 2. 2/1 means journal entry
number 2 for January. 10/6 means journal entry number 10 for June.
The journal entry must end with a narration (a description of the
transaction being recorded), and the narration must always begin
with the word Being. Just in case you are asking why - this is an
accounting convention. Since each journal entry is separate, a
total for all the debits and all the credits for January 2004 is
not necessary.

The furniture purchased on credit is recorded in the General


Journal and not the Purchases Journal, because the furniture is for
use in the office and not for resale! The electricity for January
2004 that was not paid is recorded in the General Journal, because
it is an accrued expense.

J. GOLDDIGGER GENERAL JOURNAL PAGE 1

DATE PARTICULARS NUMBER FOLIO DR. CR.

2004 $ $
Jan. 10 Furniture A/C 1/1 20,000
Other Creditors A/C 20,000
Being purchase of
furniture on credit
(invoice #95)

Jan. 31 Electricity A/C 2/1 3,000


Accruals A/C 3,000
Being electricity
payable and unpaid on
31/1/2004
91

REVIEW QUESTIONS
(1)* Cash discount is given for P_____________ P______________.

(2)* Discount allowed is an E__________________.

(3)* Trade discount is not reflected in the A_________________.

(4)* Fill in the blanks:

BOOKS OF ORIGINAL ENTRY TRANSACTIONS RECORDED

(a) Sales Journal _______________________________

(b) ______________________ Purchases returns

(c) Cash Book _______________________________

(d) ______________________ Deferred income

(e) ______________________ Sales returns

(f) ______________________ Depreciation

(g) Purchases Journal _______________________________

(h) ______________________ Accruals

(i) ______________________ Purchase of tangible non-


current asset and closing
entries
92

EXERCISE 1 *

Record the following transactions in the proper book of original


entry.
$
March 1, 2004 B.Fan started business with cash 60,000
March 1, 2004 Lodged cash to Current Account 59,000
March 2, 2004 Purchased goods by cheque 30,000
March 3, 2004 Purchased furniture with cheque 15,000
March 5, 2004 Sold goods on credit to A.Hylton 9,000
March 6, 2004 Sold goods for cash 12,000
March 7, 2004 Lodged cash to Current Account 12,000
March 10, 2004 Purchased goods on credit from R.Red 20,000
March 15, 2004 Sold goods on credit to A.Fitz 15,000
March 20, 2004 Sold goods on credit to N.Dennis 8,000
March 25, 2004 Paid telephone bill by cheque 1,000
March 25, 2004 Paid salary by cheque 5,000
March 27, 2004 Paid electricity bill by cheque 2,000
March 30, 2004 A.Hylton paid the business in cash 9,000
March 31, 2004 B.Fan withdrew cash for personal use 700

EXERCISE 2

a. Distinguish between cash discount and trade discount.

b. What is a book of original entry?

College Students

EXERCISE 3*

a. What is the meaning of the term 5 in 10 net 30?

b. List the factors you would take into consideration in


determining the amount for the petty cash float.
93

ANSWERS TO SELECTED REVIEW QUESTIONS


(1) Prompt Payment
(2) Expense
(3) Accounts

(4) (a) Credit sale of goods or services


(b) Returns Outward Journal
(c) Cash/Bank Receipts and payments
(d) General Journal
(e) Returns Inward Journal
(f) General Journal
(g) Credit purchases of goods or services
(h) General Journal
(i) General Journal

EXERCISE 1

B. FAN SALES JOURNAL

DATE PARTICULARS FOLIO INVOICE # AMOUNT

2004 $
March 5 A. Hylton 9,000
March 15 A. Fitz 15,000
March 20 N. Dennis 8,000
TOTAL CREDIT SALES FOR 32,000
MARCH 1994
94

B. FAN PURCHASES JOURNAL

DATE PARTICULARS FOLIO INVOICE # AMOUNT

1994 $
March 10 R. Red 20,000
TOTAL CREDIT PURCHASES 20,000
FOR MARCH 1994

B. FAN CASH BOOK

DR. RECEIPTS
DATE PARTICULARS FOLIO CASH A/C BANK A/C
2004 $ $
March 1 Capital A/C 60,000
March 1 Cash A/C 59,000
March 6 Cash Sales A/C 12,000
March 7 Cash A/C 12,000
March 30 A. Hylton A/C 9,000

81,000 71,000
====== ======
April 1 Balance b/d 10,000 17,300

Please NOTE that the payment side of the Cash Book is on the
following page (page 95).
95

CR. PAYMENTS
DATE PARTICULARS FOLIO CASH A/C BANK A/C
2004 $ $
March 1 Bank A/C 59,000
March 2 Cash Purchases A/C 30,000
March 3 Furniture A/C 15,000
March 7 Bank A/C 12,000
March 25 Telephone A/C 1,000
March 25 Salary A/C 5,000
March 27 Electricity A/C 2,000
March 31 Drawings A/C 700
March 31 Balance c/d 10,000 17,300
81,000 71,000
====== ======
96

Exercise 3

a. The term 5 in 10 net 30 means that a 5% cash discount is


allowed if payment is made within 10 days, but the maximum
credit period is 30 days.

b. 1. The size of the business


2. The rate of inflation
3. The frequency with which the petty cash is utilized
4. The nature of the business
97

ch5

CHAPTER FIVE

POSTING TO THE GENERAL LEDGER


AND SUBSIDIARY LEDGERS

CHAPTER OBJECTIVES
After completing this chapter you should be able to:

 Distinguish between personal accounts and impersonal


accounts.

 Develop a chart of accounts for a business.

 Post from books of original entry to the General Ledger.

 Extract a Trial Balance from the General Ledger.

 Explain when it is necessary to use a Suspense Account.

 Correct different types of errors.


98

TYPES OF ACCOUNTS
In the General Ledger two types of accounts exist, Personal
Accounts and Impersonal Accounts. Impersonal Accounts can be
further subdivided into Real Accounts and Nominal Accounts.

PERSONAL ACCOUNTS

Personal Accounts are accounts relating to transactions with


persons (for example, customer B. Proud Ltd.).

NOTE: B. Proud Ltd. is a person (an artificial person).

IMPERSONAL ACCOUNTS

Impersonal Accounts are accounts relating to tangible items and


accounts relating to income, gains, profits, expenses, losses and
costs. Impersonal Accounts can be further subdivided into Real
Accounts and Nominal Accounts.

REAL ACCOUNTS

Real Accounts are accounts relating to tangible items such as


machinery, goods and cash. The balance in these accounts is
reflected in the Balance Sheet.

NOMINAL ACCOUNTS

Nominal Accounts are accounts relating to expenses, losses, income,


profits, costs and gains (for example, Wages Account and Sales
Account).
99

EXAMPLES OF PERSONAL EXAMPLES OF IMPERSONAL ACCOUNTS


ACCOUNTS

REAL ACCOUNTS NOMINAL ACCOUNTS


CUSTOMER A/C LAND A/C RENT A/C
SUPPLIER A/C BUILDING A/C SALARIES A/C
CAPITAL A/C PLANT A/C INSURANCE A/C
STOCK A/C BAD DEBTS A/C

CHART OF ACCOUNTS
A chart of accounts is a list of ledger accounts of an accounting
entity giving the name of each account and the code for each
account (account number).

J. GOLDDIGGER GENERAL CHART OF ACCOUNTS FOR THE GENERAL LEDGER

Assets 1
Current assets 11
Non-current assets 12

Liabilities 2
Current liabilities 21
Non-current liabilities 22
Allowance 23
Owner's equity 24

Revenue 3
Sales 31
Other income 32
Trading 33
Profit and loss 34

Expenses 4
Cost of sales 41
General and administrative expenses 42
100

The general chart of accounts can be interpreted as follows:


All Asset Accounts will begin with the number 1
All Liability Accounts will begin with the number 2
All Revenue Accounts will begin with the number 3
All Expense Accounts will begin with the number 4

Any account beginning with 11 is a Current Asset Account. Any


account beginning with 22 is a Long Term Liability Account, and so
on. From the general chart of accounts it can be seen that Current
Asset Accounts must begin with 11, so the first Current Asset
Account number will be 1101, the second Current Asset Account
number 1102, and so on. The same principle will be applied to non-
current assets, current liabilities, long-term liabilities,
allowances, owner's equity, sales, other income, cost of sales, and
general and administrative expenses. Since all General and
Administrative Expense Accounts must begin with 42, the first
General and Administrative Expense Account number will be 4201, and
the second 4202, and so on. If you expect to have more than 99
General and Administrative Expense Accounts, then the first General
and Administrative Expense Account number should be 42001, and the
second 42002, and so on.
101

J. GOLDDIGGER DETAIL CHART OF ACCOUNTS FOR GENERAL LEDGER

ACCOUNT # NAME OF ACCOUNT


1101 Petty Cash Float Account
1102 Cash Account
1103 Bank Account
1104 Accounts Receivable Account
1105 Allowance for Bad Debts Account
1106 Closing Stock Account
1107 Bills Receivable Account
1108 Prepayments Account
1109 Other Receivables Account
1201 Machine Account
1202 Furniture and Fixtures Account
1203 Utility Deposit Account
2101 Accounts Payable Account
2102 Accruals Account
2103 Other Creditors Account
2201 Long Term Loan Account
2301 Allowance for Depreciation on Machine Account
2302 Allowance for Depreciation on Furniture & Fixture A/C
2401 Capital Account
2402 Drawings Account
3101 Cash Sales Account
3102 Credit Sales Account
3103 Sales Returns Account
3201 Rental Income Account
3202 Discount Received Account
3203 Profit on Disposal of Tangible Non-current Asset Account
3301 Trading Account
3401 Profit and Loss Account
4101 Cash Purchases Account
4102 Credit Purchases Account
4103 Purchases Returns Account
4104 Opening Stock Account
4201 Wages and Salaries Account
4202 Rent Expense Account
4203 Insurance Account
4204 Travelling Account
4205 Stationery Account
4206 Interest Expense Account
4207 Water Rates Account
4208 Electricity Account
4209 Discount Allowed Account
4210 Depreciation Account
4211 Postage Account
4212 Office Supplies Account
4213 Bank Charges Account
102

J. GOLDDIGGER DETAIL CHART OF ACCOUNTS FOR ACCOUNTS RECEIVABLE


(DEBTORS) SUBSIDIARY LEDGER

The Accounts Receivable Account in the General Ledger is a Control


Account (please see section on Control Accounts on pages 133 to 138
in this chapter). The Accounts Receivable Account reflects the
total for accounts receivable but not the breakdown of amounts owed
to the business by individual debtors. The individual Debtor
Account will be reflected in the Accounts Receivable Subsidiary
Ledger. The account number for accounts receivable in the detail
chart of accounts is 1104. So the first account number in the
detail chart of accounts for the Accounts Receivable Subsidiary
Ledger will be 1104001, the second account number 1104002, and so
on. The first account number is 1104001 and not 110401 because we
expect that the business will have more than 99 credit customers in
the long run. The first customer that J. Golddigger sold goods to
on credit was B. Back. So B. Back's Account number will be 1104001.

ACCOUNT # NAME OF ACCOUNT


1104001 B. Back Account
1104002 L. Hand Account
1104003 N. Foot Account
1104004 E. Head Account
103

J. GOLDDIGGER DETAIL CHART OF ACCOUNTS FOR ACCOUNTS PAYABLE


(CREDITORS) SUBSIDIARY LEDGER

The Accounts Payable Account in the General Ledger is a Control


Account. The Accounts Payable Account reflects the total for
accounts payable but not the breakdown of amounts owed to creditors
by the business. The individual Creditor Account will be reflected
in the Accounts Payable Subsidiary Ledger. The account number for
accounts payable in the detail chart of accounts is 2101. So the
first account number in the detail chart of accounts for the
Accounts Payable Subsidiary Ledger will be 210101, the second
account number 210102, and so on. The first account number is
210101 and not 2101001 because we expect to purchase goods on
credit from less than 99 suppliers. We intend to purchase goods on
credit from less than 99 suppliers because we want to build up good
credit relations with a few suppliers who deliver quality goods on
time. By developing a close relation with few suppliers we expect
to obtain the maximum credit limit and a reliable supply of goods
from these creditors. The first supplier that J. Golddigger
purchased goods on credit from was W. Dandy. So W. Dandy's Account
number will be 210101.

ACCOUNT # NAME OF ACCOUNT


210101 W. Dandy Account
210102 R. Shandy Account
210103 S. Pie Account
104

GENERAL LEDGER
From the financial accounting cycle flowchart it can be seen that
business transactions are first recorded in books of original entry
and then posted to the General Ledger and Subsidiary Ledgers. The
General Ledger is the main ledger. Both Personal and Impersonal
Accounts are kept in the General Ledger. Each account will be
assigned an account number based on the firm's chart of accounts.

If the Cash Book and the Petty Cash Book serve a dual purpose (a
book of original entry and part of the General Ledger), then no
Cash Account, no Bank Account and no Petty Cash Account will be
reflected in the General Ledger. In that case the Cash Book will
contain the Cash Account and the Bank Account, and the Petty Cash
Book will contain the Petty Cash Account. If the Cash Book and the
Petty Cash Book are utilized only as books of original entry, then
the Cash Account, the Bank Account and the Petty Cash Account will
be kept in the General Ledger.

SUBSIDIARY LEDGERS
A Subsidiary Ledger is a subset of the General Ledger. A firm can
have several Subsidiary Ledgers, but only one General Ledger.
Subsidiary Ledgers are kept in order to reduce the number of
accounts that are kept in the General Ledger and to make the
General Ledger more manageable. For example, the firm sells goods
on credit to 150 customers. Instead of keeping all the 150 Debtor
Accounts in the General Ledger the firm could create a Subsidiary
Ledger for all these debtors, and keep one account (a Control
Account) for all debtors in the General Ledger.

A Subsidiary Ledger can be created for any group of accounts (for


example, debtors, creditors, wages, salaries and property, plant &
equipment) that are kept in the General Ledger. The Subsidiary
Ledger will contain all accounts in the group, and the General
Ledger will contain one account for the entire group. The details
for each account within the group will be reflected in the
Subsidiary Ledger, and the account for the group in the General
Ledger will reflect a summary (total) of the detailed transactions
that are recorded in the Subsidiary Ledger.
105

POSTING FROM BOOKS OF ORIGINAL ENTRY


We are now ready to post the transactions that were recorded for J.
Golddigger in chapter four.

POSTING FROM THE SALES JOURNAL TO THE GENERAL LEDGER

Since an Accounts Receivable (Debtors) Subsidiary Ledger is kept,


the individual credit sale transactions are recorded in this
Subsidiary Ledger, and the total credit sales will be posted to the
General Ledger.

The individuals or firms that the business sold goods to on credit


are classified as debtors or accounts receivable. Accounts
receivable (debtor) represents an asset to the firm.

Assets DR.
R
E
If we debit the Accounts Receivable Account (Asset Account), then
we automatically credit the Credit Sales Account. The total credit
sales amounting to $47,000 is debited to Accounts Receivable
Account (A/C# 1104) and credited to the Credit Sales Account (A/C#
3102) to complete the double entry (see J. Golddigger General
Ledger on pages 113 and 117).

In the date column in the General Ledger, January 31, 2004 is


written because we are posting the total of all credit sales
transactions that occurred during the month of January 2004. In the
particulars column of the General Ledger the name of the other
account that is affected in the double entry is written. Under the
folio column in the General Ledger is written the book of original
entry and the page number from which the transaction is posted. So
on the debit side in the Accounts Receivable Account (A/C# 1104) in
the General Ledger, January 31, 2004 is recorded under the date
column, Credit Sales A/C is recorded under the particulars column
(indicating that the Credit Sales Account is credited), and SJ 1 is
recorded under the folio column (indicating that the transaction
was posted from page 1 of the Sales Journal).
106

Writing the account number in the folio column in the Sales Journal
indicates that the amount was posted and the account to which it
was posted to (see J. Golddigger Sales Journal after being posted
at the bottom of this page).

POSTING FROM THE SALES JOURNAL TO THE ACCOUNTS RECEIVABLE


SUBSIDIARY LEDGER

Since the total credit sale is debited to the Accounts Receivable


Account, the individual sums comprising this total will be debited
in the respective Debtors Account in the Accounts Receivable
Subsidiary Ledger. B. Back Account is debited with $7,000, L. Hand
Account debited with $14,000, N. Foot Account debited with $16,000,
and E. Head Account debited with $10,000 (see J. Golddigger
Accounts Receivable Subsidiary Ledger on page 118). After each
amount is posted, the account number to which the transaction was
posted is recorded in the folio column relating to that transaction
in the Sales Journal (see J. Golddigger Sales Journal after being
posted on this page, page 106). No corresponding credit entry is
needed in the Accounts Receivable Subsidiary Ledger, since all we
are recording is the detail breakdown of the $47,000 that was
debited in the Accounts Receivable Account in the General Ledger.
The double entry is done in the General Ledger and not in the
Subsidiary Ledger.

SALES JOURNAL AFTER BEING POSTED TO THE GENERAL LEDGER AND THE
ACCOUNTS RECEIVABLE SUBSIDIARY LEDGER

J. GOLDDIGGER SALES JOURNAL PAGE 1

DATE PARTICULARS FOLIO INVOICE # AMOUNT

2004 $
Jan. 13 B. Back A/C# 1104001 001 7,000
Jan. 14 L. Hand A/C# 1104002 002 14,000
Jan. 14 N. Foot A/C# 1104003 003 16,000
Jan. 18 E. Head A/C# 1104004 004 10,000
A/C# 1104 47,000
A/C# 3102
107

POSTING FROM THE PURCHASES JOURNAL TO THE GENERAL LEDGER

Since an Accounts Payable (Creditors) Subsidiary Ledger is kept,


the individual credit purchase transactions are recorded in this
Subsidiary Ledger, and the total credit purchases will be posted to
the General Ledger.

The individuals or firms from whom the business purchases goods on


credit are classified as creditors or accounts payable. Accounts
payable (creditor) represents a liability to the firm. A liability
is the opposite of asset.

Assets DR. Liability CR.


R P
E I

If we credit the Accounts Payable Account (Liability Account), then


we automatically debit the Credit Purchases Account. The total
credit purchases amounting to $49,000 is debited to the Credit
Purchases Account (A/C# 4102) and credited to the Accounts Payable
Account (A/C# 2101) to complete the double entry (see J. Golddigger
General Ledger on pages 113 and 117). On the credit side in the
Accounts Payable Account (A/C# 2101) in the General Ledger, January
31, 2004 is recorded under the date column, Credit Purchases A/C is
recorded under the particulars column (indicating that the Credit
Purchases Account is debited), and PJ 1 is recorded under the folio
column (indicating that the transaction was posted from page 1 of
the Purchases Journal).

Writing the account number in the folio column in the Purchases


Journal indicates that the amount was posted and the account to
which it was posted to (see J. Golddigger Purchases Journal after
being posted on the following page, page 108.
108

POSTING FROM THE PURCHASES JOURNAL TO THE ACCOUNTS PAYABLE


SUBSIDIARY LEDGER

Since the total credit purchase is credited to the Accounts Payable


Account, the individual sums comprising this total will be credited
in the respective Creditors Account in the Accounts Payable
Subsidiary Ledger. So W. Dandy Account will be credited with
$10,000, R. Shandy Account credited with $24,000, and S. Pie
Account credited with $15,000 (see J. Golddigger Accounts Payable
Subsidiary Ledger on page 119). After each amount is posted, the
account number to which the transaction was posted is recorded in
the folio column relating to that transaction in the Purchases
Journal (see J. Golddigger Purchases Journal after being posted on
this page, page 108). No corresponding debit entry is needed in the
Accounts Payable Subsidiary Ledger, since all we are recording is
the detail breakdown of the $49,000 that was credited in the
Accounts Payable Account in the General Ledger. The double entry is
done in the General Ledger and not in the Subsidiary Ledger.

PURCHASES JOURNAL AFTER BEING POSTED TO THE GENERAL LEDGER AND THE
ACCOUNTS PAYABLE SUBSIDIARY LEDGER

J. GOLDDIGGER PURCHASES JOURNAL PAGE 1

DATE PARTICULARS FOLIO INVOICE # AMOUNT

2004 $
Jan. 10 W. Dandy A/C# 210101 28 10,000
Jan. 13 R. Shandy A/C# 210102 500 24,000
Jan. 13 S. Pie A/C# 210103 851 15,000
A/C# 4102 49,000
A/C# 2101
109

RETURNS INWARD JOURNAL AFTER BEING POSTED TO THE GENERAL LEDGER AND
THE ACCOUNTS RECEIVABLE SUBSIDIARY LEDGER

J. GOLDDIGGER RETURNS INWARD JOURNAL PAGE 1

DATE PARTICULARS FOLIO AMOUNT

2004 $
Jan. 19 E. Head A/C# 1104004 200
Jan. 20 B. Back A/C# 1104001 100
A/C# 3103 300
A/C# 1104

RETURNS OUTWARD JOURNAL AFTER BEING POSTED TO THE GENERAL LEDGER


AND THE ACCOUNTS PAYABLE SUBSIDIARY LEDGER

J. GOLDDIGGER RETURNS OUTWARD JOURNAL PAGE 1

DATE PARTICULARS FOLIO AMOUNT

2004 $
Jan. 13 S. Pie A/C# 210103 1,000
A/C# 2101 1,000
A/C# 4103
110

CASH BOOK AFTER BEING POSTED TO THE GENERAL LEDGER

J. GOLDDIGGER CASH BOOK

DR. RECEIPTS
DATE PARTICULARS FOLIO DISCOUNT CASH BANK
ALLOWED A/C A/C
2004 $ $
Jan. 2 Capital A/C-
J. Golddigger A/C# 2401 50,000
Jan. 2 Cash A/C C 40,000
Jan.25 B. Back A/C A/C# 1104001 1,000
Jan.25 L. Hand A/C A/C# 1104002 500 13,500

-
500 51,000 53,500
=== ====== =======
Feb. 1 Balance b/d 9,000 26,500
A/C# 4209
A/C# 1104
Please NOTE that the payment side of the Cash Book is on the
following page (page 111).

PETTY CASH BOOK AFTER BEING POSTED TO THE GENERAL LEDGER

J. GOLDDIGGER PETTY CASH BOOK


DR. RECEIPTS
DATE PARTICULARS FOLIO CHEQUE PETTY
NUMBER CASH A/C
2004 $
Jan. 7 Cash A/C CB 1 - 1,000

-
1,000
=====
Feb. 1 Balance b/d 640

Please NOTE that the payment side of the Petty Cash Book is on the
following page (page 111).
111

CR. PAYMENTS PAGE 1


DATE PARTICULARS FOLIO CHEQUE DISCOUNT CASH BANK
NUMBER RECEIVED A/C A/C
2004 $ $
Jan. 2 Bank A/C C 40,000
Jan. 5 Rent A/C A/C# 4202 001 10,000
Jan. 7 Petty Cash A/C A/C# 1101 1,000
Jan.23 W. Dandy A/C A/C# 210101 002 1,000 9,000
Jan.31 Salaries A/C A/C# 4201 003 8,000
Jan.31 Drawings A/C A/C# 2402 1,000
Jan.31 Balance c/d 9,000 26,500
1,000 51,000 53,500
===== ====== ======
A/C# 2101
A/C# 3202

The discount allowed column and the discount received column are
not accounts but memorandum columns only. That is, the discount
allowed column gives a breakdown of the debtors who received a
discount during the accounting period, and the discount received
column gives a breakdown of the creditors that gave the business a
discount during the period.

CR. PAYMENTS PAGE 1


DATE PARTICULARS VOUCHER PETTY BUS POSTAGE OFFICE
NUMBER CASH A/C FARE SUPPLIES
2004 $ $ $ $
Jan. 7 Messenger 01 50 50
Jan.12 Messenger 02 40 40
Jan.12 Post Office 03 20 20
Jan.18 Supermarket 04 250 250
360 90 20 250
=======================
Jan.31 Balance c/d 640 A/C# A/C# A/C#
1,000 4204 4211 4212
=====
112

GENERAL JOURNAL AFTER BEING POSTED TO THE GENERAL LEDGER

J. GOLDDIGGER GENERAL JOURNAL PAGE 1

DATE PARTICULARS NUMBER FOLIO DR. CR.

2004 $ $
Jan 10 Furniture A/C 1/1 A/C# 1202 20,000
Other Creditors A/C A/C# 2103 20,000
Being purchase of
furniture on credit
(invoice #95)

Jan 31 Electricity A/C 2/1 A/C# 4208 3,000


Accruals A/C A/C# 2102 3,000
Being electricity
payable and unpaid
on 31/1/2004
113

J. GOLDDIGGER GENERAL LEDGER

DR. ACCOUNTS RECEIVABLE A/C (A/C# 1104) CR.


DATE PARTICULARS FOLIO AMOUNT DATE PARTICULARS FOLI0 AMOUNT
2004 $ ║ 2004 $
Jan.31 Credit Sales A/C SJ 1 47,000 ║ Jan.31 Sales Returns
║ A/C RIJ 1 300
║ Jan.31 Discount
║ Allowed A/C CB 1 500
║ Jan.31 Cash A/C CB 1 1,000
║ Jan.31 Bank A/C CB 1 13,500
║ Jan.31 Balance c/d 31,700
47,000 ║ 47,000
====== ║ ======

Feb. 1 Balance b/d 31,700 ║

DR. FURNITURE AND FIXTURES ACCOUNT (A/C# 1202) CR.


DATE PARTICULARS FOLIO AMOUNT DATE PARTICULARS FOLI0 AMOUNT
2004 $ ║ 2004 $
Jan.10 Other ║
Creditors A/C GJ 1 20,000 ║ Jan.31 Balance c/d 20,000
====== ║ =======

Feb. 1 Balance b/d 20,000 ║

DR. ACCOUNTS PAYABLE A/C (A/C# 2101) CR.


DATE PARTICULARS FOLIO AMONT DATE PARTICULARS FOLI0 AMOUNT
2004 $ ║ 2004 $
Jan.31 Purchases ║ Jan.31 Credit
Returns A/C ROJ 1 1,000 ║ Purchases A/C PJ 1 49,000
Jan.31 Discount ║
Received A/C CB 1 1,000 ║
Jan.31 Bank A/C CB 1 9,000 ║
Jan.31 Balance c/d 38,000 ║
49,000 ║ 49,000
====== ║ ======

║ Feb. 1 Balance b/d 38,000

114

DR. ACCRUALS A/C (A/C# 2102) CR.


DATE PARTICULARS FOLIO AMOUNT DATE PARTICULARS FOLI0 AMOUNT
2004 $ ║ 2004 $
Jan.31 Balance c/d 3,000║ Jan.31 Electricity A/C GJ 1 3,000
=====║ =====

║ Feb. 1 Balance b/d 3,000

DR. OTHER CREDITORS ACCOUNT (A/C# 2103) CR.


DATE PARTICULARS FOLIO AMOUNT DATE PARTICULARS FOLI0 AMOUNT
2004 $ ║ 2004 $
Jan.31 Balance c/d 20,000║ Jan.10 Furniture A/C GJ 1 20,000
======║ ======

║ Feb. 1 Balance b/d 20,000

DR. CAPITAL A/C (A/C# 2401) CR.


DATE PARTICULARS FOLIO AMOUNT DATE PARTICULARS FOLI0 AMOUNT
2004 $ ║ 2004 $
Jan.31 Balance c/d 50,000║ Jan. 2 Cash A/C CB 1 50,000
======║ ======

║ Feb. 1 Balance b/d 50,000

DR. DRAWINGS A/C (A/C# 2402) CR.


DATE PARTICULARS FOLIO AMOUNT DATE PARTICULARS FOLI0 AMOUNT
2004 $ ║ 2004 $
Jan.31 Cash A/C CB 1 1,000 ║ Jan.31 Balance c/d 1,000
===== ║ =====

Feb. 1 Balance b/d 1,000 ║

115

DR. CREDIT SALES A/C (A/C# 3102) CR.


DATE PARTICULARS FOLIO AMOUNT DATE PARTICULARS FOLI0 AMOUNT
2004 $ ║ 2004 $
║ Jan.31 Accounts
Jan.31 Balance c/d 47,000 ║ Receivable A/C SJ 1 47,000
====== ║ ======

║ Feb. 1 Balance b/d 47,000

DR. SALES RETURNS A/C (A/C# 3103) CR.


DATE PARTICULARS FOLIO AMOUNT DATE PARTICULARS FOLI0 AMOUNT
2004 $ ║ 2004 $
Jan.31 Accounts ║
Receivable A/C RIJ 1 300 ║ Jan.31 Balance c/d 300
=== ║ ===

Feb. 1 Balance b/d 300 ║

DR. DISCOUNT RECEIVED A/C (A/C# 3202) CR.


DATE PARTICULARS FOLIO AMOUNT DATE PARTICULARS FOLI0 AMOUNT
2004 $ ║ 2004 $
║ Jan.31 Accounts
Jan.31 Balance c/d 1,000 ║ Payable A/C CB 1 1,000
===== ║ =====

║ Feb. 1 Balance b/d 1,000

DR. CREDIT PURCHASES A/C (A/C# 4102) CR.


DATE PARTICULARS FOLIO AMOUNT DATE PARTICULARS FOLI0 AMOUNT
2004 $ ║ 2004 $
Jan.31 Accounts ║
Payable A/C PJ 1 49,000 ║ Jan.31 Balance c/d 49,000
====== ║ ======

Feb. 1 Balance b/d 49,000 ║

116

DR. PURCHASES RETURNS A/C (A/C# 4103) CR.


DATE PARTICULARS FOLIO AMOUNT DATE PARTICULARS FOLI0 AMOUNT
2004 $ ║ 2004 $
║ Jan.31 Accounts
Jan.31 Balance c/d 1,000 ║ Payable A/C ROJ 1 1,000
===== ║ =====

║ Feb. 1 Balance b/d 1,000

DR. WAGES AND SALARIES A/C (A/C# 4201) CR.


DATE PARTICULARS FOLIO AMOUNT DATE PARTICULARS FOLI0 AMOUNT
2004 $ ║ 2004 $
Jan.31 Bank A/C CB 1 8,000 ║ Jan.31 Balance c/d 8,000
===== ║ =====

Feb. 1 Balance b/d 8,000 ║

DR. RENT EXPENSE A/C (A/C# 4202) CR.


DATE PARTICULARS FOLIO AMOUNT DATE PARTICULARS FOLI0 AMOUNT
2004 $ ║ 2004 $
Jan. 5 Bank A/C CB 1 10,000 ║ Jan.31 Balance c/d 10,000
====== ║ ======

Feb. 1 Balance b/d 10,000 ║

DR. TRAVELLING A/C (A/C# 4204) CR.


DATE PARTICULARS FOLIO AMOUNT DATE PARTICULARS FOLI0 AMOUNT
2004 $ ║ 2004 $
Jan.31 Petty Cash A/C PCB 1 90 ║ Jan.31 Balance c/d 90
== ║ ==

Feb. 1 Balance b/d 90 ║

117

DR. ELECTRICITY A/C (A/C# 4208) CR.


DATE PARTICULARS FOLIO AMOUNT DATE PARTICULARS FOLI0 AMOUNT
2004 $ ║ 2004 $
Jan.31 Accrued A/C GJ 1 3,000 ║ Jan.31 Balance c/d 3,000
===== ║ =====

Feb. 1 Balance b/d 3,000 ║

DR. DISCOUNT ALLOWED A/C (A/C# 4209) CR.


DATE PARTICULARS FOLIO AMOUNT DATE PARTICULARS FOLI0 AMOUNT
2004 $ ║ 2004 $
Jan.31 Accounts ║
Receivable A/C CB 1 500 ║ Jan.31 Balance c/d 500
=== ║ ===

Feb. 1 Balance b/d 500 ║

DR. POSTAGE A/C (A/C# 4211) CR.


DATE PARTICULARS FOLIO AMOUNT DATE PARTICULARS FOLI0 AMOUNT
2004 $ ║ 2004 $
Jan.31 Petty Cash A/C PCB 1 20 ║ Jan.31 Balance c/d 20
== ║ ==

Feb. 1 Balance b/d 20 ║

DR. OFFICE SUPPLIES A/C (A/C# 4212) CR.


DATE PARTICULARS FOLIO AMOUNT DATE PARTICULARS FOLI0 AMOUNT
2004 $ ║ 2004 $
Jan.31 Petty Cash A/C PCB 1 250 ║ Jan.31 Balance c/d 250
=== ║ ===

Feb. 1 Balance b/d 250 ║

118

J. GOLDDIGGER ACCOUNTS RECEIVABLE SUBSIDIARY LEDGER


DR. B. BACK A/C (A/C# 1104001) CR.
DATE PARTICULARS FOLIO AMOUNT DATE PARTICULARS FOLI0 AMOUNT
2004 $ ║ 2004 $
Jan.13 Credit Sales A/C SJ 1 7,000 ║ Jan.20 Returns Inward
║ A/C RIJ 1 100
║ Jan.25 Cash A/C CB 1 1,000
║ Jan.31 Balance c/d 5,900
7,000 ║ 7,000
===== ║ =====

Feb. 1 Balance b/d 5,900 ║

DR. L. HAND A/C (A/C# 1104002) CR.


DATE PARTICULARS FOLIO AMOUNT DATE PARTICULARS FOLI0 AMOUNT
2004 $ ║ 2004 $
Jan.14 Credit Sales A/C SJ 1 14,000 ║ Jan.25 Bank A/C CB 1 13,500
║ Jan.25 Discount
║ Allowed A/C CB 1 500
14,000 ║ 14,000
====== ║ ======

DR. N. FOOT A/C (A/C# 1104003) CR.


DATE PARTICULARS FOLIO AMOUNT DATE PARTICULARS FOLI0 AMOUNT
2004 $ ║ 2004 $
Jan.14 Credit Sales A/C SJ 1 16,000║ Jan.31 Balance c/d 16,000
======║ ======

Feb. 1 Balance b/d 16,000║

DR. E. HEAD A/C (A/C# 1104004) CR.


DATE PARTICULARS FOLIO AMOUNT DATE PARTICULARS FOLI0 AMOUNT
2004 $ ║ 2004 $
Jan.18 Credit Sales A/C SJ 1 10,000 ║ Jan.19 Returns
║ Inward A/C RIJ 1 200
║ Jan.31 Balance c/d 9,800
10,000 ║ 10,000
====== ║ ======

Feb. 1 Balance b/d 9,800 ║

119

J. GOLDDIGGER ACCOUNTS PAYABLE SUBSIDIARY LEDGER


DR. W. DANDY A/C (A/C# 210101) CR.
DATE PARTICULARS FOLIO AMOUNT DATE PARTICULARS FOLI0 AMOUNT
2004 $ ║ 2004 $
Jan.23 Bank A/C CB 1 9,000 ║ Jan.10 Credit
Jan.23 Discount ║ Purchases A/C PJ 1 10,000
Received A/C CB 1 1,000 ║
10,000 ║ 10,000
====== ║ ======

DR. R. SHANDY A/C (A/C# 210102) CR.


DATE PARTICULARS FOLIO AMOUNT DATE PARTICULARS FOLI0 AMOUNT
2004 $ ║ 2004 $
║ Jan.13 Credit
Jan.31 Balance c/d 24,000║ Purchases A/C PJ 1 24,000
======║ ======

║ Feb. 1 Balance b/d 24,000

DR. S. PIE A/C (A/C# 210103) CR.


DATE PARTICULARS FOLIO AMOUNT DATE PARTICULARS FOLI0 AMOUNT
2004 $ ║ 2004 $
Jan.13 Returns ║ Jan.13 Credit
Outward A/C ROJ 1 1,000 ║ Purchases A/C PJ 1 15,000
Jan.31 Balance c/d 14,000 ║
15,000 ║ 15,000
====== ║ ======

║ Feb. 1 Balance b/d 14,000

120

BALANCING ACCOUNTS

If the value of the debit entries in an account is greater than the


value of the credit entries in that same account for the accounting
period (usually a month), then the balance in the account is a
debit balance (see E. Head A/C on page 118). This account is closed
at the end of the accounting period by crediting the account. This
credit entry is reflected in the account as balance c/d (carried
down) or as balance c/f (carried forward). This balance c/d or
balance c/f is referred to as the closing balance.

If the value of the credit entries in an account is greater than


the value of the debit entries in that same account for the
accounting period, then the balance in the account is a credit
balance (see S. Pie A/C on page 119). This account is closed at the
end of the accounting period by debiting the account. This debit
entry is reflected in the account as balance c/d or as balance c/f.
This balance c/d or balance c/f is referred to as the closing
balance.

The term balance c/f is used in an account at the end of the


financial year or if the account must be continued on a new page.
The first entry in the account in the next financial year or on the
new page is referred to as balance b/f (brought forward). This
first entry is the opening entry or opening balance.

The term balance c/d is used if the account will be continued on


the same page in the next accounting period. The first entry in the
account in the next accounting period is referred to as balance b/d
(brought down). This first entry is the opening entry or opening
balance in the new accounting period.

The accounting period or financial period is usually one month, but


this period could be one week, three months, six months or some
other time period less than twelve months. The financial year
usually refers to a twelve months period.
121

TRIAL BALANCE
A Trial Balance is a list of all accounts with debit and credit
balances in the General Ledger at the end of the accounting period,
usually in account number order.

Uses of the Trial Balance


1. To provide information on the balances in the accounts at
the end of the accounting period. For example if you wanted
to know the amount for the total debtors on December 31,
2003. The trial balance on December 31, 2003 will provide
you with that information.

2. To provide information from which the income statement and


balance sheet can be prepared.

3. To check the accuracy of the addition of the debit balances


and the addition of the credit balances.

4. To check the accuracy of the extraction of the accounts


from the ledger.

5. To check the accuracy in balancing accounts in the ledger.

6. To check the accuracy in posting from the books of original


entry to the ledger.

Please remember that the Cash Book and the Petty Cash Book are
usually utilized as a book of original entry and as part of the
General Ledger. Therefore the balance in the Petty Cash A/C (in the
Petty Cash Book), and the balances in the Cash A/C and the Bank A/C
(in the Cash Book), should be reflected in the Trial Balance. The
Trial Balance is an extraction of all accounts in the General
Ledger with debit and credit balances at the end of the accounting
period. The Unadjusted Trial Balance is the Trial Balance before
any adjustments (adjusting entries) are made.
122

J. GOLDDIGGER TRIAL BALANCE ON JANUARY 31,2004


ACCOUNT # ACCOUNT NAME DR. CR.
$ $
1101 Petty Cash Float A/C 640
1102 Cash A/C 9,000
1103 Bank A/C 26,500
1104 Accounts Receivable A/C 31,700
1202 Furniture and Fixtures A/C 20,000
2101 Accounts Payable A/C 38,000
2102 Accruals A/C 3,000
2103 Other Creditors A/C 20,000
2401 Capital A/C 50,000
2402 Drawings A/C 1,000
3102 Credit Sales A/C 47,000
3103 Sales Returns A/C 300
3202 Discount Received A/C 1,000
4102 Credit Purchases A/C 49,000
4103 Purchases Returns A/C 1,000
4201 Wages and Salaries A/C 8,000
4202 Rent Expense A/C 10,000
4204 Travelling A/C 90
4208 Electricity A/C 3,000
4209 Discount Allowed A/C 500
4211 Postage A/C 20
4212 Office Supplies A/C 250
160,000 160,000
======= =======
123

ERRORS THAT DO NOT AFFECT THE BALANCING OF


THE TRIAL BALANCE

ERROR OF OMISSION

An error of omission occurs when the debit and credit entries of a


transaction are omitted from the accounts. For example, $5,000 paid
by cheque for furniture to be used in the business is omitted from
the debit of the Furniture Account and the credit of the Bank
Account.

ERROR OF COMMISSION

An error of commission occurs when an amount is posted to the wrong


account of the same class. For example, if $1,000 cash is received
from B. Brown, a debtor, and is credited instead to J. Brown's
Account, another debtor.

ERROR OF PRINCIPLE

An error of principle occurs when an amount is posted to the wrong


class of account. For example, if $1,500 is paid for routine
repairs to machinery, an expense, and is posted to the Machinery
Account (a Tangible Non-current Asset Account) instead of the
Repairs to Machinery Account (an Expense Account).

COMPENSATING ERROR

A compensating error occurs when errors that are made cancel each
other out. For example, the Wages Account is debited with $2,500
instead of $2,000 (a debit overcast of $500) and the Sales Account
is credited with $4,500 instead of $4,000 (a credit overcast of
$500). The debit overcast of $500 cancels the credit overcast of
$500.
124

ERROR IN ORIGINAL ENTRY

An error in original entry occurs when a transaction is recorded


incorrectly in a book of original entry. For example, goods
amounting to $500 were sold on credit to J. Jack and $5,000 was
recorded in the Sales Journal instead.

ERRORS THAT AFFECT THE BALANCING OF THE TRIAL BALANCE


ONE SIDED ERROR

A one sided error occurs when the double entry is incomplete. For
example, $1,000 cash is paid for wages and the Cash Account is
credited but the Wages Account is not debited, so the double entry
is incomplete.

DOUBLE SIDED ERROR

A double sided error occurs when an amount is posted to the wrong


side of the account. For example, goods amounting to $10,000 are
sold on credit and the Sales Account is debited instead of being
credited.

ERROR IN AMOUNT

An error in amount occurs when a different amount is debited from


what is credited. For example, if $10,000 is paid by cheque for
furniture and the Furniture Account is debited with $11,000 and the
Bank Account is credited with $10,000.

ERROR IN ADDITION

An error in addition occurs when the Trial Balance is incorrectly


added.
125

ERROR IN EXTRACTION

An error in extraction occurs when the balance of an account in the


Trial Balance that is extracted from the General Ledger disagrees
with the balance in the same account in the General Ledger. For
example, the balance in the General Ledger for the Salaries Account
is $600,000 and the amount is extracted as $6,000.

SUSPENSE ACCOUNT

If the Trial Balance does not balance at the end of the accounting
period, and the accountant is unable to locate the errors
immediately, then the difference in the Trial Balance is placed in
a Suspense Account until the errors are located. The extent of the
search for the errors will depend on the materiality of the error
or errors.

If the debit balances are greater than the credit balances in the
Trial Balance at the end of the accounting period then the Suspense
Account will be credited with the difference.

DR. CR.
$ $
Bank A/C 100,000
Capital A/C 98,000
Suspense A/C 2,000
100,000 100,000
======= =======

If the credit balances are greater than the debit balances in the
Trial Balance at the end of the accounting period then the Suspense
Account will be debited with the difference.

DR. CR.
$ $
Bank A/C 151,000
Capital A/C 155,000
Suspense A/C 4,000
155,000 155,000
======= =======
126

CORRECTION OF ERRORS
Errors are corrected by means of journal entries. Errors that do
not affect the balancing of the Trial Balance (error of omission,
error of commission, error of principle, compensating error and
error in original entry) are corrected by debiting one account and
crediting another account.

If the trial balance does not balance, you should investigate


the reason(s) for the difference. The following steps will
assist you in your investigations. If one step fails to provide
you with the reason why the difference exists, move on to the
other step. Sometimes also, more than one mistake was made,
requiring you to go through more than one step in order to
determine the reasons for the difference.

The first step is to ensure that the additions of the debit


balances and the additions of the credit balances in the trial
balance are accurate.

Determine the difference between the total debits and the total
credits and see if a transaction that was posted has the same
amount. For example, if the trial balance is out by $520 and
wages for $520 were paid, then you would check to see if the
$520 was correctly posted.

If you cannot locate a transaction with the same amount as the


trial balance difference, you should divide the difference by 2
and then see if a transaction that was posted has that amount.
Let us continue to assume that the difference in the trial
balance is $520. Divide $520 by 2 ($520/2 = $260) and then see
if any of the transactions that were posted has an amount of
$260, and then check to see if this transaction was correctly
posted. The reason why we are dividing by 2 is because if an
amount is posted on the wrong side of the account (e.g. an item
debited when it should have been credited) this will result in
the error being doubled.

If a difference still exists in the trial balance then we should


ensure that all account balances were correctly extracted from
the general ledger. That is, the correct amount was extracted,
and the amount was on the correct side of the trial balance.
127

If a difference still exists, ensure that all accounts were


correctly balanced. If all of those steps do not solve the
problem, then you need to check the posting of each transaction
to ensure that each transaction was correctly debited and
credited.

One sided error, double sided error and error in amount are
subsequently corrected when located by debiting the Suspense
Account and crediting another account, or crediting the Suspense
Account and debiting another account. It should be noted that in
correcting a double sided error, the amount of the error should be
doubled in order to correct the error. If $4,000 is debited in the
Rental Income Account instead of being credited, then to correct
this error $8,000 must be credited to the Rental Income Account and
$8,000 debited in the Suspense Account.

An error in addition is corrected by adding the Trial Balance once


again using the correct figures. An error in extraction is
corrected by comparing the General Ledger balance in each account
with the corresponding balance in each account in the Trial
Balance. The balance in any account in the Trial Balance that was
extracted incorrectly is then changed to agree with the balance for
that account in the General Ledger.

At the end of the financial year, if a material balance remains in


the Suspense Account, then the accountant must search for and
correct the error or errors in order to eliminate the Suspense
Account. If the accountant is unable to locate all the errors, but
the balance in the Suspense Account is reduced to an immaterial
amount, then this balance will be reflected in the Profit and Loss
Account. If an immaterial debit balance remains at the end of the
financial year in the Suspense Account it will be added to an
Expense Account, and if an immaterial credit balance remains at the
end of the financial year in the Suspense Account it will be added
to an Income Account.

After the preparation of the Trial Balance on July 1, 2004 for the
financial year ended June 30, 2004, the credit side exceeded the
debit side by $86,000. In checking the addition of the Trial
Balance the accountant realized that the addition of the credit
side was overcast by $1,000 because the Rental Income Account was
added as $91,000 instead of $90,000. The accountant checked the
extraction of the figures from the General Ledger and found that
128

the balance in the Sales Account was extracted as $655,000 instead


of 650,000. The accountant made the necessary corrections. What is
the difference in the Trial Balance after the accountant corrected
the above errors?

The first error is an error in addition amounting to $1,000 and the


second error is an error in extraction amounting to $5,000.

Difference in Trial Balance after correction of errors


Original difference $86,000
Less overcast in addition $ 1,000
$85,000
Less overcast in extraction $ 5,000
$80,000
=======

The accountant worked until 10:00 p.m. Friday night, July 1, 2004,
but was unable to locate any other errors. The difference of
$80,000 (DR.) was placed in a Suspense Account.

Subsequent investigations during the week ending July 8, 2004


revealed the following errors:

(1) $4,800 received from a debtor was credited in the Debtors


Control Account (Accounts Receivable Account) but was not
entered in the Bank Account.

(2) Rental expense amounting to $30,000 was credited to the Rent


Expense Account (expense treated as revenue income).

(3) A payment of $3,000 to S. Stink was credited in S. Stink's


Account.

(4) Additional capital amounting $9,800 was debited in the Bank


Account but was credited in the Capital Account as $18,900.

(5) Wages amounting $500 was omitted from the Cash Account and the
Wages Account.

(6) Travelling expense amounting to $7,000 was debited to the


Insurance Expense Account instead of being debited to the
Travelling Expense Account.
129

(7) Repair to the furniture amounting to $200 was debited to the


Furniture Account instead of the Repairs to Furniture Account.

(8) The Sales Account for May 2004 was overcast by $10,000 and the
Purchases Account for May 2004 was overcast by $10,000.

(9) $19,500 paid for electricity was credited in the Bank Account
in the Cash Book as $19,000 and debited in the Electricity
Account as $19,000.

The materiality level of the firm is $300.

REQUIRED:
(a) Prepare the necessary journal entries to correct the errors
(narration not required).
(b) Prepare the Suspense Account.

The first error is a One Sided Error since the double entry is
incomplete. The correction of this error will affect the Suspense
Account.

The second error is a Double Sided Error since the Rent Expense
Account was credited instead of being debited. To correct this
error the figure of $30,000 must be doubled to $60,000. The
correction of this error will affect the Suspense Account.

The third error is a Double Sided Error since the amount should
have been debited to S. Stink Account instead of being credited. To
correct this error the figure of $3,000 must be doubled to $6,000.
The correction of this error will affect the Suspense Account.

The fourth error is an Error in Amount since the amount debited was
different from the amount that was credited. The correction of this
error (difference of $9,100) will affect the Suspense Account.

The fifth error is an Error of Omission since the debit and credit
entries are omitted from the accounts. The correction of this error
will not affect the Suspense Account.

The sixth error is an Error of Commission since the amount is


posted to the wrong account of the same class. The Travelling
Account and the Insurance Account are both Expense Accounts. The
correction of this error will not affect the Suspense Account.
The seventh error is an Error 0f Principle since the amount is
130

posted to the wrong class of account. The amount was posted to the
Furniture Account (Tangible Non-current Asset Account), instead of
the Repairs to Furniture Account (Expense Account). The correction
of this error will not affect the Suspense Account.

The eighth error is a Compensating Error since the overcast of the


Sales Account ($10,000 CR.) will cancel out the overcast of the
Purchases Account ($10,000 DR.). The correction of this error will
not affect the Suspense Account.

The ninth error is an Error in Original Entry since the payment for
electricity was incorrectly recorded in the Cash Book. The
correction of this error (difference of $500) will not affect the
Suspense account.

Since no narration is required all we need to write is:


Being ...
131

GENERAL JOURNAL FOR JUNE 2004

DATE PARTICULARS FOLIO DR. CR.

2004 $ $
June 30 Bank A/C 4,800
Suspense A/C 4,800
Being ...
June 30 Rent Expense A/C 60,000
Suspense A/C 60,000
Being ...
June 30 S. Stink A/C 6,000
Suspense A/C 6,000
Being ...
June 30 Capital A/C 9,100
Suspense A/C 9,100
Being ...
June 30 Wages A/C 500
Cash A/C 500
Being ...
June 30 Travelling Expense A/C 7,000
Insurance Expense A/C 7,000
Being ...
June 30 Repairs to Furniture A/C 200
Furniture A/C 200
Being ...
June 30 Sales A/C 10,000
Purchases A/C 10,000
Being ...
June 30 Electricity A/C 500
Bank A/C 500
Being ...
132

DR. SUSPENSE ACCOUNT CR.


DATE PARTICULARS AMOUNT DATE PARTICULARS AMOUNT
2004 $ ║ 2004 $
June 30 Balance b/f 80,000║ June 30 Bank A/C 4,800
║ June 30 Rent Expense A/C 60,000
║ June 30 S. Stink A/C 6,000
║ June 30 Capital A/C 9,100

Total Debit in the Suspense A/C = $80,000

Total Credits in the Suspense A/C = $4800 + $60000 + $6000 + $9100


= $79,900

So the Debit exceeds the Credits by $100 ($80,000 - $79,900)

The materiality level for the firm is $300 therefore any amount
less than $300 is considered immaterial. The balance of $100 in the
Suspense Account is less than $300 and is therefore immaterial.
Since this is a debit balance it can be transferred to an Expense
Account. Let us transfer this amount to the Travelling Account.
This is first recorded in the General Journal and then posted to
the General Ledger. The double entry is:

DR. Travelling A/C $100


CR. Suspense A/C $100

DR. SUSPENSE ACCOUNT CR.


DATE PARTICULARS AMOUNT DATE PARTICULARS AMOUNT
2004 $ ║ 2004 $
June 30 Balance b/f 80,000 ║ June 30 Bank A/C 4,800
║ June 30 Rent Expense A/C 60,000
║ June 30 S. Stink A/C 6,000
║ June 30 Capital A/C 9,100
║ June 30 Travelling A/C 100
80,000 ║ 80,000
====== ║ ======

133

CONTROL ACCOUNTS
Control accounts are kept in order to reduce the number of accounts
in the general ledger and make the general ledger more manageable.
Control accounts are also used to deter fraud through segregation
of duties. A total can be extracted quickly from the control
account. No need for example, to wait until all individual debtors
accounts are added up in order to obtain the total debtors balance.
The use of control accounts makes it easier to detect and correct
errors in the general ledger.

Control Accounts are kept only in the General Ledger. Only totals
are posted to the Control Account, while the individual amounts are
posted to the Subsidiary Ledger for that Control Account. For
example, the total credit sales in the Sales Book (Sales Journal)
would be posted to the Debtors Control Account while the sales to
individual debtors would be posted in the Debtors Subsidiary Ledger
in the respective Debtors Account. The Subsidiary Ledger is not
part of the General Ledger, but the closing balance in the Control
Account should be equal to the summation of the individual closing
balances in the Subsidiary Ledger. If the Control Account closing
balance does not agree with the summation of the individual closing
balances in the Subsidiary Ledger then a reconciliation similar to
a Bank Reconciliation would be done to determine the reason(s) for
the difference. (See Accounts Receivable Account in J. Golddigger's
General Ledger on page 113 and also the individual Debtors Accounts
in J. Golddigger's Accounts Receivable Subsidiary Ledger on page
118).

Some individuals are familiar with only two types of Control


Accounts, the Debtors Control Account (Accounts Receivable Account)
and the Creditors Control Account (Accounts Payable Account). But
the Wages Account, Salaries Account, Motor Vehicle Account,
Machinery Account, Furniture and Fittings Account, and Plant
Account can also be Control Accounts.
134

CONTROL ACCOUNTS SUBSIDIARY LEDGERS


Debtors A/C Debtors Subsidiary Ledger
Creditors A/C Creditors Subsidiary Ledger
Wages A/C Wages Book
Salaries A/C Salaries Book
Motor Vehicle A/C Property, Plant & Equipment Register

The Wages Book, the Salaries Book and the Property, Plant &
Equipment Register could be in the form of a computer printout. The
total salaries paid would be reflected in the Salaries Account in
the General Ledger while the salary paid to each employee would be
reflected in the Subsidiary Ledger (Salaries Book).

The total cost for machines owned by the business will be reflected
in the Machinery Account in the General Ledger, while the cost for
each machine owned by the business would be reflected in the
Subsidiary Ledger (Property, Plant & Equipment Register).

Let us now look at the items that we would expect to see in the
Debtors Account (Debtors Control Account) and the Creditors Account
(Creditors Control Account).

DR. DEBTORS CONTROL A/C CR.


Balance b/f ║ Bank A/C (receipts from debtors)
Credit Sales A/C ║ Cash A/C (receipts from debtors)
Bank A/C (returned cheques) ║ Sales Returns A/C
║ Discount Allowed A/C
║ Creditors Control A/C (set-off)
║ Bills Receivable A/C
║ Bad Debt A/C
║ Balance c/f



135

DR. CREDITOR CONTROL A/C CR.


Cash A/C (payments to creditors) ║ Balance b/f
Bank A/C (payments to creditors) ║ Credit Purchases A/C
Purchases Returns A/C ║ Bank A/C (returned cheques)
Discount Received A/C ║
Debtors Control A/C (set-off) ║
Bills Payable A/C ║
Balance c/f ║

If the business (Globe Ltd.) purchases goods on credit from


Backdoor Ltd. for $7000 and sells goods on credit to Backdoor Ltd.
amounting to $5000, then Backdoor Ltd. is both a debtor and
creditor of Globe Ltd. Instead of Globe Ltd. paying Backdoor Ltd.
$7000 and Backdoor Ltd. paying Globe Ltd. $5000, we could set-off
$5000. Globe Ltd. would then only pay Backdoor Ltd. $2000. The
lower figure is always the amount used to set-off (liquidate) the
debt.

From the following information prepare the Debtors Control Account


and the Creditors Control Account.
$
Opening balances June 1, 2004
Debtors Control A/C 120,000
Creditors Control A/C 30,000
Credit purchases 260,000
Cash purchases 20,000
Credit sales 400,000
Sales returns 5,000
Purchases returns 1,000
Cheques paid to creditors 130,000
Cheques received from debtors 270,000
Set-off of balances in the Sales Ledger
and Purchases Ledger 10,000
Bad debt 2,000
Discount allowed 4,000
Discount received 1,000
Closing balances June 30, 2004
Debtors Control A/C ?
Creditors Control A/C ?
136

DR. DEBTORS CONTROL A/C CR.


$ ║ $
Balance b/f 120,000 ║ Sales Returns A/C 5,000
Credit Sales A/C 400,000 ║ Bank A/C 270,000
║ Creditors Control A/C 10,000
║ Bad Debt A/C 2,000
║ Discount Allowed A/C 4,000
║ Balance c/f 229,000
520,000 ║ 520,000
======= ║ =======

DR. CREDITORS CONTROL A/C CR.


$ ║ $
Purchases Returns A/C 1,000 ║ Balance b/f 30,000
Bank A/C 130,000 ║ Credit Purchases A/C 260,000
Debtors Control A/C 10,000 ║
Discount Received A/C 1,000 ║
Balance c/f 148,000 ║
290,000 ║ 290,000
======= ║ =======

Please NOTE that cash purchases are not reflected in the Creditors
Control Account since only credit purchases affect this account.

It should be noted that the Debtors Control Account could contain


both a debit opening balance and a credit opening balance at the
same time. If a debtor overpays his or her account at the end of
the accounting period for example, then this Debtor's Account will
contain an opening credit balance at the beginning of the
accounting period.

DR. DEBTORS CONTROL A/C CR.


$ ║ $
Balance b/f ║ Balance b/f




137

Also the Creditors Control Account can contain both an opening


credit balance as well as an opening debit balance at the same
time. If the business overpays a Creditor's Account at the end of
the accounting period for example, then this Creditor's Account
will contain an opening debit balance at the beginning of the
accounting period.

DR. CREDITORS CONTROL A/C CR.


$ ║ $
Balance b/f ║ Balance b/f




It should be pointed out at this stage that in this textbook the


T Account format is used. The T Account format is used because I
find that students understand the double entry process much better
when this format is used. However, in several businesses the
columnar account format is used instead of the T Account format.

COLUMNAR ACCOUNT FORMAT EXAMPLES

WAGES ACCOUNT

DATE PARTICULARS FOLIO DR. CR. BALANCE


138

DEBTORS CONTROL ACCOUNT

DATE PARTICULARS FOLIO DR. CR. BALANCE

The Debtors Control Account is an Asset Account and therefore the


normal balance is a debit balance. If for any reason the Debtors
Control Account balance is a credit balance then the figure in the
balance column would be bracketed. In other words, if the balance
is not a normal balance then it should be bracketed. This principle
applies to all Asset Accounts, Liability Accounts, Income Accounts
and Expense Accounts. So in an Income Account for example, the
normal balance is a credit balance. If the balance in this Income
Account for any reason is a debit balance then this balance would
be bracketed, indicating that it is a debit balance.
139

REVIEW QUESTIONS
(1) Distinguish between Personal Accounts and Impersonal Accounts.

(2) Write short notes on the following:


Nominal Accounts
Chart of Accounts
Accounts Receivable Subsidiary Ledger
Accounts Payable Subsidiary Ledger
Trial Balance
Suspense Account

(3) Distinguish between Subsidiary Ledgers and the General Ledger.

(4) What is the difference between a one sided error and a double
sided error?

(5)* An error of ___________________ occurs when an amount is

posted to the wrong account of the same class.

(6)* A ________________ ________________ occurs when errors that

are made cancel each other out.

EXERCISE 1 *

Prepare the journal entries to correct the following errors for


Soup Ltd. for the financial year ended March 31, 2004.

(1) The total of the discount allowed column in the Cash Book
amounting to $6,500 was incorrectly credited in the Discount
Received Account.

(2) The purchase of office furniture for use in the business


amounting to $60,000 was incorrectly posted to the Purchases
Account.

(3) The total of the Sales Book was undercast by $10,000. Soup Ltd.
maintains a Sales Ledger.

(4) Payment for water rates amounting to $7,000 was correctly


entered in the Bank Account in the Cash Book but was not posted
to the Water Rates Account.
140

EXERCISE 2 *

After the preparation of the Trial Balance for Longshot Ltd. on


August 31, 2004, an unexpected difference remains, and a Suspense
Account is opened for that amount. Subsequent investigations
revealed all the errors eliminating the Suspense Account balance.
The errors discovered were:

(a) Office supplies purchased amounting to $2,500 was recorded in


the Purchases Account.

(b) The total discount received amounting to $9,500 was debited in


error to the Discount Received Account.

(c) A payment of $25,000 for cash purchases was recorded in the


Bank Account in the Cash Book, but was not posted to the Cash
Purchases Account.

(d) Interest expense amounting to $2,000 was treated as interest


income in the Interest Expense Account.

(e) Repair to machinery amounting to $2,000 was debited to the


Machinery Account instead of the Repairs to Machinery Account.

REQUIRED: Prepare the Suspense Account.


141

ANSWERS TO SELECTED REVIEW QUESTIONS


(5) Commission

(6) Compensating Error

EXERCISE 1

SOUP LTD. GENERAL JOURNAL

DATE PARTICULARS FOLIO DR. CR.

2004 $ $
March 31 Discount Received A/C 6,500
Discount Allowed A/C 6,500
Suspense A/C 13,000
Being correction of discount
allowed which was
incorrectly credited in the
Discount Received A/C.
March 31 Office Furniture A/C 60,000
Purchases A/C 60,000
Being correction of asset
purchased that was
incorrectly debited in the
Purchases A/C.
March 31 Debtors A/C 10,000
Credit Sales A/C 10,000
Being correction of the
undercast in the Sales Book.
March 31 Water Rates A/C 7,000
Suspense A/C 7,000
Being correction of an
amount, which was omitted
from the Water Rates
Account.
142

EXERCISE 2

DR. SUSPENSE ACCOUNT CR.


$ ║ $
Balance b/f ? ║ Balance b/f ?
Discount Received A/C 19,000 ║ Cash Purchases A/C 25,000
║ Interest Expense A/C 4,000

Since all the errors were discovered and eliminated


(Total Debit = Total Credit), then the opening balance in the
Suspense Account must have been:

? = [($25,000 CR. + $4,000 CR.) - $19,000 DR.] = $10,000 DR.

DR. SUSPENSE ACCOUNT CR.


$ ║ $
Balance b/f 10,000 ║ Cash Purchases A/C 25,000
Discount Received A/C 19,000 ║ Interest Expense A/C 4,000
29,000 ║ 29,000
====== ║ ======

143

ch6

CHAPTER SIX

ADJUSTMENTS

CHAPTER OBJECTIVES
After completing this chapter you should be able to:

 Distinguish between accrued expenses and prepayments.

 Distinguish between a specific allowance for bad debts and


a general allowance for bad debts.

 Distinguish between accrued income and unearned income.

 Distinguish between mark-up and margin.


144

INTRODUCTION
When the Trial Balance is extracted from the General Ledger at the
end of the accounting period it is referred to as an Unadjusted
Trial Balance. This Unadjusted Trial Balance is then adjusted to
arrive at the Adjusted Trial Balance. The adjustments made to the
Unadjusted Trial Balance refer to information received after the
preparation of the Unadjusted Trial Balance that relate to the
accounting period under review, and the correction of errors.
Adjustments include financial transactions relating to the
accounting period that were not taken into consideration, such as,
accrued expenses, prepayments, bad debts, allowance for bad debts,
allowance for discount on debtors, allowance for unrealized profit,
accrued income, unearned income, the purchase of goods for use in
the business, goods sent on a sale or return basis, drawings,
depreciation, allowance for depreciation, disposal of non-current
assets. If the item is already correctly reflected in the
Unadjusted Trial Balance then this item will not be an adjustment.
If for example, all bad debts are written off before the Unadjusted
Trial Balance is extracted, then no adjustment will be made for bad
debts.

Depreciation, allowance for depreciation and the disposal of assets


will be discussed in chapter seven. The other adjustments starting
with accrued expenses will be discussed in this chapter.

ACCRUED EXPENSES
An expense that is incurred during the accounting period and is not
paid for by the end of that same accounting period is referred to
as an accrued expense. If the rental expense for January 2004 is
$10,000 and the $10,000 is not paid by the end of January 2004,
then this unpaid amount must be accrued. The accrued expense is a
liability at the end of the accounting period, and is reflected in
the Balance Sheet under current liabilities.

Asset DR. Liability CR.


R
E
145

The unpaid amount is accrued by crediting the Accrued Expense


Account with $10,000 and debiting the Rental Expense Account with
$10,000. The firm's financial year is January 2004 to December
2004. The accrued expense is first recorded in a book of original
entry (the General Journal) and then posted to the General Ledger
Accounts (Rent Account and Accrued Expense Account).

DR. RENT ACCOUNT CR.


2004 $ ║ 2004 $
Jan.31 Accrued ║
Expense A/C 10,000 ║ Jan.31 Balance c/d 10,000
====== ║ ======

Feb. 1 Balance b/d 10,000 ║

All Expense Accounts and Income Accounts are closed and transferred
to the Profit and Loss Account at the end of the financial year. In
the Profit and Loss Account expenses are debited and income
credited. At the end of the financial year the debit balance in the
Rent Account will be transferred to the debit side of the Profit
and Loss Account.

DR. ACCRUED EXPENSE ACCOUNT CR.


2004 $ ║ 2004 $
Jan.31 Balance c/d 10,000 ║ Jan.31 Rent A/C 10,000
====== ║ ======

║ Feb. 1 Balance b/d 10,000

The $10,000 accrued will be reflected in the Balance Sheet at the


end of January 2004 as an accrued expense under current
liabilities. Any amount subsequently paid for January 2004 rental,
up to a maximum of $10,000, will be debited in the Accrued Expense
Account and not the Rent Account. A maximum of $10,000 is stated
because if $11,000 is paid in February 2004 for January 2004
146

rental, only $10,000 will be debited to the Accrued Expense


Account. The difference of $1,000 representing a retroactive
increase in the rental charge will be debited in the Rent Account,
since it was not originally accrued.

If an expense is incurred during the accounting period and part of


that expense is unpaid at the end of that same accounting period,
then the unpaid amount will be accrued. If the rental expense for
January 2004 is $10,000 and only $6,000 is paid by the end of
January 2004, then the unpaid amount of $4,000 will be accrued. The
unpaid amount is accrued by increasing (debiting) the Rental
Expense Account with $4,000 and increasing (crediting) the Accrued
Expense Account with $4,000. The $6,000 was paid on January 28,
2004 by cheque. The firm's financial year is January 2004 to
December 2004. The accrued expense is first recorded in the General
Journal and then posted to the General Ledger.

DR. RENT ACCOUNT CR.


2004 $ ║ 2004 $
Jan.28 Bank A/C 6,000 ║ Jan.31 Balance c/d 10,000
Jan.31 Accrued ║
Expense A/C 4,000 ║
10,000 ║ 10,000
====== ║ ======

Feb. 1 Balance b/d 10,000 ║

DR. ACCRUED EXPENSE ACCOUNT CR.


2004 $ ║ 2004 $
Jan.31 Balance c/d 4,000 ║ Jan.31 Rent A/C 4,000
===== ║ =====

║ Feb. 1 Balance b/d 4,000

The $4,000 accrued will be reflected in the Balance Sheet at the


end of January 2004 as an accrued expense under current
liabilities. Any amount subsequently paid for January 2004 rental,
up to a maximum of $4,000, will be debited in the Accrued Expense
Account and not the Rent Account.
147

Suppose $14,000 was paid by cheque on February 10, 2004 for rent,
broken down as follows:

January 2004 rent $ 4,000


February 2004 rent $10,000
$14,000
=======

The $4,000 will be debited in the Accrued Expense Account and the
$10,000 will be debited in the Rent Account.

DR. RENT ACCOUNT CR.


2004 $ ║ 2004 $
Jan.28 Bank A/C 6,000 ║ Jan.31 Balance c/d 10,000
Jan.31 Accrued ║
Expense A/C 4,000 ║
10,000 ║ 10,000
====== ║ ======

Feb. 1 Balance b/d 10,000 ║ Feb.28 Balance c/d 20,000
Feb.10 Bank A/C 10,000 ║
20,000 ║ 20,000
====== ║ ======

Mar. 1 Balance b/d 20,000 ║

DR. ACCRUED EXPENSE ACCOUNT CR.


2004 $ ║ 2004 $
Jan.31 Balance c/d 4,000 ║ Jan.31 Rent A/C 4,000
===== ║ =====

Feb.10 Bank A/C 4,000 ║ Feb. 1 Balance b/d 4,000
===== ║ =====

148

Instead of having one Accrued Expense Account, the firm could have
a separate Accrued Expense Account for each expense. For example,
an Accrued Expense Account for rent, one for travelling, and so on.

PREPAYMENTS
An expense that relates to a future accounting period and is paid
for before the beginning of this future accounting period is
referred to as a prepayment. If the insurance for January 2004 to
December 2004 is $24,000 and it is paid during January 2004, then
at the end of January 2004 the insurance paid for February 2004 to
December 2004 (11 months) will be a prepayment. The prepayment
represents an asset to the firm at the end of the accounting
period, and is reflected in the Balance Sheet under current assets.

Asset DR.
R
E

Therefore in order to record the prepayment, we debit the


Prepayment (Asset) Account and credit the Expense Account with the
amount prepaid. Let us assume that the insurance was paid by cheque
on January 25, 2004 and the firm's financial year end is December
31, 2004.

The insurance prepaid at the end of January 2004


= $24,000 x 11/12
= $22,000
149

The prepayment would be reflected in the accounts as follows at the


end of January 2004.

DR. INSURANCE ACCOUNT CR.


2004 $ ║ 2004 $
Jan.25 Bank A/C 24,000 ║ Jan.31 Prepayment A/C 22,000
║ Jan.31 Balance c/d 2,000
24,000 ║ 24,000
====== ║ ======

Feb. 1 Balance b/d 2,000 ║

At the end of the financial year the debit balance in the Insurance
Account will be transferred to the debit side of the Profit and
Loss Account.

DR. PREPAYMENT ACCOUNT CR.


2004 $ ║ 2004 $
Jan.31 Insurance A/C 22,000 ║ Jan.31 Balance c/d 22,000
====== ║ ======

Feb. 1 Balance b/d 22,000 ║

The prepayment is recorded by debiting the Prepayment Account with


$22,000 and crediting the Insurance Account with $22,000. The
prepayment is first recorded in a book of original entry (the
General Journal) and then posted to the General Ledger Accounts
(Insurance Account and Prepayment Account). The prepayment of
$22,000 will be reflected in the Balance Sheet at the end of
January 2004 as a prepayment under current assets.

At the end of February 2004 the prepayment will be reduced to 10


months (March 2004 to December 2004).

Prepayment (unexpired cost) = $24,000 x 10/12 = $20,000


150

Expired cost for February 2004 = $24,000 x 1/12 = $ 2,000


Since the insurance for February 2004 is $2,000, the Insurance
Account (Expense Account) will be debited with $2,000 and the
Prepayment Account will be credited with $2,000. This entry is
first be recorded in the General Journal and then posted to the
General Ledger. The accounts would then appear as follows:

DR. INSURANCE ACCOUNT CR.


2004 $ ║ 2004 $
Jan.25 Bank A/C 24,000 ║ Jan.31 Prepayment A/C 22,000
║ Jan.31 Balance c/d 2,000
24,000 ║ 24,000
====== ║ ======

Feb. 1 Balance b/d 2,000 ║ Feb.28 Balance c/d 4,000
Feb.28 Prepayment A/C 2,000 ║
4,000 ║ 4,000
===== ║ =====

Mar. 1 Balance b/d 4,000 ║

DR. PREPAYMENT ACCOUNT CR.


2004 $ ║ 2004 $
Jan.31 Insurance A/C 22,000 ║ Jan.31 Balance c/d 22,000
====== ║ ======

Feb. 1 Balance b/d 22,000 ║ Feb.28 Insurance A/C 2,000
║ Feb.28 Balance b/d 20,000
22,000 ║ 22,000
====== ║ ======

Mar. 1 Balance b/d 20,000 ║

The prepayment of $20,000 will be reflected as a current asset in


the Balance Sheet at the end of February 2004. Instead of one
Prepayment Account, the firm could have a separate Prepayment
Account for each expense. For example, a Prepayment Account for
rent, one for travelling, and so on.
151

BAD DEBTS
If after making several efforts over an extended period to collect
an amount owing to the firm, by writing, telephoning, personal
visits, etcetera, then the firm will write off the debt if it is
reasonable certain that the debtor will never pay the firm. This
debt that is written off by the firm is referred to as a bad debt.

For example, M. Samfie owes the firm $10,000 and the firm is unable
to collect the amount from M. Samfie after one year. Letters,
telephone calls and telegrams all fail to get any response from M.
Samfie. Information reaching the firm indicates that M. Samfie had
migrated to the United States of America. Since it is reasonable
certain that M. Samfie will never pay the firm, M. Samfie's debt to
the firm becomes a bad debt. To write off the debt we debit the Bad
Debts Account (Expense Account) in the General Ledger with $10,000
and credit the Accounts Receivable Account in the General Ledger
with $10,000. The double entry will therefore be:
DR. Bad Debts Account $10,000
CR. Accounts Receivable Account $10,000

M. Samfie Account in the Accounts Receivable Subsidiary Ledger will


be credited with $10,000. This is not part of the double entry. To
write off any bad debt the entry must first be recorded in the
General Journal and then posted to the General Ledger (Bad Debts
Account and Accounts Receivable Account), and the individual
Debtor's Account in the Debtors Subsidiary Ledger. The debit
balance in the Bad Debts Account (Expense Account) at the end of
the financial year is transferred to the Profit and Loss Account.

If the business does not maintain a Subsidiary Ledger for Debtors


then an individual account for each debtor will be kept in the
General Ledger, and no Accounts Receivable Account (Debtors Control
Account) will be kept in the General Ledger. If the business does
not maintain a Subsidiary Ledger for Debtors, then M. Samfie
Account will be kept in the General Ledger. In that case the double
entry to write off the bad debt would be to debit the Bad Debts
Account with $10,000 and to credit M. Samfie Account with $10,000.
DR. Bad Debts Account $10,000
CR. M. Samfie Account $10,000
152

When an allowance for bad debt account is maintained, the bad debt
is written off in the allowance account and not the Profit and Loss
Account. Since the allowance was created in the event of a bad
debt, the bad debt is written off against the allowance account.

Where a subsidiary ledger is maintained, the accounting entry to


write off the bad debt will be:

Dr. Allowance for Bad debts Account $10,000


Cr. Accounts Receivable Account $10,000

M. Samfie Account in the Accounts Receivable Subsidiary Ledger will


be credited with $10,000.

Where a subsidiary ledger is not maintained, the accounting entry


to write off the bad debt will be:

Dr. Allowance for Bad debts Account $10,000


Cr. M. Samfie Account $10,000

If a debt is written off as a bad debt, and the debtor subsequently


pays then the amount received will be reflected in a Bad Debts
Recovered Account (an Income Account). For example, M. Samfie
subsequently pays the firm $10,000 cash. Then the double entry will
be:
CR. Bad Debts Recovered A/C $10,000
DR. Cash A/C $10,000
153

ALLOWANCE FOR BAD DEBTS

An allowance for bad debt (formerly provision for bad debt) is


usually made for debts (amounts owed to the firm), the recovery of
which is doubtful. The allowance can be a specific allowance or a
general allowance. Let us assume that the firm's Accounts
Receivable Account contains a balance of $100,000, broken down as
follows:
A. Flat $ 10,000
T. Loop $ 24,000
L. Scoop $ 15,000
F. Fool $ 30,000
J. Sand $ 21,000
$100,000
========

If it is doubtful that the firm will ever collect from A. Flat and
L. Scoop, then a specific allowance can be created for $10,000 +
$15,000 = $25,000. The specific allowance is based on specific
debts owed by A. Flat $10,000 and L. Scoop $15,000.

Instead of a specific allowance the firm could create a general


allowance. The general allowance is based on a percentage of the
total debts outstanding ($100,000), and not on any specific debt.
If historically 10% of the total debts prove to be bad, then the
allowance will be 10% of $100,000 that is, $10,000.

If an allowance for bad debts already exists, then a comparison of


the existing allowance must be made with the new allowance to
determine whether to increase or decrease the allowance. Let us
assume that the existing allowance on January 1, 2003 is $5,000,
and the closing balance for accounts receivable on December 31,
2003 is $100,000. Suppose the allowance for bad debts is 8% of
debtors (accounts receivable). The new allowance is $100,000 x 0.08
= $8,000. Therefore the allowance for Bad Debts Account will
increase by $8,000 - $5,000 = $3,000.

To create, increase or decrease the allowance for bad debts


(doubtful debts), the entry must first be recorded in the General
Journal and then posted to the General Ledger.
154

To create the allowance


DR. Increase in Allowance for Bad Debts A/C (Expense)
CR. Allowance for Bad Debts A/C

To increase the allowance


DR. Increase in Allowance for Bad Debts A/C (Expense)
CR. Allowance for Bad Debts A/C

To decrease the allowance


DR. Allowance for Bad Debts A/C
CR. Decrease in Allowance for Bad Debts A/C (Income)

The debit balance in the Increase in Allowance for Bad Debts


Account (Expense Account) will be transferred to the debit side of
the Profit and Loss Account at the end of the financial year. The
credit balance in the Decrease in Allowance for Bad Debts Account
(Income Account) will be transferred to the credit side of the
Profit and Loss Account at the end of the financial year. The
credit balance in the Allowance for Bad Debts Account is subtracted
from the accounts receivable (debtors) balance in the Balance Sheet
at the end of the financial year.

OTHER ALLOWANCE

ALLOWANCE FOR DISCOUNT ON DEBTORS

The allowance for discount on debtors (formerly provision for


discount on debtors) is a general allowance. It is usually made for
cash discounts on debts that are expected to be paid promptly by
customers. It is based on a percentage of debtors net of allowance
for bad debts. The accounts receivable is $100,000 and the
allowance for bad debts is 8%.

Accounts receivable $100,000


Less allowance for bad debts $ 8,000
$ 92,000
========
155

So the allowance for discount on debtors will be based on $92,000


and not $100,000, since it is doubtful that the $8,000 will be paid
to the firm. So it is not necessary to make any allowance for
discount on debtors in respect of this $8,000.

If an allowance for discount on debtors already exists, then a


comparison of the existing allowance with the new allowance must be
made to determine whether to increase or decrease the allowance.

To create, increase or decrease the allowance for discount on


debtors, the entry must first be recorded in the General Journal
and then posted to the General Ledger.

To create the allowance


DR. Increase in Allowance for Discount on Debtors A/C (Expense)
CR. Allowance for Discount on Debtors A/C

To increase the allowance


DR. Increase in Allowance for Discount on Debtors A/C (Expense)
CR. Allowance for Discount on Debtors A/C

To decrease the allowance


DR. Allowance for Discount on Debtors A/C
CR. Decrease in Allowance for Discount on Debtors A/C (Income)

The debit balance in the Increase in Allowance for Discount on


Debtors Account (Expense Account) will be transferred to the debit
side of the Profit and Loss Account at the end of the financial
year. The credit balance in the Decrease in Allowance for Discount
on Debtors Account (Income Account) will be transferred to the
credit side of the Profit and Loss Account at the end of the
financial year. The credit balance in the Allowance for Discount on
Debtors Account is subtracted from the accounts receivable
(debtors) balance in the Balance Sheet at the end of the financial
year.
156

ALLOWANCE FOR UNREALIZED PROFIT

When goods are sold by one department to another department in the


same firm at a profit, then the goods transferred at a profit that
remain unsold at the end of the accounting period contain
unrealized profits. An allowance (formerly provision) is created
for this unrealized profit. If an allowance already exists, then a
comparison of the existing allowance with the new allowance must be
made to determine whether the allowance should be increased or
decreased. Unrealized profit on goods can also occur in Branch
Accounts and Group Accounts, which are dealt with in advanced
accounting.

Goods costing $20,000 were sold by the manufacturing department to


the trading department for $24,000. The goods remain unsold at the
end of the accounting period. The allowance for unrealized profit
will be $24,000 - $20,000 = $4,000

To create, increase or decrease the allowance for unrealized


profit, the accounting entry must first be recorded in the General
Journal and then posted to the General Ledger.

To create the allowance


DR. Increase in Allowance for Unrealized Profit A/C (Expense)
CR. Allowance for Unrealized Profit A/C

To increase the allowance


DR. Increase in Allowance for Unrealized Profit A/C (Expense)
CR. Allowance for Unrealized Profit A/C

To decrease the allowance


DR. Allowance for Unrealized Profit A/C
CR. Decrease in Allowance for Unrealized Profit A/C (Income)

The debit balance in the Increase in Allowance for Unrealized


Profit Account (Expense Account) will be transferred to the debit
side of the Profit and Loss Account at the end of the financial
year. The credit balance in the Decrease in Allowance for
157

Unrealized Profit Account (Income Account) will be transferred to


the credit side of the Profit and Loss Account at the end of the
financial year. The credit balance in the Allowance for Unrealized
Profit Account is subtracted from the closing stock of finished
goods balance in the Balance Sheet at the end of the financial
year.

Allowance Accounts are Liability Accounts, but these accounts are


deducted from the Asset Account that they relate to in the Balance
Sheet.

ASSET ACCOUNT RELATED ALLOWANCE ACCOUNT


Accounts Receivable Allowance for Bad Debts
Accounts Receivable Allowance for Discount on Debtors
Closing Stock of Finished Allowance for Unrealized Profit
Goods

ACCRUED INCOME
Income that is earned during an accounting period but is not paid
to the firm by the end of that same accounting period is referred
to as accrued income. If the interest earned for March 2004 is
$2,000 and the $2,000 is not paid to the firm by the end of March
2004, then the amount must be accrued. The accrued income
receivable represents an asset to the firm at the end of the
accounting period, and is reflected in the Balance Sheet under
current assets.

Asset DR.
R
E
158

The income not yet paid to the firm is debited to the Accrued
Income Receivable Account (Asset Account) and credited to the
Income Account. The accrued income is first recorded in a book of
original entry (the General Journal) and then posted to the General
Ledger.

The Interest Receivable Account (Accrued Income Account) is debited


with $2,000 and the Interest Income Account is credited with
$2,000. The accounts would appear as follows:

DR. INTEREST INCOME ACCOUNT CR.


2004 $ ║ 2004 $
║ Mar.31 Interest
Mar.31 Balance c/d 2,000 ║ Receivable A/C 2,000
===== ║ =====

║ Apr. 1 Balance b/d 2,000

At the end of the financial year the credit balance in the Interest
Income Account will be transferred to the credit side of the Profit
and Loss Account.

DR. INTEREST RECEIVABLE ACCOUNT CR.


2004 $ ║ 2004 $
Mar.31 Interest ║
Income A/C 2,000 ║ Mar.31 Balance c/d 2,000
===== ║ =====

Apr. 1 Balance b/d 2,000 ║

The $2,000 interest receivable will be reflected in the Balance


Sheet at the end of March 2004 under current assets.
159

When the interest that was accrued is paid to the firm the Interest
Receivable Account is credited. If the firm received the $2,000 for
interest receivable on April 20, 2004, the accounts would appear as
follows:

DR. INTEREST INCOME ACCOUNT CR.


2004 $ ║ 2004 $
║ Mar.31 Interest
Mar.31 Balance c/d 2,000 ║ Receivable A/C 2,000
===== ║ =====

Apr.30 Balance c/d 2,000 ║ Apr. 1 Balance b/d 2,000
===== ║ =====

║ May 1 Balance b/d 2,000

DR. INTEREST RECEIVABLE ACCOUNT CR.


2004 $ ║ 2004 $
Mar.31 Interest ║
Income A/C 2,000 ║ Mar.31 Balance c/d 2,000
===== ║ =====

Apr. 1 Balance b/d 2,000 ║ Apr.20 Bank A/C 2,000
===== ║ =====

If the interest earned for March 2004 was $2,000 and the firm
received $1,600, then the accrued income would only be $400.

UNEARNED INCOME
Where the firm receives payment in the current period in respect of
future sales or service, the amount received in advance is referred
to as unearned income (deferred income). For example, on January 5,
2004 a tenant pays $30,000 by cash for rent to the firm for January
2004 to March 2004, that is, $10,000 for each month. The $30,000
160

cash received is debited in the Cash Account and credited in the


Rental Income Account. The unearned income represents a liability
to the firm at the end of the accounting period, and is reflected
in the Balance Sheet under current liability.

Asset DR. Liability CR.


R P
E I
Therefore in order to record the income received in advance, we
credit the Income Received in Advance Account (liability) and debit
the Income Account with the income received in advance. At the end
of January 2004, the rental income for February 2004 and March 2004
($20,000) represents unearned income. The unearned income is
recorded by crediting the Rent Received in Advance Account with
$20,000 and debiting the Rental Income Account with $20,000. The
accounts would then appear as follows:

DR. RENTAL INCOME ACCOUNT CR.


2004 $ ║ 2004 $
Jan.31 Rent Received in ║ Jan. 5 Cash A/C 30,000
Advance A/C 20,000 ║
Jan.31 Balance c/d 10,000 ║
30,000 ║ 30,000
====== ║ ======

║ Feb. 1 Balance b/d 10,000

At the end of the financial year the credit balance in the Rental
Income Account will be transferred to the credit side of the Profit
and Loss Account.

DR. RENT RECEIVED IN ADVANCE ACCOUNT CR.


2004 $ ║ 2004 $
Jan.31 Balance c/d 20,000 ║ Jan.31 Rental Income A/C 20,000
====== ║ ======

║ Feb. 1 Balance b/d 20,000
161

The unearned income is first recorded in a book of original entry


(the General Journal) and then posted to the General Ledger
Accounts (Rental Income Account and Rent Received in Advance
Account).

The unearned income of $20,000 will be reflected in the Balance


Sheet at the end of January 2004 as rent received in advance under
current liabilities.

At the end of February 2004 the unearned income will be reduced


from 2 months (February and March) to 1 month (March). Since the
rental income for February 2004 is $10,000, the Rental Income
Account (Income Account) will be credited with $10,000 and the Rent
Received in Advance Account will be debited with $10,000. The entry
will first be recorded in the General Journal and then posted to
the General Ledger. The accounts would then appear as follows:

DR. RENTAL INCOME ACCOUNT CR.


2004 $ ║ 2004 $
Jan.31 Rent Received ║ Jan. 5 Cash A/C 30,000
in Advance A/C 20,000 ║
Jan.31 Balance c/d 10,000 ║
30,000 ║ 30,000
====== ║ ======

Feb.28 Balance c/d 20,000 ║ Feb. 1 Balance b/d 10,000
║ Feb.28 Rent Received
║ in Advance A/C 10,000
20,000 ║ 20,000
====== ║ ======
║ Mar. 1 Balance b/d 20,000

DR. RENT RECEIVED IN ADVANCE ACCOUNT CR.


2004 $ ║ 2004 $
Jan.31 Balance c/d 20,000 ║ Jan.31 Rental Income A/C 20,000
====== ║ ======

Feb.28 Rental ║ Feb. 1 Balance b/d 20,000
Income A/C 10,000 ║
Feb.28 Balance c/d 10,000 ║
20,000 ║ 20,000
====== ║ ======
║ Mar. 1 Balance b/d 10,000
162

The rent received in advance of $10,000 will be reflected as a


current liability in the Balance Sheet at the end of February 2004.
Instead of having an individual account for each unearned income
(Rent Received in Advance Account) we could have just one Unearned
Income Account, for all unearned income.

MARK-UP AND MARGIN


When the profit is expressed as a percentage of cost, it is
referred to as the mark-up. When the profit is expressed as a
percentage of selling price it is referred to as the margin.

Selling Price = Cost + Profit

If for example, the selling price for a product is $120 and the
cost of the product is $100, then the profit will be $20.
Therefore, the mark-up and the margin for this product can be
calculated as follows:

MARK-UP = PROFIT 100


COST X 1

= $ 20 100
$100 X 1

= 20%

MARGIN = PROFIT 100


SELLING PRICE X 1

= $ 20 100
$120 X 1

= 16 2/3 %

Multiplying by 100 converts the fraction to a percentage.


1
163

CONVERSION OF MARK-UP TO MARGIN

To convert the mark-up to margin we add the numerator to the


denominator. The numerator is the number at the top of the fraction
and the denominator is the number at the bottom of the fraction.

NOTE: The denominator of the margin will always be greater than the
denominator of the mark-up. While the denominator changes,
the numerator remains the same.

Mark-up = 20% = 1
5

Therefore Margin = 1
5 + 1

= 1
6

Mark-up = 2
5

Therefore Margin = 2
5 + 2

= 2
7
164

CONVERSION OF MARGIN TO MARK-UP

To convert the margin to mark-up we subtract the numerator from the


denominator. While the denominator changes, the numerator remains
the same.

Margin = 25% = 1
4

Therefore Mark-up = 1
4 - 1

= 1
3

The selling price is $125 and the mark-up is 25%.


Calculate the profit and the cost.

We must compare like items, we cannot add mangoes and bananas. The
mark-up relates to cost and we are given the selling price.
Therefore we must convert the mark-up to margin, since margin
relates to selling price.

Mark-up = 25% = 1
4
Therefore Margin = 1
4 + 1

= 1
5

Profit = $125 x 1 = $25


5

Cost = $125 - $25 = $100


165

OTHER ADJUSTMENTS
PURCHASE OF GOODS FOR USE IN THE BUSINESS

Where goods are purchased for use in the business and material
amounts remain unused at the end of the accounting period, the
unused amount must be reflected in the accounts. Examples of goods
purchased for use in the business are office supplies and
stationery.

Let us look at an example for stationery.

Stock of stationery on January 1, 2003 $20,000


Stationery purchased by cheque
March 1, 2003 $40,000
June 10, 2003 $50,000
Stock of stationery on December 31, 2003 $25,000
Financial year end is December 31, 2003.

The stationery expense for the financial year ended December 31,
2003 will be $85,000.

Opening stock of stationery January 1, 2003 $ 20,000


Add stationery purchases for 2003 $ 90,000
Stationery available for use $110,000
Less closing stock of stationery December 31, 2003 $ 25,000
Stationery used (expense reflected in the $ 85,000
Profit and Loss A/C) ========

The stock of stationery on December 31, 2003 amounting to $25,000


represents an asset to the firm. The $25,000 will be reflected
under current assets in the Balance Sheet at December 31, 2003.
166

GOODS SENT ON A SALE OR RETURN BASIS

If goods are sent by firm X to firm Y on a sale or return basis,


then until firm Y sells the goods, firm Y will owe firm X $0. If
the goods are not sold, then firm Y can send back the goods to firm
X without incurring any liability. If firm Y sells 80 per cent of
the goods, then firm Y will only incur a liability to firm X for
the 80 per cent of the goods that were sold. Firm Y will put its
own mark-up on the goods before selling them. Even though firm Y
has possession of the goods, firm X remains the legal owner of the
goods. Therefore any unsold goods at the end of the accounting
period must be included in the closing stock of firm X, at cost,
and not the closing stock of firm Y.

For example, firm X sent goods to firm Y on a sale or return basis


amounting to $125,000 (selling price). The mark-up on the goods by
firm X is 25%. Firm Y sold 80% of the goods. How much will be
included in the closing inventory of firm X in respect of goods
sent on a sale or return basis? Since 20% of the goods remain
unsold we must calculate the cost price of the unsold goods.

Selling price of unsold goods = $125,000 x 0.20


= $25,000

Mark-up = 25% = 1
4

Therefore Margin = 1
4 + 1

= 1
5

Profit on unsold goods = $25,000 x 1


5
= $5,000
167

Cost of unsold goods = Selling price of unsold goods - Profit of


unsold goods

= $25,000 - $5,000

= $20,000

Therefore, $20,000 will be included in the closing inventory of


firm X in respect of goods sent on a sale or return basis to
Firm Y. Please NOTE that firm Y cannot include the $20,000 in its
closing inventory since firm Y does not own the goods.

If the goods were sent on consignment to firm Y by firm X, then


firm Y would be an agent for firm X. Firm X will set the selling
price for the goods and Firm Y will receive a commission from firm
X for selling the goods.

DRAWINGS

When the owner of the business takes cash or goods from the
business for his or her personal use, then this transaction is
referred to as drawings.

TRANSACTION DOUBLE ENTRY


Cash Drawings DR. Drawings A/C
CR. Cash A/C
Cheque Drawings DR. Drawings A/C
CR. Bank A/C
Drawings of goods at cost DR. Drawings A/C
CR. Purchases A/C
Drawings of goods at selling price DR. Drawings A/C
CR. Sales A/C

Please NOTE that in all cases above the Drawings Account is


debited.
168

CONTINGENT LIABILITY

A contingent liability is a potential liability at the end of the


financial period that may or may not materialize (become a
liability), for example, a pending lawsuit against the firm. The
contingent liability if material is reflected as a note to the
Balance Sheet. The contingent liability is not reflected in the
accounts, that is, no account is debited and no account is
credited.

COMMISSION BASED ON PROFIT

Commission is an incentive given to employees or managers for


achieving a certain level of sales or profits for a particular
period. The commission is based on the sales performance or profit
performance for that period.

Profit before commission for 2003 = $27,720

Calculate the following commissions:


(a) Employees to receive a commission of 5% of the profits.
(b) Employees to receive a commission of 5% of the profit after
charging the commission.

(a) Employees Commission = $27,720 x 5


100
= $1,386

(b) Employees Commission

= Profit before Commission Percentage


Commission x 100 + Commission Percentage

= $27,720 x 5
100 + 5
= $1,320
169

REVIEW QUESTIONS
(1) Distinguish between accrued expense and accrued income.

(2) Distinguish between a bad debt and an allowance for bad debt.

(3)* Fill in the blanks -

(a) An accrued expense is a c________________ _________________


in the Balance Sheet.

(b) A prepayment is a c_________________ ___________________ in


the Balance Sheet.

(c) Allowance for bad debts is deducted from a___________________


__________________ in the Balance Sheet.

(d) Allowance for unrealized profit is deducted from c___________


_______________ in the Balance Sheet.

(e) Accrued income is a c________________ __________________ in


the Balance Sheet.

(f) Unearned income is a c_________________ __________________


in the Balance Sheet.

(4)* If the mark-up is 3, what is the margin?


8

EXERCISE 1 *

From the following information prepare the Rent Account and the
Rent Accrued Account for the financial year ended December 31,
2003.

Rent accrued January 1, 2003 $ 5,000


Total rent for January 2003 to December 2003 $160,000
Total rent paid by cheque on January 31, 2003 $ 15,000
Total rent paid by cheque February to December 2003 $135,000
170

EXERCISE 2 *

From the following information prepare the journal entries, the


Allowance for Bad Debts Account and the Allowance for Discount on
Debtors Account for the financial year ended April 30, 2004.

May 1, 2003 Allowance for bad debts balance $ 16,000


Accounts receivable balance on April 30, 2004 $200,000

The allowance for bad debts should be 5 per cent of debtors.

Create a 1% Allowance for Discount on Debtors Account.

EXERCISE 3 *

From the following information calculate the cost price and the
profit.

Selling Price $15,000


Mark-Up 25%

College Question

EXERCISE 4 *

2002 2003 2004


$ $ $
Trade debtors 350,000 380,000 260,000
Allowance for bad debts 17,500

The financial year end is December 31. The allowance for bad debts
should be 5% of trade debtors.

Required:
a. Calculate the allowance for bad debt for 2003 and 2004.

b. Calculate the increase or decrease in allowance for bad debt


for 2003 and 2004 and indicate whether the amount should be
debited or credited in the Profit and Loss Account.
171

ANSWERS TO SELECTED REVIEW QUESTIONS


(3) (a) Current Liability
(b) Current Asset
(c) Accounts Receivable
(d) Closing Stock
(e) Current Asset
(f) Current Liability

(4) 3
11

EXERCISE 1

DR. RENT ACCOUNT CR.


2003 $ ║ 2003 $
Jan-Dec. Bank A/C 145,000 ║ Dec. 31 Profit & Loss
Dec. 31 Rent Accrued ║ A/C 160,000
A/C 15,000 ║
160,000 ║ 160,000
======= ║ =======

The total rental expense of $160,000 is transferred to the Profit


and Loss Account at the end of the financial year.

DR. RENT ACCRUED ACCOUNT CR.


2003 $ ║ 2003 $
Jan. 31 Bank A/C 5,000 ║ Jan. 1 Balance b/f 5,000
===== ║ =====

Dec. 31 Balance c/f 15,000 ║ Dec. 31 Rent A/C 15,000
====== ║ ======

172

WORKINGS

Total rent paid in 2003 ($15,000 + $135,000) $150,000


Less payment for 2002 accrual $ 5,000
Total rent paid for 2003 $145,000
========

Accrued rent for 2003 = Total rent for 2003 - Rent paid for 2003
= $160,000 - $145,000
= $15,000

EXERCISE 2

Existing allowance for bad debts $16,000


New allowance for bad debts ($200,000 x 0.05) $10,000
Decrease in allowance for bad debts $ 6,000
=======

Debtors $200,000
Less new allowance for bad debts $ 10,000
$190,000
========

Allowance for discount on debtors = $190,000 x 0.01 = $1,900


173

GENERAL JOURNAL FOR APRIL 2004

DATE PARTICULARS FOLIO DR. CR.

$ $
30/4/2004 Allowance for Bad Debts A/C 6,000
Decrease in Allowance for Bad
Debts A/C 6,000
Being reduction in allowance
for bad debts. (See
supporting workings attached)
30/4/2004 Decrease in Allowance for Bad
Debts A/C 6,000
Profit and Loss A/C 6,000
Being transfer to the Profit
and Loss A/C.
30/4/2004 Increase in Allowance for
Discount on Debtors A/C 1,900
Allowance for Discount on
Debtors A/C 1,900
Being creation of an
allowance for Discount on
Debtors A/C. (See supporting
workings attached)
30/4/2004 Profit and Loss A/C 1,900
Increase in Allowance for
Discount on Debtors A/C 1,900
Being transfer to the Profit
and Loss A/C.

PLEASE NOTE: The financial year begins on May 1, 2003 and ends on
April 30, 2004. So the General Journal and the General
Ledger of this business can contain entries for 2003
and 2004.
174

GENERAL LEDGER

DR. ALLOWANCE FOR BAD DEBTS ACCOUNT CR.


$ ║ $
30/4/04 Decrease in Allow. ║ 1/5/03 Balance b/f 16,000
for Bad Debts A/C 6,000 ║
30/4/04 Balance c/f 10,000 ║
16,000 ║ 16,000
====== ║ ======

DR. ALLOWANCE FOR DISCOUNT ON DEBTORS ACCOUNT CR.


$ ║ $
║ 30/4/04 Increase in Allow.
║ for Discount on
30/4/04 Balance c/f 1,900 ║ Debtors A/C 1,900
===== ║ =====

EXERCISE 3

Mark-up = 25 = 1
100 4

Therefore margin = 1
5

Profit = $15,000 x 1 = $3,000


5

Cost Price = Selling Price - Profit


= $15,000 - $3,000
= $12,000

EXERCISE 4

(a) 2003 allowance for bad debt $19,000


2004 allowance for bad debts $13,000.
(b) 2003 increase $1,500 debit Profit and Loss Account
2004 decrease $6,000 credit Profit and Loss Account
175

ch7
CHAPTER SEVEN

PROPERTY, PLANT AND EQUIPMENT

CHAPTER OBJECTIVES
After completing this chapter you should be able to:

 Distinguish between capital expenditure and revenue


expenditure.

 State the purpose of providing depreciation.

 Describe the various methods of providing for


depreciation.

 Calculate depreciation using the straight-line method and


the reducing balance method.

 Calculate the profit or loss on the disposal of an item


of property, plant and equipment.
176

CAPITAL EXPENDITURE AND REVENUE EXPENDITURE


Capital expenditure refers to costs incurred in the acquisition of
tangible non-current assets (property, plant and equipment), and in
material modification and additions to these assets. Material
modifications and additions to these assets are capitalised, that
is, added to the cost of the asset. The following items are usually
capitalised:

(1) Costs incurred in the extension of the asset’s life.


(2) Costs incurred in improving the efficiency of the asset.
(3) Costs incurred in replacing a major component of the asset.
(4) Costs incurred in transporting a new tangible non-current asset
to the location where it will be used.
(5) Costs incurred in installing a new tangible non-current asset.

Capital expenditure is allocated over successive accounting


financial years during the lifetime of the property, plant and
equipment. The costs allocated for that financial year are charged
as depreciation in the Income Statement in that financial year.
Capital expenditure results in the acquisition of items that are of
a long lasting and permanent nature (for example, buildings)
whereas revenue expenditure is not of a long lasting or permanent
nature (for example, payment for wages).

Revenue expenditure refers to costs incurred in relation to cost of


sales and operating expenses. These costs are charged in the Income
Statement for the relevant financial period. Revenue expenditure is
a recurrent expenditure, while capital expenditure is a non-
recurrent expenditure.

If costs are incurred amounting to $500,000 for the purchase of an


item of property, plant and equipment which is expected to last for
10 years, and $500,000 for wages and salaries, then the $500,000
for the purchase of the item of property, plant and equipment will
be classified as capital expenditure and the $500,000 for the wages
and salaries will be classified as revenue expenditure. The
$500,000 for the item of property, plant and equipment minus any
scrap value will be allocated to the Income Statement by means of
depreciation each year over a period of ten years. The $500,000 for
the wages and salaries is charged in the Income Statement for the
financial year that it relates to as wages and salaries. Property,
plant and equipment were formerly referred to as fixed assets.
177

EXAMPLES OF CAPITAL EXAMPLES OF REVENUE


EXPENDITURE EXPENDITURE
Purchase of furniture for use Costs incurred for travelling
in the business
Purchase of machines for use Costs incurred for rent
in the business
Purchase of motor vehicles for Costs incurred for stationery
use in the business
Purchase of land for use in Costs incurred for wages
the business
Purchase of buildings for use Costs incurred for audit fees
in the business
Purchase of fixtures for use Costs incurred for electricity
in the business
Purchase of plant for use in Costs incurred for goods for
the business resale
178

DEPRECIATION
Depreciation is an allocation of the capitalised costs of an item
of property, plant and equipment (minus the salvage value where
necessary) over the estimated useful life of the item of property,
plant and equipment. The capitalised cost minus the salvage value
is referred to as the depreciable cost. The salvage value or
residual value is the revenue that the firm expects to receive when
it sells the asset at the end of the asset’s useful life. Several
methods of providing for depreciation exist. The most important
methods are stated below.

(1) Straight line method


(2) Reducing balance method
(3) Sum of the years' digit method
(4) Revaluation method
(5) Depletion method

Depreciation is calculated on a monthly basis and is an expense for


the firm. The depreciation is first recorded in the General Journal
(book of original entry) and then posted to the General Ledger. The
double entry is, debit the Depreciation Account and credit the
Allowance for Depreciation Account (formerly Provision for
Depreciation Account) with the depreciation charge.

The two methods of depreciation that are frequently used by firms


are the straight line method and the reducing balance method.

DEPRECIATION POLICY
The depreciation policy of the firm will state the method of
depreciation to be applied to each category of property, plant and
equipment and the rate of depreciation to be charged. The firm may
for example apply the straight line method of depreciation to all
machines at 20 per cent per annum on costs, and apply the reducing
balance method of depreciation to all motor vehicles at 25 per cent
per annum.

The firm's depreciation policy will also state if depreciation is


to be charged on a pro rata basis or if a full month's depreciation
is to be charged for a fraction of a month in the year of
acquisition or disposal.
179

METHODS OF PROVIDING FOR DEPRECIATION

STRAIGHT LINE OR FIXED INSTALMENT METHOD


The straight line method of depreciation charges a fixed amount for
depreciation each financial year over the useful life of an item of
property, plant and equipment, ceteris paribus (all things being
equal).

Annual Depreciation Charge = Capitalised Cost - Residual Value


Estimated Useful Life

On January 1, 2002 Alicia Limited started business. Several items


of furniture amounting to $140,000 were purchased by cheque on
January 1, 2002 for use in the business. The items of furniture
have an estimated useful life of 10 years with a residual value of
$20,000. On June 16, 2002 furniture amounting to $90,000 was
purchased on credit from Beast Limited for use in the business. The
furniture has an estimated life of 15 years with a residual value
of $0. The firm's financial year end is June 30.

The depreciation policy of the firm stated the following:

(1) Depreciation on furniture is to be provided using the straight


line method of depreciation.
(2) Depreciation is to be charged on a pro rata basis.

REQUIRED:

(a) The journal entry for depreciation for January 2002 to June
2002.
(b) The Furniture Account for January 2002 to June 2002.
(c) The Depreciation on Furniture Account for January 2002 to June
2002.
(d) The Allowance for Depreciation on Furniture Account for January
2002 to June 2002.
(e) The Balance Sheet extract for Non-current assets relating to
furniture as at June 30, 2002.
(f) The total depreciation charge for the financial year ended June
30, 2003 and the financial year ended June 30, 2004.
180

WORKINGS
Calculation of the depreciation charge on the $140,000 furniture
using the straight line method of depreciation.

Annual Depreciation Charge = Capitalised Cost - Residual Value


Estimated Useful Life

= $140,000 - $20,000
10

= $12,000

Since 12 months make one year, in order to obtain the monthly


depreciation charge we divide the annual depreciation charge by 12.

Monthly Depreciation Charge = $12,000


12
= $1,000

So each month the depreciation charge will be $1,000.

Calculation of depreciation charge on the $90,000 furniture using


the straight line method of depreciation.

Annual Depreciation Charge = Capitalised Cost - Residual Value


Estimated Useful Life

= $90,000 - $0
15

= $6,000

The furniture was purchased on June 16, 2002. Since depreciation is


to be charged on a pro rata basis, for the month of June 2002
depreciation will be charged for 15 days (June 16, 2002 to June 30,
2002). So the depreciation charge for June 2002 will be for
(15 days/30 days) half of a month. (Remember - 30 days for
September, April, June and November).

Monthly Depreciation Charge = $6,000


12
= $500
181

Depreciation charge for June 2002 = $500 x 1/2


= $250

The depreciation charge for July 2002 and each month thereafter
will be $500.

The Depreciation Account is an Expense Account, therefore the


Expense Account will be debited and the Allowance Account credited.

JANUARY 2002 GENERAL JOURNAL

DATE PARTICULARS FOLIO DR. CR.

2002 $ $
Jan. 31 Depreciation on Furniture A/C 1,000
Allow. for Dep. on Furn. A/C 1,000
Being depreciation charge on
furniture for January 2002.

FEBRUARY 2002 GENERAL JOURNAL

DATE PARTICULARS FOLIO DR. CR.

2002 $ $
Feb. 29 Depreciation on Furniture A/C 1,000
Allow. for Dep. on Furn. A/C 1,000
Being depreciation charge on
furniture for February 2002.
182

MARCH 2002 GENERAL JOURNAL

DATE PARTICULARS FOLIO DR. CR.

2002 $ $
Mar. 31 Depreciation on Furniture A/C 1,000
Allow. for Dep. on Furn. A/C 1,000
Being depreciation charge on
furniture for March 2002.

APRIL 2002 GENERAL JOURNAL

DATE PARTICULARS FOLIO DR. CR.

2002 $ $
Apr. 30 Depreciation on Furniture A/C 1,000
Allow. for Dep. on Furn. A/C 1,000
Being depreciation charge on
furniture for April 2002.

MAY 2002 GENERAL JOURNAL

DATE PARTICULARS FOLIO DR. CR.

2002 $ $
May 31 Depreciation on Furniture A/C 1,000
Allow. for Dep. on Furn. A/C 1,000
Being depreciation charge on
furniture for May 2002.
183

JUNE 2002 GENERAL JOURNAL

DATE PARTICULARS FOLIO DR. CR.

2002 $ $
June 16 Furniture A/C 90,000
Beast Ltd. A/C 90,000
Being furniture purchased on
credit from Beast Limited for
use in the business.
June 30 Depreciation on Furniture A/C 1,250
Allow. for Dep. on Furn. A/C 1,250
Being depreciation charge on
furniture for June 1992.

($1,000 + $250)
June 30 Profit & Loss A/C 6,250
Depreciation on Furniture A/C 6,250
Being depreciation on
furniture transferred to the
Profit & Loss A/C.

January 1, 2002

The Furniture Account is an Asset Account, so the Asset Account


will be debited with $140,000, and the Bank Account credited with
$140,000 since the payment was made by cheque.

June 16, 2002

The Furniture Account is debited with $90,000, and Beast Limited


Account credited with $90,000 since the furniture was purchased on
credit from Beast Limited. Since the furniture is purchased on
credit for use in the business, it is first recorded in the General
Journal (see June 2002 General Journal above).
184

DR. FURNITURE ACCOUNT CR.


DATE PARTICULARS AMOUNT DATE PARTICULARS AMOUNT
2002 $ ║ 2002 $
Jan. 1 Bank A/C 140,000 ║ Jan.31 Balance c/d 140,000
======= ║ =======

Feb. 1 Balance b/d 140,000 ║ Feb.29 Balance c/d 140,000
======= ║ =======

Mar. 1 Balance b/d 140,000 ║ Mar.31 Balance c/d 140,000
======= ║ =======

Apr. 1 Balance b/d 140,000 ║ Apr.30 Balance c/d 140,000
======= ║ =======

May 1 Balance b/d 140,000 ║ May 31 Balance c/d 140,000
======= ║ =======

June 1 Balance b/d 140,000 ║ Jun.30 Balance c/f 230,000
Jun.16 Beast Ltd. A/C 90,000 ║
230,000 ║ 230,000
======= ║ =======

The balance in the account is carried down (c/d) at the end of each
accounting period (at the end of each month) to the next accounting
period. At the end of the financial year (June 30, 2002) the
balance in the Furniture Account is not carried down (c/d) but
carried forward (c/f) to the next financial year beginning July 1,
2002. On July 1, 2002 the opening balance in the Furniture Account
will be referred to as the balance b/f (brought forward) and not as
the balance b/d (brought down).
185

Since we are dealing with furniture, instead of saying Depreciation


Account we will be specific and say Depreciation on Furniture
Account.

DR. DEPRECIATION ON FURNITURE ACCOUNT CR.


DATE PARTICULARS AMOUNT DATE PARTICULARS AMOUNT
2002 $ ║ 2002 $
Jan.31 Allow. for Dep. ║
on Furn. A/C 1,000 ║ Jan.31 Balance c/d 1,000
===== ║ =====

Feb. 1 Balance b/d 1,000 ║ Feb.29 Balance c/d 2,000
Feb.29 Allow. for Dep. ║
on Furn. A/C 1,000 ║
2,000 ║ 2,000
===== ║ =====

Mar. 1 Balance b/d 2,000 ║ Mar.31 Balance c/d 3,000
Mar.31 Allow. for Dep. ║
on Furn. A/C 1,000 ║
3,000 ║ 3,000
===== ║ =====

Apr. 1 Balance b/d 3,000 ║ Apr.30 Balance c/d 4,000
Apr.30 Allow. for Dep. ║
on Furn. A/C 1,000 ║
4,000 ║ 4,000
===== ║ =====

May 1 Balance b/d 4,000 ║ May 31 Balance c/d 5,000
May 31 Allow. for Dep. ║
on Furn. A/C 1,000 ║
5,000 ║ 5,000
===== ║ =====

June 1 Balance b/d 5,000 ║ Jun.30 Profit & Loss
Jun.30 Allow. for Dep. ║ A/C 6,250
on Furn. A/C 1,250 ║
6,250 ║ 6,250
===== ║ =====
186

At the end of the financial year, (June 30, 2002 in this case), the
Depreciation Account is closed by transferring the balance in the
account to the Profit and Loss Account (see June 2002 General
Journal on page 183).

Since we are dealing with furniture, instead of saying Allowance


for Depreciation Account, we will be specific and say Allowance for
Depreciation on Furniture Account.

DR. ALLOWANCE FOR DEPRECIATION ON FURNITURE ACCOUNT CR.


DATE PARTICULARS AMOUNT DATE PARTICULARS AMOUNT
2002 $ ║ 2002 $
Jan.31 Balance c/d 1,000║ Jan.31 Dep. on Furn.A/C 1,000
=====║ =====

Feb.29 Balance c/d 2,000 ║ Feb. 1 Balance b/d 1,000
║ Feb.29 Dep. on Furn. A/C 1,000
2,000 ║ 2,000
===== ║ =====

Mar.31 Balance c/d 3,000 ║ Mar. 1 Balance b/d 2,000
║ Mar.31 Dep. on Furn. A/C 1,000
3,000 ║ 3,000
===== ║ =====

Apr.30 Balance c/d 4,000 ║ Apr. 1 Balance b/d 3,000
║ Apr.30 Dep. on Furn. A/C 1,000
4,000 ║ 4,000
===== ║ =====

May 31 Balance c/d 5,000 ║ May 1 Balance b/d 4,000
║ May 31 Dep. on Furn. A/C 1,000
5,000 ║ 5,000
===== ║ =====

Jun.30 Balance c/f 6,250 ║ June 1 Balance b/d 5,000
║ Jun.30 Dep. on Furn. A/C 1,250
6,250 ║ 6,250
===== ║ =====

187

At the end of the financial year (June 30, 2002) the balance in the
Allowance Account is not carried down (c/d) but carried forward
(c/f) to the next financial year beginning July 1, 2002.

BALANCE SHEET (EXTRACT) OF ALICIA LTD. AS AT JUNE 30, 2002


$
Non-current Assets
Furniture 230,000
Less allowance for depreciation 6,250
Net book value or carrying value 223,750

Since a complete Balance Sheet is not prepared, but only a section


of the Balance Sheet, it is referred to as an extract. The $230,000
represents the balance in the Furniture Account on June 30, 2002,
and the $6,250 represents the balance in the Allowance for
Depreciation on Furniture Account on June 30, 2002. The $223,750 is
referred to as the net book value (NBV) or the carrying value.

The total depreciation charge for the financial year ended June 30,
2003 = $12,000 + $6,000 = $18,000.

The total depreciation charge for the financial year ended June 30,
2004 = $18,000.

Assuming that the furniture costing $140,000 is not sold, then it


will be depreciated over the years down to a net book value of
$20,000 and then no further depreciation will be charged on this
furniture. The $20,000 represents the expected residual value.

Assuming that the furniture costing $90,000 is not sold. Then it


will be depreciated over the years down to a net book value of $1,
instead of its expected residual value of $0. This is done to keep
the furniture on the books and to prevent fraud.
188

REDUCING BALANCE OR DIMINISHING BALANCE METHOD


The reducing balance method of depreciation charges a different
amount for depreciation each accounting period over the useful life
of an item of property, plant and equipment. In order to calculate
the depreciation charge, a fixed percentage is applied to the net
book value (NBV) of the asset. This percentage is calculated as
follows:

P = 1 - N S/C

Where P = The percentage rate of depreciation

N = The estimated useful life of the asset (in years)

S = The salvage value (this value should be material or the


percentage rate of depreciation will be meaningless)

C = The capitalised cost for the asset

The percentage should be rounded to the nearest whole number.

A motor van is purchased for US$10,000. Freight, insurance and


wharfage fees for shipping the motor van to Jamaica amounted to
US$500. The government charged a duty of 40% on the cost of the
motor van. The rate of exchange is US$1 to J$10.
J$
Purchase price of motor van (10,000 x 10) 100,000
Freight, insurance and wharfage fees (500 x 10) 5,000
Duty (10,000 x 0.40) x 10 40,000
Capitalised cost 145,000
=======

The motor van is expected to last for 5 years with an expected


salvage value of $24,500.
189

Calculate the depreciation charge for each of the 5 years using the
reducing balance method. The motor van is received on January 1,
1980. Round all calculations to the nearest whole number.

P = 1 - 5 $ 24,500
$145,000

= 1 - 5 0.1689655

= 0.2992556

 30%

In order to find the fifth root for 0.1689655 you should use a
scientific calculator.

0.1689655 INV x1 5 = 0.7007443


y

1 - 0.7007443 = 0.2992556

Please NOTE that some scientific calculators use a SHIFT key


instead of an INV key.

1980 Depreciation = $145,000 x 0.30 = $ 43,500


1980 NBV = $145,000 - $ 43,500 = $101,500

1981 Depreciation = $101,500 x 0.30 = $ 30,450


1981 NBV = $101,500 - $ 30,450 = $ 71,050

1982 Depreciation = $ 71,050 x 0.30 = $ 21,315


1982 NBV = $ 71,050 - $ 21,315 = $ 49,735

1983 Depreciation = $ 49,735 x 0.30 = $ 14,921


1983 NBV = $ 49,735 - $ 14,921 = $ 34,814

The final year's depreciation charge is calculated as follows:


1984 Depreciation = NBV on Jan.1,1984 - Salvage value of motor van
= $34,814 - $24,500
= $10,314
PROOF

1984 NBV = $34,814 - $10,314


= $24,500 (expected salvage value)
190

Because we rounded the percentage to apply to the net book value,


in the final year the depreciation calculation must be calculated
as stated above.

The motor van is depreciated down to its salvage value, and no


further depreciation is charged. So if the firm continues to use
the motor van in 1985 no depreciation will be charged on the motor
van in 1985. The depreciation charge is first recorded in the
General Journal and then posted to the General Ledger.

SUM OF THE YEARS' DIGIT METHOD


The sum of the years' digit method of depreciation charges a
different amount for depreciation each financial year over the
useful life of an item of property, plant and equipment. The
expected life of the asset provides the basis for the depreciation
charge. The highest depreciation charge is in the first year, and
the lowest depreciation charge in the last year.

A machine is purchased on January 1, 2000 for $190,000.


Installation cost amounted to $10,000 for the modification in the
electrical system in the firm's factory. The residual value is
estimated at $50,000 and the useful life of the machine is
estimated at 5 years. The financial year ends on December 31.
Calculate the depreciation charge for each of the five years using
the sum of the years' digit method.

Sum of the years = 5 + 4 + 3 + 2 + 1 = 15

Depreciable Cost = Capitalised Cost - Residual Value


= ($190,000 + $10,000) - $50,000
= $150,000

2000 Depreciation = 5/15 x $150,000 = $ 50,000


2001 Depreciation = 4/15 x $150,000 = $ 40,000
2002 Depreciation = 3/15 x $150,000 = $ 30,000
2003 Depreciation = 2/15 x $150,000 = $ 20,000
2004 Depreciation = 1/15 x $150,000 = $ 10,000
$150,000
========

No depreciation would be charged on this asset in 2005 or any year


thereafter, since at the end of 2004 it would have been depreciated
down to $50,000, the expected residual value, that is, ($190,000 +
$10,000) - $150,000.
191

If an item of property, plant and equipment useful life is expected


to be 4 years, then the depreciation charge under the sum of the
years' digit method would be calculated as follows:

Sum of the years = 4 + 3 + 2 + 1 = 10

Year 1 Depreciation Charge = 4/10 x Depreciable Cost


Year 2 Depreciation Charge = 3/10 x Depreciable Cost
Year 3 Depreciation Charge = 2/10 x Depreciable Cost
Year 4 Depreciation Charge = 1/10 x Depreciable Cost

REVALUATION METHOD
Under the revaluation method the depreciation charge for each
financial year is based on the net book value (NBV) of the asset at
the beginning of the financial year and the revalued net book value
at the end of the financial year. This method is usually applied to
tools and depreciation is usually charged on an annual basis
instead of monthly because the monthly charge is usually not
material.

Tools were purchased for use in the business on January 1, 2001 for
$10,000. The firm's financial year ends on December 31. At the end
of December 2001 the tools were valued at $9,000. At the end of
December 2002 the tools were valued at $8,500. Tools costing $2,000
were purchased on March 15, 2003. At the end of December 2003 the
tools were valued at $9,800. Calculate the depreciation charge for
2001, 2002 and 2003 using the revaluation method.

Depreciation Charge for 2001 = NBV on 1/1/01 - NBV on 31/12/01


= $10,000 - $9,000
= $1,000

Depreciation Charge for 2002 = NBV on 1/1/02 - NBV on 31/12/02


= $9,000 - $8,500
= $500

Depreciation Charge for 2003


= (NBV on 1/1/03 + Additions) - NBV on 31/12/03
= $8,500 + $2,000 - $9,800
= $700
192

DEPLETION METHOD
The depletion method is usually applied to mines and quarries. The
depreciation charge is based on the estimated quantity of material
in the mine or quarry and the quantity of material extracted during
the accounting period.

A mine costing $120 million was purchased on January 1, 2004. The


surveyor's report revealed that material amounting to 200,000 tons
could be extracted from the mine.

Extraction of material for January 2004 = 10,000 tons


Extraction of material for February 2004 = 20,000 tons
The residual value of the mine is expected to be $20 million.

Calculate the depreciation charge for January 2004 and February


2004 using the depletion method.

Depreciable Cost = Capitalised Cost - Residual Value


= $120,000,000 - $20,000,000
= $100,000,000

Depreciation Charge for January 2004

= Material Extracted for January 2004 x Depreciable Cost


Total Material in the Mine

= 10,000 x $100,000,000
200,000

= $5,000,000

Depreciation Charge for February 2004

= Material Extracted for February 2004 x Depreciable Cost


Total Material in the Mine

= 20,000 x $100,000,000
200,000

= $10,000,000
193

DISPOSAL OF PROPERTY, PLANT AND EQUIPMENT


When an item of property, plant and equipment is disposed of, the
cost of the asset, the accumulated depreciation to the date of
disposal and the proceeds from the disposal of the asset are all
reflected in the Disposal Account. The balance in the Disposal
Account is transferred to the Profit and Loss Account.

Steps to follow in the disposal of a property, plant and equipment:

(1) DR. Disposal A/C


CR. Asset A/C
With the cost of the asset being disposed of.

(2) DR. Allowance for Depreciation A/C


CR. Disposal A/C
With the accumulated depreciation to the date of sale
provided on the asset being disposed of.

(3) DR. Bank A/C or Cash A/C


CR. Disposal A/C
With the money received for selling the asset.

OR
(4) DR. Other Debtors A/C
CR. Disposal A/C
With the sales price of the asset if it is sold on
credit.

(5) DR. Disposal A/C


CR. Profit and Loss A/C
With the profit on disposal of the asset.

OR
(6) DR. Profit and Loss A/C
CR. Disposal A/C
With the loss on disposal of the asset.

Each step, except for step 3 is first recorded in the General


Journal and then posted to the General Ledger. Step 3 is first
recorded in the Cash Book and then posted to the General Ledger.
194

A motor vehicle that was purchased on January 1, 2001 at a


capitalised cost of $120,000 is being disposed of. Residual value
is $0. The motor vehicle was disposed of on April 30, 2004 for
$70,000 cash. Depreciation is being provided at a rate of 20 per
cent per annum on cost. The firm's financial year ends December 31.
Calculate the profit or loss on disposal.

Accumulated Depreciation

January 1, 2001 to December 31, 2003 - 3 years

January 1, 2004 to April 30, 2004 - 4 months

($120,000 x 0.20) x 3 = $72,000


($120,000 x 0.20) x 4/12 = $ 8,000
$80,000
=======

Profit/(Loss) on Disposal
Capitalised cost $120,000
Less accumulated depreciation $ 80,000
Net book value $ 40,000
Proceeds from sale $ 70,000
Net profit on disposal $ 30,000
========

If the proceeds from sale are greater than the net book value, then
the firm will make a profit on disposal of the asset. If the
proceeds from sale are less than the net book value, then the firm
will make a loss on the disposal of the asset.

The profit or loss on the disposal of the asset could also be


determined by preparing a Disposal Account.

DR. DISPOSAL ACCOUNT CR.


$ ║ $
1994 ║ 1994
Apr.30 Motor Vehicle A/C 120,000 ║ Apr.30 Allow. For Dep.
Apr.30 Profit & Loss A/C 30,000 ║ on Motor
║ Vehicle A/C 80,000
║ Apr.30 Cash A/C 70,000
150,000 ║ 150,000
======= ║ =======
195

REVIEW QUESTIONS
(1) What is the meaning of the term `capitalise'?

(2) Distinguish between capital expenditure and revenue


expenditure.

(3) Write short notes on the following:


Depreciable costs
Salvage value
Depreciation policy
Depreciation

(4)* If the proceed from the disposal of an asset is less


than the NBV of the asset being disposed of, then the firm
will make a ____________________ on __________________
of the asset.

EXERCISE 1 *

Better Manufacturing Ltd. acquired three (3) machines for use in


its factory. All the machines were acquired on August 1, 1987 at a
cost of $300,000 each. The machines are being depreciated on the
straight line basis at a rate of 10 per cent per annum on cost. On
November 30, 1991 two (2) of the machines were sold for $200,000.
On August 1, 1992 the remaining machine was modified at a cost of
$138,000. The modification will extend the useful life of the
machine by three (3) years. Cheques are used to make all payments.
The company's financial year ends on July 31.

Prepare the following accounts for the financial years 1991/1992


and 1992/1993:

(1) Machinery Account.


(2) Allowance for Depreciation on Machinery Account.
(3) Disposal Account.

EXERCISE 2 *

From the following information calculate the depreciation charge


for each of the four(4) years using the sum of the years' digit
method. The firm's financial year ends on December 31.
Cost of plant purchased on January 1, 2001 $220,000
Expected salvage value at the end of 4 years $ 20,000
196

ANSWERS TO SELECTED REVIEW QUESTIONS


(4) Loss
Disposal

EXERCISE 1

WORKINGS
Accumulated dep. for 1/8/87 to 31/7/91 (4 years) on all machines

= [Total cost of asset x depreciation rate] x number of years

= [ ($300,000 x 3) x 0.10 ] x 4

= $90,000 x 4

= $360,000

Depreciation for August 1, 1991 to November 30, 1991 (4 months) for


machines sold.

Accumulated depreciation on machines sold on November 30, 1991.

1/8/87 to 31/07/91 = [ ($300,000 x 2) x 0.10 ] x 4 = $240,000


1/8/91 to 30/11/91 = [ ($300,000 x 2) x 0.10 ] x 4/12 = $ 20,000
$260,000
========

DISPOSAL OF MACHINES
$
Cost ($300,000 x 2) 600,000
Less accumulated depreciation 260,000
Net book value 340,000
Proceeds from sale 200,000
Loss on disposal 140,000
=======
197

Depreciation charge for each month for the period August 1, 1991 to
November 30, 1991

Monthly depreciation charge on machines sold


= ($600,000 x 0.10)  12 $5,000
Monthly depreciation charge on remaining machine
= ($300,000 x 0.10)  12 $2,500
$7,500
======

Depreciation charge for each month for the period December 1, 1991
to July 31, 1992

Monthly depreciation charge on remaining machine


= ($300,000 x 0.10)  12
= $2,500

Original life of machine = 100%  10% = 10 years

Depreciation for August 1, 1992 to July 31, 1993

Original Cost + Modification Cost = Capitalised Cost


$300,000 + $138,000 = $438,000

Annual depreciation = capitalised cost - accumulated depreciation


original life - expired life + extended life

= $438,000 - ($300,000 x 0.10 x 5)


(10 - 5) + 3

= $288,000
8

= $36,000

Monthly depreciation charge = $36,000


12

= $3,000
198

FINANCIAL YEAR AUGUST 1, 1991 TO JULY 31, 1992

DR. MACHINERY ACCOUNT CR.


$ ║ $
1/08/91 Balance b/f 900,000 ║ 31/08/91 Balance c/d 900,000
======= ║ =======

1/09/91 Balance b/d 900,000 ║ 30/09/91 Balance c/d 900,000
======= ║ =======

1/10/91 Balance b/d 900,000 ║ 31/10/91 Balance c/d 900,000
======= ║ =======

1/11/91 Balance b/d 900,000 ║ 30/11/91 Disposal A/C 600,000
║ 30/11/91 Balance c/d 300,000
900,000 ║ 900,000
======= ║ =======

1/12/91 Balance b/d 300,000 ║ 31/12/91 Balance c/d 300,000
======= ║ =======

1/01/92 Balance b/d 300,000 ║ 31/01/92 Balance c/d 300,000
======= ║ =======

1/02/92 Balance b/d 300,000 ║ 29/02/92 Balance c/d 300,000
======= ║ =======

1/03/92 Balance b/d 300,000 ║ 31/03/92 Balance c/d 300,000
======= ║ =======

1/04/92 Balance b/d 300,000 ║ 30/04/92 Balance c/d 300,000
======= ║ =======

1/05/92 Balance b/d 300,000 ║ 31/05/92 Balance c/d 300,000
======= ║ =======

1/06/92 Balance b/d 300,000 ║ 30/06/92 Balance c/d 300,000
======= ║ =======

1/07/92 Balance b/d 300,000 ║ 31/07/92 Balance c/f 300,000
======= ║ =======

199

DR. ALLOWANCE FOR DEPRECIATION ON MACHINERY ACCOUNT CR.


$ ║ $
31/08/91 Balance c/d 367,500 ║ 1/08/91 Balance b/f 360,000
║ 31/08/91 Dep. A/C 7,500
367,500 ║ 367,500
======= ║ =======

30/09/91 Balance c/d 375,000 ║ 1/09/91 Balance b/d 367,500
║ 30/09/91 Dep. A/C 7,500
375,000 ║ 375,000
======= ║ =======

31/10/91 Balance c/d 382,500 ║ 1/10/91 Balance b/d 375,000
║ 31/10/91 Dep. A/C 7,500
382,500 ║ 382,500
======= ║ =======

30/11/91 Disposal A/C 260,000 ║ 1/11/91 Balance b/d 382,500
30/11/91 Balance c/d 130,000 ║ 30/11/91 Dep. A/C 7,500
390,000 ║ 390,000
======= ║ =======

31/12/91 Balance c/d 132,500 ║ 1/12/91 Balance b/d 130,000
║ 31/12/91 Dep. A/C 2,500
132,500 ║ 132,500
======= ║ =======

31/01/92 Balance c/d 135,000 ║ 1/01/92 Balance b/d 132,500
║ 31/01/92 Dep. A/C 2,500
135,000 ║ 135,000
======= ║ =======

29/02/92 Balance c/d 137,500 ║ 1/02/92 Balance b/d 135,000
║ 29/02/92 Dep. A/C 2,500
137,500 ║ 137,500
======= ║ =======

31/03/92 Balance c/f 140,000 ║ 1/03/92 Balance b/d 137,500
║ 31/03/92 Dep. A/C 2,500
140,000 ║ 140,000
======= ║ =======

200

DR. ALLOWANCE FOR DEPRECIATION ON MACHINERY ACCOUNT CR.


$ ║ $
30/04/92 Balance c/d 142,500 ║ 1/04/92 Balance b/f 140,000
║ 30/04/92 Dep. A/C 2,500
142,500 ║ 142,500
======= ║ =======

31/05/92 Balance c/d 145,000 ║ 1/05/92 Balance b/d 142,500
║ 31/05/92 Dep. A/C 2,500
145,000 ║ 145,000
======= ║ =======

30/06/92 Balance c/d 147,500 ║ 1/06/92 Balance b/d 145,000
║ 30/06/92 Dep. A/C 2,500
147,500 ║ 147,500
======= ║ =======

31/07/92 Balance c/f 150,000 ║ 1/07/92 Balance b/d 147,500
║ 31/07/92 Dep. A/C 2,500
150,000 ║ 150,000
======= ║ =======

DR. DISPOSAL ACCOUNT CR.


$ ║ $
30/11/91 Machinery A/C 600,000 ║ 30/11/91 Allowance for
║ Dep. A/C 260,000
║ 30/11/91 Bank A/C 200,000
║ 31/07/92 Profit and
║ Loss A/C 140,000
600,000 ║ 600,000
======= ║ =======

201

FINANCIAL YEAR AUGUST 1, 1992 TO JULY 31, 1993

DR. MACHINERY ACCOUNT CR.


$ ║ $
1/08/92 Balance b/f 300,000 ║ 31/08/92 Balance c/d 438,000
1/08/92 Bank A/C 138,000 ║
438,000 ║ 438,000
======= ║ =======

1/09/92 Balance b/d 438,000 ║ 30/09/92 Balance c/d 438,000
======= ║ =======

1/10/92 Balance b/d 438,000 ║ 31/10/92 Balance c/d 438,000
======= ║ =======

1/11/92 Balance b/d 438,000 ║ 30/11/92 Balance c/d 438,000
======= ║ =======

1/12/92 Balance b/d 438,000 ║ 31/12/92 Balance c/d 438,000
======= ║ =======

1/01/93 Balance b/d 438,000 ║ 31/01/93 Balance c/d 438,000
======= ║ =======

1/02/93 Balance b/d 438,000 ║ 28/02/93 Balance c/d 438,000
======= ║ =======

1/03/93 Balance b/d 438,000 ║ 31/03/93 Balance c/d 438,000
======= ║ =======

1/04/93 Balance b/d 438,000 ║ 30/04/93 Balance c/d 438,000
======= ║ =======

1/05/93 Balance b/d 438,000 ║ 31/05/93 Balance c/d 438,000
======= ║ =======

1/06/93 Balance b/d 438,000 ║ 30/06/93 Balance c/d 438,000
======= ║ =======

1/07/93 Balance b/d 438,000 ║ 31/07/93 Balance c/f 438,000
======= ║ =======

202

DR. ALLOWANCE FOR DEPRECIATION ON MACHINERY ACCOUNT CR.


$ ║ $
31/08/92 Balance c/d 153,000 ║ 1/08/92 Balance b/f 150,000
║ 31/08/92 Dep. A/C 3,000
153,000 ║ 153,000
======= ║ =======

30/09/92 Balance c/d 156,000 ║ 1/09/92 Balance b/d 153,000
║ 30/09/92 Dep. A/C 3,000
156,000 ║ 156,000
======= ║ =======

31/10/92 Balance c/d 159,000 ║ 1/10/92 Balance b/d 156,000
║ 31/10/92 Dep. A/C 3,000
159,000 ║ 159,000
======= ║ =======

30/11/92 Balance c/d 162,000 ║ 1/11/92 Balance b/d 159,000
║ 30/11/92 Dep. A/C 3,000
162,000 ║ 162,000
======= ║ =======

31/12/92 Balance c/d 165,000 ║ 1/12/92 Balance b/d 162,000
║ 31/12/92 Dep. A/C 3,000
165,000 ║ 165,000
======= ║ =======

31/01/93 Balance c/d 168,000 ║ 1/01/93 Balance b/d 165,000
║ 31/01/93 Dep. A/C 3,000
168,000 ║ 168,000
======= ║ =======

28/02/93 Balance c/d 171,000 ║ 1/02/93 Balance b/d 168,000
║ 28/02/93 Dep. A/C 3,000
171,000 ║ 171,000
======= ║ =======

31/03/93 Balance c/f 174,000 ║ 1/03/93 Balance b/d 171,000
║ 31/03/93 Dep. A/C 3,000
174,000 ║ 174,000
======= ║ =======

203

DR. ALLOWANCE FOR DEPRECIATION ON MACHINERY ACCOUNT CR.


$ ║ $
30/04/93 Balance c/d 177,000 ║ 1/04/93 Balance b/f 174,000
║ 30/04/93 Dep. A/C 3,000
177,000 ║ 177,000
======= ║ =======

31/05/93 Balance c/d 180,000 ║ 1/05/93 Balance b/d 177,000
║ 31/05/93 Dep. A/C 3,000
180,000 ║ 180,000
======= ║ =======

30/06/93 Balance c/d 183,000 ║ 1/06/93 Balance b/d 180,000
║ 30/06/93 Dep. A/C 3,000
183,000 ║ 183,000
======= ║ =======

31/07/93 Balance c/f 186,000 ║ 1/07/93 Balance b/d 183,000
║ 31/07/93 Dep. A/C 3,000
186,000 ║ 186,000
======= ║ =======

204

EXERCISE 2

4 + 3 + 2 +1 = 10

Depreciable cost = Capitalised Cost – Residual Value


= $220,000 - $20,000
= $200,000

Depreciation charge:

2001 = $200,000 X 4/10 = $80,000

2002 = $200,000 X 3/10 = $60,000

2003 = $200,000 X 2/10 = $40,000

2004 = $200,000 X 1/10 = $20,000


205

Ch8
CHAPTER EIGHT

FINAL ACCOUNTS AND FINANCIAL


STATEMENTS

CHAPTER OBJECTIVES
After completing this chapter you should be able to:

 Prepare an Adjusted Trial Balance.

 Distinguish between closing entries and opening entries.

 Explain the purpose of an Accountant's Worksheet.

 Prepare a Trading Account.

 Prepare a Profit and Loss Account.

 Prepare a Balance Sheet.

 Explain the term accounting equation.


206

INTRODUCTION
The financial statements for a manufacturing enterprise include the
Manufacturing Account, Trading Account, Profit and Loss Account, and
Balance Sheet. The financial statements for a trading enterprise
include the Trading Account, Profit and Loss Account, and Balance
Sheet. The financial statements for a service enterprise include the
Profit and Loss Account, and Balance Sheet.

Final Accounts for a manufacturing enterprise refer to the


Manufacturing Account, the Trading Account, and the Profit and Loss
Account. Final Accounts for a trading enterprise refer to the
Trading Account, and the Profit and Loss Account. The Final Account
for a service enterprise refers to the Profit and Loss Account.
Please note that the Balance Sheet is not an account.

ADJUSTED TRIAL BALANCE


The first Trial Balance that is extracted from the General Ledger at
the end of the accounting period is referred to as the Unadjusted
Trial Balance. The Trial Balance extracted from the General Ledger
at the end of the accounting period after the adjusting entries have
been posted is referred to as the Adjusted Trial Balance. The
Adjusted Trial Balance is the Trial Balance used to prepare the
financial statements.

CLOSING ENTRIES
All Nominal Accounts in the General Ledger are closed by means of
journal entries at the end of the financial year. The process of
closing these accounts is called closing entries. Let us assume that
J. Golddigger's financial year ended on January 31, 2004.

All accounts relating to the Trading Account will be closed and


transferred to the Trading Account at the end of the financial year,
such as the Opening Stock Account, the Cash Purchases Account, the
Credit Purchases Account, the Purchases Returns Account, the
Carriage Inward Account, the Cash Sales Account, the Credit Sales
Account and the Sales Returns Account. The closing stock is brought
into the accounts by debiting the Closing Stock Account and
crediting the Trading Account.
207

All Expense Accounts and Income Accounts are closed and transferred
to the Profit and Loss Account at the end of the financial year. The
Drawings Account in a sole trader type of business is closed and
transferred to the Capital Account. A Capital Account is a Liability
Account reflecting the amount of investment in the business by it
owner(s). For a company the Capital Account is referred to as the
Issued Share Capital Account. The capital invested by shareholders
in a company includes the capital reserves and revenue reserves.

J. Golddigger's closing stock on January 31, 2004 amount to $23,000.


The furniture was purchased on January 10, 2004 for use in the
business. The furniture has an estimated useful life of 10 years
with an estimated residual value of $2,000. It is the policy of the
business to depreciate furniture on the straight line basis. A full
month’s depreciation is to be charged for a fraction of a month in
the year of acquisition or disposal. No depreciation was provided on
the furniture. During January 2004, J. Golddigger withdrew goods
from the business at cost for personal use amounting to $3,000 and
this transaction was not recorded in the books. Please refer to J.
Golddigger's Trial Balance on page 122.

Annual Depreciation charge = $20,000 - $2,000


10
= $18,000

Depreciation for January 2004 = $18,000


12
= $150
208

Adjusting entries for J. Golddigger.

J. GOLDDIGGER GENERAL JOURNAL

DATE PARTICULARS FOLIO DR. CR.


2004 $ $
Jan. 31 Closing Stock A/C 1106 23,000
Trading A/C 3301 23,000
Being closing stock for
January 2004 now reflected
in the accounts.
Jan. 31 Depreciation A/C 4210 150
Allowance for depreciation
On Furn. & Fixt. A/C 2302 150
Being depreciation charge
on furniture for the month
of January 2004.
Jan. 31 Drawings A/C 2402 3,000
Credit Purchases A/C 4102 3,000
Being goods withdrawn at
cost by the owner now
reflected in the accounts.
209

The following journal entries represent the closing entries for J.


Golddigger assuming the financial year ended on January 31, 2004.
Please remember that when the adjusting entries are posted the
balance in the Drawings Account will be $4,000 and the balance in
the Credit Purchases Account will be $46,000.

J. GOLDDIGGER GENERAL JOURNAL

DATE PARTICULARS FOLIO DR. CR.


2004 $ $
Jan. 31 Trading A/C 3301 46,300
Sales Returns A/C 3103 300
Credit Purchases A/C 4102 46,000
Being Trading A/C closing
entries.
Jan. 31 Credit Sales A/C 3102 47,000
Purchases Returns A/C 4103 1,000
Trading A/C 3301 48,000
Being Trading A/C closing
entries.
Jan. 31 Discount Received A/C 3202 1,000
Profit & Loss A/C 3401 1,000
Being Profit & Loss A/C
income closing entry.
Jan. 31 Profit & Loss A/C 3401 22,010
Wages & Salaries A/C 4201 8,000
Rent Expense A/C 4202 10,000
Travelling A/C 4204 90
Electricity A/C 4208 3,000
Discount Allowed A/C 4209 500
Depreciation A/C 4210 150
Postage A/C 4211 20
Office Supplies A/C 4212 250
Being Profit & Loss A/C
expense closing entries.
Jan. 31 Capital A/C 2401 4,000
Drawings A/C 2402 4,000
Being Drawings A/C
transferred to Capital A/C.
210

ACCOUNTANT'S WORKSHEET
The Accountant's Worksheet is extremely important and helpful, but
it is not part of the permanent accounting records of a firm. The
Accountant's Worksheet is prepared in pencil so that any errors
detected can be easily corrected before preparing the permanent
financial statements. Accountants prepare worksheets in order to
organize the end of period accounting procedures in a logical
manner. This makes the adjusting and closing processes more
straightforward and easier to accomplish at the end of the financial
year. The Accountant's Worksheet is a trial run for the preparation
of the permanent financial statements.
211

J. Golddigger Accountant's Worksheet on January 31,2004

An analysis sheet is obtained (preferably a 14 column) and the


Unadjusted Trial Balance is written in pencil on this sheet or we
can use a spreadsheet. The next step is to record the adjustments
(see adjusting entries for J. Golddigger on page 208).

If an account has no adjustment and the balance in the Unadjusted


Trial Balance is a debit balance, then this amount is transferred to
the Adjusted Trial Balance on the debit side. If an account has no
adjustment and the balance in the Unadjusted Trial Balance is a
credit balance, then this amount is transferred to the Adjusted
Trial Balance on the credit side.
212

J. GOLDDIGGER ACCOUNTANT'S
A/C NAME OF ACCOUNT UNADJUSTED ADJUSTMENTS
# TRIAL BALANCE
DR. CR. DR. CR.
$ $ $ $
1101 Petty Cash Float A/C 640
1102 Cash A/C 9000
1103 Bank A/C 26500
1104 Accounts Receivable A/C 31700
1202 Furniture & Fixtures A/C 20000
2101 Accounts Payable A/C 38000
2102 Accruals A/C 3000
2103 Other Creditors A/C 20000
2401 Capital A/C 50000
2402 Drawings A/C 1000 3000
3102 Credit Sales A/C 47000
3103 Sales Returns A/C 300
3202 Discount Received A/C 1000
4102 Credit Purchases A/C 49000 3000
4103 Purchases Returns A/C 1000
4201 Wages & Salaries A/C 8000
4202 Rent Expense A/C 10000
4204 Travelling A/C 90
4208 Electricity A/C 3000
4209 Discount Allowed A/C 500
4211 Postage A/C 20
4212 Office Supplies A/C 250
160000 160000
1106 Closing Stock A/C 23000
3301 Trading A/C 23000
4210 Depreciation A/C 150
2302 Allowance for Depr. A/C 150
26150 26150
Net Profit
213

WORKSHEET ON JANUARY 31, 2004


ADJUSTED TRIAL INCOME STATEMENT BALANCE SHEET
BALANCE
DR. CR. DR. CR. ASSETS LIABILITIES
$ $ $ $ $ $
640 640
9000 9000
26500 26500
31700 31700
20000 20000
38000 38000
3000 3000
20000 20000
50000 50000
4000 4000
47000 47000
300 300
1000 1000
46000 46000
1000 1000
8000 8000
10000 10000
90 90
3000 3000
500 500
20 20
250 250

23000 23000
23000 23000
150 150
150 150
183150 183150
3690 3690
72000 72000 114840 114840
214

STEP BY STEP EXPLANATION OF J. GOLDDIGGER ACCOUNTANT'S WORKSHEET

The Drawings Account has a debit of $1,000 in the Unadjusted Trial


Balance and a $3,000 debit adjusting entry, making a total of $4,000
debit. The $4,000 debit is reflected in the Adjusted Trial Balance.

The Credit Purchases Account has a debit of $49,000 in the


Unadjusted Trial Balance and a $3,000 credit adjusting entry, making
a net debit of $46,000. This $46,000 debit is reflected in the
Adjusted Trial Balance. Accounts not in the Trial Balance are
created at the bottom of the Trial Balance, for example,
Depreciation Account.

Once the Adjusted Trial Balance is prepared you are no longer


concerned with the Unadjusted Trial Balance or the Adjustments.

An Income Statement reflects the net profit or net loss for a


particular accounting period. Expenses are debited in the Income
Statement and any income is credited in the Income Statement. For a
trading enterprise the Income Statement includes the Trading
Account, and the Profit and Loss Account. For a manufacturing
enterprise the Income Statement includes the Manufacturing Account,
the Trading Account, and the Profit and Loss Account. For a service
enterprise the Income Statement includes only the Profit and Loss
Account.

The Balance Sheet is not an account but for the purpose of preparing
the Accountant's Worksheet we can consider the asset side as the
debit side and the liability side the credit side. This question is
asked about each item in the Adjusted Trial Balance:

IS THIS ITEM AN INCOME STATEMENT ITEM OR A BALANCE SHEET ITEM ?

If the item is a Balance Sheet item and it has a debit balance in


the Adjusted Trial Balance the amount is transferred to the asset
(DR.) side of the Balance Sheet.

If the item is a Balance Sheet item and it has a credit balance in


the Adjusted Trial Balance the amount is transferred to the
liability (CR.) side of the Balance Sheet.

If the item is an Income Statement item and it has a credit balance


in the Adjusted Trial Balance the amount is transferred to the
credit side of the Income Statement.

If the item is an Income Statement item and it has a debit balance


in the Adjusted Trial Balance the amount is transferred to the debit
side of the Income Statement.
215

The closing stock represents an asset and it is transferred to the


asset side of the Balance Sheet.

After all the items are transferred from the Adjusted Trial Balance,
the debit side of the Income Statement is compared with the credit
side of the Income Statement. If the credit side exceeds the debit
side a net profit is made and this net profit is debited in the
Income Statement and credited (liability side) in the Balance Sheet.
In the Income Statement the total debits will now equal the total
credits, and in the Balance Sheet the total assets will now equal
the total liabilities. If the debit side of the Income Statement
exceeds the credit side a net loss is incurred and this net loss is
credited in the Income Statement and debited (asset side) in the
Balance Sheet. In the Income Statement the total debits will now
equal the total credits, and in the Balance Sheet the total assets
will now equal the total liabilities.
216

PREPARATION OF TRADING AND PROFIT AND LOSS ACCOUNT

TRADING ACCOUNT
The Trading Account is used by wholesalers and retailers to
determine the gross profit or the gross loss for the accounting
period.

PREPARATION OF TRADING ACCOUNT IN THE GENERAL LEDGER

DR. TRADING ACCOUNT CR.


$ ║ $
Opening stock ║ Cash Sales
Cash purchases ║ Credit sales
Credit purchases ║ Purchases returns
Carriage inwards ║ Closing stock
Sales returns ║

If the amount on the credit side of the Trading Account is greater


than the amount on the debit side of the Trading Account then the
firm will make a gross profit.

DR. TRADING ACCOUNT CR.


$ ║ $
Opening stock X ║ Cash Sales XX
Cash purchases X ║ Credit sales XX
Credit purchases X ║ Purchases returns X
Carriage inwards X ║ Closing stock X
Sales returns X ║
Gross profit X ║
XXXXXX ║ XXXXXX
====== ║ ======

217

If the amount on the debit side of the Trading Account is greater


than the amount on the credit side of the Trading Account then the
firm will incur a gross loss.

DR. TRADING ACCOUNT CR.


$ ║ $
Opening stock XX ║ Cash Sales XX
Cash purchases X ║ Credit sales X
Credit purchases X ║ Purchases returns X
Carriage inwards X ║ Closing stock X
Sales returns X ║ Gross loss X
XXXXXX ║ XXXXXX
====== ║ ======

TRADITIONAL METHOD FOR THE PREPARATION OF THE TRADING ACCOUNT

DR. TRADING ACCOUNT CR.


$ $ ║ $
Opening stock X ║ Cash sales XX
Cash purchases X ║ Credit sales XX
Credit purchases X ║ XXXX
xx ║ Less sales returns X
less purchases returns X ║ Net sales XXX
X ║
Add carriage inwards X ║
XX ║
XXX ║
Less closing stock X ║
Cost of sales XX ║
Gross profit X ║
XXX ║ XXX
=== ║ ===

If the cost of sales is less than the net sales, then the firm will
make a gross profit. If the cost of sales is greater than the net
sales, then the firm will incur a gross loss.

In order to determine the cost of sales, the sales returns is


deducted from the sales instead of being debited in the Trading
Account, the purchases returns is deducted from the purchases
instead of being credited in the Trading Account, and the closing
stock is deducted on the debit side of the Trading Account instead
of being credited in the Trading Account.
218

Carriage inward represents transportation cost paid for by the


business for transporting goods purchased from suppliers. Carriage
inward is reflected on the debit side of the Trading Account, since
it increases the cost of the goods purchased.

Carriage outward represents transportation cost paid for by the


business for transporting goods to customers. Carriage outward is a
selling expense and will therefore be reflected on the debit side of
the Profit and Loss Account and not in the Trading Account.

Both carriage inward and carriage outward are transportation costs


but carriage inward is reflected in the Trading Account, while
carriage outward is reflected in the Profit and Loss Account.

PROFIT AND LOSS ACCOUNT


The Profit and Loss Account is used to determine the net profit or
the net loss for the accounting period. In the Profit and Loss
Account all relevant expenses are debited and all relevant income is
credited. If the amount on the credit side is greater than the
amount on the debit side then the firm will make a profit (net
profit). If the amount on the debit side is greater than the amount
on the credit side then the firm will incur a loss (net loss).

If the firm makes a gross profit in the Trading Account then this
gross profit is credited in the Profit and Loss Account. If the firm
incurs a gross loss in the Trading Account then this gross loss is
debited in the Profit and Loss Account.

EXAMPLE OF PROFIT AND LOSS ACCOUNT WITH A NET PROFIT

DR. PROFIT AND LOSS ACCOUNT CR.


$ ║ $
Wages & salaries 10,000 ║ Gross profit (transfer
Discount allowed 500 ║ from Trading A/C) 20,000
Office expenses 2,500 ║ Discount received 1,000
Electricity 2,000 ║ Rental income 5,000
Postage 100 ║ Commission income 2,000
Depreciation 1,900 ║ Dividend income 1,000
Bad debt 2,000 ║
Net profit 10,000 ║
29,000 ║ 29,000
====== ║ ======

Please NOTE that the figures used above are chosen arbitrarily.
219

EXAMPLE OF PROFIT AND LOSS ACCOUNT WITH A NET LOSS

DR. PROFIT AND LOSS ACCOUNT CR.


$ ║ $
Wages & salaries 70,000 ║ Gross profit (transfer
Depreciation 10,000 ║ from Trading A/C) 50,000
Postage 1,000 ║ Rental income 10,000
║ Net loss 21,000
81,000 ║ 81,000
====== ║ ======

Please NOTE that the figures used above are chosen arbitrarily.
The Trading Account and the Profit and Loss Account can be combined
or shown separately.

Combined Trading and Profit and Loss Account for J. Golddigger.

J. GOLDDIGGER TRADING AND PROFIT AND LOSS ACCOUNT


FOR THE YEAR ENDED JANUARY 31, 2004

$ ║ $
Credit purchases (net) 46,000 ║ Credit sales 47,000
Less purchases returns 1,000 ║ Less sales returns 300
45,000 ║ 46,700
Less closing stock 23,000 ║
Cost of sales 22,000 ║
Gross profit c/d 24,700 ║
46,700 ║ 46,700
====== ║ ======

Wages and salaries 8,000 ║ Gross profit b/d 24,700
Rent expense 10,000 ║ Discount received 1,000
Travelling 90 ║
Electricity 3,000 ║
Depreciation 150 ║
Discount allowed 500 ║
Postage 20 ║
Office Supplies 250 ║
Net profit 3,690 ║
25,700 ║ 25,700
====== ║ ======

220

The combined Trading and Profit and Loss Account for a business that
is involved in the buying and selling of goods is sometimes referred
to as an Income Statement. If the business manufactures goods then
the Income Statement would comprise the Manufacturing Account, the
Trading Account, and the Profit and Loss Account. If the business
enterprise is strictly a service business then the Income Statement
would reflect only the Profit and Loss Account.

Did you realize that the Income Statement and the Final Accounts are
one and the same?

If yes, congrats! You deserve ten (10) minutes break.

If no, you need to re-read the chapter before you take a break.
Remember, never give up.
221

PREPARATION OF BALANCE SHEET


The Balance Sheet shows the financial position of the business at
the end of the accounting period. All assets and liabilities at the
end of the accounting period are reflected in the Balance Sheet.
Several different formats can be used to prepare the Balance Sheet.
The traditional format for preparing the Balance Sheet is to have
all assets on the right hand side and all liabilities on the left
hand side.

The Balance Sheet is a financial statement and not an account. Since


the Balance Sheet is a financial statement and not an account, the
assets can be on the left hand side and the liabilities on the right
hand side or the assets can be on the right hand side and the
liabilities can be on the left hand side.

SAMPLE BALANCE SHEET


$ $ ║ $ $
Capital XXXX ║ Non-current Assets
Add net profit XX ║ Machinery XX
XXXXXX ║ Less allow. for dep. X
Less drawings X ║ X
XXXXX ║ Furniture XX
Current Liabilities ║ Less allow. for dep. X
Trade creditors X ║ X
Accruals X ║ XX
Bills Payable X ║ Current Assets
XXX ║ Trade debtors XXX
║ Less allow. for
║ for bad debts X
║ XX
║ Closing stock X
║ Cash and bank X
║ Bills receivable X
║ Prepayments X
║ XXXXXX
XXXXXXXX ║ XXXXXXXX
======== ║ ========

If a net loss is incurred it is subtracted from the Capital Account.


222

A bills receivable represents a current asset to the firm at the end


of the accounting period while a bills payable represents a current
liability to the firm at the end of the accounting period. Bills
receivable and bills payable are both bills of exchange.

A bills receivable is a written undertaking by a trade debtor to pay


a trade debt within a specific period of time. This period is less
than one year, for example, 30 days or 60 days. When the debtor
signs the bills receivable the trade debt is converted into another
form of current asset, a bills receivable. For example, if T. Heidi
is a trade debtor owing the business $1,000 and signs a bills
receivable to pay the $1,000 in 90 days, then instead of reflecting
a trade debtor of $1,000 in the Balance Sheet a bills receivable of
$1,000 is reflected.

A bills payable is a written undertaking by the business to pay a


trade debt within a specific period of time. This period is less
than one year. When the agent for the business signs the bills
payable the trade debt (trade creditor) is converted into another
form of current liability, a bills payable.

Impact of transactions on the balance sheet

Every transaction will affect the composition of the balance


sheet. In order to demonstrate this we will start with a simple
balance sheet and see how the following transactions affect this
balance sheet. We will modify this balance sheet one transaction
at a time.

John Sweet Balance Sheet as at November 29, 2003


$ $
Capital 50,000 Non-current Assets
Furniture 20,000
Current Liabilities
Trade Creditors 15,000 Current Assets
Accruals 5,000 Stock 12,000
20,000 Bank 30,000
Cash 8,000
50,000

70,000 70,000
223

Transactions on November 29, 2003


1. Purchase of goods on credit for $2,000.
2. Purchase of goods by cheque for $1,000.
3. Paid $4,000 cash for accruals.
4. Purchase of motor vehicle for $15,000 by cheque for use in the
business.
5. Additional capital introduced by the owner in cash amounting to
$20,000.

Transaction number 1 the purchase of goods on credit for $2,000.

John Sweet Balance Sheet as at November 29, 2003


$ $
Capital 50,000 Non-current Assets
Furniture 20,000
Current Liabilities
Trade Creditors 17,000 Current Assets
Accruals 5,000 Stock 14,000
22,000 Bank 30,000
Cash 8,000
52,000

72,000 72,000

The purchase of $2,000 worth of goods on credit has resulted in


the increase of liabilities by $2,000 and an increase of assets
by $2,000. The balance sheet items affected are:
Liability item - Trade creditors increase by $2,000
Asset item – Stock increase by $2,000

Transaction number 2 the purchase of goods by cheque for $1,000.

John Sweet Balance Sheet as at November 29, 2003


$ $
Capital 50,000 Non-current Assets
Furniture 20,000
Current Liabilities
Trade Creditors 17,000 Current Assets
Accruals 5,000 Stock 15,000
22,000 Bank 29,000
Cash 8,000
52,000

72,000 72,000
224

The purchase of $1,000 worth of goods by cheque has not resulted


in any change in the total assets or any change in the total
liabilities. The balance sheet items affected are:
Asset item - stock increase by $1,000
Asset item - bank decrease by $1,000

Transaction number 3 paid $4,000 cash for accruals.

John Sweet Balance Sheet as at November 29, 2003


$ $
Capital 50,000 Non-current Assets
Furniture 20,000
Current Liabilities
Trade Creditors 17,000 Current Assets
Accruals 1,000 Stock 15,000
18,000 Bank 29,000
Cash 4,000
48,000

68,000 68,000

The payment of $4,000 cash for accruals has resulted in the


decrease of liabilities by $4,000 and the decrease of assets by
$4,000. The balance sheet items affected are:

Liability item – accruals decrease by $4,000


Asset item – cash decrease by $4,000

Transaction number 4 purchase of motor vehicle for $15,000 by


cheque for use in the business.

John Sweet Balance Sheet as at November 29, 2003


$ $
Capital 50,000 Non-current Assets
Furniture 20,000
Current Liabilities Motor Vehicle 15,000
Trade Creditors 17,000 35,000
Accruals 1,000
18,000 Current Assets
Stock 15,000
Bank 14,000
Cash 4,000
33,000

68,000 68,000
225

The purchase of a motor vehicle for $15,000 by cheque has not


resulted in any change in the total assets or any change in the
total liabilities. The balance sheet items affected are:

Asset item - motor vehicle increase by $15,000


Asset item – bank decrease by $15,000

Transaction number 5 $20,000 additional capital introduced by


the owner in cash.

John Sweet Balance Sheet as at November 29, 2003


$ $
Capital 70,000 Non-current Assets
Furniture 20,000
Current Liabilities Motor Vehicle 15,000
Trade Creditors 17,000 35,000
Accruals 1,000
18,000 Current Assets
Stock 15,000
Bank 14,000
Cash 24,000
53,000

88,000 88,000

The introduction of $20,000 cash in additional capital has


resulted in the increase of liabilities by $20,000 and an
increase of assets by $20,000. The balance sheet items affected
are:

Liability item – capital increase by $20,000


Asset item – cash increase by $20,000
226

The Balance Sheet for J. Golddigger is shown below.

BALANCE SHEET OF J.
GOLDDIGGER AS AT JANUARY 31,2004
$ $ ║ $ $
Non-current Asset ║
Furniture & fixtures 20,000 ║ Capital 50,000
Less Accumulated depreciation 150 ║ Add net profit 3,690
19,850 ║ 53,690
Current Assets ║ Less drawings 4,000
Petty cash 640 ║ 49,690
Cash 9,000 ║
Bank 26,500 ║ Current Liabilities
Accounts receivable 31,700 ║ Accounts
Closing stock 23,000 ║ payable 38,000
90,840 ║ Accruals 3,000
║ Other creditor 20,000
║ 61,000
110,690 ║ 110,690
======= ║ =======

The $49,690 represents the closing balance in the Capital Account


after debiting drawings and crediting net profit in this account.
Accountants prefer to show in the Balance Sheet the opening capital
balance and the debit and credit entries in the Capital Account that
resulted in the closing Capital Account balance.
227

VERTICAL FORMAT FOR J. GOLDDIGGER BALANCE SHEET

BALANCE SHEET OF J. GOLDDIGGER AS AT JANUARY 31, 2004


$ $ $
Non-current Asset
Furniture & fixtures 20,000
Less Allowance for depreciation 150
19,850
Current Assets
Petty cash 640
Cash 9,000
Bank 26,500
Accounts receivable 31,700
Closing stock 23,000
90,840
Less Current Liabilities
Accounts payable 38,000
Accruals 3,000
Other creditor 20,000
61,000
Net current assets (working capital) 29,840
49,690
======

Financed By:
Capital 50,000
Add net profit 3,690
53,690
Less drawings 4,000
49,690
======

ACCOUNTING EQUATION
The Balance Sheet contains information in respect of the assets,
owners' equity and other liabilities. This information is often
expressed in the form of an equation known as the accounting
equation:

Assets = Liabilities + Owners' Equity

OR
Assets - Liabilities = Owners' Equity

The owners’ equity is the capital invested by the owners or owner.


J. Golddigger’s equity is $49,690.
228

OPENING ENTRIES
All Asset and Liability Accounts with balances at the end of the
financial year in the General Ledger are opened at the beginning of
the financial year by means of a journal entry. The journal entry is
then posted to the General Ledger in the new accounting period. The
process of opening the Asset Accounts and Liability Accounts at the
beginning of the financial year is called opening entries.

The following journal entry represents the opening entry for J.


Golddigger assuming the financial year begins on February 1, 2004.

J. GOLDDIGGER GENERAL JOURNAL FOR FEBRUARY 2004

DATE PARTICULARS FOLI DR. CR.


O
2004 $ $
Feb. 1 Furniture & Fixtures A/C 1202 20,000
Petty Cash A/C 1101 640
Cash A/C 1102 9,000
Bank A/C 1103 26,500
Accounts Receivable A/C 1104 31,700
Opening Stock A/C 4104 23,000
Capital A/C 2401 49,690
Accounts Payable A/C 2101 38,000
Accruals A/C 2102 3,000
Other Creditors A/C 2103 20,000
Allowance for Depreciation A/C 2302 150
110,840 110,840
======= =======
Being the opening balances in
the Asset and Liability
Accounts at the beginning of
the financial year.
229

LIMITATIONS OF THE INCOME STATEMENT

1. The Income Statement includes estimates, such as, allowance for


bad debts and depreciation. So the net profit or net loss
reflected in the Income Statement could be more or less
depending on the estimate.
2. When the Income Statement reflects a net profit some people
automatically assume that the business has enough cash to pay
its debts. Since non-cash income, such as, credit sales is
reflected in the Income Statement, a net profit does not mean
that cash is available to pay debts.
3. The Income Statement is a historical statement. It reflects the
net profit or net loss for a period in the past. The Income
Statement does not provide information on the future
profitability of the business.

LIMITATIONS OF THE BALANCE SHEET


1. The Balance Sheet is at a particular point in time, for
example, as at December 31, 2004. The user of the Balance
Sheet will receive it after this date. Let us assume that the
user receives the Balance Sheet on January 10, 2005. The
Balance Sheet will provide the user with the financial
position of the business on December 31, 2004 but not with
the current financial position on January 10, 2005.

2. The Balance Sheet does not reflect all the assets of the
business; for example, it does not reflect good customer
relations.
230

REVIEW QUESTIONS
(1) Distinguish between opening entries and closing entries.

(2) Distinguish between the Unadjusted Trial Balance and the


Adjusted Trial Balance.

(3)* Assets = $560,000


Liabilities = $410,000
What is the owners' equity?

(4) What is the purpose of an Accountant's Worksheet?

(5)* The Trading Account is used to determine the G_____________


P________________ or the G____________ L_____________.

(6)* The Profit and Loss Account is used to determine the


N________________ P______________ or the N_______________
L_________________.

(7)* The assets of a sole trader total $100,000 and the


liabilities other than capital amounted to $60,000.
Calculate the capital invested in the business by the owner.
231

EXERCISE 1 *

From the following information prepare a Trading Account for Grenada


Record Shop, for the period ended December 31, 2004.

$
Sales 235,400
Returns inward 400
Purchases 105,000
Returns outward 4,800
Carriage inward 2,700
Carriage outward 5,400
Opening inventory 25,000
Closing inventory 40,000

EXERCISE 2 *

From the following information prepare the Trading and Profit and
Loss Account for Gimme Me Bit Shop, for the year ended March 31,
2004.

$
Sales 850,000
Returns outward 10,000
Carriage outward 15,000
Purchases 300,000
Opening stock 100,000
Closing stock 50,000
Advertising 30,000
Depreciation 25,000
Insurance 18,000
Repairs and maintenance 12,000
Salaries 200,000
Bad debts 1,000
Interest expense 25,000
Rental income 60,000
232

EXERCISE 3 *

Trial Balance of Good Behaviour Enterprises on June 30, 2004.

ACCOUNT # NAME OF ACCOUNT DEBIT CREDIT


$ $
1101 Petty Cash Float A/C 1,000
1102 Bank Account #1 39,000
1103 Bank Account #2 80,000
1104 Accounts Receivable A/C 180,000
1105 Allowance for Bad Debts A/C 1,500
1107 Bills Receivable A/C 10,000
1201 Machines A/C 200,000
1202 Allowance for Depreciation
on Machines A/C 113,333
1203 Furniture & Fixtures A/C 70,000
1204 Allowance for Depreciation
on Furniture & Fixtures A/C 10,000
2101 Creditors A/C 125,000
2201 Loan A/C 100,000
2301 Capital A/C 168,167
2302 Drawings A/C 30,000
3101 Cash Sales A/C 280,000
3102 Credit Sales A/C 430,000
3103 Sales Returns A/C 5,000
3201 Rental Income A/C 26,000
3202 Discount Received A/C 4,000
4102 Credit Purchases A/C 400,000
4104 Opening Stock A/C 80,000
4201 Wages and Salaries A/C 60,000
4202 Rent Expense A/C 48,000
4203 Insurance A/C 47,000
4204 Travelling A/C 7,000
4205 Stationery A/C 22,000
4206 Interest Expense A/C 20,000
4207 Water Rates A/C 12,000
4208 Electricity A/C 24,000
4209 Discount Allowed A/C 1,000
1,297,000 1,297,000
========= =========
233

The following transactions have not been recorded or adjusted for in


the books of Good Behaviour Enterprises.

(1) Debtors include $10,000 in respect of goods sent to Popeye on


a sale or return basis at a mark-up of 33⅓%, and the goods
remained unsold at the year end date.

(2) The owner withdrew goods costing $5,000 for her own personal
use.

(3) A sales invoice for $30,000 was omitted from the Sales
Journal.

(4) A cheque for $1,000 received from Con Artist a debtor, and
lodged in Bank Account #1, was dishonoured by the bank.

(5) The allowance for bad debts should be 1% of debtors.

(6) Rental income for July 2004 amounting to $2,000 was received
in June 2004.

(7) A machine costing $50,000 that was purchased on March 10,


2001 was sold for $90,000 cash on June 26, 2004. The cash was
lodged in Bank Account #1.

(8) Travelling expense to be accrued $2,500.

(9) Insurance paid for January 2004 to December 1994 $40,000.

(10) Closing stock on June 30, 2004 $50,000.

(11) Provide for depreciation on machines and furniture and


fixtures.

ADDITIONAL DATA:
(a) The machines are being depreciated on the straight line basis
at a rate of 20% per annum on cost.

(b) The furniture and fixtures were purchased July 1, 2001 and are
being depreciated on the straight line basis.

(c) Depreciation policy - a full month's depreciation is charged


on property, plant and equipment acquired or sold during the
month.
234

(d) Loan repayment (principal) due within the next 12 months


$60,000.
(e) Round all calculations to the nearest dollar.

The following information was extracted from Good Behaviour


Enterprises chart of accounts.

Assets 1
Current assets 11
Non-current assets 12

Liabilities 2
Current liabilities 21
Non-current liabilities 22
Capital 23

Revenue 3
Sales 31
Other income 32

Expenses 4
Cost of sales 41
General and administrative expenses 42

Other Accounts

ACCOUNT # NAME OF ACCOUNT


1106 Closing Stock Account
1108 Prepayments Account
1109 Other Receivables Account
2102 Accruals Account
2103 Other Creditors Account
3203 Profit on Disposal of Assets Account
4101 Cash Purchases Account
4103 Purchases Returns Account
4210 Depreciation Account

REQUIRED:

(a) Prepare the Trading Statement and Profit and Loss Statement
for the financial year ended June 30, 2004 (vertical format).

(b) Prepare the Balance Sheet as at June 30, 2004 (vertical


format).
235

EXERCISE 4 *

From the following information prepare the Accountant's Worksheet


for Christmas Breeze on December 31, 2003, showing the Unadjusted
Trial Balance, the Adjustments, the Adjusted Trial Balance, the
Income Statement and the Balance Sheet. Identify three limitations
of an Income Statement.

ACCOUNT # NAME OF ACCOUNT $


2301 Capital Account 59,000
2302 Drawings Account 5,000
4102 Purchases Account 42,000
4103 Purchases Returns Account 2,000
3102 Sales Account 101,000
4104 Opening Stock Account 23,000
4201 Salaries Account 20,000
4202 Rent Account 5,000
4203 Insurance Account 4,000
4210 Other Expenses Account 1,000
1203 Office Furniture Account 20,000
1204 Allowance for Depreciation Account 4,000
1104 Debtors Account 50,000
1102 Bank Account 21,000
2101 Creditors Account 25,000

Additional information:

(a) Depreciation is on the straight line basis at 10% per annum on


cost.

(b) Travelling expenses not paid at December 31, 2003 $1,000.

(c) Insurance prepaid $500.

(d) Stock at December 31, 2003 $22,000.

Other Accounts

ACCOUNT # NAME OF ACCOUNT


1106 Closing Stock Account
1108 Prepayment Account
2102 Accrual Account
3301 Trading Account
4204 Travelling Expense Account
4209 Depreciation Account
236

ANSWERS TO SELECTED REVIEW QUESTIONS


(3) $150,000

(5) Gross Profit


Gross Loss

(6) Net Profit


Net Loss

(7) $40,000

EXERCISE 1

GRENADA RECORD SHOP TRADING ACCOUNT FOR THE PERIOD ENDED


DECEMBER 31,2004
$ $ ║ $
Opening stock 25,000 ║ Sales 235,400
Purchases 105,000 ║ Less returns
Less returns outward 4,800 ║ inward 400
100,200 ║ 235,000
Add carriage inward 2,700 ║
102,900 ║
127,900 ║
Less closing stock 40,000 ║
Cost of sales 87,900 ║
Gross profit 147,100 ║
235,000 ║ 235,000
======= ║ =======

Carriage outward is a selling expense and will be reflected in the


Profit and Loss Account, and not the Trading Account.
237

EXERCISE 2

GIMME ME BIT SHOP TRADING AND PROFIT AND LOSS ACCOUNT FOR THE YEAR
ENDED MARCH 31,2004
-------------------------------------------------------------------

$ $ ║ $
Opening stock 100,000 ║ Sales 850,000
Purchases 300,000 ║
Less returns outward 10,000 ║
290,000 ║
390,000 ║
Less closing stock 50,000 ║
Cost of sales 340,000 ║
Gross profit c/d 510,000 ║
850,000 ║ 850,000
======= ║ =======

Carriage outward 15,000 ║ Gross profit b/d 510,000
Advertising 30,000 ║ Rental income 60,000
Depreciation 25,000 ║ 570,000
Insurance 18,000 ║
Repairs & maintenance 12,000 ║
Salaries 200,000 ║
Bad debts 1,000 ║
Interest expense 25,000 ║
Net profit 244,000 ║
570,000 ║ 570,000
======= ║ =======
238

EXERCISE 3

WORKINGS
Let us look at the effect of each working to the Trial Balance
starting with working number 1.

WORKINGS NUMBER 1

Accounts affected:
Decrease Accounts Receivable A/C by $10,000
Decrease Credit Sales A/C by $10,000
Increase Closing Stock A/C by $ 7,500

The closing stock is increased by the cost price of the goods unsold
at the year end date.

Mark-up = 33⅓% = ⅓
Therefore Margin = ¼
Cost Price = Selling Price – Profit
= $10,000 - ($10,000 x ¼)
= $10,000 - $2,500
= $7,500

WORKINGS NUMBER 2
Increase Drawings A/C by $5,000
Decrease Credit Purchases A/C by $5,000

WORKINGS NUMBER 3
Increase Credit Sales A/C by $30,000
Increase Accounts Receivable A/C by $30,000

WORKINGS NUMBER 4
Increase Bank A/C #1 overdraft by $1,000 (in other words,
credit Bank A/C #1 with $1,000)
Increase Accounts Receivable A/C by $1,000
239

WORKINGS NUMBER 5

New allowance for bad debts


= ($180,000 - $10,000 + $30,000 + $1,000) x 1
100
= $201,000 x 1
100
= $2,010

To calculate the new allowance for bad debts, the accounts


receivable balance in the Trial Balance is first adjusted by all
items affecting the Accounts Receivable Account. $180,000 relates to
the accounts receivable balance in the Trial Balance; $10,000
relates to the Accounts Receivable Account in working number 1;
$30,000 relates to the Accounts Receivable Account in working number
3; $1,000 relates to the Accounts Receivable Account in working
number 4.

Increase in Allowance = New Allowance - Existing Allowance


= $2,010 - $1,500
= $510

WORKINGS NUMBER 6

Decrease Rental Income A/C by $2,000


Increase Other Creditors A/C by $2,000

You need to create an account for other creditors since none exists
in the Trial Balance.

WORKINGS NUMBER 7

Decrease Machine A/C by $50,000


Dr. Bank A/C #1 with $90,000

Disposal of Machine
$ $
Cost of machine 50,000
Less accumulated depreciation
March 2001 to June 2001 dep (4 months)
(4/12 x $50,000 x 20/100) 3,333
July 2001 to June 2004 dep (3 years)
(3 x $50,000 x 20/100) 30,000
33,333
Net book value (NBV) 16,667
Sales proceeds 90,000
Profit on disposal 73,333
======
240

Decrease Allowance for Depreciation on Machine A/C by $33,333

WORKINGS NUMBER 8

Increase Travelling Expense A/C by $2,500


Increase Accruals A/C by $2,500

You need to create an Accruals Account since none exists in the


Trial Balance.

WORKINGS NUMBER 9

Decrease Insurance A/C by $20,000


Increase Prepayments A/C by $20,000

The insurance paid for July 2004 to December 2004 (6 months)


represents a prepayment at the end of the financial year (June 30,
2004).

Prepayment = $40,000 x 6/12


= $20,000

WORKINGS NUMBER 10

Closing stock = $50,000 + $7,500


= $57,500

Refer to working number 1.


241

WORKINGS NUMBER 11

Depreciation on balance of machines = $150,000 x 20/100 = $30,000


Depreciation on machine sold = $ 50,000 x 20/100 = $10,000
$40,000
=======
A full month’s depreciation is charged on the machine that was sold
during the month of June. Therefore twelve (12) months depreciation
is charged on the machine that was sold on June 26, 2004.

Depreciation on furniture and fixtures

2 years depreciation has been provided to date - July 1, 2001 to


June 30, 2003. Therefore annual depreciation charge equals allowance
for depreciation divided by number of years.

Annual Depreciation Charge = $10,000  2


= $5,000

Total depreciation = Machine dep. + Furniture & fixtures dep.


= $40,000 + $5,000
= $45,000

Closing balance in Allowance for Depreciation on Machines Account

Opening balance $113,333


Less accumulated depreciation on machine sold $ 33,333
$ 80,000
Add depreciation charge for this financial year $ 40,000
$120,000
========

Closing balance in Allowance for Depreciation on Furniture and


Fixtures Account

Opening balance $10,000


Add depreciation charge for this financial year $ 5,000
$15,000
=======
ADDITIONAL DATA (d)
Total loan $100,000
Less current portion of long term loan $ 60,000
Non-current liability portion of long term loan $ 40,000
========
The $60,000 represents a current liability in the Balance Sheet.
242

GOOD BEHAVIOUR ENTERPRISES TRADING STATEMENT FOR THE YEAR ENDED


JUNE 30, 2004
------------------------------------------------------------------
$ $
Cash sales 280,000
Credit sales 450,000
730,000
Less sales returns 5,000
725,000
Less cost of sales
Opening stock 80,000
+ Purchases (net of drawings) 395,000
475,000
- Closing stock 57,500
417,500
Gross profit 307,500
=======

GOOD BEHAVIOUR ENTERPRISES PROFIT AND LOSS STATEMENT FOR THE YEAR
ENDED JUNE 30, 2004
------------------------------------------------------------------
$ $
Gross profit 307,500
Discount received 4,000
Rental income 24,000
335,500
Less operating expenses
Wages and salaries 60,000
Rent expense 48,000
Insurance 27,000
Travelling 9,500
Stationery 22,000
Interest expense 20,000
Water rates 12,000
Electricity 24,000
Discount allowed 1,000
Increase in allowance for bad debts 510
Depreciation 45,000
269,010
Trading profit 66,490
Profit on disposal of machine 73,333
Profit 139,823
=======

The disposal of the machine a non-current asset is not considered as


part of the trading activities of the business and is therefore
shown separately from the trading profit.
243

GOOD BEHAVIOUR ENTERPRISES BALANCE SHEET AS AT JUNE 30, 2004


-----------------------------------------------------------------
$ $ $
Non-current Assets
Machines 150,000
Less allow. for depreciation 120,000
30,000
Furniture and fixtures 70,000
Less allow. for depreciation 15,000
55,000
85,000
Current Assets
Petty cash float 1,000
Bank A/C #1 50,000
Bank A/C #2 80,000
Accounts receivable 201,000
Less allow. for bad debts 2,010
198,990
Bills receivable 10,000
Prepayments 20,000
Closing stock 57,500
417,490
Less Current Liabilities
Creditors 125,000
Current portion of long term loan 60,000
Other creditors 2,000
Accruals 2,500
189,500
Net current assets 227,990
Total assets less current liabilities 312,990
=======

Financed By:
Capital 168,167
Less drawings 35,000
133,167
Add net profit 139,823
272,990

Non-current liability
Long term loan 40,000
312,990
=======
244

EXERCISE 4

CHRISTMAS BREEZE ACCOUNTANT'S


A/C NAME OF ACCOUNT UNADJUSTED ADJUSTMENTS
# TRIAL BALANCE
DR. CR. DR. CR.
$ $ $ $
2301 Capital A/C 59000
2302 Drawings A/C 5000
4102 Purchases A/C 42000
4103 Purchases Returns A/C 2000
3102 Sales A/C 101000
4104 Opening Stock A/C 23000
4201 Salaries A/C 20000
4202 Rent A/C 5000
4203 Insurance A/C 4000 500
4210 Other Expenses A/C 1000
1203 Office Furniture A/C 20000
1204 Allow. for Dep. on Office 4000 2000
Furniture A/C
1104 Debtors A/C 50000
1102 Bank A/C 21000
2101 Creditors A/C 25000
191000 191000
4209 Depreciation A/C 2000
4204 Travelling Expense A/C 1000
2102 Accrual A/C 1000
1108 Prepayment A/C 500
1106 Closing Stock A/C 22000
3301 Trading A/C 22000
25500 25500
Net Profit
245

WORKSHEET ON DECEMBER 31, 2003


ADJUSTED TRIAL INCOME STATEMENT BALANCE SHEET
BALANCE
DR. CR. DR. CR. ASSETS LIABILITIES
$ $ $ $ $ $
59000 59000
5000 5000
42000 42000
2000 2000
101000 101000
23000 23000
20000 20000
5000 5000
3500 3500
1000 1000
20000 20000
6000 6000

50000 50000
21000 21000
25000 25000

2000 2000
1000 1000
1000 1000
500 500
22000 22000
22000 22000
216000 216000
27500 27500
125000 125000 118500 118500
246

EXERCISE 4 CONTINUED

The Income Statement includes estimates, such as, allowance for bad
debts and depreciation. So the net profit or net loss reflected in
the Income Statement could be more or less depending on the
estimate.

When the Income Statement reflects a net profit some people


automatically assume that the business has enough cash to pay its
debts. Since non-cash income, such as, credit sales is reflected in
the Income Statement, a net profit does not mean that cash is
available to pay debts.

The Income Statement is a historical statement. It reflects the net


profit or net loss for a period in the past. The Income Statement
does not provide information on the future profitability of the
business.
247

ch9
CHAPTER NINE

CASH FLOW STATEMENT

CHAPTER OBJECTIVES
After completing this chapter you should be able to:

 Distinguish between a Cash Flow Statement and a Funds Flow


Statement.

 Prepare a Cash Flow Statement using the direct method

 Prepare a Cash Flow Statement using the indirect method.


248

INTRODUCTION
In addition to the Final Accounts and Balance Sheet, accountants
usually prepare the Cash Flow Statement at the end of the financial
year. A Cash Flow Statement reflects the sources of cash and the
uses of cash. It indicates the increase or decrease in the cash and
cash equivalents balance for the accounting period. The Cash Flow
Statement replaces the Funds Flow Statement because the Funds Flow
Statement was not a good predictor of business failure. A Funds Flow
Statement reflects the sources of working capital and the
application of working capital. An adequate amount of working
capital (positive working capital) does not necessarily mean that
the business has enough cash to pay its debts, since current assets
include non-cash items such as stock and debtors. So accountants
prefer to prepare a Cash Flow Statement rather than a Funds Flow
Statement. The Cash Flow Statement will indicate all significant
sources and uses of cash, and the ability of the business to
generate positive future net cash inflows. It will also indicate the
ability of the business to pay short-term and long-term debts, and
the need to invest surplus funds or the need to obtain additional
financing.

Cash consists of cash on hand, cash at bank and cash demand


deposits. Cash equivalents are highly liquid short-term investments
with original maturities of three months or less, that can be
converted quickly into known amounts of cash and will be subject to
an insignificant amount of risk of changes in value, e.g. 30 day
fixed deposit. A short-term investment of stocks listed on a Stock
Exchange would not be a cash equivalent. The stocks may be readily
converted into cash but stocks listed on a Stock Exchange are
subject to a significant amount of risks.

The Cash Flow Statement can be prepared using the Direct Method or
the Indirect Method. Whichever method is used cash flows should be
classified into three headings, Operating Activities, Investing
Activities, and Financing Activities.

Operating Activities are the principal revenue generating activities


of a business and other activities that cannot be classified under
Investing Activities or Financing Activities.

Investing Activities relate to the acquisition and disposal of non-


current assets. Financing Activities relate to receipts of loan
capital and capital from owner(s) and the repayment of loan capital
and share capital. It also includes cash distribution of profits to
the owner(s), e.g. drawings and cash dividends.
249

DIRECT METHOD
The Direct Method of presentation is easier to understand by users
without any accounting knowledge. Let us prepare the Cash Flow
Statement for J. Golddigger for the month of January 2004. In order
to prepare this Cash Flow Statement we need the current period
Income Statement and Balance Sheet, and the opening Balance Sheet
for the period under review. Since this is the first month of
operation for the business each item in the opening Balance Sheet on
January 1, 2004 has a value of $0.

J. Golddigger Income Statement for January 2004


$
Sales (net) 46,700

Less Cost of Sales


Opening stock 0
Purchases (net) 45,000
45,000
Less closing stock 23,000
22,000

Gross profit 24,700


Discount received 1,000
25,700

Less Operating Expenses


Wages and salaries 8,000
Rent expense 10,000
Travelling 90
Electricity 3,000
Discount allowed 500
Postage 20
Depreciation 150
Office supplies 250
22,010

Profit 3,690
======
250

J. Golddigger Balance Sheet for 2004


January 31 January 1
$ $
Non-current Assets
Furniture and fixtures 20,000 0
Less allowance for depreciation 150 0
19,850 0

Current Assets
Closing stock 23,000 0
Accounts receivable 31,700 0
Bank 26,500 0
Cash 9,640 0
90,840 0

Total assets 110,690 0


======= ======

Current Liabilities
Accounts payable 38,000 0
Accruals 3,000 0
Other creditor 20,000 0
61,000 0

Owner’s Equity
Capital 50,000 0
Add profit 3,690 0
53,690 0
Less drawings 4,000 0
49,690 0

Total liabilities 110,690 0


======= ======
251

Workings

Operating Activities

Cash received from customers (see Cash Book page 82)


$
B. Back 1,000
L. Hand 13,500
14,500
======

Cash paid to suppliers (see Cash Book page 83)


$
W. Dandy 9,000
=====

Cash paid to and on behalf of employees (see Cash Book page 83)
$
Salaries 8,000
Wages 0
Income tax and other deductions 0
8,000
=====

Other payments
$
Rent (see Cash Book page 83) 10,000
Sundries (see Petty Cash Book page 87) 360
10,360
======

Investing Activities

Comparing the Balance Sheets for the two periods, furniture and
fixtures increased by, $20,000 - $0 = $20,000. Since no cash was
paid for this item during the current period, we will not reflect an
outflow of cash for this item under investing activities. Investing
and financing transactions that do not involve the use of cash or
cash equivalents in the current period should be excluded from the
Cash Flow Statement. Such transactions if material can be reflected
at the bottom of the Cash Flow Statement or in the Notes to the
Financial Statements.
252

Financing Activities

Comparing the Balance Sheets for the two periods, capital increased
by, $50,000 -$0 = $50,000. This increase represents an inflow.
Drawings increased by, $4,000 -$0 =$4,000. Only $1,000 of this
amount is cash drawings. The $1,000 represents an outflow. Any
outflow in the Cash Flow Statement is bracketed.

Comparing the Balance Sheets for the two periods:


Cash equivalents increase/(decrease) = $ 0
Cash increased by $ 9,640 - $0 = $ 9,640
Bank increased by $26,500 - $0 = $26,500
Net increase in cash and cash equivalents $36,140
=======

If the net cash flow for an activity is negative, then cash was used
by this activity. If the net cash flow for an activity is positive,
then cash was provided by this activity.
253

J. Golddigger Cash Flow Statement for the month of January 2004


$
Cash Flows from Operating Activities
Cash received from customers 14,500
Cash paid to suppliers ( 9,000)
Cash paid to and on behalf of employees ( 8,000)
Other cash payments (10,360)
Net cash used by operating activities (12,860)
======

Cash Flows from Investing Activities


Long-term receivables made 0
Long-term receivables liquidated 0
Acquisition of property, plant and equipment 0
Disposal of property, plant and equipment 0
Net cash (used)/provided by investing activities 0

Cash Flows from Financing Activities


Loan received 0
Loan payment 0
Additional capital received 50,000
Cash drawings ( 1,000)
Net cash provided by financing activities 49,000

Net increase in cash and cash equivalents 36,140


Opening cash and cash equivalents 0
Closing cash and cash equivalents 36,140
======

Represented by
Cash 9,640
Bank 26,500
Cash equivalents 0
Closing cash and cash equivalents 36,140
======

Furniture amounting to $20,000 was acquired on credit during January


2004.
254

The cash used or provided by each activity is combined to determine


the amount by which cash and cash equivalents increased or decreased
during the period. That is, -$12,860 + $0 + $49,000 = $ $36,140.

Cash Flow Statement Summary


Net cash used by operating activities ($12,860)
Net cash used/provided by investing activities 0
Net cash provided by financing activities $49,000
Net increase in cash and cash equivalents $36,140
=======

INDIRECT METHOD
The more popular method of presentation by accountants is the
Indirect Method. This method requires the user to have some
financial accounting knowledge in order to fully understand the
statement. Since the indirect method is more popular, we will
concentrate on this method of presentation.

If closing stock for 2003 is $25,000 and the closing stock for 2004
is $30,000, this will be an increase of $5,000 and an outflow of
cash of $5,000. Therefore any increase in a current asset item other
than cash and cash equivalents, represent an outflow of cash.

If closing trade debtors for 2003 is $50,000 and the closing trade
debtors for 2004 is $30,000, this will be a decrease in trade
debtors of $20,000 and an inflow of cash of $20,000. Therefore any
decrease in a current asset item other than cash and cash
equivalents, represent an inflow of cash.

If Closing trade creditors for 2003 is $10,000 and the closing trade
creditors for 2004 is $12,000, this will be an increase of $2,000
and an inflow of cash of $2,000. It is an inflow because trade
creditors are providing $2,000 additional short-term financing.
Therefore any increase in a current liability other than bank
overdraft, represent an inflow of cash.
255

If closing accrued expense payable for 2003 is $100,000 and the


closing accrued expense payable for 2004 is $60,000, this will be a
decrease of $40,000 and an outflow of cash of $40,000. Therefore any
decrease in a current liability item other than bank overdraft,
represent an outflow of cash.

Let us now summarize the treatment of changes in current asset and


current liability items other than cash, cash equivalent and bank
overdraft.

Increase in current asset Outflow


Decrease in current asset Inflow
Increase in current liability Inflow
Decrease in current liability Outflow

Depreciation represents a non-cash expense and it is debited in the


Profit and Loss Account and therefore reduces the profit.

$
Cash sales 100,000
Less cash purchases 60,000
Cash gross profit 40,000
Less depreciation 10,000
Profit 30,000
=======

The non-cash expense is added back to the profit in order to


determine the cash flow from operations, that is,
$30,000 + $10,000 = $40,000. Therefore any non-cash expense such as
loss on disposal is added to the profit in order to determine the
cash flow from operating activities. Non-cash income such as
gain/profit on disposal is credited in the Profit and Loss Account
and increases profit. This non-cash income is subtracted from profit
in order to determine the cash flow from operating activities.
256

Let us now prepare the Cash Flow Statement for J. Golddigger for the
month of January 2004 using the indirect method. Please refer to the
Income Statement on page 249 and the Balance Sheets on page 250.We
will start with the profit of $3,690 and add back the depreciation
of $150 a non-cash item. If a loss was made for the period then this
would represent and outflow and the loss figure would be in
brackets. Comparing the Balance Sheets for the two periods, drawings
increased by, $4,000 -$0 =$4,000. Only $1,000 of this amount is cash
drawings the balance represents drawings of goods. The drawings of
goods at cost amounting to $3,000 (refer to adjusting entries on
page 208) represent an outflow under operating activities.

Let us now determine the increase or decrease in operating assets.


Operating assets are all current assets except for, interest and
dividend receivable, cash and cash equivalents. Comparing the
Balance Sheets for the two periods, closing stock increased by,
$23,000 - $0 = $23,000. This increase represents an outflow so the
$23,000 will be in brackets in the Cash Flow Statement. Comparing
the Balance Sheets for the two periods, accounts receivable
increased by, $31,700 - $0 = $31,700. This increase represents an
outflow.

Let us now determine the increase or decrease in operating


liabilities. Operating liabilities are all current liabilities
except for interest payable, corporation tax payable and bank
overdraft. Comparing the Balance Sheets for the two periods,
accounts payable increased by, $38,000 - $0 = $38,000. This increase
represents an inflow so the $38,000 is not bracketed in the Cash
Flow Statement. Comparing the Balance Sheets for the two periods,
accruals increased by, $3,000 - $0 = $3,000. This increase
represents an inflow. We will ignore the increase in other creditors
amounting to $20,000 because this was a liability directly related
to the acquisition of furniture for $20,000. No cash was paid for
the furniture.

Let us now look at financing activities. Comparing the Balance


Sheets for the two periods, capital increased by, $50,000 - $0 =
$50,000. This increase represents an inflow. Drawings increased by,
$4,000 - $0 = $4,000. This increase represents an outflow. The
outflow is allocated as follows: Drawings of good $3,000 operating
activities and Drawings of cash $1,000 financing activities
257

J. Golddigger Cash Flow Statement for the month of January 2004


$
Cash Flows from Operating Activities
Profit 3,690
Adjustment for non-cash items
Depreciation 150
Drawings of goods (3,000)
(Increase)/Decrease in operating assets
Stock (23,000)
Accounts receivable (31,700)
Increase/(Decrease) in operating liabilities
Accounts payable 38,000
Accruals 3,000
Net cash used by operating activities (12,860)

Cash Flows from Investing Activities


Long-term receivables made 0
Long-term receivables liquidated 0
Acquisition of property, plant and equipment 0
Disposal of property, plant and equipment 0
Net cash (used)/provided by investing activities 0

Cash Flows from Financing Activities


Loan received 0
Loan payment 0
Additional capital received 50,000
Cash drawings ( 1,000)
Net cash provided by financing activities 49,000

Net increase in cash and cash equivalents 36,140


Opening cash and cash equivalents 0
Closing cash and cash equivalents 36,140
======
Represented by
Cash 9,640
Bank 26,500
Cash equivalents 0
Closing cash and cash equivalents 36,140
======
Furniture amounting to $20,000 was acquired on credit during January
2004.
258

Whichever method is used to prepare the Cash Flow Statement, the net
cash used/provided by each activity should be the same. The
components of operating activities will be different for each
method, but the net cash flow will be the same. The components of
investing activities and financing activities will be the same for
both methods.
259

REVIEW QUESTIONS

(1) What is the difference between a Cash Flow Statement and a Funds
Flow Statement?

(2) Explain the term cash equivalents.

(3)* What are the three main headings in a Cash Flow Statement?

(4)* Complete the following sentences with outflow or inflow.

An increase in trade debtors is an ___________.

A decrease in closing stock is an ___________.

An increase in trade creditors is an ___________.

A decrease in accruals is an ___________.

A profit is an ___________.

A loss is an ___________.

(5) Explain the two methods of preparing a Cash Flow Statement.

Exercise 1 *

Jack Flash Income Statement for the year ended December 31, 2003
$
Sales 971,700
Less cost of sales 501,000
Gross profit 470,700
Less operating expenses 176,485
Profit 294,215
=======
260

Jack Flash Balance Sheet as at December 31


2003 2002
$ $

Non-current Assets
Furniture 100,000 80,000
Less allowance for depreciation 48,800 36,000
51,200 44,000

Current Assets
Closing stock 82,800 95,200
Insurance prepaid 45,000 40,000
Trade debtors (net) 267,795 160,500
Bank 132,500 121,600
Cash 10,200 6,800
538,295 424,100

589,495 468,100
======= =======

Current Liabilities
Trade creditors 45,600 210,000
Unearned income 10,000 -
Accruals 2,180 8,100
57,780 218,100

Owner’s Equity
Capital 250,000 120,000
Less drawings 12,500 20,000
237,500 100,000
Add profit 294,215 150,000
531,715 250,000

589,495 468,100
======= =======

Additional Information:
a. Depreciation provided for 2003 amounted to $12,800.
b. All drawings for 2003 were cash drawings.

Required: Prepare the Cash Flow Statement for the year ended
December 31, 2003.
261

EXERCISE 2 *

J. Golddigger Balance Sheet for 2004


February 29 January 1
$ $
Non-current Assets
Furniture and fixtures 22,000 20,000
Less allowance for depreciation 315 150
21,685 19,850

Current Assets
Closing stock 12,200 23,000
Accounts receivable 18,360 31,700
Investments in stocks 5,000 0
Deposit on call 10,000 0
Bank 2,200 26,500
Cash 2,030 9,640
49,790 90,840

Total assets 71,475 110,690


====== =======

Current Liabilities
Accounts payable 16,400 38,000
Accruals 3,100 3,000
Other creditor 0 20,000
19,500 61,000

Owner’s Equity
Capital 49,690 50,000
Add profit 3,285 3,690
52,975 53,690
Less drawings 1,000 4,000
51,975 49,690

Total liabilities 71,475 110,690


====== =======
262

J. Golddigger Income Statement for February 2004


$
Sales (net) 51,370

Less Cost of Sales


Opening stock 23,000
Purchases (net) 15,500
38,500
Less closing stock 12,200
26,300

Gross profit 25,070


Discount received 200
25,270

Less Operating Expenses


Wages and salaries 8,000
Rent expense 10,000
Travelling 100
Electricity 3,200
Discount allowed 400
Postage 20
Depreciation 165
Office supplies 100
21,985

Profit 3,285
======

Additional information:
a. Investment in stocks listed on the stock exchange amounted
to $5,000.
b. Cash drawings during the period amounted to $1,000.
c. The $20,000 paid to the other creditor was for furniture
acquired on credit in January 2004.

Required: Prepare the Cash Flow Statement for the month ended
February 29, 2004.
263

ANSWERS TO SELECTED REVIEW QUESTIONS


(3) Operating Activities
Investing Activities
Financing Activities

(4) Outflow
Inflow
Inflow
Outflow
Inflow
Outflow
264

EXERCISE 1

Jack Flash Cash Flow Statement for the year ended December 31, 2003
$
Cash Flows from Operating Activities
Profit 294,215
Adjustment for non-cash items
Depreciation 12,800

(Increase)/Decrease in operating assets


Stock 12,400
Insurance prepaid ( 5,000)
Trade debtors (107,295)

Increase/(Decrease) in operating liabilities


Trade creditors (164,400)
Unearned income 10,000
Accruals ( 5,920)
Net cash provided by operating activities 46,800

Cash Flows from Investing Activities


Acquisition of furniture ( 20,000)
Net cash used by investing activities ( 20,000)

Cash Flows from Financing Activities


Cash drawings (12,500)
Net cash used by financing activities (12,500)

Net increase in cash and cash equivalents 14,300


Opening cash and cash equivalents 128,400
Closing cash and cash equivalents 142,700
=======

Represented by
Cash 10,200
Bank 132,500
Cash equivalents 0
Closing cash and cash equivalents 142,700
=======
265

EXERCISE 2

J. Golddigger Cash Flow Statement for the month of February 2004


$
Cash Flows from Operating Activities
Profit 3,285
Adjustment for non-cash items
Depreciation 165

(Increase)/Decrease in operating assets


Stock 10,800
Accounts receivable 13,340
Investments in stocks ( 5,000)

Increase/(Decrease) in operating liabilities


Accounts payable (21,600)
Accruals 100
Net cash provided by operating activities 1,090

Cash Flows from Investing Activities


Acquisition of furniture and fixtures (22,000)
Net cash used by investing activities (22,000)

Cash Flows from Financing Activities


Cash drawings ( 1,000)
Net cash used by financing activities ( 1,000)

Net decrease in cash and cash equivalents (21,910)


Opening cash and cash equivalents 36,140
Closing cash and cash equivalents 14,230
======

Represented by
Cash 2,030
Bank 2,200
Cash equivalents 10,000
Closing cash and cash equivalents 14,230
======
266

Workings

Acquisition of Furniture and Fixtures


See Balance Sheet $22,000 - $20,000 = $ 2,000
Amount paid to other creditors = $20,000

The cash equivalent is the $10,000 deposit on call.


267

Ch10
CHAPTER TEN

INCOMPLETE RECORDS

CHAPTER OBJECTIVES
After completing this chapter you should be able to:

 State the features of incomplete records.

 Calculate credit sales from incomplete records.

 Calculate credit purchases from incomplete records.

 Prepare an Income Statement from incomplete records.

 Prepare a Balance Sheet from incomplete records.


268

INTRODUCTION
According to the financial accounting cycle, business transaction
are recorded in books of original entry and then posted to the
general ledger. A trial balance is extracted from the general
ledger at the end of the accounting period. It is from this trial
balance that the Income Statement and Balance Sheet are prepared.
If any part of this cycle is not completed, the accounting records
will be incomplete. The following are some of the features of
incomplete accounting records:

1. Not all business transactions are recorded in a book of


original entry.

2. A complete general ledger with all personal and impersonal


accounts is not kept.

3. The double entry principle is violated. For example, a cash


account may be kept, but there is no corresponding debit or
credit.

4. Some transactions are not recorded, but retained only in the


memory of the owner.

5. The total debtors are obtained from adding all the unpaid
sales invoices in a folder.

6. The total creditors are obtained from adding all the unpaid
purchase invoices in a folder.

7. Some of the owner’s personal expenses are paid from the


business cash account and bank account.

8. No trial balance is extracted.

Some operations of small and medium size businesses keep little or


no accounting records. For such businesses, if we can establish the
assets and liabilities at the end of the accounting period, the
drawings during the accounting period, the capital introduced
during the accounting period, and the capital at the beginning of
the accounting period, it will be possible to determine the profit
or loss for the accounting period. Depending on how much
269

information is available it might be possible to prepare an Income


Statement for the accounting period.

CALCULATION OF PROFIT OR LOSS

Profit/(loss) = closing capital + drawings – additional capital introduced – opening capital

Let us assume that J. Golddigger did not keep any accounting


records for January 2004, but the following is known.

Opening Capital $50,000


Additional Capital Introduced $0
Drawings $4,000
Closing Capital $49,690

Profit/(loss) = closing capital + drawings – additional capital introduced – opening capital


= $49,690 + $4,000 – $0 - $50,000
= $3,690

The profit for J. Golddigger for January 2004 is $3,690 (see


page 219).
270

PREPARATION OF FINAL ACCOUNTS


Merchant XX started business with $50,000 cash in January 2004. The
merchant did not keep proper accounting records for January 2004,
but you are able to obtain the following information.

Furniture purchased on credit for use in the business $20,000. This


amount is still outstanding on January 31,2004. Electricity bill
unpaid on January 31, 2004 amounted to $3,000.

Expenses paid for in January 2004


$
Wages and Salaries 8,000
Rent 10,000

Travelling 90
Postage 20
Office Supplies 250
18,360

Sales returns during the period amounted to $300, and purchases


returns amounted to $1,000. Discount allowed to trade debtors
amounted to $500 and discounted received from trade creditors
amounted to $1,000. The merchant withdrew $1,000 cash and $3,000
goods at cost for personal use. The amount paid to trade creditors
amounted to $9,000 and $14,500 was received from trade debtors. The
merchant kept two books, one for credit customers and one for
credit suppliers. A perusal of the books indicated the following.

Trade debtors with balances on January 31, 2004


$
B. Back 5,900
N. Foot 16,000
E. Head 9,800
Total Account Receivable 31,700
271

Trade creditors with balances on January 31, 2004


$
R. Shandy 24,000
S. Pie 14,000
Total Accounts Payable 38,000

Depreciation of $150 should be charged. Petty cash on hand amounted


to $640 and cash on hand amounted to $9,000. Closing Stock on
January 31, 2004 amounted to $23,000.

From this information we will prepare a trading and profit and loss
account, and a balance sheet. Assume that the financial year end
date is January 31, 2004.

CALCULATION OF CREDIT SALES

Since the amount of credit sales for the month of January is not
known we will utilize the accounts receivable control account
principle to determine the credit sales. Since the business was
started in January 2004 the opening accounts receivable balance
would be $0.

This is how the Accounts Receivable Control Account would look


before the credit sales figure is calculated.

Accounts Receivable Control A/c


$ $
Balance B/f 0 Sales Returns 300
Credit Sales ? Discount Allowed 500
Cash and Bank 14,500
Balance C/f 31,700

The difference between the total debits and the total credits will
give us the missing credit sales figure.
272

Credit Sales = $47,000 - $0 = $47,000


This is how the Accounts Receivable Control Account would look
after the credit sales figure is calculated.

Accounts Receivable Control A/c


$ $
Balance B/f 0 Sales Returns 300
Credit Sales 47,000 Discount Allowed 500
Cash and Bank 14,500
Balance C/f 31,700
47,000 47,000

CALCULATION OF CREDIT PURCHASES

We will utilize the accounts payable control account principle to


determine the amount of credit purchases. Since the business was
started in January 2004 the opening accounts payable balance would
be $0.

This is how the Accounts Payable Control Account would look before
the credit purchases figure is calculated.

Accounts Payable Control Account A/c


$ $
Purchases Returns 1,000 Balance B/f 0
Discount Received 1,000 Credit Purchases ?
Cash and Bank 9,000
Balance C/f 38,000

The difference between the total debits and the total credits will
give us the missing credit purchases figure.
273

Credit Purchases = $49,000 - $0 = $49,000

This is how the Accounts Payable Control Account would look after
the credit purchases figure is calculated.

Accounts Payable Control Account A/c


$ $
Purchases Returns 1,000 Balance B/f 0
Discount Received 1,000 Credit Purchases 49,000
Cash and Bank 9,000
Balance C/f 38,000
49,000 49,000

Net credit purchases = credit purchases - drawings of goods


= $49,000 - $3,000
= $46,000

Receipts = $50,000 + $14,500 = $64, 500

Payments = $18,360 + $1,000 + $9,000 = $28,360

Cash and Bank Balance = $64,500 - $28,360


= $36,140

Bank balance = total balance – petty cash on hand – cash on hand


Bank balance = $36,140 - $640 - $9,000
= $26,500
274

PREPARATION OF TRADING AND PROFIT AND LOSS ACCOUNT

Merchant XX Trading and Profit and Loss Account


For the year ended January 31, 2004
$ $
Credit Purchases (Net) 46,000 Credit Sales 47,000
Less Returns 1,000 Less Returns 300
45,000 46,700
Less Closing Stock 23,000
Cost of Sales 22,000
Gross Profit C/d 24,700
46,700 46,700
Electricity 3,000 Gross Profit B/d 24,700
Wages and Salaries 8,000 Discount Received 1,000
Rent 10,000
Travelling 90
Postage 20
Office Supplies 250
Depreciation 150
Discount Allowed 500
Net Profit 3,690
25,700 25,700
275
PREPARATION OF BALANCE SHEET

Merchant XX Balance Sheet as at January 31,2004


$ $
Capital 50,000 Non-current
Asset
Less Drawings 4,000 Furniture 20,000
46,000 Less allowance
for depreciation 150
Add Profit 3,690 19,850
49,690
Current Assets
Cash & Bank 36,140
Current Trade Debtors 31,700
Liabilities
Other Creditors 20,000 Closing Stock 23,000
Accrued 90,840
Electricity 3,000
Trade Creditors 38,000

61,000

110,690 110,690

The single entry records have now been converted to the double
entry system. The balances in the balance sheet will form the
opening debit and credit balances on February 1, 2004.

Did you realize that Merchant XX is in fact J. Golddigger? The net


profit for Merchant XX is the same as the net profit for J.
Golddigger on page 219. The assets and liabilities for Merchant XX
are the same as the assets and liabilities for J. Golddigger on
page 226. It should be clear by now that even if proper accounting
records are not kept, it is still possible to prepare an Income
Statement and a Balance Sheet if certain vital information is
available.
276

If proper accounting records are not kept it is unlikely that the


operator(s) of the business will remember the details of every
transaction that was not recorded.
277

REVIEW QUESTIONS
(1) What are the features of incomplete records?

(2) Which account is used to determine credit purchases?

(3) Which account is used to determine credit sales?

EXERCISE 1 *

From the following information determine the net profit or net loss
for the financial year ended June 30, 2004.

Opening Capital $65,200


Additional Capital Introduced $50,000
Drawings $ 2,000
Closing Capital $98,200

EXERCISE 2 *

From the following information determine the net profit or net loss
for the financial year ended December 31, 2004.

Opening Capital $120,600


Additional Capital Introduced $100,000
Drawings $ 10,000
Closing Capital $285,600
278

EXERCISE 3 *

Christine Dell started business on April 1, 2004. At that date she


purchased for cash:
Motor vehicle $400,000
Furniture $ 30,000

On April 1, 2004 after purchasing the motor vehicle and furniture


she opened a business bank current account with $100,000.

On June 30, 2004 the assets of the business in addition to the motor
vehicle and furniture were:
Cash $ 5,000
Bank $38,600
Trade debtors $12,750
Stock $47,800

On June 30, 2004 the liabilities were:


Trade creditors $ 8,172
Loan (2006) $50,000

The loan that is to be repaid in the year 2006 was received on June
1, 2004. The interest rate on the loan is 15% per annum. Interest is
to be paid semi-annually on November 30, 2004 and on May 31, 2005.

Christine withdrew $4,000 per month from the business for her
private use. Insurance paid for April 1, 2004 to March 31, 2005
amounted to $24,000.

Depreciation should be provided on the straight line basis as


follows:
Motor vehicle 20% per annum on cost
Furniture 10% per annum on cost

Required:

a. Calculate Christine Dell capital on April 1, 2004.

b. Calculate the profit or loss for the business for the quarter
ended June 30, 2004.

c. Prepare Christine Dell Balance Sheet as at June 30, 2004.


279

EXERCISE 4 *

On April 1, 2003 Bob Expo capital was $181,000. Bob withdrew $1,000
cash each month for his personal use for the financial year ended
March 31, 2004. On November 13, 2003 Bob introduced additional
capital of $50,000.

List of balances on March 31, 2004


Cash 10,000
Bank 27,500
Trade debtors 12,000
Prepayments 1,950
Stock 26,150
Furniture & fittings 20,000
Computer equipment 50,000
Trade creditors 6,700
Accruals 1,100

Bob did not remember to charge 10% depreciation on the cost of


furniture and fittings and 20% depreciation on the cost of computer
equipment.

Required:

a. Prepare a statement to show Bob Expo profit or loss for the


year ended March 31, 2004.

b. Prepare Bob Expo Statement of Affairs on March 31, 2004.


280

ANSWERS TO SELECTED REVIEW QUESTIONS

EXERCISE 1

Profit/loss) = $98,200 + $2,000 - $50,000 - $65,200


Loss = ($15,000)

EXERCISE 2

Profit/loss) = $285,600 + $10,000 - $100,000 - $120,600


Profit = $75,000

EXERCISE 3

(a) Capital on April 1, 2004


Motor vehicle $400,000
Furniture $ 30,000
Bank $100,000
$530,000
========

(b) WORKINGS

Interest on loan accrued = ($50,000 X 0.15)/12 = $625

Drawings = $4,000 X 3 = $12,000

Insurance prepaid = ($24,000/12) X 9 = $18,000

Allowance for depreciation


Motor vehicle = ($400,000 X 0.20) X ¼ = $20,000

Furniture = ($30,000 X 0.10) X ¼ = $750


281

Assets = $400,000 + $30,000 + $5,000 + $38,600 + $12,750 + $47,800 + $18,000


= $552,150

Liabilities = $8,172 + $50,000 + $625 + $20,000 + $750


= $79,547

Closing capital = Assets – Liabilities


= $552,150 - $79,547
= $472,603

Profit/loss) for the quarter ended June 30, 2004

Profit/loss) = $472,603 + $12,000 - $530,000


Loss = ($45,397)
282

Christine Dell Balance Sheet as at June 30, 2004


----------------------------------------------------------
$ $
Non-current Assets
Motor Vehicle 400,000
Less Allowance for Depreciation 20,000
380,000

Furniture 30,000
Less Allowance for Depreciation 750
29,250
409,250

Current Assets
Prepayment 18,000
Stock 47,800
Trade Debtors 12,750
Bank 38,600
Cash 5,000
122,150

Less Current Liabilities


Trade creditors 8,172
Accrued interest 625
8,797

Net current assets 113,353


Total assets less current liabilities 522,603

Less Non-current Liabilities


Loan 50,000
472,603
=======

Financed by
Capital 530,000
Less Loss 45,397
484,603
Less Drawings 12,000
472,603
=======
283

EXERCISE 4

WORKINGS

Allowance for depreciation


Furniture & Fittings = $20,000 X 0.10 = $ 2,000

Computer Equipment = $50,000 X 0.20 = $10,000

Assets = $10,000 + $27,500 + $12,000 + $1,950 + $26,150 + $20,000 + $50,000


= $147,600

Liabilities = $6,700 + $1,100 + $2,000 + $10,000


= $19,800

Closing capital = Assets – Liabilities


= 147,600 - $19,800
= $127,800

Drawings = $1,000 X 12 = $12,000

Statement of profit/(loss) for the year ended March 31, 2004


$
Closing Capital 127,800
Add Drawings 12,000
139,800
Less Capital Introduced 50,000
89,800
Less Opening Capital 181,000
Loss ( 91,200)
=======
284

Bob Expo Statement of Affairs as at March 31, 2004


----------------------------------------------------------
$ $
Non-current Assets
Furniture & Fittings 20,000
Less Allowance for Depreciation 2,000
18,000

Computer Equipment 50,000


Less Allowance for Depreciation 10,000
40,000
58,000

Current Assets
Prepayment 1,950
Stock 26,150
Trade Debtors 12,000
Bank 27,500
Cash 10,000
77,600

Less Current Liabilities


Trade creditors 6,700
Accruals 1,100
7,800

Net current assets 69,800


127,800
=======

Financed by
Capital 181,000
Add Capital Introduced 50,000
231,000
Less Loss 91,200
139,800
Less Drawings 12,000
127,800
=======
285

Ch11
CHAPTER ELEVEN

BANK RECONCILIATIONS

CHAPTER OBJECTIVES
After completing this chapter you should be able to:

 State the reasons why the Bank Statement balance may be


different from the bank balance per Cash Book.

 Distinguish between the different types of cheques.

 Prepare a Bank Reconciliation Statement.


286

INTRODUCTION
Firms that operate a Current Account (Chequing Account) receive a
Bank Statement from the bank on a monthly basis. The bank balance
per Bank Statement at the end of the month usually does not agree
with the bank balance per Cash Book (Bank Account balance) at the
end of the same month. Bank reconciliation is done to reconcile the
Bank Statement balance to the General Ledger Bank Account balance.

REASONS FOR DIFFERENCES BETWEEN THE BANK BALANCE PER BANK STATEMENT
AND THE BANK BALANCE PER CASH BOOK

PAYMENTS RECORDED ON THE BANK STATEMENT BUT NOT YET RECORDED


IN THE CASH BOOK

(a) Bank charges for operating the account not yet recorded in the
firm's Cash Book.

(b) Interest on overdraft charged by the bank not yet recorded in


the firm's Cash Book.

(c) Standing order payments made by the bank on behalf of the firm
(for example, insurance payments) not yet recorded in the
firm's Cash Book.

(d) Payments made by the bank by means of direct transfer from the
firm's Current Account, which are not yet recorded in the
firm's Cash Book.

(e) Cheques lodged to the firm's Current Account that are


subsequently dishonoured and debited on the Bank Statement but
not yet recorded in the firm's Cash Book.
287

PAYMENTS RECORDED IN THE CASH BOOK BUT NOT YET RECORDED


ON THE BANK STATEMENT

Cheques drawn and credited in the Cash Book that have not yet been
presented to the bank for payment. These cheques are referred to as
unpresented cheques or outstanding cheques.

RECEIPTS RECORDED IN THE CASH BOOK BUT NOT YET RECORDED


ON THE BANK STATEMENT

Deposits made by the firm at the bank (deposits through the night
safe) and debited in the firm's Cash Book, but not yet recorded by
the bank. These deposits are usually referred to as deposits not yet
credited.

RECEIPTS RECORDED ON THE BANK STATEMENT BUT NOT YET RECORDED


IN THE CASH BOOK

Amounts received by the bank by means of direct transfer to the


firm's Current Account not yet recorded in the firm's Cash Book.

Errors in the firm's Cash Book or errors on the Bank Statement can
also cause the bank balance per Cash Book to disagree with the bank
balance per Bank Statement.

In the firm's Cash Book, receipts are debited and payments credited.
At the bank in the Current Account kept for the firm the opposite is
done, that is, receipts (lodgements) are credited and payments are
debited. This is so because if the firm lodges $10,000 to its
Current Account, from the bank's point of view it now owes the firm
$10,000. So when the bank receives the $10,000, the bank credits the
firm's Current Account. The credit is indicating that the bank owes
the firm $10,000.
288

When the Bank Statement is received,

(1) The important question to ask is, what am I reconciling from


to? Usually the bank balance per Bank Statement is being
reconciled to the bank balance per Cash Book.

In this case:
(a) Subtract payments recorded in the Cash Book but not yet
recorded on the Bank Statement.

(b) Add payments recorded on the Bank Statement but not yet
recorded in the Cash Book.

(c) Add lodgements recorded in the Cash Book but not yet
credited on the Bank Statement.

(d) Subtract lodgements recorded on the Bank Statement but not


yet recorded in the Cash Book.

(2) In reconciling from the bank balance per Bank Statement, if the
bank balance per Bank Statement is an overdraft (O/D), then the
opposite of (a) to (d) in 1 above is done. For example, add
payments recorded in the Cash Book but not yet recorded on the
Bank Statement.

(3) In reconciling from the bank balance per Cash Book to the bank
balance per Bank Statement then the opposite of (a) to (d) in
1 above is done.

(4) In reconciling from the bank balance per Cash Book, if the bank
balance per Cash Book is an overdraft, then the same principles
in (a) to (d) in 1 above apply. For example, subtract payments
recorded in the Cash Book but not yet recorded on the Bank
Statement.
289

SUMMARY OF THE PRINCIPLES ENUNCIATED IN 1 TO 4

(1) (2) (3) (4)


Reconciling Reconciling Reconciling Reconciling
From Bank From Bank From Cash From Cash
Statement Statement (O/D) Book Book (O/D)
Subtract Add Add Subtract
Add Subtract Subtract Add
Add Subtract Subtract Add
Subtract Add Add Subtract

All you need to remember is column 1, because column 4 is the same


as column 1, and columns 2 and 3 are the opposite of column 1.

The following four sentences are completed with information from the
relevant column.

__________________ payments recorded in the Cash Book only.

__________________ payments recorded on the Bank Statement only.

__________________ lodgements recorded in the Cash Book only.

__________________ lodgements recorded on the Bank Statement only.


290

So if we are reconciling from the Bank Statement, and the bank


balance per Bank Statement is an overdraft, then column 2 is
relevant and the sentences will be completed as follows:

Add payments recorded in the Cash Book only.

Subtract payments recorded on the Bank Statement only.

Subtract lodgements recorded in the Cash Book only.

Add lodgements recorded on the Bank Statement only.

RELEVANT TERMINOLOGY

Drawer - The person who signs the cheque.

Drawee - The bank on which the cheque is drawn.

Payee - The person or firm that should receive the money


from the cheque.

TYPES OF CHEQUES

OPEN CHEQUE

An open cheque is a cheque that is not crossed and can be `cashed'


at the bank.

CROSSED CHEQUE

A crossed cheque is a cheque that must be lodged in an account. It


cannot be `cashed'. The cross // is indicated at the top left hand
corner of the cheque.
291

RESTRICTED CROSSED CHEQUE

A restricted crossed cheque is a cheque that must be lodged in the


account of the payee only. So the restricted crossed cheque will
indicate /A/C payee only/ or /& CO/.

CANCELLED CHEQUE

If a mistake is made in writing a cheque, then this cheque must be


cancelled. A cancelled cheque is where the word 'cancelled' is
written across the cheque and it is stapled to the cheque stub. The
word 'cancelled' is also written on the cheque stub.

STALE DATED CHEQUE

A stale dated cheque is a cheque that is not lodged or cashed after


six (6) months. This cheque must be written back. If for example, a
cheque for $10,000 is drawn on January 1, 2004 by BB Ltd. and paid
to John Crow Ltd. for furniture to be used in the business, then the
Bank Account will be credited with $10,000 and the Furniture Account
debited with $10,000. If on July 1, 2004 (6 months later) the cheque
is not lodged or cashed by John Crow Ltd., then BB Ltd. must write
back the cheque, that is, debit the Bank Account. BB Ltd. will debit
the Bank Account with $10,000 and credit John Crow Ltd. Account with
$10,000 (indicating that $10,000 is still owed to John Crow Ltd.) A
new cheque will be issued to John Crow Ltd. only if John Crow Ltd.
requests a new cheque. A new cheque is not automatically issued when
the stale dated cheque is written back.

POSTDATED CHEQUE

A postdated cheque is a cheque that is issued now but bears some


future date. This cheque cannot be recorded in the Cash Book, or
cashed, or lodged until that future date. If BB Ltd. issued a cheque
today (September 1, 2004) to Insurance Ltd. for $60,000, with the
cheque bearing the date December 31, 2004, then Insurance Ltd.
cannot record this cheque in its Cash Book, or lodge, or cash this
cheque until December 31, 2004. BB Ltd. cannot record this cheque as
a payment in its cash Book until the future date, December 31, 2004.
292

DISHONOURED CHEQUE

A dishonoured cheque is a cheque which is not paid by the bank on


which it is drawn. The drawee can refuse to honour (to pay) a cheque
for several reasons that are listed below:

(1) The cheque is stale-dated.

(2) The cheque is post-dated.

(3) The words (amount stated in words) and figures disagree.

(4) Insufficient funds in the drawer's account to pay the cheque.


So the bank will state that you should `refer to drawer'. This
cheque is also referred to as a `Bounced Cheque'.

(5) Unauthorized signature. Where the person or one of the persons


signing the cheque is not authorized to do so, based on the
bank's records of the authorized signatures for the firm for
that Bank Account.

(6) Where the cheque is incomplete, for example, the date is


omitted.

CERTIFIED CHEQUE

A certified cheque is a cheque that is guaranteed payment by the


bank. When the bank certifies a cheque it takes the funds from the
firm's account and set it aside to honour (pay) the cheque. If BB
Ltd. had a Current Account balance of $100,000 at the bank on
January 31, 2004 and BB Ltd. certified a cheque for M. Williams for
$60,000, then the bank would deduct $60,000 from BB Ltd. Current
Account and set it aside to pay M. Williams.

M. Williams cashed the cheque on March 2, 2004. Even though M.


Williams did not cash the cheque on January 31, 2004, the balance in
BB Ltd. Current Account after the cheque is certified on January 31,
2004 (assuming that no other transaction takes place) will be
$40,000.
293

MANAGERS’ CHEQUE

Many banks no longer certify cheques, instead they require you to


purchase a managers’ cheque. A managers’ cheque is similar to a
certified cheque since it guarantees payment to the payee. The
difference between the certified cheque and the managers’ cheque is
that the certified cheque is drawn by the firm making the payment,
whereas the managers’ cheque is drawn by a bank and is used by the
firm to make a payment.

RECONCILIATION
From the following information prepare a Bank Reconciliation
Statement for BW Ltd. on August 31, 2004.

Bank balance as per Bank Statement on 31/8/04 $50,000


Bank balance as per Cash Book on 31/8/04 $63,100

Unpresented cheques
Cheque # Date of Cheque Amount
750 16/7/04 $1,000
810 21/8/04 $6,000
$7,000
======

Cheques and cash deposited on 31/8/04 and not yet credited by the
bank $20,000. Bank charges for August 2004 amounted to $100.

We will reconcile from the bank balance per Bank Statement to the
bank balance per Cash Book.

Subtract payments recorded in the Cash Book only.


Add payments recorded on the Bank Statement only.
Add lodgements recorded in the Cash Book only.
Subtract lodgements recorded on the Bank Statement only.
294

BANK RECONCILIATION STATEMENT FOR BW LTD. ON AUGUST 31, 2004


$
Bank balance as per Bank Statement 50,000
Less unpresented cheques (see listing below) 7,000
43,000
Add bank charges 100
43,100
Add deposits not yet credited 20,000
Bank balance per Cash Book 63,100
======

LISTING OF UNPRESENTED CHEQUES ON AUGUST 31, 1994


Cheque # Date of Cheque Amount
750 16/7/04 $1,000
810 21/8/04 $6,000
$7,000
======

Let us assume that the financial year for J. Golddigger ends on


December 31. The Bank Statement is usually received by the firm
about two weeks after the month end date. So J. Golddigger's Bank
Statement for January 2004 will be received sometime in February
2004. J. Golddigger's Bank Statement can be collected from the bank
on February 14, 2004, based on information received from the bank.
The management meeting for J. Golddigger's business is scheduled for
February 6, 2004. The Final Accounts and Balance Sheet for January
2004 is an item on the agenda (list of items to be dealt with at the
meeting). You are the accountant for J. Golddigger's business and
you are faced with two alternatives:

ALTERNATIVE 1

Postpone the balancing of the Cash Book for January 2004 until you
receive the Bank Statement. Inform the management committee that you
are awaiting the Bank Statement so the management meeting will be
rescheduled for a later date in February 2004.
295

ALTERNATIVE 2

Balance the Cash Book for January 2004 on February 1, 2004 and
prepare the Final Accounts and Balance Sheet for the meeting.

Since the Final Accounts and Balance Sheet are being prepared for
the period ended January 31, 2004 (which is during the financial
year) and not for the financial year ended December 31, 2004, then
the non-receipt of the Bank Statement is not material for the
preparation of the Final Accounts and Balance Sheet. If you were
preparing the Final Accounts and Balance Sheet for the financial
year ended December 31, 2004, then the receipt of the Bank Statement
would be relevant for making your decision. This is due to the fact
that you need to close the accounts for that financial year and such
things as bank charges and interest on overdraft for that financial
year (December 2004 bank charges and interest on overdraft) must be
reflected in the accounts before you close the accounts for that
financial year.

If you were preparing the Final Accounts and Balance Sheet for the
financial year ended December 31, 2004, alternative 1 would be
selected. But since you are preparing the Final Accounts and Balance
Sheet for January 2004 alternative 2 is selected. So January 2004
bank charges will be reflected in the Cash Book in February 2004.

Since this chapter deals with Bank Reconciliation the Final Accounts
and Balance Sheet will not be shown. Only relevant material relating
to the preparation of the Bank Reconciliation will be shown.
296

ALTERNATIVE 2

Let us refer to the Cash Book of J. Golddigger for January 2004.

J. GOLDDIGGER CASH BOOK

DR. RECEIPTS
DATE PARTICULARS FOLIO DISCOUNT CASH BANK
ALLOWED A/C A/C
2004 $ $
Jan. 2 Capital A/C-
J. Golddigger A/C# 2401 50,000
Jan. 2 Cash A/C C 40,000
Jan.25 B. Back A/C A/C# 1104001 1,000
Jan.25 L. Hand A/C A/C# 1104002 500 13,500

500 51,000 53,500


=== ====== =======
Feb. 1 Balance b/d 9,000 26,500
A/C# 4209
A/C# 1104

Please NOTE that the payment side of the Cash Book is on the
following page (page 297).
297

CR. PAYMENTS PAGE 1


DATE PARTICULARS FOLIO CHEQUE DISCOUNT CASH BANK
NUMBER RECEIVED A/C A/C
2004 $ $
Jan. 2 Bank A/C C 40,000
Jan. 5 Rent A/C A/C# 4202 001 10,000
Jan. 7 Petty Cash A/C A/C# 1101 1,000
Jan.23 W. Dandy A/C A/C# 210101 002 1,000 9,000
Jan.31 Salaries A/C A/C# 4201 003 8,000
Jan.31 Drawings A/C A/C# 2402 1,000
Jan.31 Balance c/d 9,000 26,500
1,000 51,000 53,500
===== ====== ======
A/C# 2101
A/C# 3202
298

On February 14, 2004, the accountant for J. Golddigger received the


following Bank Statement for January 2004 from the bank.

FOG COMMERCIAL BANK LTD.

DR. CR. BALANCE


$ $ $
02/1/04 Lodgement 40,000 40,000
25/1/04 Lodgement 13,500 53,500
26/1/04 002 9,100 44,400
31/1/04 Bank charge 400 44,000
31/1/04 003 7,800 36,200

When the Bank Statement for January 2004 is received it is compared


with the Cash Book for January 2004. The following symbols are used
to make the comparison:

/ Items recorded in the Cash Book and reflected on the Bank


Statement.

O/S Items recorded in the Cash Book but not yet reflected on the
Bank Statement.

C/B Items recorded on the Bank Statement but not yet reflected in
the Cash Book or errors that should be corrected in the Cash
Book.
299

The credits on the Bank Statement are compared with the debits in
the Bank Account in the Cash Book. The lodgement of $40,000 on
January 2 and the lodgement of $13,500 on January 25, are both
reflected in the Cash Book.

The debits on the Bank Statement are compared with the credits in
the Bank Account in the Cash Book. Cheque #002 is reflected on the
Bank Statement; therefore it is not an outstanding cheque. In the
Cash Book $9,000 is recorded for cheque #002 and $9,100 is reflected
on the Bank Statement for cheque #002. An examination of the paid
cheque which is returned to the business by the bank along with
January 2004 Bank Statement will reveal whether the bank or the firm
made the mistake. The examination of the returned cheque revealed
that the Cash Book is incorrect; the amount on the cheque is $9,100.
Since an adjustment must be made in the Cash Book in February 2004
for the difference of $100, beside the $9,100 on the Bank statement
C/B will be written. The difference of $100 represents a payment
recorded on the Bank Statement only.

The bank charge is not yet reflected in the Cash Book so beside the
$400 on the Bank Statement C/B will be written. The bank charge is a
payment recorded on the Bank Statement only.

Cheque #003 is reflected on the Bank Statement; therefore it is not


an outstanding cheque. In the Cash Book $8,000 is recorded for
cheque #003 and $7,800 is reflected on the Bank Statement for cheque
#003. An examination of the paid cheque revealed that the Cash Book
is incorrect; the amount on the cheque is $7,800. Since an
adjustment will be made in February 2004 for the difference of $200,
C/B is written beside the $7,800 on the Bank Statement. From the
point of view of the bank reconciliation the difference of $200
represents a payment recorded in the Cash Book only.

Cheque #001 is not recorded on the Bank Statement, so it is an


outstanding cheque. So beside the $10,000 for cheque #001 in the
Cash Book, O/S is written. The outstanding cheque or unpresented
cheque is a payment recorded in the Cash Book only.
300

After the comparison, the Cash Book and the Bank Statement will
appear as follows.

J. GOLDDIGGER CASH BOOK

DR. RECEIPTS
DATE PARTICULARS FOLIO DISCOUNT CASH BANK
ALLOWED A/C A/C
2004 $ $
Jan. 2 Capital A/C-
J. Golddigger A/C# 2401 50,000
Jan. 2 Cash A/C C 40,000 /
Jan.25 B. Back A/C A/C# 1104001 1,000
Jan.25 L. Hand A/C A/C# 1104002 500 13,500 /

500 51,000 53,500


=== ====== =======
Feb. 1 Balance b/d 9,000 26,500
A/C# 4209
A/C# 1104

Please NOTE that the payment side of the Cash Book is on the
following page (page 301).
301

CR. PAYMENTS PAGE 1


DATE PARTICULARS FOLIO CHEQUE DISCOUNT CASH BANK
NUMBER RECEIVED A/C A/C
2004 $ $
Jan. 2 Bank A/C C 40,000
Jan. 5 Rent A/C A/C# 4202 001 10,000 o/s
Jan. 7 Petty Cash A/C A/C# 1101 1,000
Jan.23 W. Dandy A/C A/C# 210101 002 1,000 9,000 c/b
Jan.31 Salaries A/C A/C# 4201 003 8,000 c/b
Jan.31 Drawings A/C A/C# 2402 1,000
Jan.31 Balance c/d 9,000 26,500
1,000 51,000 53,500
===== ====== ======
A/C# 2101
A/C# 3202
302

FOG COMMERCIAL BANK LTD.

DR. CR. BALANCE


$ $ $
02/1/04 Lodgement 40,000 / 40,000
25/1/04 Lodgement 13,500 / 53,500
26/1/04 002 9,100 C/B 44,400
31/1/04 Bank charge 400 C/B 44,000
31/1/04 003 7,800 C/B 36,200

Since we are reconciling from the Bank Statement, the principles in


1 apply.

Subtract payments recorded in the Cash Book only.


Add payments recorded on the Bank Statement only.
Add lodgements recorded in the Cash Book only.
Subtract lodgements recorded on the Bank Statement only.

BANK RECONCILIATION STATEMENT FOR J. GOLDDIGGER ON JANUARY 31, 2004


$ $
Bank balance per Bank Statement 36,200
Less
Unpresented cheque #001 10,000
Error in recording cheque #003 200
10,200
26,000
Add
Bank charge 400
Error in recording cheque #002 100
500
Bank balance per Cash Book 26,500
======
303

J. Golddigger's Cash Book transactions to February 14, 2004.

February 2, 2004 paid rent by cheque #004 $10,000


February 13, 2004 drawings by cheque #005 $ 800

J. Golddigger's Cash Book for February 2004, with transactions


recorded up to February 14, 2004 after the Bank Statement is
received will appear as follows.
304

J. GOLDDIGGER CASH BOOK

DR. RECEIPTS
DATE PARTICULARS FOLIO DISCOUNT CASH BANK
ALLOWED A/C A/C
2004 $ $
Jan. 2 Capital A/C-
J. Golddigger A/C# 2401 50,000
Jan. 2 Cash A/C C 40,000
Jan.25 B. Back A/C A/C# 1104001 1,000
Jan.25 L. Hand A/C A/C# 1104002 500 13,500

500 51,000 53,500


=== ====== ======
Feb. 1 Balance b/d 9,000 26,500
A/C# 4209
A/C# 1104
Feb.14 Salaries A/C 200

Please NOTE that the payment side of the Cash Book is on the
following page (page 305).
305

CR. PAYMENTS PAGE 1


DATE PARTICULARS FOLIO CHEQUE DISCOUNT CASH BANK
NUMBER RECEIVED A/C A/C
2004 $ $
Jan. 2 Bank A/C C 40,000
Jan. 5 Rent A/C A/C# 4202 001 10,000
Jan. 7 Petty Cash A/C A/C# 1101 1,000
Jan.23 W. Dandy A/C A/C# 210101 002 1,000 9,000
Jan.31 Salaries A/C A/C# 4201 003 8,000
Jan.31 Drawings A/C A/C# 2402 1,000
Jan.31 Balance c/d 9,000 26,500
1,000 51,000 53,500
===== ====== ======
A/C# 2101
A/C# 3202
Feb. 2 Rent A/C 004 10,000
Feb.13 Drawings A/C 005 800
Feb.14 Bank Charges
A/C 400
Feb.14 W. Dandy A/C 100

Let us assume that the accountant decided to go along with


alternative one (1). Therefore, the Cash Book for January 2004 would
not be closed until the Bank statement is received on February 14,
2004 and the relevant transactions reflected (for example, bank
charges).
306

The Cash Book for January 2004 would appear as follows.

ALTERNATIVE 1

J. GOLDDIGGER CASH BOOK

DR. RECEIPTS
DATE PARTICULARS FOLIO DISCOUNT CASH BANK
ALLOWED A/C A/C
2004 $ $
Jan. 2 Capital A/C-
J. Golddigger A/C# 2401 50,000
Jan. 2 Cash A/C C 40,000 /
Jan.25 B. Back A/C A/C# 1104001 1,000
Jan.25 L. Hand A/C A/C# 1104002 500 13,500 /
Jan.31 Salaries A/C A/C# 4201 200

500 51,000 53,700


=== ====== ======
Feb. 1 Balance b/d 9,000 26,200
A/C# 4209
A/C# 1104

Please NOTE that the payment side of the Cash Book is on the
following page (page 307).
307

CR. PAYMENTS PAGE 1


DATE PARTICULARS FOLIO CHEQUE DISCOUNT CASH BANK
NUMBER RECEIVED A/C A/C
2004 $ $
Jan. 2Bank A/C C 40,000
Jan. 5Rent A/C A/C#4202 001 10,000 O/S
Jan. 7Petty Cash A/C A/C#1101 1,000
Jan. 23
W. Dandy A/C A/C#210101 002 1,000 9,000 C/B
Jan. 31
Salaries A/C A/C#4201 003 8,000 C/B
Jan. 31
Drawings A/C A/C#2402 1,000
Jan. 31
Bank Charges
A/C A/C#4213 400
Jan. 31 W. Dandy A/C A/C#210101 100
Jan. 31 Balance c/d 9,000 26,200
1,000 51,000 53,700
===== ====== ======
A/C#2101
A/C#3202

The Bank Reconciliation will therefore appear as follows.

BANK RECONCILIATION STATEMENT FOR J. GOLDDIGGER ON JANUARY 31, 2004


$
Bank balance per Bank Statement 36,200
Less unpresented cheque #001 10,000
Bank balance per Cash Book 26,200
======
308

REVIEW QUESTIONS
(1)* The drawer is the person who ________________ the cheque.

(2) Distinguish between an open cheque and a crossed cheque.

(3) Write short notes on the following:


Postdated cheque
Dishonoured cheque
Stale dated cheque
Managers’ cheque

(4)* The ___________________ refers to the bank on which the cheque


is drawn.

EXERCISE 1 *

Bank balance as per Cash Book on Jan.31, 2004 $30,000

Bank balance as per Bank Statement on Jan.31, 2004 $27,000

Unpresented cheques
Cheque #05 S. Smith $2,000
Cheque #10 T. Jones $5,000

Cheques deposited and not yet credited by the bank $10,000

REQUIRED:
(a) Prepare a Bank Reconciliation Statement
reconciling from the Cash Book balance to the
bank statement balance.

(b) Prepare a Bank Reconciliation Statement


reconciling from the bank statement balance to
the Cash Book balance.
309

EXERCISE 2 *

Overdraft as per Bank Statement on Feb.29, 2004 $85,000


Unpresented cheques $ 1,000
Deposits not yet credited $ 4,000
Interest on overdraft $ 400
Overdraft as per Cash Book on Feb.29, 2004 $81,600

REQUIRED:
(c) Prepare a Bank Reconciliation Statement
reconciling from the Cash Book balance to the
bank statement balance.

(d) Prepare a Bank Reconciliation Statement


reconciling from the bank statement balance to
the Cash Book balance.

College Questions

EXERCISE 3 *

Bank balance as per Cash Book on August 31, 2004 $100,000


Unpresented cheques $ 30,000
Deposits not yet credited by the bank $ 5,000

Items on the Bank Statement not yet reflected


in the Cash Book
Standing order paid by the bank $ 3,000
Transfer from S. James a debtor in settlement
of his debt $ 20,000
Transfer to SY Ltd. a creditor in full settlement $ 15,000
Bank charges $ 100
Cheques deposited and dishonoured $ 1,000

What is the bank balance per the Bank Statement on August 31, 2004?
310

EXERCISE 4 *

Use the data in question 3 to prepare a Bank Reconciliation


Statement, but adjust the Cash Book balance before preparing the
Statement.
311

ANSWERS TO SELECTED REVIEW QUESTIONS


(1) Signs

(4) Drawee

EXERCISE 1(a)

BANK RECONCILIATION STATEMENT ON JANUARY 31, 2004


$
Bank balance as per Cash Book 30,000
Add unpresented cheques
Cheque # Amount
05 $2,000
10 $5,000
7,000
37,000
Less deposits not yet credited 10,000
Bank balance as per Bank Statement 27,000
======

EXERCISE 1(b)

BANK RECONCILIATION STATEMENT ON JANUARY 31, 2004


$
Bank balance as per Bank Statement 27,000
Less unpresented cheques
Cheque # Amount
05 $2,000
10 $5,000
7,000
20,000
Add deposits not yet credited 10,000
Bank balance as per Cash Book 30,000
======
312

EXERCISE 2(a)

BANK RECONCILIATION STATEMENT ON FEBRUARY 29, 2004


$ $
Overdraft as per Cash Book 81,600
Less unpresented cheques 1,000
80,600
ADD
Deposits not yet credited 4,000
Interest on overdraft 400
4,400
Overdraft as per Bank Statement 85,000
======

EXERCISE 2(b)

BANK RECONCILIATION STATEMENT ON FEBRUARY 29, 2004


$ $
Overdraft as per Bank Statement 85,000
Add unpresented cheques 1,000
86,000
LESS
Deposits not yet credited 4,000
Interest on overdraft 400
4,400
Overdraft as per Cash Book 81,600
======
313

EXERCISE 3

BANK BALANCE PER BANK STATEMENT ON AUGUST 31, 2004


$ $
Bank Balance per Cash Book 100,000
ADD
Unpresented cheques 30,000
Transfer from S. James 20,000
50,000
150,000

LESS
Deposits not yet credited 5,000
Standing order payment 3,000
Transfer to SY Ltd. 15,000
Bank charges 100
Cheques deposited and dishonoured 1,000
24,100
125,900
=======
314

EXERCISE 4

ADJUSTED CASH BOOK BANK BALANCE


$ $
Bank balance as per Cash Book 100,000

LESS
Standing order 3,000
Transfer to SY Ltd. 15,000
Bank charges 100
Cheques deposited and dishonoured 1,000
19,100
80,900
Add transfer from S. James 20,000
100,900
=======

BANK RECONCILIATION STATEMENT ON AUGUST 31, 2004


$
Adjusted bank balance per Cash Book 100,900
Add unpresented cheques 30,000
130,900
Less deposits not yet credited 5,000
Bank balance per Bank Statement 125,900
=======
315

Ch12
CHAPTER TWELVE

PARTNERSHIP ACCOUNTS

CHAPTER OBJECTIVES
After completing this chapter you should be able to:

 Prepare a Current Account for a partnership business.

 Prepare a Statement of Changes in Equity for a partnership.

 Prepare the Final Accounts for a partnership.


316

INTRODUCTION
A partnership is a business owned by two or more persons. If no
partnership agreement exists whether implied or in writing, or if no
agreement exists in relation to the sharing of profits and losses,
or remuneration to a partner for working in the partnership
business, or the payment of interest on each partner's capital, or
the payment of interest on any loan made by a partner, then the
following provisions of the Partnership Act of 1890 will apply:

(a) The partners will share profits and losses equally;

(b) No partner is entitled to any remuneration (salary) for working


in the partnership business;

(c) No partner is entitled to any interest on capital; and

(d) A partner should receive interest at the rate of 5 per cent per
annum on any loan made to the business by him or her.

PROFIT AND LOSS APPROPRIATION ACCOUNT


In addition to the normal Profit and Loss Account, a Profit and Loss
Appropriation Account is prepared to distribute the profits or
losses of the partnership business to the partners.

DEBITS in the Profit and Loss Appropriation Account


Loss
Interest on capital for each partner
Salary paid to a partner
Each partner's share of the profits
CREDITS in the Profit and Loss Appropriation Account
Profit
Interest on drawings from each partner
Each partner's share of the losses
317

The interest on any loan from a partner is debited in the Profit and
Loss Account, whereas interest on a partner's capital is debited in
the Profit and Loss Appropriation Account.

CAPITAL ACCOUNT
A Capital Account is opened for each partner with the capital that
he or she contributes. Each Capital Account will remain constant
from year to year unless a partner increases the capital that he or
she has invested in the business. Profits or losses and drawings are
not transferred to each partner's Capital Account, but to each
partner's Current Account.

CURRENT ACCOUNT
A Current Account is opened for each partner. Interest is charged on
drawings by the firm to discourage the partners from withdrawing
funds or goods at cost from the business during the financial year.
The interest on drawings is usually calculated at the end of the
financial year. If the interest on drawings for a partner for the
financial year is $10,000 and the partner paid $8,000 to the
partnership business, then only $2,000 would be debited to the
Current Account of the partner. The salary for a partner is to
compensate the partner for working in the partnership business. If a
partner is to receive a salary of $600,000 for the financial year
and $550,000 is paid to the partner, then only the $50,000
outstanding will be reflected in the Partner’s Current Account. The
purpose of interest on capital is to compensate each partner for the
amount of capital contributed to the partnership business especially
where the partners share profits and losses equally. Let us assume
that Michael and Susan are in partnership sharing profits and losses
equally. Michael contributed capital of $100,000 and Susan
contributed capital of $200,000. Interest on capital is at a rate of
10% per annum. Michael would receive $10,000 and Susan $20,000
interest on capital. The more capital that a partner contributes the
more interest that partner will receive.
318

DEBITS in the Current Account


The balance from the Drawings Account
Interest on drawings
The partner's share of losses
CREDITS in the Current Account
Partner's salary
Interest on capital
The partner's share of profits

Interest on any loan made by the partner to the partnership business


not paid at the end of the accounting period can be credited in the
Current Account. If the partnership business is being liquidated the
balance in the partner’s Current Account is transferred to that
partner’s Capital Account. The Capital Account rank last on the
payment list in a liquidation and any interest on loan made by the
partner rank’s above the Capital Accounts for payment. Therefore,
any interest on loan payable to a partner should be reflected under
current liabilities in the Balance Sheet.

The balance in the Current Account is reflected on the liability


side of the Balance Sheet. If a DEBIT balance exists it is shown as
a deduction from other liabilities and if a CREDIT balance exists it
is shown as an addition to other liabilities.
319

FINAL ACCOUNTS AND BALANCE SHEET


From the following information prepare the Final Accounts and
Balance Sheet (vertical format).

TRIAL BALANCE OF ROAD, STREET AND LANE PARTNERSHIP ON


December 31, 2003
------------------------------------------------------------------

NAME OF ACCOUNT DEBIT CREDIT


$ $
Property, plant and equipment (net) 195,000
Stock December 31, 2003 10,000
Trade Debtors 20,000
Bank 90,000
Capital Accounts
Road 50,000
Street 50,000
Lane 100,000
Current Accounts
Road 2,000
Street 1,000
Lane 4,000
Trade Creditors 13,000
Cost of Sales 140,000
Sales 288,000
Operating Expenses 68,000
Loan from Lane (2007) 20,000
Drawings
Road 1,500
Street 1,500
Lane 2,000 -
528,000 528,000
======= =======

Interest of 10 per cent per annum is to be paid on each partner's


fixed capital and the loan from Lane. Road is to receive a salary of
$14,000 for working in the business. Interest of 20 per cent is to
be charged on the closing balance of each partner's Drawings
Account.
320

Final Accounts in this case mean Trading Account, Profit and Loss
Account, and Profit and Loss Appropriation Account. Property, plant
and equipment (net) mean the net book value of property, plant and
equipment. The cost of sales is already calculated in the Trial
Balance therefore, the stock in the Trial Balance must be the
closing stock. Loan from Lane (2007), this means that the loan must
be repaid in the year 2007.

Since no profit/loss sharing ratio is stated, the partners will


share any profit or loss equally. The vertical format will be used
to prepare the Final Accounts and the Balance Sheet.

ROAD, STREET AND LANE TRADING ACCOUNT FOR THE YEAR ENDED
December 31, 2003
------------------------------------------------------------------------
$
Sales 288,000
Less cost of sales 140,000
Gross profit 148,000
=======

ROAD, STREET AND LANE PROFIT AND LOSS ACCOUNT FOR THE YEAR
ENDED DECEMBER 31,2003
-------------------------------------------------------------------
$ $
Gross profit 148,000
Less operating costs
Operating expenses 68,000
Interest on loan from Lane ($20,000 x 0.10) 2,000
70,000
Profit 78,000
=======
321

ROAD, STREET AND LANE PROFIT AND LOSS APPROPRIATION ACCOUNT


FOR THE YEAR ENDED DECEMBER 31,2003
------------------------------------------------------------------
$ $ $
Profit 78,000
Add interest on drawings
Road ($1,500 x 0.20) 300
Street ($1,500 x 0.20) 300
Lane ($2,000 x 0.20) 400
1,000
79,000
Less
Interest on capital
Road ($50,000 x 0.10) 5,000
Street ($50,000 x 0.10) 5,000
Lane ($100,000 x 0.10) 10,000
20,000
Road's salary 14,000
34,000
45,000
======

Share of profits
Road 15,000
Street 15,000
Lane 15,000
45,000
======
322

Before preparing the Balance Sheet, the new balance in each


partner's Current Account must be calculated.

DR. ROAD CURRENT ACCOUNT CR.


$ ║ $
Drawings 1,500 ║ Balance b/f 2,000
Interest on drawings 300 ║ Interest on capital 5,000
Balance c/f 34,200 ║ Salary 14,000
║ Profits 15,000
36,000 ║ 36,000
====== ║ ======

DR. STREET CURRENT ACCOUNT CR.


$ ║ $
Drawings 1,500 ║ Balance b/f 1,000
Interest on drawings 300 ║ Interest on capital 5,000
Balance c/f 19,200 ║ Profits 15,000
21,000 ║ 21,000
====== ║ ======

DR. LANE CURRENT ACCOUNT CR.


$ ║ $
Drawings 2,000 ║ Balance b/f 4,000
Interest on drawings 400 ║ Interest on capital 10,000
Balance c/f 26,600 ║ Profits 15,000
29,000 ║ 29,000
====== ║ ======

323

BALANCE SHEET OF ROAD, STREET AND LANE AS AT DECEMBER 31, 2003


$ $ $
Non-current Assets
Property, plant and equipment (net) 195,000

Current Assets
Stock 10,000
Trade debtors 20,000
Bank 90,000
120,000
Current liabilities
Interest on loan accrued ( 2,000)
Trade creditors (13,000)
(15,000)
Net current assets 105,000
Total assets less current liabilities 300,000
=======

Financed By:
Capital Accounts
Road 50,000
Street 50,000
Lane 100,000
200,000
Current Accounts
Road 34,200
Street 19,200
Lane 26,600
80,000
280,000
Non-current Liabilities
Long term loan from Lane 20,000
300,000
=======
324

STATEMENT OF CHANGES IN PARTNERS’ EQUITY


The Statement of Changes in Partners’ Equity will include the
information that is reflected in the Profit and Loss Appropriation
Section.

Hand and Foot partnership list of Balances on December 31, 2004


$
Furniture 780,000
Allowance for depreciation on furniture 145,000
Stock December 31, 2004 35,000
Trade Debtors 115,000
Bank 140,000
Capital Accounts
Hand 300,000
Foot 300,000
Current Accounts
Hand 6,400
Foot 7,600
Trade Creditors 24,000
Cost of Sales 280,000
Sales 647,000
Rent 72,000
Utilities 24,000
Office supplies 2,000
Salaries 28,000
Depreciation 39,000
Loan from Hand 85,000

Profits and losses are to be shared equally. Interest of 10 per cent


per annum is to be paid on the loan from Hand and it was agreed that
$20,000 (principal) should be paid in 2005. Provide for interest on
capital at 10 per cent per annum. Hand is to receive a salary of
$30,000 and Foot is to receive a salary of $36,000 for working in
the partnership business. Prepare the following:

a. The Income Statement for the year ended December 31, 2004 (an
appropriation section is not required).
b. The Statement of Changes in Partners’ Equity for the year
ended December 31, 2004.
c. The Balance Sheet as at December 31, 2004.
325

Hand and Foot


Income Statement for the year ended December 31, 2004
------------------------------------------------------------
$
Sales 647,000
Less Cost of Sales 280,000
Gross profit 367,000

Less Operating Expenses


Rent 72,000
Utilities 24,000
Office Supplies 2,000
Salaries 28,000
Depreciation 39,000
Interest on loan 8,500
173,500

Profit 193,500
=======

Hand and Foot Statement of Changes in Partners’ Equity


For the year ended December 31, 2004
------------------------------------------------------------------
Hand Foot Hand Foot Total
Capital Capital Current Current
Account Account Account Account
$ $ $ $ $
Balance 1/01/04 300,000 300,000 6,400 7,600 614,000
Interest on capital 0 0 30,000 30,000 60,000
Salary 0 0 30,000 36,000 66,000
Share of profits 0 0 33,750 33,750 67,500
Balance 31/12/04 300,000 300,000 100,150 107,350 807,500
======= ======= ======= ======= =====

The Statement of Changes in Partners’ Equity reflects the


distribution of profits in terms of interest on capital, salary for
a partner, interest on drawings, and share of profits or losses.
Interest on drawings and drawings are deducted in the Statement of
Changes in Partners’ Equity under the current account section, in
other words the figure will be in brackets.
326

Hand and Foot


Balance Sheet as at December 31, 2004
-----------------------------------------------------------
$
Non-current Assets
Furniture 780,000
Less Allowance for Depreciation 145,000
635,000

Current Assets
Stock 35,000
Trade Debtors 115,000
Bank 140,000
290,000

Less Current Liabilities


Current Portion of Loan 20,000
Trade creditors 24,000
Interest accrued 8,500
52,500

Net Current Assets 237,500


Total assets less current liabilities 872,500
Less Non-current Liabilities
Loan 65,000
807,500
=======

Financed By
Capital 600,000
Current Accounts 207,500
807,500
=======

The Statement of Changes in Partners’ Equity provides us with


detailed information about the capital balance for each partner and
the current account balance for each partner that is why the total
for capital and the total for current accounts are reflected in the
Balance Sheet. But you can still show the break down in the Balance
Sheet.
327

REVIEW QUESTIONS
(1)* If no partnership agreement exists profits and losses are
shared ______________________.

(2) Distinguish between a Profit and Loss Account, and a Profit


and Loss Appropriation Account.

(3) Distinguish between a Capital Account and a Current Account in


a partnership business.

(4)* Interest on a partner's loan is debited in the


_______________ _____________ ________________ Account.

EXERCISE 1 *

From the following information prepare the Profit and Loss


Appropriation Statement for Billy and Kid partnership for the year
ended June 30, 2004.

$
Profit 60,000
Interest on capital for Billy 2,000
Interest on capital for Kid 1,000
Interest on drawings - Kid 5,000
Billy's salary 20,000
328

EXERCISE 2 *

TRIAL BALANCE OF HOT AND COLD ON DECEMBER 31, 2003


DR. CR.
$ $
Capital Accounts
Hot 100,000
Cold 30,000
Current Accounts
Hot 20,000
Cold 10,000
Furniture & fittings 50,000
Motor vehicle 200,000
Allowance for depreciation
Furniture & fittings 10,000
Motor vehicle 30,000
Bank 60,000
Debtors 40,000
Creditors 25,000
Accruals 5,000
Closing stock 10,000
Profit 150,000
370,000 370,000
======= =======

Additional Information:

(1) Provide for interest on capital at 20% per annum.


(2) Cold is to receive a salary of $50,000 for working in the
business.
(3) Profits and losses are shared in the ratio of Hot 75% and
Cold 25%.

Prepare the following:

(a) The Profit and Loss Appropriation Statement.


(b) The Partners Current Accounts.
(c) The Balance Sheet.
329

College Questions

EXERCISE 3 *

Trial Balance of Fat and Thin on December 31, 2003


Debit Credit
$ $
Capital Accounts
Fat 60,000
Thin 40,000
Current Accounts
Fat 15,000
Thin 2,000
Furniture and Fittings 60,000
Motor Vehicle 280,000
Allowance for Depreciation
Furniture and Fittings 18,000
Motor Vehicle 120,000
Fat Drawings 5,000
Cash and Bank 30,720
Trade Debtors 105,000
Trade Creditors 28,650
Accruals 16,275
Closing Stock 70,265
Net profit - 255,060
552,985 552,985
======= =======

Additional Information:

(1) Provide for interest on capital at 15% per annum.


(2) Fat is to receive a salary of $72,000 for working in the
partnership business.
(3) Profits and losses are shared in the ratio of Fat 60% and
Thin 40%.

Prepare the following:

(a) The Profit and Loss Appropriation Statement.


(b) The Statement of Changes in Partners’ Equity.
(c) The Balance Sheet.
330

EXERCISE 4 *

Cat and Dog are in a partnership business sharing profits and losses
equally. On March 31, 2003 the balances in the partnership general
ledger showed the following:
Cat Dog
$ $
Capital Account 100,000 100,000
Current Account 1,000 Debit 2,000 Credit

On April 1, 2003 Cat and Dog admitted Rat into partnership. Rat had
the following assets and liabilities that were incorporated into the
new partnership business of Cat, Dog and Rat.
$
Furniture 20,000
Trade Debtors 12,500
Cash 20,000
Trade Creditors 2,500

The new partnership agreement included the following:


1. 10 per cent per annum interest on capital.
2. 20 per cent per annum interest on drawings.
3. The Capital Accounts should remain fixed.
4. Rat is to receive a salary of $96,000 for working
in the partnership business.
5. Profits and losses should be shared Cat 40%, Dog
40% and Rat 20%.

During the period in anticipation of profits Dog withdrew $40,000 on


June 1, 2003 and Rat withdrew $30,000 on November 1, 2003.

Profit for the period April 1, 2003 to December 31, 2003 amounted to
$720,000.

Required:
a. State the fixed capital for each partner at the commencement of
the new partnership on April 1, 2003.
b. Prepare the Profit and Loss Appropriation Statement for the 9
months ended December 31, 2003.
c. Prepare the partners’ Current Accounts.
d. State the amount of investment that each partner has in the
business on December 31, 2003.
331

ANSWERS TO SELECTED REVIEW QUESTIONS


(1) Equally

(4) Profit and Loss

EXERCISE 1

BILLY AND KID PROFIT AND LOSS APPROPRIATION STATEMENT


FOR THE YEAR ENDED JUNE 30, 2004
---------------------------------------------------------------
$ $ $
Net profit 60,000
Add interest on drawings from Kid 5,000
65,000
LESS
Interest on capital
Billy 2,000
Kid 1,000
3,000
Billy's salary 20,000
23,000
42,000
======

Share of profits
Billy ($42,000 x 0.50) 21,000
Kid ($42,000 x 0.50) 21,000
42,000
======

PLEASE NOTE: Since no profit sharing agreement is stated, the


partners share profits and losses equally.
332

EXERCISE 2

HOT AND COLD PROFIT AND LOSS APPROPRIATION STATEMENT


FOR THE YEAR ENDED DECEMBER 31, 2003
----------------------------------------------------------------
$ $ $
Profit 150,000
Less
Interest on capital
Hot ($100,000 x 0.20) 20,000
Cold ($ 30,000 x 0.20) 6,000
26,000
Cold's salary 50,000
76,000
74,000
======
Share of profits
Hot ($74,000 x 0.75) 55,500
Cold ($74,000 x 0.25) 18,500
74,000
======

DR. HOT CURRENT ACCOUNT CR.


$ ║ $
Balance c/f 95,500 ║ Balance b/f 20,000
║ Interest on capital 20,000
║ Profits 55,500
95,500 ║ 95,500
====== ║ ======

DR. COLD CURRENT ACCOUNT CR.


$ ║ $
Balance b/f 10,000 ║ Interest on capital 6,000
Balance c/f 64,500 ║ Salary 50,000
║ Profits 18,500
74,500 ║ 74,500
====== ║ ======
333

BALANCE SHEET OF HOT AND COLD AS AT DECEMBER 31, 2003


$ $ $
Non-current Assets
Furniture and fittings 50,000
Less allowance for depreciation 10,000
40,000
Motor vehicle 200,000
Less allowance for depreciation 30,000
170,000
210,000
Current Assets
Bank 60,000
Debtors 40,000
Closing stock 10,000
110,000
Current Liabilities
Creditors (25,000)
Accruals ( 5,000)
(30,000)
Net current assets 80,000
Total assets less current liabilities 290,000
=======

Financed By:
Capital Accounts
Hot 100,000
Cold 30,000
130,000

Current Accounts
Hot 95,500
Cold 64,500
160,000
290,000
=======
334

EXERCISE 3

FAT AND THIN PROFIT AND LOSS APPROPRIATION STATEMENT


FOR THE YEAR ENDED DECEMBER 31, 2003
----------------------------------------------------------------
$ $ $
Profit 255,060
Less
Interest on capital
Fat ($60,000 x 0.15) 9,000
Thin ($40,000 x 0.15) 6,000
15,000
Fat’s salary 72,000
87,000
168,060
=======
Share of profits
Fat ($168,060 x 0.60) 100,836
Thin ($168,060 x 0.40) 67,224
255,060
=======

Fat and Thin Statement of Changes in Partners’ Equity


For the year ended December 31, 2003
------------------------------------------------------------------
Fat Thin Fat Thin Total
Capital Capital Current Current
Account Account Account Account
$ $ $ $ $
Balance 1/01/03 60,000 40,000 15,000 ( 2,000) 113,000
Drawings 0 0 ( 5,000) 0 (5,000)
Interest on capital 0 0 9,000 6,000 15,000
Salary 0 0 72,000 0 72,000
Share of profits 0 0 100,836 67,224 168,060
Balance 31/12/03 60,000 40,000 191,836 71,224 363,060
======= ======= ======= ======= =====
335

BALANCE SHEET OF FAT AND THIN AS AT DECEMBER 31, 2003


$ $ $
Non-current Assets
Furniture and fittings 60,000
Less allowance for depreciation 18,000
42,000
Motor vehicle 280,000
Less allowance for depreciation 120,000
160,000
202,000
Current Assets
Cash and Bank 30,720
Debtors 105,000
Closing stock 70,265
205,985
Current Liabilities
Trade Creditors (28,650)
Accruals (16,275)
(44,925)
Net current assets 161,060
Total assets less current liabilities 363,060
=======

Financed By:
Capital Accounts
Fat 60,000
Thin 40,000
100,000

Current Accounts
Fat 191,836
Thin 71,224
263,060
363,060
=======
336

EXERCISE 4

Fixed capital for each partner on April 1, 2003

Cat Capital
Capital Account $100,000
Less Current Account debit balance $ 1,000
$ 99,000
=======

Dog Capital
Capital Account $100,000
Add Current Account credit balance $ 2,000
$102,000
=======

Rat Capital
Assets
Furniture $20,000
Trade debtors $12,500
Cash $20,000
$52,500
Less Liabilities
Trade Creditors $ 2,500
$50,000
=======
337

CAT, DOG AND RAT PROFIT AND LOSS APPROPRIATION STATEMENT


FOR THE YEAR ENDED DECEMBER 31,2003
------------------------------------------------------------------
$ $ $
Profit 720,000
Add interest on drawings
Dog ($40,000 x 0.20 X 6/12) 4,000
Rat ($30,000 x 0.20 X 2/12) 1,000
5,000
725,000
Less
Interest on capital
Cat ($ 99,000 x 0.10 X 9/12) 7,425
Dog ($102,000 x 0.10 X 9/12) 7,650
Rat ($ 50,000 x 0.10 X 9/12) 3,750
18,825
Rat's salary 96,000
114,825
610,175
=======

Share of profits
Cat 244,070
Dog 244,070
Rat 122,035
610,175
=======
338

DR. CAT CURRENT ACCOUNT CR.


$ ║ $
║ Balance b/f 0
║ Interest on capital 7,425
Balance c/f 251,495 ║ Profits 244,070
251,495 ║ 251,495
======= ║ =======

DR. DOG CURRENT ACCOUNT CR.


$ ║ $
Drawings 40,000 ║ Balance b/f 0
Interest on drawings 4,000 ║ Interest on capital 7,650
Balance c/f 207,720 ║ Interest on capital 244,070
251,720 ║ 251,720
======= ║ =======

DR. RAT CURRENT ACCOUNT CR.


$ ║ $
Drawings 30,000 ║ Balance b/f 0
Interest on drawings 1,000 ║ Interest on capital 3,750
Balance c/f 190,785 ║ Salary 96,000
║ Profits 122,035
221,785 ║ 221,785
======= ║ =======

Investment in the business by each partner on December 31, 2003

Partner Capital Account Current Account Total


Cat $ 99,000 $251,495 $350,495
Dog $102,000 $207,720 $309,720
Rat $ 50,000 $190,785 $240,785
339

Ch13
CHAPTER THIRTEEN

COMPANY ACCOUNTS

CHAPTER OBJECTIVES
After completing this chapter you should be able to:

 Distinguish between authorized share capital and issued


share capital.

 Describe the features of ordinary shares.

 Describe the features of preference shares.

 Distinguish between capital reserves and revenue reserves.

 Prepare the Final Accounts for a company.

 Explain the procedure for forming a company.

 Distinguish between a private company and a public company.

 Explain the role of the Board of Directors.

 Explain the purpose of a general meeting.


340

INTRODUCTION
Shareholders own a company. These shareholders are also referred to
as members. A company is an artificial person. A company has a
separate legal existence from its members. Therefore, the company
can enter into a contract with anyone of its members and it can also
sue a member. A member can enter into a contract with the company
and the member can sue the company. A company will continue in
existence regardless of the transfer of shares or the death of a
member until the company is wound up or is struck off the register
of companies at the Registrar of Companies office.

Two main types of companies exist, private companies and public


companies. Private companies cannot advertise to the general public
to subscribe for its shares and debentures, whereas a public company
can. Private companies restrict the right of members to transfer
(sell) its shares whereas a public company does not.

A company can have limited liability or unlimited liability. The


majority of companies are limited liability companies. Therefore,
unlike a sole trader or a general partner, a shareholder's liability
does not extend to his or her personal assets in a limited liability
company, but is limited to the capital that he or she contributes or
agrees to contribute.

FORMATION OF A COMPANY
In order to form a company you must file certain documents with the
Registrar of Companies. The Registrar of Companies is a Statutory
Body created by the government to administer the Companies Act. The
two main documents to be filed with the Registrar are the Articles
of Incorporation and the Declaration of Compliance. The Articles of
Incorporation must state certain important facts about the proposed
company (clauses), including the name of the company with "limited"
as the last word if the company is a limited company, whether it is
a private company or a public company, the proposed address of the
registered office of the company, the amount of the authorized share
capital, that the liability of its members is limited if the company
is to be a limited liability company, the voting rights of
341

shareholders, the powers of directors, the period of notice required


for an annual general meeting. Another important clause that should
be stated in the Articles of Incorporation is the "subscription
clause" stating the desire of the signatories to acquire shares and
to form a company. For a private company at least one (1) signatory
is required, whereas seven (7) signatories are required for a public
company. Each signatory is required to state opposite his or her
signature the number of shares that he or she agrees to take.

The Declaration of Compliance is a sworn statement by the Company


Secretary before a Justice of the Peace, indicating that all the
relevant regulations of the Companies Act were complied with.

A Certificate of Incorporation will be issued by the Registrar if he


or she is satisfied that all the relevant documents were filed, the
stamp duty paid, and that the proposed name of the company is not
identical to any existing registered company. When the Registrar
issues the Certificate of Incorporation he or she certifies that the
company has been incorporated under the Companies Act and the
company becomes a legal entity (an artificial person). The name of
the company, whether it is a public or private company, and the date
of incorporation will be stated on the Certificate of Incorporation.

REGISTERED COMPANY
A registered company is a company incorporated under the Companies
Act. A company is registered from the first date on its Certificate
of Incorporation and continues to be a registered company if it
satisfies the requirements (filing of annual return) laid down in
the Companies Act. A registered company may be unlimited, limited by
guarantee or limited by shares. The registered company may be a
private company or a public company. The public company must be a
limited company. However the private company can either be a limited
company or unlimited company.
342

PRIVATE COMPANY
The following are the distinguishing features of a private company:

(1) It restricts the right of its members to transfer shares.

(2) It limits the number of its members to a maximum of fifty 50).


The limit of fifty 50) does not include current employees and
ex-employees.

(3) It prohibits any invitation to the general public to subscribe


for its shares or debentures.

(4) The minimum number of members is one (1).

(5) The company cannot be listed on the Stock Exchange.

PUBLIC COMPANY
The following are the distinguishing features of a public company:

(1) It does not restrict the right of its members to transfer


shares.

(2) It places no limit on the maximum number of members.

(3) It can invite the general public to subscribe for its shares
and debentures, but does not necessarily do so. The invitation
to the general public is usually done by way of a prospectus.
A prospectus is an offer to the general public to subscribe for
a company's shares or debentures.

(4) The minimum number of members is seven (7).

(5) The company can be listed on the Stock Exchange at the


discretion of the Stock Exchange Board.
343

BOARD OF DIRECTORS
The responsibility for managing the affairs of a company is vested
in a Board of Directors. The Board of Directors is a group of
individuals appointed by the members at an annual general meeting
and the responsibility for running the company is delegated to this
board by the members. Therefore the members have no power to enter
into contracts on behalf of the company. The powers of the directors
can be located in the company's Articles of Incorporation. The Board
of Directors has the right to decide whether to declare dividends.

The directors usually appoint a Chairman and a Company Secretary.


The Chairman is responsible for convening the board meetings. The
Company Secretary usually maintains the register of members and the
register of mortgages. The register of members contains the
following:

(1) The name and address of each member.


(2) The distinctive numbers of each member's shares.
(3) The amount of shares held by each member.
(4) The date on which each person was entered in the register as a
member.
(5) The date when a person ceases to be a member.

The register of mortgages contains the following:

(1) The name and address of each mortgagee (the person or business
to whom the money is owed).
(2) The type of charge, whether fixed (specific) or floating.
(3) The amount of the charge.
(4) A description of the property charged.
(5) The date the charge was registered.
(6) The date the charge is discharged.

The charge can be a fixed charge that is a charge on a specific


identifiable asset giving the chargee (the person or organization
from whom the mortgage was obtained) the right to sell that specific
asset if certain conditions are not fulfilled. On the other hand, a
floating charge is a charge on a group of assets giving the chargee
the right to sell whatever asset within the group if certain
conditions are not fulfilled.
344

GENERAL MEETING
A general meeting is a meeting of shareholders of the company
convened by the Board of Directors, to discuss the operation of the
company and to pass or reject resolution(s). The Companies Act
requires the convening of an annual general meeting (AGM) for its
members to deal with the following matters:

(1) Election of directors for those retiring.


(2) Approving and fixing the remuneration for directors.
(3) Declaration of dividends.
(4) Consideration of the annual audited accounts.
(5) Consideration of the auditors' and directors' report.
(6) Appointment of auditors.
(7) Fixing of remuneration for auditors.
(8) Consideration of ordinary resolutions, extra-ordinary
resolutions and special resolutions.

A resolution is a written statement about a decision that must be


made at a general meeting, such as the appointment of auditors and
increasing the authorized share capital of the company.

An ordinary resolution is a resolution that can be passed by a


simple majority of votes cast at a general meeting without any
minimum period of notice to members that the resolution is to be
proposed at the general meeting.

An extra-ordinary resolution is a resolution that must be passed by


at least three-quarters of the votes cast at a general meeting.
Notice must be given to members that the resolution is to be
proposed as an extra-ordinary resolution but no minimum period of
notice is required.

A special resolution is a resolution that must be passed by at least


three-quarters of the votes cast at a general meeting. At least
twenty-one (21) days notice must be given to members that the
resolution is to be proposed as a special resolution at the general
meeting.

Let us assume that the company wishes to increase its authorized


share capital and the next annual general meeting will be in five
(5) months. Instead of waiting on the annual general meeting the
345

company may call an extra-ordinary general meeting. An extra-


ordinary general meeting is usually called in the event of an
emergency or when important decisions have to be made by members
within a short time period.

If a member is unable to attend an annual general meeting or an


extra-ordinary general meeting the member may appoint another person
(a proxy) to represent him or her at the meeting to cast his or her
vote.

AUTHORIZED SHARE CAPITAL AND ISSUED SHARE CAPITAL


The authorized share capital is stated in the company's Articles of
Incorporation. The authorized share capital indicates the maximum
amount of each class of share the company can legally issue to raise
capital. The procedure for increasing the authorized share capital
usually includes the passing of a resolution at an annual general
meeting or an extra-ordinary general meeting. Shares cannot be
issued in excess of the authorized share capital. The authorized
share capital is reflected as a note in the financial statements.
The issued share capital is the value of shares issued to date.

TYPES OF SHARES
The main types of share capital are ordinary shares and preference
shares. Shareholders receive dividends for investing in the shares
of a company.

ORDINARY SHARES

A company issues ordinary shares in order to raise capital for the


company. A person who buys an ordinary share is referred to as an
ordinary shareholder. An ordinary share (common stock) receives
dividends after preference shares receive dividends. Ordinary shares
are not entitled to any fixed percentage in dividends. Dividends are
346

declared at the discretion of the Board of Directors. Ordinary


shareholders usually have voting rights at the annual general
meeting and extra-ordinary general meeting.

PREFERENCE SHARES

A company issues preference shares in order to raise capital for the


company. A preference share (preferred stock) is a share that is
entitled to a fixed percentage in respect of dividends. This fixed
percentage is applied to the issue value of the share in order to
obtain the monetary value of the preference dividend. Preference
shares are entitled to receive dividends before ordinary shares.
Dividends are declared at the discretion of the Board of Directors.

CUMULATIVE PREFERENCE SHARES

These shares accumulate dividends in arrears. If no dividends are


declared in one year or over several years, then the cumulative
preference dividends in arrears must be paid before any other
dividends can be paid. The cumulative preference dividends in
arrears will be reflected as a contingent liability.

REDEEMABLE PREFERENCE SHARES

These shares are repayable in a given year, for example 2009, or


when the company chooses within a given period, for example 2008 to
2010.

PARTICIPATING PREFERENCE SHARES

In addition to receiving a fixed percentage dividend, these shares


also participate in `super profits'. In other words, they receive
additional dividends when the company makes `super profits'.

CONVERTIBLE PREFERENCE SHARES

Holders of these shares have the option of converting their


preference shares into ordinary shares.
347

DIVIDENDS
Dividends are paid out of profits at the discretion of the Board of
Directors. Dividends do not represent a tax deductible expense.
Dividends represent a distribution of profits. Dividends are paid to
shareholders who are listed in the register of members at a certain
cut off date. Let us assume that you purchased 2,000 Super Limited
shares from Frank Williams on June 30, 2004 but the documents in
relation to the transfer of ownership were received by the company
on July 17, 2004. If the company declares a dividend in respect of
the financial year ended June 30, 2004 payable to shareholders
listed in the register of members on July 15, 2004 (cut off date),
then the dividend cheque would be sent to Frank Williams even though
you were the owner of the shares on July 15, 2004. The cheque would
not be made payable to you since on July 15, 2004 your name would
not have been recorded in the company's register of members.

Dividends are usually declared for a financial year after the end of
that financial year. But dividends can be declared in respect of a
financial year before the end of that financial year. In other
words, the dividends are declared during the financial year and
these dividends are referred to as interim dividends. These
dividends are interim in the sense that we expect additional
dividends to be declared after the end of the financial year. These
additional dividends are referred to as final dividends. Dividends
declared after the end of the financial year represent a non-
adjusting post balance sheet event if international accounting
standards are being used. In other words these dividends are not
reflected in the accounts in the financial year to which they
relate. Under international accounting standards these dividends are
reflected in the financial year that it was declared. Interim
dividends are usually declared after the company un-audited
accounts, showing the company after tax profits for the first six
months (half year) of the financial year, are prepared. Final
dividends are usually declared after the company's draft audited
financial statements, showing the company after tax profits for the
financial year, are received from the auditors.
348

CAPITAL RESERVE AND REVENUE RESERVE


CAPITAL RESERVES

Capital reserves are non-distributable reserves, that is, these


reserves cannot be used to pay cash dividends. Examples are Share
Premium Account and Revaluation Reserve Account. These reserves can
be used to pay non-cash dividends in the form of bonus ordinary
shares.

The share premium is the excess of the issue price of a share over
the par value of that share. The par value is the nominal value for
the share. Companies can issue shares at a value in excess of its
nominal value (par value). If the issue price of an ordinary share
is $10 and the par value is $8, then the share premium is $2. Please
note that companies can decide to have no par shares. That is, no
par value is stated for the share.

It is now common practice to recognize a permanent increase in the


value of land and buildings in the Balance Sheet. This permanent
increase in the value of the asset represents an unrealized gain,
and is therefore not recognized as revenue in the Profit and Loss
Account. The unrealized gain is reflected in a Revaluation Reserve
Account. For example, unrealized gain $40,000 and the corporation
tax rate is 25%.

DR. Asset Account $40,000


CR. Deferred Taxation Account ($40,000 X 0.25) $10,000
CR. Revaluation Reserve Account ($40,000 - $10,000) $30,000

REVENUE RESERVES

Revenue reserves are reserves that can be used to pay cash dividends
(distributable). Examples are Retained Earnings Account and General
Reserve Account. Retained earnings represent accumulated
undistributed profits. General reserve represents undistributed
profits which are set aside.
349

DEBENTURES AND BONDS


Debentures and bonds are securities representing money borrowed from
the investing public. Debentures and bonds are long term loans made
to the company. The debenture or bond agreement will contain the
following information:

(a) The rate of interest on the loan, if the interest rate is


fixed or variable, if the interest will be paid monthly,
quarterly, semi-annually or annually;

(b) The terms for repaying the loan; and

(c) The action that can be taken by the debenture-holder or the


bondholder if the firm default on its principal payments or
interest payments.

PROFIT AND LOSS APPROPRIATION ACCOUNT


In addition to the normal Profit and Loss Account, a Profit and Loss
Appropriation Account is prepared to distribute the profits or
losses of the company.

DEBITS in the Profit and Loss Appropriation Account


Accumulated losses brought forward
Loss before tax
Corporation tax
Transfer to capital reserve
Transfer to revenue reserve
Preference dividend
Ordinary dividend
Retained earnings carried forward
CREDITS in the Profit and Loss Appropriation Account
Retained earnings brought forward
Profit before tax
Accumulated losses carried forward
350

FINAL ACCOUNTS AND BALANCE SHEET


From the following information prepare the Final Accounts and the
Balance Sheet.

TRIAL BALANCE OF WILLIAMS LTD. ON SEPTEMBER 30, 2004


DR. CR.
$ $
20% Preference shares 100,000
Ordinary shares 500,000
Share premium 100,000
30% Debentures 200,000
Profit and Loss Appropriation A/C 10,000
Cost of sales 1,500,000
Sales 2,600,000
Interest on debentures 60,000
Other operating expenses 940,000
Plant and machinery 800,000
Allowance for dep. on plant & machinery 100,000
Closing stock 60,000
Trade debtors 200,000
Bank 30,000
Preference dividends paid 20,000
Ordinary dividends paid 50,000
Trade creditors 40,000
Interest accrued 10,000
3,660,000 3,660,000
========= =========

Provide for corporation tax amounting to $25,000.


351

WILLIAMS LIMITED
TRADING STATEMENT FOR THE YEAR ENDED SEPTEMBER 30, 2004
$
Sales 2,600,000
Less cost of sales 1,500,000
Gross profit 1,100,000
=========

WILLIAMS LIMITED PROFIT AND LOSS STATEMENT FOR THE YEAR ENDED
SEPTEMBER 30, 2004
-------------------------------------------------------------------
$ $
Gross profit 1,100,000
Less
Interest on debentures 60,000
Other operating expenses 940,000
1,000,000
Profit 100,000
=========

WILLIAMS LIMITED PROFIT AND LOSS APPROPRIATION STATEMENT FOR


THE YEAR ENDED SEPTEMBER 30, 2004
---------------------------------------------------------------
$ $
Retained profit b/f 10,000
Profit 100,000
110,000
Less
Corporation tax 25,000
Preference dividend 20,000
Ordinary dividend 50,000
95,000
Retained profit c/f 15,000
=======
352

BALANCE SHEET OF WILLIAMS LIMITED AS AT SEPTEMBER 30, 2004


$ $
Non-current Assets
Plant and machinery 800,000
Less allowance for depreciation 100,000
700,000
Current Assets
Closing stock 60,000
Trade debtors 200,000
Bank 30,000
290,000

Less Current Liabilities


Trade creditors 40,000
Interest accrued 10,000
Corporation tax payable 25,000
75,000
Net current assets 215,000
Total assets less current liabilities 915,000
=======
Financed By:
20% Preference share capital 100,000
Ordinary share capital 500,000
Share premium 100,000
Retained earnings 15,000
Ordinary shareholders net worth 615,000

Non-current Liabilities
30% Debentures 200,000
915,000
=======

STATEMENT OF CHANGES IN EQUITY


The Statement of Changes in Equity will include the information that
is reflected in the Profit and Loss Appropriation Section.
Therefore, if a Statement of Changes in Equity is prepared for
publication the Profit and Loss Appropriation Section is not
required. The Income Statement will end at after tax profit/loss.
353

Greek Limited list of balances on December 31, 2003


$
Ordinary Share Capital 200,000
Share Premium 37,500
General Reserves 20,000
Retained Profits 1/1/2003 85,000
Long Term Loan 200,000
Cost of Sales 750,000
Sales 1,305,000
Other Operating Income 20,000
Distributions Costs 145,800
Administrative Costs 274,200
Other Operating Costs 60,000
Interest on Loan 20,000
Furniture 512,500
Allowance for Depreciation on Furniture 50,000
Closing Stock 30,000
Trade Debtors 60,000
Bank 90,000
Trade Creditors 20,000
Rent Accrued 5,000

Additional Information:
1. Ordinary dividends amounting to $30,000 was paid in
December 2003, but this dividend is not yet reflected
in the 2003 accounts.
2. Transfer $10,000 to general reserves.
3. Corporation tax is estimated at $12,800 for the 2003
financial year.
4. $50,000 of the long-term loan will be paid on December
31, 2004.
5. The weighted average number of ordinary shares in issue
during 2003 was 200,000.

Required:
a. Prepare the Income Statement for the year ended December 31,
2003.

b. Prepare the Statement of Changes in Equity for the year ended


December 31, 2003.

c. Prepare the Balance Sheet as at December 31, 2003.


354

Greek Limited
Income Statement for the year ended December 31, 2003
------------------------------------------------------------
$
Sales 1,305,000
Cost of Sales ( 750,000)
Gross Profit 555,000
Other Operating Income 20,000
Distribution Costs ( 145,800)
Administrative Costs ( 274,200)
Other Operating Costs ( 60,000)
Operating Profit 95,000
Finance Costs ( 20,000)
Profit Before Taxation 75,000
Taxation ( 12,800)
Profit After Taxation 62,200
======

Earnings Per Share $0.31

Greek Limited Statement of Changes in Equity


For the year ended December 31, 2003
------------------------------------------------------------------
Ordinary Share General Retained Total
Share Premium Reserves Profits
Capital
$ $ $ $ $
Balance 1/01/03 200,000 37,500 20,000 85,000 342,500
After Tax Profits 0 0 0 62,200 62,200
Dividends 0 0 0 (30,000) (30,000)
Transfer 0 0 10,000 (10,000) 0
Balance 31/12/03 200,000 37,500 30,000 107,200 374,700
======= ======= ======= ======= =====
355

Greek Limited
Balance Sheet as at December 31, 2003
---------------------------------------------------------------
$
Non-current Assets
Furniture 512,500
Less Allowance for Depreciation 50,000
462,500

Current Assets
Stock 30,000
Trade Debtors 60,000
Bank 60,000
150,000

Less Current Liabilities


Trade Creditors 20,000
Rent Accrued 5,000
Corporation Tax Payable 12,800
Current Portion of Long-term Loan 50,000
87,800

Net Current Assets 62,200


Total Assets Less Current Liabilities 524,700

Less Non-current Liabilities


Long-term Loan 150,000
374,700
=======

Financed by
Ordinary Share Capital 200,000
Share Premium 37,500
General Reserves 30,000
Retained Profits 107,200
374,700
=======
356

WORKINGS
Adjusted Bank Balance = Closing Bank Balance – Dividends Paid
= $90,000 - $30,000
= $60,000

Earnings per share = After Tax Profits/Ordinary Shares Issued


= $62,200/200,000
= $0.31

BALANCE SHEET ARRANGEMENT

Assets and liabilities can be arranged in order of permanency or in


order of liquidity in the Balance Sheet.

The current assets in order of liquidity are


Cash
Bank
Bills Receivable
Trade Debtors
Closing Stock
Prepayments

The Non-current assets in order of permanency


Land
Buildings
Furniture and Fittings
Plant & Machinery
Motor Vehicles
Long-term investments
Long-term receivables

Organizations normally arrange their assets in the balance sheet in


order of permanency. Financial institutions are the exception. They
normally arrange their assets in the balance sheet in order of
liquidity. Individuals who deposit money in a bank will more be
interested in the liquidity of the bank’s assets (ability of the
bank to repay the deposit) than the permanency of its assets. From
a theoretical standpoint if assets are arranged in order of
357

permanency land and buildings would be the first item and cash
would be the last item. If assets are arranged in order of
liquidity land and buildings would be the last item and cash would
be the first item. From a practical standpoint it is not that
simple you may have to examine the nature of the asset. Which is
the least liquid of the following assets that are being disposed
of?

Land and Buildings


Machinery (obsolete)
Stock

Because the machinery is obsolete it might be more difficult to


liquidate than land and buildings. Thus, from a practical
standpoint the machinery by its nature, being obsolete, may be the
least liquid asset of the three.

Rank the following assets in order of permanency and then in


order of liquidity.
Current Assets
Non-current Assets

Liquidity
Current Assets
Non-current Assets

Permanency
Non-current Assets
Current Assets

Rank the following liabilities in order of permanency and then


in order of liquidity.
Current Liabilities
Capital
Non-current Liabilities

Permanency
Capital
Non-current Liabilities
Current Liabilities
358

Liquidity
Current Liabilities
Non-current Liabilities
Capital

Permanency refers to the length of time the asset or liability will


remain in the business before changing its form. For example stocks
are more likely to be sold and transformed into cash or trade
debtors in a shorter period of time (usually within one year) than
plant and machinery. The plant and machinery would normally be used
in the business for several years before it is disposed of and
transformed into cash or other debtor. Therefore, plant and
machinery is more permanent to the business than stock. A debenture
that is to be paid in 5 years time is more permanent than trade
creditors, since the trade creditors are normally paid within one
year.

Liquidity in terms of an asset refers to the length of time the


asset will be in the business before it is transformed into cash.
Liquidity in terms of a liability refers to the length of time the
liability will be in the business before it is paid. The following
is an example of a Bank’s Balance Sheet in order of liquidity.

Sure Bank Limited Balance Sheet as at December 31, 2004

Assets $’000
Cash resources
Cash on hand and at the central bank 4,140
Amounts due from other banks 6,512

Investments
Government securities 2,318

Loans
Loans and advances less allowance for losses 28,216

Leased assets 414


Customer’s liability under acceptances,
guarantees and letter of credit 1,500
Other assets 600
Property, plant and equipment 2,700
46,400
359

Liabilities $’000
Deposits
Deposits and current account of customers 32,800
Amounts due to other banks & financial 40
institutions

Other Liabilities
Cheques and other instruments in transit 725
Acceptances, guarantees, indemnities and
letters of credit 5,660
Obligation under repurchase agreements 200
Other liabilities 2,475

Shareholder’s equity
Share capital 2,000
Capital reserve 500
Banking reserve fund 250
Retained earnings reserve 550
Un-appropriated profits 1,200
46,400

CASH FLOW STATEMENT


The Cash Flow Statement for a company can be prepared using the
Direct Method or the Indirect Method. Whichever method is used cash
flows should be classified into three headings, Operating
Activities, Investing Activities, and Financing Activities.

Operating Activities are the principal revenue generating activities


of a business and other activities that cannot be classified under
Investing Activities or Financing Activities. Under Operating
Activities we will start with the profit after taxation.

Investing Activities relate to the acquisition and disposal of non-


current assets. Financing Activities relate to receipts of loan
capital and proceeds from the issue of shares and the repayment of
loan capital and preference share capital. It also includes cash
payment of dividends.
360

The indirect method of presentation is the more popular method. This


was stated in the chapter on Cash Flow Statement (chapter 9). This
is the method we will use to prepare a Cash Flow Statement for a
company.

Greek Limited
Income Statement for the year ended December 31, 2003
------------------------------------------------------------
$
Sales 1,305,000
Cost of Sales ( 750,000)
Gross Profit 555,000
Other Operating Income 20,000
Distribution Costs ( 145,800)
Administrative Costs ( 274,200)
Other Operating Costs ( 60,000)
Operating Profit 95,000
Finance Costs ( 20,000)
Profit Before Taxation 75,000
Taxation ( 12,800)
Profit After Taxation 62,200
======

Earnings Per Share $0.31

Greek Limited Statement of Changes in Equity


For the year ended December 31, 2003
------------------------------------------------------------------
Ordinary Share General Retained Total
Share Premium Reserves Profits
Capital
$ $ $ $ $
Balance 1/01/03 200,000 37,500 20,000 85,000 342,500
After Tax Profits 0 0 0 62,200 62,200
Dividends 0 0 0 (30,000) (30,000)
Transfer 0 0 10,000 (10,000) 0
Balance 31/12/03 200,000 37,500 30,000 107,200 374,700
======= ======= ======= ======= =====
361

Greek Limited
Balance Sheet as at December 31
2003 2002
$ $
Non-current Assets
Furniture 512,500 500,000
Less Allowance for Depreciation 50,000 25,000
462,500 475,000

Current Assets
Stock 30,000 47,000
Trade Debtors 60,000 40,000
Bank 60,000 73,000
150,000 160,000

Less Current Liabilities


Trade Creditors 20,000 12,400
Rent Accrued 5,000 10,000
Corporation Tax Payable 12,800 20,100
Current Portion of Long-term Loan 50,000 50,000
87,800 92,500

Net Current Assets 62,200 67,500


Total Assets Less Current Liabilities 524,700 542,500

Less Non-current Liabilities


Long-term Loan 150,000 200,000
374,700 342,500
======= =======

Financed by
Ordinary Share Capital 200,000 200,000
Share Premium 37,500 37,500
General Reserves 30,000 20,000
Retained Profits 107,200 85,000
374,700 342,500
======== ========

Prepare Greek Limited Cash Flow Statement for the year ended
December 31, 2004 using the indirect method.
362

Greek Limited
Cash Flow Statement for the year ended December 31, 2003
$
Cash Flows from Operating Activities
Profit before taxation 75,000
Add interest expense 20,000
Operating profit 95,000
Adjustment for non-cash items
Depreciation 25,000

(Increase)/Decrease in operating assets


Stock 17,000
Trade debtors (20,000)

Increase/(Decrease) in operating liabilities


Trade creditors 7,600
Rent accrued ( 5,000)
Cash generated from operations 119,600
Corporation tax paid ( 20,100)
Net cash provided by operating activities 99,500

Cash Flows from Investing Activities


Acquisition of long term investments 0
Disposal of long term investments 0
Acquisition of furniture (12,500)
Net cash used by investing activities (12,500)

Cash Flows from Financing Activities


Interest paid (finance costs) (20,000)
Dividends paid (30,000)
Loan payments (50,000)
Net cash used by financing activities (100,000)

Net decrease in cash and cash equivalents (13,000)


Opening cash and cash equivalents 73,000
Closing cash and cash equivalents 60,000
======

Represented by
Bank 60,000
======
363

Workings

Acquisition of Furniture
See Balance Sheet $512,500 - $500,000 = $12,500

Dividends Paid
The dividend of $30,000 reflected in the Statement of Changes in
Equity is not reflected under current liabilities in the 2003
Balance Sheet; therefore the $30,000 must have been paid.

Loan Payment
The current portion of the long-term loan is added to the non-
current portion of the long-term loan for each year in order to
determine the loan payment.

Loan payment = ($200,000 + $50,000) – ($150,000 + $50,000)


= $250,000 - $200,000
= $50,000

Opening corporation tax payable $20,100


Add Corporation tax charge for 2003 $12,800
$32,900
Less closing corporation tax payable $12,800
Corporation tax paid in 2003 $20,100
364

REVIEW QUESTIONS
(1)* A company is an person.

(2) Distinguish between authorized share capital and issued share


capital.

(3) Distinguish between ordinary shares and preference shares.

(4) Write short notes on the following:

(a) Capital reserve


(b) Revenue reserve
(c) Redeemable preference shares
(d) Convertible preference shares

(5) Describe the process for registering a company.

(6) What is the purpose of an annual general meeting?

(7) At an Annual General Meeting of a company a shareholder voted by


proxy. What is a proxy?

(8) Distinguish between a special resolution and an ordinary


resolution.

(9) What is the purpose of a register of members?

EXERCISE 1 *

From the following information prepare a Profit and Loss


Appropriation Statement for the year ended December 31, 2004.

$
Profit and Loss Appropriation A/C balance 1/1/04 80,000
Profit for 2004 700,000
Corporation tax for 2004 150,000
Transfer to general reserve 100,000
Ordinary dividend paid 200,000
365

EXERCISE 2 *

TRIAL BALANCE OF FRANCE LIMITED ON DECEMBER 31, 2004


DR. CR.
$ $
10% Preference shares @ $5 5,000
Ordinary shares @ $10 25,000
Share premium 5,000
15% Debentures 10,000
Profit and Loss Appropriation A/C 500
Cost of sales 75,000
Sales 130,000
Other operating expenses 47,000
Plant and machinery 40,000
Allowance for dep. on plant & machinery 5,000
Closing stock 3,000
Trade debtors 10,000
Preference dividend paid 500
Ordinary dividend paid 3,000
Bank 4,500
Trade creditors 2,000
Rent accrued 500
183,000 183,000
======= =======

Additional Information:

(1) Provide for corporation tax amounting to $1,000.


(2) Transfer $1,000 to general reserve.
(3) Accrue one year’s interest on the Debentures.

Required:

a. Prepare the Trading and Profit and Loss Statement for the year
ended December 31, 2004.

b. Prepare the Balance Sheet as at December 31, 2004.


366

COLLEGE STUDENT

EXERCISE 3 *

Pear Limited list of balances on December 31, 2003


$’000
Ordinary Share Capital 250,000
Capital Reserves 20,000
Retained Profits 1/1/2003 35,000
Long Term Loan 100,000
Cost of Sales 750,700
Sales 1,350,000
Other Operating Income 9,000
Distributions Costs 120,100
Administrative Costs 300,200
Other Operating Costs 51,600
Interest on Loan 30,000
Closing Stock 28,400
Trade Debtors 417,500
Bank 99,000
Trade Creditors 16,000
Ordinary Dividends Paid 25,000

Additional Information:
(1) Transfer $10,000,000 to a General Reserve Account.
(2) Corporation tax is estimated at $13,800,000 for the 2003
financial year.
(3) Bonus issue of ordinary shares out of Capital Reserves Account
on January 1, 2003 amounted to $5,000,000. This transaction is
not yet reflected in the accounts.
(4) The weighted average number of ordinary shares in issue during
2003 is 255,000,000.

Required:
a. Prepare the Income Statement for the year ended December 31,
2003.
b. Prepare the Statement of Changes in Equity for the year ended
December 31, 2003.

Please note: A Balance Sheet is not required.


367

EXERCISE 4 *
$
Ordinary Share Capital ($1 each) 350,000
10% Preference Share Capital ($1 each) 100,000
Profit and Loss Appropriation A/C Balance 1/1/04 2,127,200
Profit Before Taxation for 2004 1,620,500
Corporation Tax for 2004 470,820
Transfer to General Reserve 250,000

Additional Information:
1. Ordinary dividends declared and paid in 2004 $0.50 per
share.
2. Preference dividends of 10% per share declared and paid
in 2004.
3. On January 9,2005 the Board of Directors declared an
additional ordinary dividend of $24,500 in respect of
the 2004 financial year.

Required:

Prepare the Profit and Loss Appropriation Statement for Penny


Limited for the year ended December 31, 2004.
368

EXERCISE 5 *

Time Limited Balance Sheet as at December 31


2003 2002
$’000 $’000

Non-current Assets
Furniture 520 500
Less allowance for depreciation 242 200
278 300

Current Assets
Closing stock 713 605
Insurance prepaid 18 16
Trade debtors 147 130
Bank 830 744
Cash 210 10
1,918 1,505

2,196 1,805
===== =====

Current Liabilities
Trade creditors 102 115
Corporation tax payable 94 260
Accruals 12 20
208 395

Shareholder’s Equity
Ordinary share capital 1,200 1,000
Retained profits 788 410
1,988 1,410

2,196 1,805
===== =====
369

Time Limited Income Statement for the year ended December 31, 2003
$’000
Sales 10,195
Less cost of sales 6,120
Gross profit 4,075
Add other operating income 5
4,080
Less operating expenses 3,158
Profit before taxation 922
Taxation 244
Profit after taxation 678
Less ordinary dividends paid 300
Retained profit for the year 378
Retained profit brought forward 410
Retained profit carried forward 788
======

Additional information:

Depreciation provided for 2003 amounted to $51,000.

Furniture costing $20,000 with accumulated depreciation of $9,000


was sold for $16,000 cash.

Required: Prepare the Cash Flow Statement for the year ended
December 31, 2003 using the indirect method.
370

ANSWERS TO SELECTED REVIEW QUESTIONS


(1) Artificial

EXERCISE 1

PROFIT AND LOSS APPROPRIATION STATEMENT FOR THE YEAR


ENDED DECEMBER 31, 2004
---------------------------------------------------------------
$ $
Retained profit b/f 80,000
Profit 700,000
780,000
Less
Corporation tax 150,000
General reserve 100,000
Ordinary dividend 200,000
450,000
Retained profit c/f 330,000
=======

EXERCISE 2

WORKINGS
Interest accrued (debentures) = $10,000 x 0.15
= $1,500
371

FRANCE LIMITED
TRADING STATEMENT FOR THE YEAR ENDED December 31, 2004
----------------------------------------------------------
$
Sales 130,000
Less cost of sales 75,000
Gross profit 55,000
=======

FRANCE LIMITED
PROFIT AND LOSS STATEMENT FOR THE YEAR ENDED December 31, 2004
-------------------------------------------------------------------
$ $
Gross profit 55,000
Less
Interest on debentures 1,500
Other operating expenses 47,000
48,500
Profit 6,500
======

FRANCE LIMITED PROFIT AND LOSS APPROPRIATION STATEMENT


FOR THE YEAR ENDED December 31, 2004
-------------------------------------------------------------------
$ $
Retained profit b/f 500
Profit 6,500
7,000
Less
Corporation tax 1,000
Preference dividend 500
Ordinary dividend 3,000
General reserve 1,000
5,500
Retained profit c/f 1,500
=====
372

FRANCE LIMITED
BALANCE SHEET AS AT December 31, 2004
-------------------------------------------------------------------
$ $
Non-current Assets
Plant and machinery 40,000
Less allowance for depreciation 5,000
35,000
Current Assets
Closing stock 3,000
Trade debtors 10,000
Bank 4,500
17,500

Less Current Liabilities


Trade creditors 2,000
Interest accrued 1,500
Rent accrued 500
Corporation tax payable 1,000
5,000
Net current assets 12,500
47,500
======

Financed By:
10% Preference share capital 5,000
Ordinary share capital 25,000
Share premium 5,000
General reserve 1,000
Retained earnings 1,500
Ordinary shareholders net worth 32,500

Non-current Liabilities
15% Debentures 10,000
47,500
======
373

EXERCISE 3

Pear Limited
Income Statement for the year ended December 31, 2003
------------------------------------------------------------
$’000
Sales 1,350,000
Cost of Sales ( 750,700)
Gross Profit 599,300
Other Operating Income 9,000
Distribution Costs ( 120,100)
Administrative Costs ( 300,200)
Other Operating Costs ( 51,600)
Operating Profit 136,400
Finance Costs ( 30,000)
Profit Before Taxation 106,400
Taxation ( 13,800)
Profit After Taxation 92,600
======

Earnings Per Share $0.36

Pear Limited
Statement of Changes in Equity
For the year ended December 31, 2003
------------------------------------------------------------------
Ordinary Capital General Retained Total
Share Reserves Reserves Profits
Capital
$’000 $’000 $’000 $’000 $’000
Balance 1/01/03 250,000 20,000 0 35,000 305,000
After Tax Profits 0 0 0 92,600 92,600
Bonus Issue 5,000 ( 5,000) 0 0 0
Dividends 0 0 0 (25,000) (25,000)
Transfer 0 0 10,000 (10,000) 0
Balance 31/12/03 255,000 15,000 10,000 92,600 372,600
======= ======= ======= ====== ======
374

EXERCISE 4

WORKINGS

Number of ordinary shares issued = $350,000/$1


= 350,000

Preference dividends = $100,000 X 0.10


= $10,000

Ordinary dividends = 350,000 X $0.50


= $175,000

The dividends declared on January 9, 2005 represent a non-adjusting


post balance sheet event and is not reflected in the 2004 Profit and
Loss Appropriation Statement.

PENNY LIMITED PROFIT AND LOSS APPROPRIATION STATEMENT FOR


THE YEAR ENDED SEPTEMBER 30, 2004
---------------------------------------------------------------
$ $
Retained Profit b/f 2,127,200
Profit 1,620,500
3,747,700
Less
Taxation 470,820
General Reserve 250,000
Preference Dividend 10,000
Ordinary dividend 175,000
905,820
Retained profit c/f 2,841,880
=========
375

EXERCISE 5
Time Limited
Cash Flow Statement for the year ended December 31, 2003
$’000
Cash Flows from Operating Activities
Profit before taxation 922
Adjustment for non-cash items
Depreciation 51
Profit on disposal of furniture ( 5)

(Increase)/Decrease in operating assets


Stock (108)
Insurance prepaid ( 2)
Trade debtors ( 17)

Increase/(Decrease) in operating liabilities


Trade creditors ( 13)
Accruals ( 8)
Cash generated from operations 820
Corporation tax paid (410)
Net cash provided by operating activities 410

Cash Flows from Investing Activities


Proceeds from disposal of furniture 16
Acquisition of furniture ( 40)
Net cash used by investing activities ( 24)

Cash Flows from Financing Activities


Dividends paid (300)
Proceeds from issue of shares 200
Net cash used by financing activities (100)

Net increase in cash and cash equivalents 286


Opening cash and cash equivalents 754
Closing cash and cash equivalents 1,040
=====

Represented by
Cash 210
Bank 830
Closing cash and cash equivalents 1,040
=====
376

WORKINGS ($’000)

Proceeds from issue of shares = 1,200 – 1,000


= 200

Profit/(loss) on disposal of furniture = 16 – (20 – 9)


= 16 – 11
= 5

Furniture acquired = 520 – (500 – 20)


= 520 - 480
= 40

Opening corporation tax payable 260


Add Corporation tax charge for 2003 244
504
Less closing corporation tax payable 94
Corporation tax paid in 2003 410
377

ch14
CHAPTER FOURTEEN

DEPARTMENTAL ACCOUNTS

CHAPTER OBJECTIVES
After completing this chapter you should be able to:

 Identify the most appropriate method for allocating a


specific indirect cost.

 Prepare a Departmental Trading and Profit and Loss


Statement.

 Prepare Room Department Income Statement.

 Prepare Food Department Income Statement.

 Prepare Beverage Department Income Statement.


378

INTRODUCTION
The accounting system should provide management with useful
information for them to make rational decisions. In order to do
this, the accounting system must reflect the structure of the
organization. If several profit oriented departments are in a
company (Clear Water Limited), and the company only produces
financial statements for the business as a whole (profit for Clear
Water Limited $600,000), then the loss of one department will not be
known if it is more than covered by the profit of other departments.
The preparation of Departmental Accounts will reveal those
departments that are making a profit and those departments that are
making a loss.

Profit for Box Drink Department $1,800,000


Add Profit of Bag Juice Department $ 200,000
$2,000,000
Less Loss of Stout Department $1,400,000
Profit for Clear Water Limited $ 600,000
==========

By preparing Departmental Accounts for an organization that is so


structured, enables management to monitor and track the progress of
each department. The financial statement of each department will
provide management with a basis to award incentives to department
managers and staff. The preparation of Departmental Accounts may
also encourage `healthy competition' between the various
departments, and management will also be better able to develop the
most appropriate strategies for each department. The main profit
oriented departments for a hotel are Rooms Department, Food
Department and Beverage Department. The departmental Income
Statement for a hotel should be prepared in accordance with the
Uniform Systems of Accounts for the Lodging Industry (USALI).
379

INDIRECT COSTS AND DIRECT COSTS


In order to prepare accounts for each department, management must
establish a policy for allocating indirect costs. Indirect costs are
common costs incurred for the benefit of all departments. Direct
costs are costs that can be charged directly to the relevant
department. Any method devised for allocating indirect costs is
usually arbitrary, but the most appropriate method should be
selected.

ALLOCATION OF INDIRECT COSTS

SUGGESTED METHODS OF ALLOCATING INDIRECT COSTS


INDIRECT COSTS BASIS OF ALLOCATION
Rent and rates Floor area
Insurance Value of assets insured
Advertising Dollar sales
Selling expenses Dollar sales
Administrative expenses Number of employees or dollar sales

Electricity Unit production or floor area

Please NOTE that costs can be direct or indirect depending on the


organization. If each department is given a separate advertising
budget for example, then advertising in this case will be a direct
cost.
380

DEPARTMENTAL TRADING AND PROFIT AND LOSS STATEMENT


From the following information prepare Drink Limited Departmental
Trading and Profit and Loss Statement for the year ended March 31,
2004.

Sales - Soft Drink Department $400,000


Beer Department $500,000
Shandy Department $100,000

Cost of sales - Soft Drink Department $300,000


Beer Department $250,000
Shandy Department $ 75,000

Advertising - Soft Drink Department $ 10,000


Beer Department $ 20,000
Shandy Department $ 10,000

Rent $120,000
Administrative costs $200,000
Other costs $ 60,000
INDIRECT COSTS BASIS OF ALLOCATION
Rent Square feet
Administrative costs Number of employees
Other costs 1/3 to each department

┌─────────────────┬─────────────┬──────────────┐
│ SOFT DRINK DEPT.│ BEER DEPT. │ SHANDY DEPT. │
┌───────────────┼─────────────────┼─────────────┼──────────────┤
│ SQUARE FEET │ 2,000 │ 2,000 │ 1,000 │
│ EMPLOYEES │ 30 │ 50 │ 20 │
└───────────────┴─────────────────┴─────────────┴──────────────┘

WORKINGS
Total square feet = 2,000 + 2,000 + 1,000 = 5,000
Total number of employees = 30 + 50 + 20 = 100

Soft Drink Department % of total square feet = 2,000/5,000 = 40%


Beer Department % of total square feet = 2,000/5,000 = 40%
Shandy Department % of total square feet = 1,000/5,000 = 20%
381

Soft Drink Department % of total number of employees = 30/100 = 30%


Beer Department % of total number of employees = 50/100 = 50%
Shandy Department % of total number of employees = 20/100 = 20%

RENT ALLOCATION
Soft Drink Department = 0.40 x $120,000 = $48,000
Beer Department = 0.40 x $120,000 = $48,000
Shandy Department = 0.20 x $120,000 = $24,000

ADMINISTRATIVE COSTS ALLOCATION


Soft Drink Department = 0.30 x $200,000 = $ 60,000
Beer Department = 0.50 x $200,000 = $100,000
Shandy Department = 0.20 x $200,000 = $ 40,000

Allocation of other costs to each department = $60,000  3


= $20,000.

So $20,000 will be allocated to each department.

Drink Limited Trading and Profit and Loss Statement


for the year ended March 31, 2004
---------------------------------------------------------------
SOFT DRINK BEER SHANDY TOTAL

$ $ $ $
Sales 400,000 500,000 100,000 1,000,000
Cost of sales 300,000 250,000 75,000 625,000
Gross profit/(loss) 100,000 250,000 25,000 375,000
Departmental cost
Advertising 10,000 20,000 10,000 40,000
Departmental profit 90,000 230,000 15,000 335,000

Operating costs
Rent 48,000 48,000 24,000 120,000
Administrative costs 60,000 100,000 40,000 200,000
Other costs 20,000 20,000 20,000 60,000
128,000 168,000 84,000 380,000

Profit/loss) ( 38,000) 62,000 ( 69,000) ( 45,000)


========= ====== ========= =========
382

ROOMS DEPARTMENT INCOME STATEMENT


US$’000
Revenue (gross) 11,000
Sales allowances 100
Expenses
Salaries and wages 2,000
Employee benefits (excluding free meals) 250
Cable and satellite television 420
Commissions 50
Complimentary guest services 10
Contract services 40
Guest relocation 7
Guest transportation 22
Laundry and dry cleaning 60
Linen 16
Operating supplies 48
Reservations 32
Telecommunications 11
Training 18
Uniforms 26
Miscellaneous expenses 4

Additional information:
a. The gross revenue includes a 10% tax.

b. Commission amounting to US$12,000 payable to a travel agent


should be accrued.

c. Meals provided free of cost for the Rooms Department employees


amounted to US$50,000.

Prepare the Rooms Department Income Statement for the year ended
December 31, 2004 for Love Bug Hotel Limited in accordance with the
Uniform Systems of Accounts for the Lodging Industry.
383

Sales allowances should be subtracted from revenue in order to


obtain the net revenue for the department. Meals provided free of
cost to employees should be included in employee benefits. Even
though uniforms are an employee benefit the amount for uniforms is
not included in employee benefits but disclosed as a separate line
item expense under the Uniform Systems of Accounts for the Lodging
Industry. Miscellaneous expenses are referred to as ‘Other’ in the
Departmental Income Statement.

WORKINGS ($’000)

Revenue = [11,000/(100 + 10)] X 100


= 10,000

Commission = 50 + 12 = 62

Employee benefits = 250 + 50


= 300
384

Love Bug Hotel Limited


Rooms Department Income Statement
for the year ended December 31, 2004

US$’000
Revenue 10,000
Less Sales allowances 100
Net revenue 9,900

Expenses
Salaries and wages 2,000
Employee benefits 300
Total payroll and related expenses 2,300

Other Expenses
Cable and satellite television 420
Commissions 62
Complimentary guest services 10
Contract services 40
Guest relocation 7
Guest transportation 22
Laundry and dry cleaning 60
Linen 16
Operating supplies 48
Reservations 32
Telecommunications 11
Training 18
Uniforms 26
Other 4
Total other expenses 776

Total expenses 3,076

Department profit 6,824


=====
385

FOOD DEPARTMENT INCOME STATEMENT

US$’000 US$’000
Revenue (including sweets and cigarettes) 4,790
Sales allowances 40
Opening stock of food 80
Purchases of food 990
Closing stock of food 70
Costs of sweets and cigarettes 5
Beverage transfers to Food Department 23
Food transfers to Beverage Department 31
Promotional meals 10
Employee meals (provided free of cost)
Food Department 70
Beverage Department 40
Rooms Department 50
Administrative Department 30
190
Visual equipment rental 8
Meeting room rental income 780
Service charge income 470
Expenses
Salaries and wages 2,826
Employee benefits (excluding free meals) 480
China, glassware, silver, and linen 20
Contract services 48
Laundry and dry cleaning 50
Licenses 3
Cost of renting visual equipment 6
Music and entertainment 40
Operating supplies 10
Telecommunications 15
Training 12
Uniforms 240
Miscellaneous expenses 5

Prepare the Food Department Income Statement for the year ended
December 31, 2004 for Love Bug Hotel Limited in accordance with the
Uniform Systems of Accounts for the Lodging Industry.
386

Once you have other income a line item for total revenue will be at
the top of the Income Statement. The double line below the figure
for total revenue in the Income Statement is indicating that this
figure is not used in calculating the Department profit or loss.

Since revenue includes sweets and cigarettes, the cost of sweets and
cigarettes is referred to as other cost of sales. The net revenue
minus the total cost of sales will give us the gross profit. Visual
equipment rental income is referred to as miscellaneous banquet
income and the cost of renting visual equipment is referred to as
miscellaneous banquet expense. The department profit or loss is
obtained after deducting total expenses from gross profit and other
income.

WORKINGS ($’000)
Cost of Food
Opening stock of food 80
Add purchases of food 990
1,070
Less closing stock of food 70
1,000
=====

Employee benefits = 480 + 70 = 550

Total revenue = net revenue + total other income


= 4,750 + 1,258
= 6,008
387

Love Bug Hotel Limited


Food Department Income Statement
for the year ended December 31, 2004

US$’000
Total revenue 6,008
=====

Revenue 4,790
Less Sales allowances 40
Net revenue 4,750

Cost of sales
Cost of food 1,000
Add: Beverage transfers to Food Department 23
Less: Food transfers to Beverage Department ( 31)
Less: Promotional meals ( 10)
Less: Cost of employee meals ( 190)
Net cost of food 792
Other cost of sales 5
Total cost of sales 797

Gross profit on food sales 3,953

Other Income
Miscellaneous banquet income 8
Meeting room rental income 780
Service charge income 470
Total other income 1,258

Gross profit and other income carried forward 5,211


388

US$’000

Gross profit and other income brought forward 5,211

Expenses
Salaries and wages 2,826
Employee benefits 550
Total payroll and related expenses 3,376

Other Expenses
China, glassware, silver, and linen 20
Contract services 48
Laundry and dry cleaning 50
Licenses 3
Miscellaneous banquet expense 6
Music and entertainment 40
Operating supplies 10
Telecommunications 15
Training 12
Uniforms 240
Other 5
Total other expenses 449

Total expenses 3,825

Department profit 1,386


=====
389

BEVERAGE DEPARTMENT INCOME STATEMENT


US$’000
Revenue (including peanuts) 1,470
Sales allowances 2
Opening stock of beverage 70
Purchases of beverage 590
Closing stock of beverage 60
Costs of peanuts 2
Beverage transfers to Food Department 23
Food transfers to Beverage Department 31
Cover charge income 20
Service charge income 145
Expenses
Salaries and wages 470
Employee benefits (excluding free meals) 26
China, glassware, silver, and linen 31
Contract services 6
Laundry and dry cleaning 14
Licenses 2
Music and entertainment 15
Operating supplies 26
Telecommunications 5
Training 10
Uniforms 20
Miscellaneous expenses 3

Additional information:
a. Meal provided free of cost for the Beverage Department
employees amounted to US$40,000.

b. Food transfers to the Beverage Department include US$29,000 for


food used in the preparation of beverages and US$2,000 for
appetizers, served as complimentary food during happy hour at
the bar.

Prepare the Beverage Department Income Statement for the year ended
December 31, 2004 for Love Bug Hotel Limited in accordance with the
Uniform Systems of Accounts for the Lodging Industry.
390

Of the US$31,000 food transfers to the Beverage Department only


US$29,000 was used in the preparation of beverages. Therefore only
US$29,000 is added to the cost of beverage. The other US$2,000 is an
expense since this food was served free of costs to customers. This
expense is referred to as gratis food expense in the Income
Statement. Since revenue includes peanuts, the cost of peanuts is
referred to as other cost of sales.

WORKINGS ($’000)
Cost of Beverage
Opening stock of beverage 70
Add purchases of beverage 590
660
Less closing stock of beverage 60
600
===

Employee benefits = 26 + 40 = 66

Total revenue = net revenue + total other income


= 1,468 + 165
= 1,633
391

Love Bug Hotel Limited


Beverage Department Income Statement
for the year ended December 31, 2004

US$’000
Total revenue 1,633
=====

Revenue 1,470
Less Sales allowances 2
Net revenue 1,468

Cost of sales
Cost of beverage 600
Less: Beverage transfers to Food Department ( 23)
Add: Food transfers to Beverage Department 29
Net cost of beverage 606
Other cost of sales 2
Total cost of sales 608

Gross profit on beverage sales 860

Other Income
Cover charge income 20
Service charge income 145
Total other income 165

Gross profit and other income carried forward 1,025


392

US$’000

Gross profit and other income brought forward 1,025

Expenses
Salaries and wages 470
Employee benefits 66
Total payroll and related expenses 536

Other Expenses
China, glassware, silver, and linen 31
Contract services 6
Laundry and dry cleaning 14
Licenses 2
Music and entertainment 15
Operating supplies 26
Telecommunications 5
Training 10
Uniforms 20
Gratis food expense 2
Other 3
Total other expenses 134

Total expenses 670

Department profit 355


===
393

REVIEW QUESTIONS
(1)* Indirect costs are _________________ costs incurred for the
benefit of all departments.

(2)* Direct costs are costs that can be charged ______________ to


the relevant department.

(3)* Fill in the blanks.

INDIRECT COSTS BASIS OF ALLOCATION

(a) Electricity __________________________

(b) ____________________ Floor area

(c) Administrative expenses __________________________

(d) Advertising __________________________

(e) Insurance __________________________

(4) What are the advantages of Departmental Accounts?


394

EXERCISE 1 *

From the following information prepare the Departmental Trading and


Profit and Loss Statement for Sweet Limited for the year ended
December 31, 2003.
$
Sales - Perfume Department 1,250,000
Appliance Department 9,375,000
Clothing Department 1,875,000

Cost of sales - Perfume Department 460,000


Appliance Department 5,700,000
Clothing Department 900,000

Wages and salaries - Perfume Department 130,000


Appliance Department 450,000
Clothing Department 360,000

Insurance 50,000
Electricity 100,000
Rent 200,000
Administrative costs 960,000
Other costs 240,000

INDIRECT COSTS BASIS OF ALLOCATION

Insurance Value of assets insured


Electricity Floor area
Rent Floor area
Administrative costs Number of employees
Other costs Dollar sales
395

┌─────────────┬───────────────┬───────────────┐
│ PERFUME │ APPLIANCE │ CLOTHING │
│ DEPT. │ DEPT. │ DEPT. │
┌────────────────┼─────────────┼───────────────┼───────────────┤
│ ASSETS INSURED │ $800,000 │ $14,000,000 │ $5,200,000 │
│ │ │ │ │
│ FLOOR AREA │ 480 sq. ft. │ 3,720 sq. ft. │ 1,800 sq. ft. │
│ │ │ │ │
│ # OF EMPLOYEES │ 8 │ 16 │ 16 │
│ │ │ │ │
└────────────────┴─────────────┴───────────────┴───────────────┘
396

COLLEGE STUDENTS

EXERCISE 2 *

$
Revenue 10,668,000
Sales allowances 5,000
Expenses
Salaries and wages 3,602,200
Employee benefits 580,000
Cable and satellite television 48,000
Commissions 120,000
Complimentary guest services 2,700
Contract services 30,000
Guest transportation 28,600
Laundry and dry cleaning 430,100
Linen 10,000
Operating supplies 260,400
Reservations 26,700
Telecommunications 5,260
Training 4,000
Uniforms 50,000
Miscellaneous expenses 1,000

Additional information not yet reflected in the accounts:


a. Cable and satellite television prepaid amounted to $12,000.
b. Contract services to be accrued amounted to $10,000.

Prepare the Rooms Department Income Statement for the year ended
June 30, 2004 in accordance with the Uniform Systems of Accounts for
the Lodging Industry.
397

EXERCISE 3 *

$
Revenue (including cigarettes) 7,540,400
Sales allowances 8,400
Opening stock of food 400,000
Purchases of food 3,500,000
Closing stock of food 700,000
Costs of cigarettes 10,000
Beverage transfers to Food Department 3,000
Food transfers to Beverage Department 25,600
Promotional meals 17,000
Employee meals (provided free of cost) 136,000
Meeting room rental income 260,000
Service charge income 602,560
Shop rental income 710,000
Expenses
Salaries and wages 1,830,700
Employee benefits (excluding free meals) 520,800
China, glassware, silver, and linen 16,700
Contract services 46,400
Laundry and dry cleaning 230,000
Licenses 4,600
Music and entertainment 75,000
Operating supplies 102,900
Telecommunications 21,000
Training 40,000
Uniforms 80,000
Miscellaneous expenses 2,860

Additional information:
a. Shop rental income amounting to $10,000 should be accrued.
b. Meals provided free of cost to the Food Department employees
amounted to $32,000.

Prepare the Food Department Income Statement for the year ended
December 31, 2004 in accordance with the Uniform Systems of Accounts
for the Lodging Industry.
398

EXERCISE 4 *

$
Revenue (including peanuts) 2,740,700
Sales allowances 1,200
Opening stock of beverage 160,800
Purchases of beverage 530,100
Closing stock of beverage 150,400
Costs of peanuts 5,000
Beverage transfers to Food Department 2,000
Food transfers to Beverage Department 10,000
Games room income 164,370
Service charge income 630,400
Expenses
Salaries and wages 860,000
Employee benefits (excluding free meals) 240,000
China, glassware, silver, and linen 36,000
Contract services 5,000
Laundry and dry cleaning 12,600
Licenses 8,000
Music and entertainment 100,000
Operating supplies 45,000
Telecommunications 4,700
Training 10,000
Uniforms 40,000
Miscellaneous expenses 1,900

Additional information:
a. Meals provided free of cost for the Beverage Department
employees amounted to $20,000.
b. $60,000 should be accrued for retroactive salaries and wages.
c. Food transfers to the Beverage Department include $4,000 for
food used in the preparation of beverages and $6,000 for
appetizers, served as complimentary food during happy hour.

Prepare the Beverage Department Income Statement for the year ended
December 31, 2004 in accordance with the Uniform Systems of Accounts
for the Lodging Industry.
399

ANSWERS TO SELECTED REVIEW QUESTIONS


(1) Common

(2) Directly

(3) (a) Unit Production or Floor Area


(b) Rent and Rates or Electricity
(c) Number of Employees or Dollar Sales
(d) Dollar Sales
(e) Value of Assets Insured

EXERCISE 1

WORKINGS

Total assets insured = $800,000 + $14,000,000 + $5,200,000


= $20,000,000

Total floor area = 480 + 3,720 + 1,800 = 6,000

Total number of employees = 8 + 16 + 16 = 40

Total dollar sales = $1,250,000 + $9,375,000 + $1,875,000


= $12,500,000

INSURANCE ALLOCATION
Perfume = $50,000 x 0.04 = $ 2,000
Appliance = $50,000 x 0.70 = $35,000
Clothing = $50,000 x 0.26 = $13,000
400

ELECTRICITY ALLOCATION
Perfume = $100,000 x 0.08 = $ 8,000
Appliance = $100,000 x 0.62 = $62,000
Clothing = $100,000 x 0.30 = $30,000

RENT ALLOCATION
Perfume = $200,000 x 0.08 = $ 16,000
Appliance = $200,000 x 0.62 = $124,000
Clothing = $200,000 x 0.30 = $ 60,000

ADMINISTRATIVE COSTS ALLOCATION


Perfume = $960,000 x 0.20 = $192,000
Appliance = $960,000 x 0.40 = $384,000
Clothing = $960,000 x 0.40 = $384,000

OTHER COSTS ALLOCATION


Perfume = $240,000 x 0.10 = $ 24,000
Appliance = $240,000 x 0.75 = $180,000
Clothing = $240,000 x 0.15 = $ 36,000
401

Sweet Limited Trading and Profit and Loss Statement


for the year ended December 31, 2003
---------------------------------------------------------------
PERFUME APPLIANCE CLOTHING TOTAL

$'000 $'000 $'000 $'000


Sales 1,250 9,375 1,875 12,500
Less cost of sales 460 5,700 900 7,060
Gross profit 790 3,675 975 5,440
Departmental cost
Wages & salaries 130 450 360 940
Departmental profit 660 3,225 615 4,500

Operating costs
Insurance 2 35 13 50
Electricity 8 62 30 100
Rent 16 124 60 200
Administrative costs 192 384 384 960
Other costs 24 180 36 240
242 785 523 1,550

Profit 418 2,440 92 2,950


=== ===== == =====
402

EXERCISE 2

Rooms Department Income Statement


for the year ended June 30, 2004

$
Revenue 10,668,000
Less Sales allowances 5,000
Net revenue 10,663,000

Expenses
Salaries and wages 3,602,200
Employee benefits 580,000
Total payroll and related expenses 4,182,200

Other Expenses
Cable and satellite television 36,000
Commissions 120,000
Complimentary guest services 2,700
Contract services 40,000
Guest transportation 28,600
Laundry and dry cleaning 430,100
Linen 10,000
Operating supplies 260,400
Reservations 26,700
Telecommunications 5,260
Training 4,000
Uniforms 50,000
Other 1,000
Total other expenses 1,014,760

Total expenses 5,196,960

Department profit 5,466,040


=========
403

EXERCISE 3

Food Department Income Statement


for the year ended December 31, 2004

$
Total revenue 9,114,560
=========

Revenue 7,540,400
Less Sales allowances 8,400
Net revenue 7,532,000

Cost of sales
Cost of food 3,200,000
Add: Beverage transfers to Food Department 3,000
Less: Food transfers to Beverage Department ( 25,600)
Less: Promotional meals ( 17,000)
Less: Cost of employee meals ( 136,000)
Net cost of food 3,024,400
Other cost of sales 10,000
Total cost of sales 3,034,400

Gross profit on food sales 4,497,600

Other Income
Meeting room rental income 260,000
Service charge income 602,560
Shop rental income 720,000
Total other income 1,582,560

Gross profit and other income carried forward 6,080,160


404

Gross profit and other income brought forward 6,080,160

Expenses
Salaries and wages 1,830,700
Employee benefits 552,800
Total payroll and related expenses 2,383,500

Other Expenses
China, glassware, silver, and linen 16,700
Contract services 46,400
Laundry and dry cleaning 230,000
Licenses 4,600
Music and entertainment 75,000
Operating supplies 102,900
Telecommunications 21,000
Training 40,000
Uniforms 80,000
Other 2,860
Total other expenses 619,460

Total expenses 3,002,960

Department profit 3,077,200


=========
405

EXERCISE 4

Beverage Department Income Statement


for the year ended December 31, 2004

$
Total revenue 3,534,270
=========

Revenue 2,740,700
Less Sales allowances 1,200
Net revenue 2,739,500

Cost of sales
Cost of beverage 540,500
Less: Beverage transfers to Food Department ( 2,000)
Add: Food transfers to Beverage Department 4,000
Net cost of beverage 542,500
Other cost of sales 5,000
Total cost of sales 547,500

Gross profit on beverage sales 2,192,000

Other Income
Games room income 164,370
Service charge income 630,400
Total other income 794,770

Gross profit and other income carried forward 2,986,770


406

Gross profit and other income brought forward 2,986,770

Expenses
Salaries and wages 920,000
Employee benefits 260,000
Total payroll and related expenses 1,180,000

Other Expenses
China, glassware, silver, and linen 36,000
Contract services 5,000
Laundry and dry cleaning 12,600
Licenses 8,000
Music and entertainment 100,000
Operating supplies 45,000
Telecommunications 4,700
Training 10,000
Uniforms 40,000
Gratis food expense 6,000
Other 1,900
Total other expenses 269,200

Total expenses 1,449,200

Department profit 1,537,570


=========
407

Ch15
CHAPTER FIFTEEN

ACCOUNTING FOR

NON-PROFIT ORGANIZATIONS

CHAPTER OBJECTIVES
After completing this chapter you should be able to:

 Distinguish between subscriptions in arrears and


subscriptions in advance.

 Distinguish between an Income and Expenditure Account and a


Receipts and Payments Account.

 Prepare an Income and Expenditure Account.

 Prepare a Balance Sheet for a non-profit organization.

 Prepare financial statements for a statutory body.


408

INTRODUCTION
The main objective of a non-profit organization such as a sports
club or a Kiwanis club is to provide service. Club members pay a
subscription each year. The club will decide whether the
subscription should be paid monthly, or quarterly, or semi-annually,
or annually. When members owe the club subscriptions, this is
referred to as subscriptions in arrears. Subscription in arrears is
reflected in the Balance Sheet as a current asset. When members pay
subscriptions before the due date, this is referred to as
subscriptions in advance. In this case the club owes the members the
subscriptions they have paid in advance and this is reflected as a
current liability in the Balance Sheet.

In order to become a member of some clubs, such as the Kiwanis Club


of Liguanea, a member of the club must invite you to attend a
meeting. The Board of Directors of the club will examine your
application for membership and if you satisfy certain requirements
the Board of Directors will approve your application for membership.
You may be required to pay a membership fee (a one time charge) that
is separate from the annual fee (dues) that you and other members
are required to pay. In some cases the annual fee is calculated on a
pro rata basis where the individual's membership is approved during
the financial year. Non-profit organizations other than clubs exist
that do not require subscriptions, such as consumer affair groups,
government statutory bodies, and organizations to help the poor.

Since the objective of a non-profit organization is to provide


service and not to make a profit the organization prepares an Income
and Expenditure Account instead of a Profit and Loss Account. The
excess of income over expenditure is referred to as a surplus
instead of net profit and the excess of expenditure over income is
referred to as a deficit instead of a net loss. If the club makes a
surplus, this surplus is added to the accumulated fund, in other
words, you credit the Accumulated Fund Account with the surplus. If
the club makes a deficit, this deficit is subtracted from the
accumulated fund, in other words, you debit the Accumulated Fund
Account with the deficit. It is also important to note that the
organization prepares a Receipts and Payments Account instead of a
Cash Book and the capital is referred to as the accumulated fund.
409

Non-profit organizations usually apply to the Minister of Finance


for tax exemption status, so that the organization can receive
exemption from certain taxation charges.

PROFIT ORIENTED ORGANIZATION N0N-PROFIT ORIENTED ORGANIZATION


Profit and Loss Account Income and Expenditure Account
Sales Subscriptions/Fund-raising
revenue/Donations
Net Profit Surplus
Net Loss Deficit
Cash Book Receipts and Payments Account
Capital Accumulated Fund

Receipts and Payments A/C - DR. Receipts and CR. Payments

Income and Expenditure A/C - DR. Expenses and CR. Income

Only cash transactions are recorded in the receipts and payments


account, whereas, cash and non-cash transactions (such as
deprecation) are recorded in the income and expenditure account.
The balance in the receipts and payments account at the end of the
financial year is carried forward to the next financial year. The
balance in the income and expenditure account (surplus or deficit)
is transferred to the accumulated fund. The income and expenditure
account does not start with a balance brought forward, whereas the
receipts and payments account starts with a balance brought
forward.

DONATIONS RECEIVED BY THE CLUB


The double entry for a cash donation received is as follows:
DR. Receipts and Payments (Cash A/C)
CR. Income and Expenditure A/C
410

The double entry for a cheque donation received as follows:


DR. Receipts and Payments (Bank A/C)
CR. Income and Expenditure A/C

The double entry for a donation of material received for use on a


project or projects is as follows:
DR. Material A/C
CR. Income and Expenditure A/C
With the estimated value of the material received.

DR. Project Expense A/C


CR. Material A/C
With the estimated value of the material used.

The estimated value of any material on hand at the end of the


accounting period will be reflected as a current asset in the
Balance Sheet.

LIFE MEMBERSHIP FUND


If a member pays a lump sum subscription to obtain life membership
in the club, then the double entry will be:
DR. Receipts and Payments A/C
CR. Life Membership Fund A/C
With cash received for life membership.

The lump sum received for life membership is usually transferred to


the Income and Expenditure Account over the estimated life of the
member. The double entry will be:
DR. Life Membership Fund A/C
CR. Income and Expenditure A/C
With the amount transferred to the Income and Expenditure
Account during the financial year.

The balance in the Life Membership Fund A/C at the end of the
accounting period is shown as a non-current liability in the Balance
Sheet.
411

ACCOUNTING POLICY
The accounting policy of the organization may state that
subscriptions in arrears should not be reflected in the accounts. If
subscriptions in arrears are subsequently paid, then the amount
received will be treated as income in the financial year in which it
was received.

Let us assume that the Speculation District Football Club was


started on January 1, 2002 with 100 members. Each member is required
to pay $200 for the year. Subscriptions were received from 90
members. It is the club's policy to ignore any subscriptions in
arrears. The club's financial year ends on December 31.

The subscription income for 2002 will be 90 x $200 = $18,000. The


$2,000 subscriptions in arrears will not be recorded in the Income
and Expenditure Account or the Balance Sheet. Two of the ten members
who did not pay any subscription in 2002, paid their subscription
for 2002 in 2004. The subscriptions received in respect of 2002
($200 x 2 = $400) will be treated as income for 2004. The
subscription for each member in 2004 amounted to $250. A total of
100 members paid their subscription for 2004.

TOTAL SUBSCRIPTION INCOME FOR 2004


100 x $250 = $25,000
2 x $200 = $ 400
$25,400
=======

Please NOTE that the accounting policy of the club must clearly
state that subscription in arrears should not be reflected in the
accounts. If the accounting policy does not state this, then
subscription in arrears must be taken into account.
412

FINAL ACCOUNTS AND BALANCE SHEET


The following is a summary of Playgirl Sports Club Receipts and
Payments Account.

DR. RECEIPTS AND PAYMENTS A/C CR.


RECEIPTS PAYMENTS
$ ║ $
Bank balances 1/1/03 ║ Bar creditors 393,000
Current A/C 56,600 ║ Wages 63,000
Deposit A/C 115,000 ║ Electricity 28,000
Subscriptions ║ Rent 56,000
For 2002 6,000 ║ General expenses 6,000
For 2003 114,800 ║ New furniture 92,000
For 2004 9,600 ║ New refrigerator 24,000
Dance entrance fees 10,000 ║ Bank balances 31/12/03
Bar takings 481,000 ║ Current A/C 36,800
Deposit account interest 5,200 ║ Deposit A/C 99,400
798,200 ║ 798,200
======= ║ =======

ADDITIONAL INFORMATION:

(1) The assets and liabilities were: 31.12.02 31.12.03


$ $
Furniture (at cost) 88,000 ?
Refrigerator (at cost) 24,000 ?
Allowance for dep. - furniture 8,800 ?
Allowance for dep. - refrigerator 20,000 ?
Bar stock at cost 41,800 35,600
Creditors for bar purchases 37,200 49,600
Electricity accrued 1,800 2,200
Rent prepaid 2,400 5,200
Subscriptions in arrears 6,000 10,000

(2) The payment of $24,000 for the new refrigerator was obtained
after deducting the trade in allowance of $6,000 for the old
refrigerator.
413

(3) The barman is to receive a bonus of 5 per cent of the gross


profit for the bar.

(4) Depreciation on furniture should be provided at a rate of 10


per cent on cost. The new refrigerator is expected to last for
10 years with a residual value of $5,000. A full year's
depreciation is to be provided on the new furniture and the new
refrigerator.

REQUIRED: Prepare the Income and Expenditure Account for the year
ended December 31, 2003 and a Balance Sheet as at that
date.

WORKINGS
BAR PURCHASES
Payment to bar creditors $393,000
Less creditors for bar purchases on 31.12.02 $ 37,200
$355,800
Add creditors for bar purchases on 31.12.03 $ 49,600
$405,400
========

BAR TRADING ACCOUNT FOR THE YEAR ENDED DECEMBER 31, 2003
$ ║ $
Opening bar stock 41,800 ║ Bar takings (sales) 481,000
Bar purchases 405,400 ║
447,200 ║
Less closing bar stock 35,600 ║
Cost of sales 411,600 ║
Gross profit 69,400 ║
481,000 ║ 481,000
======= ║ =======
414

We must calculate the opening accumulated fund balance (capital)


since this figure was not given and it is needed for the Balance
Sheet.

Opening Accumulated Fund Balance


= Opening Asset Balances - Opening Liability Balances

Net book value for furniture = Cost - Allowance


= $88,000 - $ 8,800 = $79,200

Net book value for refrigerator = $24,000 - $20,000 = $ 4,000

The closing asset balances on 31/12/02 become the opening asset


balances on 1/1/03. The closing liability balances on 31/12/02
become the opening liability balances on 1/1/03.

The Bank Current Account balance and the Bank Deposit Account
balance of the Receipts and Payments Account for 1/1/03 represent
assets on 1/1/03.

Bank Current Account $ 56,600


Bank Deposit Account $115,000

Opening Asset Balances


= $56,600 + $115,000 + $41,800 + $2,400 + $6,000 + $79,200 + $4,000
= $305,000

Opening Liability Balances


= $37,200 + $1,800
= $39,000

Opening Accumulated Fund Balance


= Opening Asset Balances 1/1/03 - Opening Liability Balances 1/1/03
= $305,000 - $39,000
= $266,000
415

Depreciation on furniture = ($88,000 + $92,000) x 0.10


= $18,000

Cost of new refrigerator = Amount paid + Trade in allowance


= $24,000 + $6,000
= $30,000

Depreciation on refrigerator = $30,000 - $5,000


10
= $2,500

Profit on disposal of old refrigerator = Trade in allowance - NBV


= $6,000 - $4,000
= $2,000

Electricity expense
= Amount paid - Accrual 1/1/03 + Accrual 31/12/03
= $28,000 - $1,800 + $2,200
= $28,400

Rent expense
= Amount paid + Prepayment 1/1/03 - Prepayment 31/12/03
= $56,000 + $2,400 - $5,200
= $53,200

Barman bonus = $69,400 x 0.05


= $3,470

2003 Subscription income = Subscription received for 2003 +


Subscriptions in arrears for 2003
= $114,800 + $10,000
= $124,800
416

PLAYGIRL SPORTS CLUB INCOME AND EXPENDITURE ACCOUNT FOR THE YEAR
ENDED December 31, 2003
------------------------------------------------------------------
$ ║ $
Wages 63,000 ║ Subscriptions 124,800
Electricity 28,400 ║ Dance entrance fees 10,000
Rent 53,200 ║ Deposit account
General expenses 6,000 ║ interest 5,200
Barman bonus 3,470 ║ Bar gross profit 69,400
Depreciation on furniture 18,000 ║ Profit on disposal 2,000
Depreciation on refrigerator 2,500 ║
Surplus 36,830 ║
211,400 ║ 211,400
======= ║ =======

DR. ACCUMULATED FUND ACCOUNT CR.


$ ║ $
Balance c/f 302,830 ║ Balance b/f 266,000
║ Income & Exp A/C (surplus) 36,830
302,830 ║ 302,830
======= ║ =======

417

PLAYGIRL SPORTS CLUB BALANCE SHEET AS AT DECEMBER 31,2003


$ $ $
Non-current Assets
Furniture 180,000
Less allow. for dep. 26,800
153,200
Refrigerator 30,000
Less allow. for dep. 2,500
27,500
180,700
Current Assets
Bank Current A/C 36,800
Bank Deposit A/C 99,400
Bar stock 35,600
Rent prepaid 5,200
Subscriptions in arrears 10,000
187,000
Less Current Liabilities
Bar creditors 49,600
Electricity accrued 2,200
Barman bonus accrued 3,470
Subscriptions paid in advance 9,600
64,870
Net current assets 122,130
Total assets less current liabilities 302,830
=======
Financed By:
Accumulated fund 266,000
Add surplus 36,830
302,830
=======

STATUTORY BODY
A statutory body is an organization created by an Act of Parliament.
The financial year end for the statutory body is the same as the
fiscal year end for Central Government. For example, if Central
Government year end is March 31, then the statutory body year end
will be March 31. An Income and Expenditure Statement is prepared
for the statutory body comparing revenue with expenditure. If
revenue is greater than expenditure a surplus is made. If revenue is
less than expenditure a deficit is made.
418

National Water Supply Trial Balance on March 31, 2002


Debit Credit
$’000 $’000
Inventories 210,400
Other receivables and prepayments 35,624
Due from Government 30,650
Trade accounts receivable (net) 820,466
Cash and bank 180,235
Deposits and retentions 280,625
Trade accounts payable 31,400
Other payables 41,655
Long term loan 18,000
Capital reserve 850,000
Accumulated surplus April 1, 2001 534,583
Property, plant and equipment 2,751,200
Accumulated depreciation 403,630
Long term receivable 400,000
Operating revenue 3,756,219
Project management fees 10,216
Interest income 14,410
Profit on disposal of land 1,100,000
Salaries, wages and related costs 1,456,100
Repairs and maintenance 398,755
Utilities 613,712
Fuel and lubrication 23,176
Bank charges 6,120
Depreciation 85,100
Water purchases 26,500
Interest expense 2,700 -
7,040,738 7,040,738
========= =========
Additional information:
1. Current portion of long term loan $5,000,000.
2. Current portion of long term receivable $100,000,000.

Required:
a. Prepare the Income and Expenditure Statement for the year
ended March 31, 2002.
b. Prepare the Statement of Changes in Equity for the year
ended March 31, 2002
c. Prepare the Balance Sheet as at March 31, 2002.
419

National Water Supply


Income and Expenditure Statement for the year ended March 31, 2002
$’000
Revenue
Operating revenue 3,756,219
Project management fees 10,216
Interest income 14,410
Total revenue 3,780,845

Expenditure
Salaries, wages and related cost 1,456,100
Repairs and maintenance 398,755
Utilities 613,712
Fuels and lubrication 23,176
Bank charges 6,120
Depreciation 85,100
Water purchases 26,500
Interest expense 2,700
Total expenditure 2,612,163

Operating surplus 1,168,682


Profit on disposal of land 1,100,000
Surplus for the year 2,268,682
=========

National Water Supply


Statement of Changes in Equity
for the year ended March 31, 2002

Capital Accumulated Total


Reserve Surplus
$’000 $’000 $’000
Balance April 1, 2001 850,000 534,583 1,384,583
Surplus for the year - 2,268,682 2,268,682
Balance March 31, 2002 850,000 2,803,265 3,653,265
======= ========= =========
420

National Water Supply


Balance Sheet as at March 31, 2002
$’000
Non-current Assets
Property, plant and equipment 2,751,200
Less accumulated depreciation 403,630
2,347,570
Long term receivable 300,000
2,647,570

Current Assets
Inventories 210,400
Other receivables and prepayments 35,624
Due from Government 30,650
Trade accounts receivable (net) 820,466
Current portion of long term receivable 100,000
Cash and bank 180,235
1,377,375

Less Current Liabilities


Deposits and retentions 280,625
Trade accounts payable 31,400
Other payables 41,655
Current portion of long term loan 5,000
358,680

Net current assets 1,018,695


Total assets less current liabilities 3,666,265

Less Non-current Liabilities


Long term loan 13,000
3,653,265
=========

Financed by
Capital reserve 850,000
Accumulated surplus 2,803,265
Total accumulated fund 3,653,265
=========
421

REVIEW QUESTIONS
(1) Distinguish between subscriptions in advance and subscriptions
in arrears.

(2) Distinguish between a Profit and Loss Account and an Income


and Expenditure Account.

(3)* Subscription in arrears is a _______________ _____________


in the Balance Sheet.

(4)* Subscription in advance is a _______________ _____________


in the Balance Sheet.

(5) Distinguish between a Receipts and Payments Account and an


Income and Expenditure Account.

EXERCISE 1 *

From the following information prepare the Subscription Account for


the financial year ended December 31, 2004.

Number of members 100


Annual subscription per member $ 1,000
Subscriptions in arrears on December 31, 2003 $10,000
Subscriptions in arrears on December 31, 2004 $15,000
Subscriptions paid in advance on December 31, 2003 $ 2,000
Subscriptions paid in advance on December 31, 2004 $ 5,000
422

EXERCISE 2 *

The following information relates to Happy Go Lookers Club for the


financial year ended December 31, 2003.

DR. RECEIPTS AND PAYMENTS A/C CR.


RECEIPTS PAYMENTS
$ ║ $
Bank Balance b/f 10,000 ║ Fundraising expenses 25,000
Subscriptions for 2002 1,000 ║ Wages 5,000
Subscriptions for 2003 49,000 ║ Secretarial expenses 4,000
Subscriptions for 2004 5,000 ║ Electricity 8,000
Fundraising 60,000 ║ Rent 30,000
║ Bank balance c/f 53,000
125,000 ║ 125,000
======= ║ =======

ADDITIONAL INFORMATION:

(1) Subscriptions in arrears at December 31, 2002 $2,000.


(2) The subscription for each member is $1,000. Fifty (50) members
were registered with the club in 2003.
(3) Wages outstanding at December 31, 2003 amounted to $1,000.
(4) Rent paid in advance for January 2004 amounted to $2,000.
(5) Furniture and fittings on January 1, 2003 $45,000.
(6) Furniture and fittings on December 31, 2003 $43,000.

REQUIRED: Prepare the Income and Expenditure Account for the


financial year ended December 31, 2003 and a Balance
Sheet as at that date.
423

College Student

EXERCISE 3 *

The Surrey Coffee Growers Co-operative has a membership of 100


coffee farmers. The co-operative surplus for the year ended June 30,
2004 amounted to $7,850,000. Each member owns 1,000 shares and is
entitled to dividends after the following are provided for:

1. 10% Hurricane Crop Insurance.


2. 20% Coffee Growers Pension Fund.
3. 5% Scholarship Fund.

Required:

a. Prepare Surrey Coffee Growers Co-operative Income and


Expenditure Appropriation section for the year ended June 30,
2004.

b. What is the dividend per share?

c. How much will each coffee farmer receive as dividend?


424

EXERCISE 4 *

National Water Supply Trial Balance on March 31, 2003


Debit Credit
$’000 $’000
Inventories 231,440
Other receivables and prepayments 39,180
Due from Government 536,780
Trade accounts receivable (net) 738,420
Cash and bank 144,188
Deposits and retentions 290,850
Trade accounts payable 43,601
Other payables 17,218
Long term loan 13,000
Capital reserve 850,000
Accumulated surplus April 1, 2001 2,803,265
Property, plant and equipment 2,751,200
Accumulated depreciation 482,880
Long term receivable 300,000
Operating revenue 3,540,990
Project management fees 9,620
Interest income 10,808
Salaries, wages and related costs 2,043,466
Repairs and maintenance 462,555
Utilities 681,220
Fuel and lubrication 25,957
Bank charges 6,426
Depreciation 79,250
Water purchases 20,200
Interest expense 1,950 -
8,062,232 8,062,232
========= =========
Additional information:
1. Current portion of long term loan $8,000,000.
2. Current portion of long term receivable $140,000,000.

Required:
a. Prepare the Income and Expenditure Statement for the
year ended March 31, 2003.
b. Prepare the Statement of Changes in Equity for the year
ended March 31, 2003.
c. Prepare the Balance Sheet as at March 31, 2003.
425

ANSWERS TO SELECTED REVIEW QUESTIONS


(3) Current Asset

(4) Current Liability

EXERCISE 1

DR. SUBSCRIPTION ACCOUNT CR.


$ ║ $
Balance b/f ║ Balance b/f
(arrears 1/1/04) 10,000 ║ (advance 1/1/04) 2,000
Income & Expenditure A/C 100,000 ║ Bank A/C 98,000
Balance c/f ║ Balance c/f
(advance 31/12/04) 5,000 ║ (arrears 31/12/04) 15,000
115,000 ║ 115,000
======= ║ =======

WORKINGS
Income & Expenditure Account amount
= Annual Subscription x Number of Members
= $1,000 x 100
= $100,000

Bank = $10,000 + $100,000 + $5,000 - $2,000 - $15,000


= $98,000

EXERCISE 2

WORKINGS
Wages = $ 5,000 + $ 1,000 = $ 6,000
Rent = $30,000 - $ 2,000 = $28,000
Depreciation = $45,000 - $43,000 = $ 2,000
Subscriptions = 50 x $1,000 = $50,000
426

HAPPY GO LOOKERS CLUB INCOME AND EXPENDITURE ACCOUNT FOR THE YEAR
ENDED December 31, 2003
------------------------------------------------------------------
$ ║ $
Fundraising expenses 25,000 ║ Subscriptions 50,000
Wages 6,000 ║ Fundraising 60,000
Secretarial expenses 4,000 ║
Electricity 8,000 ║
Rent 28,000 ║
Depreciation 2,000 ║
Surplus 37,000 ║
110,000 ║ 110,000
======= ║ =======

SUBSCRIPTIONS IN ARREARS AT 31/12/03


$
2002 arrears 2,000
Less amount received 1,000
1,000
2003 arrears ($50,000 - $49,000) 1,000
2,000
=====

OPENING ACCUMULATED FUND BALANCE ON 1/1/03

Assets = Bank balance 1/1/03 + Subscriptions in arrears 1/1/03 +


Furniture and fittings 1/1/03

Assets = $10,000 + $2,000 + $45,000 = $57,000

Liabilities = $0

Accumulated fund = $57,000 - $0 = $57,000


427

HAPPY GO LOOKERS CLUB BALANCE SHEET AS AT DECEMBER 31,2003


$ $
Non-current Asset
Furniture and fittings (net) 43,000

Current Assets
Bank 53,000
Rent prepaid 2,000
Subscriptions in arrears 2,000
57,000
Less Current Liabilities
Wages accrued 1,000
Subscriptions paid in advance 5,000
6,000
Net current assets 51,000
Total assets less current liabilities 94,000
======

Financed By:
Accumulated fund 57,000
Add surplus 37,000
94,000
======

EXERCISE 3

WORKINGS

Hurricane Crop Insurance = $7,850,000 X 0.10 = $785,000

Coffee Growers Pension Fund = $7,850,000 X 0.20 = $1,570,000

Scholarship fund = $7,850,000 X 0.05 = $392,500


428

SURREY COFFEE GROWERS CO-OPERATIVE INCOME AND EXPENDITURE


APPROPRIATION SECTION FOR THE YEAR ENDED JUNE 30, 2004
----------------------------------------------------------------
$ $
Surplus 7,850,000
Less
Hurricane Crop Insurance 785,000
Coffee Growers Pension Fund 1,570,000
Scholarship Fund 392,500
Dividends 5,102,500
7,850,000
-
=========

Dividend per share = $5,102,500/100,000 = $51.025

Dividend received by each farmer = $51.025 X 1,000 = $51,025


429

EXERCISE 4

National Water Supply


Income and Expenditure Statement for the year ended March 31, 2003
$’000
Revenue
Operating revenue 3,540,990
Project management fees 9,620
Interest income 10,808
Total revenue 3,561,418

Expenditure
Salaries, wages and related cost 2,043,466
Repairs and maintenance 462,555
Utilities 681,220
Fuels and lubrication 25,957
Bank charges 6,426
Depreciation 79,250
Water purchases 20,200
Interest expense 1,950
Total expenditure 3,321,024

Surplus for the year 240,394


=======

National Water Supply


Statement of Changes in Equity Fund
for the year ended March 31, 2003

Capital Accumulated Total


Reserve Surplus
$’000 $’000 $’000
Balance April 1, 2002 850,000 2,803,265 3,653,265
Surplus for the year - 240,394 240,394
Balance March 31, 2003 850,000 3,043,659 3,893,659
======= ========= =========
430

National Water Supply


Balance Sheet as at March 31, 2003
$’000
Non-current Assets
Property, plant and equipment 2,751,200
Less accumulated depreciation 482,880
2,268,320
Long term receivable 160,000
2,428,320

Current Assets
Inventories 231,440
Other receivables and prepayments 39,180
Due from Government 536,780
Trade accounts receivable (net) 738,420
Current portion of long term receivable 140,000
Cash and bank 144,188
1,830,008

Less Current Liabilities


Deposits and retentions 290,850
Trade accounts payable 43,601
Other payables 17,218
Current portion of long term loan 8,000
359,669

Net current assets 1,470,339


Total assets less current liabilities 3,898,659

Less Non-current Liabilities


Long term loan 5,000
3,893,659
=========

Financed by
Capital reserve 850,000
Accumulated surplus 3,043,659
Total accumulated fund 3,893,659
=========
431

ch16
CHAPTER SIXTEEN

MANUFACTURING ACCOUNTS

CHAPTER OBJECTIVES
After completing this chapter you should be able to:

 Distinguish between a Manufacturing Account and a Trading


Account.

 Calculate the cost of raw material consumed.

 Calculate the prime cost.

 Calculate the total manufacturing cost or production cost.

 Calculate the manufacturing cost per unit.

 Explain the difference between a manufacturing cost center


and a manufacturing profit center.

 Explain the nature of unrealized profit.

 Prepare the Final Accounts of a manufacturing business.

 Prepare a Job Cost Sheet.


432

INTRODUCTION
The management of a business that is involved in the manufacture of
goods would certainly need to know the total manufacturing cost or
production cost of the goods that they produce during the accounting
period. Management would also need to know the unit manufacturing
cost. This information will assist management in controlling the
unit cost of production and in pricing the product.

In order to determine the manufacturing costs of the goods produced


during the accounting period, the firm prepares a Manufacturing
Account. In the Manufacturing Account, the accountant first
calculates the cost of raw material consumed. All direct
manufacturing costs are then added to the cost of raw material
consumed to arrive at the prime costs. All costs that are traceable
to the finished product for a particular period are direct costs,
such as, material and labour costs.

All indirect manufacturing costs and opening stock of work in


progress are added to the prime costs and the closing stock of work
in progress is then subtracted to arrive at the total manufacturing
costs. Indirect costs are costs that are not traceable to the
finished product for a particular period. Indirect cost is usually
referred to as overhead. If the manufacturing department is a cost
center, then the total manufacturing cost is transferred to the
Trading Account and is added to the opening stock of finished goods.
If the manufacturing department is a profit center, then a profit is
added to the total manufacturing cost and then this grand total
(transfer price) is added to the opening stock of finished goods in
the Trading Account.
433

COST OF RAW MATERIALS CONSUMED


The cost of raw materials consumed is calculated in the
Manufacturing Account as follows:

$
Opening stock of raw materials XX
Add purchases of raw materials X
Add carriage inwards on raw materials X
XXXX
Less returns outward of raw materials X
XXX
Less closing stock of raw materials X
Cost of raw materials consumed XX
====

The X above represents any arbitrary figure and is used for


illustration purposes only. The format for calculating the cost of
raw materials consumed will vary depending on the needs of the
business. If for example, the business does not return any raw
materials during the accounting period, then that section (less
returns outward of raw materials) will not be needed.

PRIME COST
The prime cost is calculated in the Manufacturing Account by adding
all direct manufacturing costs to the costs of raw materials
consumed.
$
Cost of raw materials consumed XX
Add direct wages X
Add royalty X
Add other direct manufacturing costs X
Prime cost XXXXX
=====
The calculation may vary depending on the needs of the business, for
example, if no royalty is paid by the business then that section
(add royalty) would not be relevant. Royalty is a fee paid by a
manufacturing firm (let us say firm X) to another firm (let us say
firm Y) for the right to manufacture and sell the other firm's (firm
Y) brand name product(s).
434

MANUFACTURING COST OR COST OF PRODUCTION


The total manufacturing cost is calculated in the Manufacturing
Account as follows:

$
Opening stock of raw materials XX
Add purchases of raw materials X
Add carriage inwards on raw materials X
XXXX
Less returns outward of raw materials X
XXX
Less closing stock of raw materials X
Cost of raw materials consumed XX
Add direct wages X
Add royalty X
Add other direct manufacturing costs X
Prime cost XXXXX
Add indirect costs X
XXXXXX
Add opening stock of WIP X
XXXXXXX
Less closing stock of WIP X
Total manufacturing costs or cost of production XXXXXX
=======

Even though I included only one figure for indirect costs in the
Manufacturing Account, each item of indirect costs would be stated
separately.

MANUFACTURING COST PER UNIT


The manufacturing cost per unit or production cost per unit is
determined by the following equation:

Manufacturing Cost Per Unit = Total Manufacturing Cost


Total Number of Units Produced
435

MANUFACTURING ACCOUNT

COST CENTER
When the manufacturing department is a cost center, it accumulates
the costs for the units produced. The total manufacturing cost for
the finished goods produced is then transferred to the trading
department and added to the value of the opening stock of finished
goods.

From the following information prepare a Manufacturing Account for


Happy Hippo Ltd. for the year ended December 31, 2004, and calculate
the manufacturing cost per unit if 100,000 units are produced for
the financial year. Happy Hippo Ltd. manufacturing department is
classified as a cost center.

$
Opening stock of raw materials 10,000
Purchases of raw materials 260,000
Returns outward of raw materials 20,000
Carriage inwards on raw materials 5,000
Closing stock of raw materials 15,000
Direct wages 850,000
Royalty 100,000
Electricity 250,000
Water 30,000
Factory rent 240,000
Depreciation on factory machines 20,000
Repairs to factory machines 10,000
Opening stock of work in progress 50,000
Closing stock of work in progress 30,000
436

HAPPY HIPPO LTD. MANUFACTURING ACCOUNT FOR THE YEAR ENDED


DECEMBER 31, 2004
-----------------------------------------------------------------------------
$ ║ $
Opening stock of raw ║ Trading A/C (total
materials 10,000 ║ manufacturing
Add purchases of raw ║ cost) 1,760,000
materials 260,000 ║
Add carriage inwards ║
on raw materials 5,000 ║
275,000 ║
Less returns outward ║
of raw materials 20,000 ║
255,000 ║
Less closing stock ║
of raw materials 15,000 ║
Cost of raw materials ║
consumed 240,000 ║
Direct wages 850,000 ║
Royalty 100,000 ║
Prime cost 1,190,000 ║
Electricity 250,000 ║
Water 30,000 ║
Factory rent 240,000 ║
Dep. on factory ║
machines 20,000 ║
Repairs to factory ║
machines 10,000 ║
1,740,000 ║
Add opening stock WIP 50,000 ║
1,790,000 ║
Less closing stock WIP 30,000 ║
1,760,000 ║ 1,760,000
========= ║ =========

Manufacturing Cost Per Unit = Total Manufacturing Cost


Number of Units Produced

= $1,760,000
100,000
= $17.60
437

PROFIT CENTER
When the manufacturing department is a profit center, it adds a
profit to the total manufacturing cost. In other words, the
manufacturing department sells the goods manufactured to the trading
department at a profit. Since the trading department is within the
same firm, then this profit is an internal profit. The manufacturing
profit (internal profit) is debited in the Manufacturing Account and
credited in the Profit and Loss Account. The manufacturing profit
plus the manufacturing cost is referred to as the transfer price.
The transfer price is added to the opening stock of finished goods
in the Trading Account.

Let us use the same data from the previous problem and modify it for
the profit element.

From the following information prepare a Manufacturing Account for


Happy Hippo Ltd. for the year ended December 31, 2004, and calculate
the manufacturing cost per unit and the transfer price per unit if
100,000 units are produced for the financial year. Goods are
transferred from the manufacturing department to the trading
department at a mark-up of 20%. Happy Hippo Ltd. manufacturing
department is classified as a profit center.

$
Opening stock of raw materials 10,000
Purchases of raw materials 260,000
Returns outward of raw materials 20,000
Carriage inwards on raw materials 5,000
Closing stock of raw materials 15,000
Direct wages 850,000
Royalty 100,000
Electricity 250,000
Water 30,000
Factory rent 240,000
Depreciation on factory machines 20,000
Repairs to factory machines 10,000
Opening stock of work in progress 50,000
Closing stock of work in progress 30,000
438

HAPPY HIPPO LTD. MANUFACTURING ACCOUNT FOR THE YEAR ENDED


DECEMBER 31, 2004
-----------------------------------------------------------------------------
$ ║ $
Opening stock of raw ║ Trading A/C (total
materials 10,000 ║ manufacturing
Add purchases of raw ║ cost) 1,760,000
materials 260,000 ║
Add carriage inwards ║
on raw materials 5,000 ║
275,000 ║
Less returns outward ║
of raw materials 20,000 ║
255,000 ║
Less closing stock ║
of raw materials 15,000 ║
Cost of raw materials ║
consumed 240,000 ║
Direct wages 850,000 ║
Royalty 100,000 ║
Prime cost 1,190,000 ║
Electricity 250,000 ║
Water 30,000 ║
Factory rent 240,000 ║
Dep. on factory ║
machines 20,000 ║
Repairs to factory ║
machines 10,000 ║
1,740,000 ║
Add opening stock WIP 50,000 ║
1,790,000 ║
Less closing stock WIP 30,000 ║
1,760,000 ║
Manufacturing profit 352,000 ║ -
2,112,000 ║ 2,112,000
========= ║ =========
439

Manufacturing Profit = Total Manufacturing Cost x 20


100
= $1,760,000 x 20
100
= $352,000

Manufacturing Cost Per Unit = Total Manufacturing Cost


Number of Units Produced

= $1,760,000
100,000

= $17.60

Transfer Price Per Unit = Total Transfer Price


Number of Units Produced

= $2,112,000
100,000

= $21.12
440

WORK IN PROGRESS VALUED AT PRIME COST


If work in progress (WIP) or work in process (partly manufactured
goods) were valued at prime cost (but this must be clearly stated),
then prime cost would be calculated as follows:

$
Cost of raw materials consumed XX
Add direct wages X
Add royalty X
Add opening stock of WIP (valued at prime cost) X
Add other direct manufacturing costs X
XXXXXX
Less closing stock of WIP (valued at prime cost) X
Prime cost XXXXX
======

From the following information for Ice Skating Ltd. prepare the
Manufacturing Account for the year ended November 30, 2004.

$
Opening stock of raw materials 25,000
Closing stock of raw materials 15,000
Purchases of raw materials 200,000
Direct wages 350,000
Electricity 100,000
Plant repairs 30,000
Water 40,000
Plant depreciation 50,000
Opening stock WIP (valued at prime cost) 40,000
Closing stock WIP (valued at prime cost) 25,000
441

ICE SKATING LTD. MANUFACTURING ACCOUNT FOR THE YEAR ENDED


NOVEMBER 30, 2004
-----------------------------------------------------------------
$ ║ $
Opening stock of raw ║ Trading A/C (manu-
materials 25,000 ║ facturing cost) 795,000
Purchases of raw materials 200,000 ║
225,000 ║
Less closing stock of ║
raw materials 15,000 ║
Cost of raw materials ║
consumed 210,000 ║
Direct wages 350,000 ║
Add opening stock of WIP 40,000 ║
600,000 ║
Less closing stock of WIP 25,000 ║
Prime cost 575,000 ║
Electricity 100,000 ║
Plant repairs 30,000 ║
Water 40,000 ║
Plant depreciation 50,000 ║ -
795,000 ║ 795,000
======= ║ =======
442

COST CENTER TRADING ACCOUNT


When the manufacturing department is a cost center, the Trading
Account will appear as follows:

DR. TRADING ACCOUNT CR.


$ ║ $
Opening stock of finished goods X ║ Sales XXX
Manufacturing costs X ║
XX ║
Add purchases of finished goods X ║
XXX ║
Less closing stock of finished ║
goods X ║
Cost of sales XX ║
Gross profit X ║
XXX ║ XXX
=== =====

PLEASE NOTE: The business can also make a gross loss.

PROFIT CENTER TRADING ACCOUNT


When the manufacturing department is a profit center, the Trading
Account will appear as follows:

DR. TRADING ACCOUNT CR.


$ ║ $
Opening stock of finished goods X ║ Sales XXX
Transfer price X ║
XX ║
Add purchases of finished goods X ║
XXX ║
Less closing stock of finished ║
goods X ║
Cost of sales XX ║
Gross profit X ║
XXX ║ XXX
=== ║ ===

PLEASE NOTE: The business can also make a gross loss.


443

Under certain circumstances, finished goods are purchased from


outside the firm. This may occur because of a strike or electrical
power supply interruptions.

UNREALIZED PROFIT
When the manufacturing department is a profit center the goods
manufactured (finished goods) are transferred to the trading
department at a profit. This profit is an internal profit and is
only realized by the business when the goods are sold by the trading
department to outside customers. If the internal profit is not
realized at the end of the accounting period, it is referred to as
unrealized profit.

MANUFACTURING TRADING OUTSIDE


DEPARTMENT DEPARTMENT CUSTOMERS
PROFIT CENTER

Let us assume that we just started business and the manufacturing


department manufactured 100 units at a manufacturing cost of $10 per
unit and adds a profit of $2 per unit. The 100 units are then
transferred to the trading department.

Total Manufacturing Costs = 100 x $10 = $1,000


Total Profit = 100 x $ 2 = $ 200
Transfer Price $1,200

Transfer Price Per Unit = $10 + $2 = $12

Let us assume that the trading department at the end of the


accounting period sold 90 units to outside customers. Therefore, the
number of units not yet sold to outside customers equals ten units
(100 - 90). Since these ten (10) units have not been sold to outside
customers, the manufacturing profit on these goods is unrealized at
the end of the accounting period.
444

Unrealized Manufacturing Profit


= Number of units unsold x profit per unit
= 10 units x $2
= $20

This unrealized profit of $20 (allowance for unrealized profit) is


subtracted from the value of finished goods in the Balance Sheet.

BALANCE SHEET (EXTRACT)


--------------------------------------------------------------
Current Assets
Closing stock of finished goods (10 units x $12) $120
Less allowance for unrealized profit $ 20
$100

The $100 represents the manufacturing cost of the 10 units, that is,
10 units x $10 = $100.

If an allowance for unrealized profit already exists, then a


comparison of the existing allowance (opening allowance balance)
must be made with the new allowance (closing allowance balance) to
determine whether to increase or decrease the allowance. Any
increase or decrease in the allowance for unrealized profit is
reflected in the Profit and Loss Account, and the closing balance in
the allowance for unrealized profit is subtracted from the value of
finished goods in the Balance Sheet. The opening balance in the
Allowance Account is based on the opening stock of finished goods
and the closing balance in the Allowance Account is based on the
closing stock of finished goods.

To create the allowance account


DR. Increase in Allowance for Unrealized Profit A/C (Expense)
CR. Allowance for Unrealized Profit A/C

To increase the allowance account


DR. Increase in Allowance for Unrealized Profit A/C (Expense)
CR. Allowance for Unrealized Profit A/C

To decrease the allowance account


DR. Allowance for Unrealized Profit A/C
CR. Decrease in Allowance for Unrealized Profit A/C (Income)
445

FINAL ACCOUNTS FOR A MANUFACTURING ENTERPRISE


The Final Accounts of a manufacturing business include the
Manufacturing Account, The Trading Account and the Profit and Loss
Account.

From the following information prepare the Final Accounts of Santa


for the year ended June 30, 2004, and the Balance Sheet at that
date.
$
Opening Stock
Raw materials 60,000
Partly manufactured goods (WIP) 120,000
Finished goods 200,000
Purchases
Raw materials 560,000
Finished goods 30,000
Closing Stock
Raw materials 40,000
Partly manufactured goods (WIP) 100,000
Finished goods 250,000
Raw material returns 15,000
Direct manufacturing wages and salaries 790,000
Discount allowed 2,000
Discount received 6,000
Office salaries and wages 320,000
Electricity 200,000
Water 60,000
Depreciation
Factory machines 20,000
Factory furniture 5,000
Office furniture 10,000
Factory machines (net of depreciation) 200,000
Factory furniture (net of depreciation) 50,000
Office furniture (net of depreciation) 100,000
Sales 1,900,000
Creditors 25,000
Debtors 60,000
Cash and bank 70,000
Capital 901,000
Accruals 30,000
General expenses 20,000
446

ADDITIONAL INFORMATION:

(a) Electricity is to be apportioned as follows:


Factory 80%
Office 20%

(b) Water should be apportioned as follows:


Factory 90%
Office 10%

SANTA MANUFACTURING ACCOUNT FOR THE YEAR ENDED JUNE 30, 2004
$ ║ $
Opening stock of raw ║ Trading A/C (cost
materials 60,000 ║ of goods manu-
Purchases of raw materials 560,000 ║ factured) 1,614,000
620,000 ║
Less raw material returns 15,000 ║
605,000 ║
Less closing stock of ║
raw materials 40,000 ║
Cost of raw materials ║
consumed 565,000 ║
Direct manufacturing ║
wages and salaries 790,000 ║
Prime cost 1,355,000 ║
Electricity ║
($200,000 x 0.80) 160,000 ║
Water ($60,000 x 0.90) 54,000 ║
Depreciation - factory ║
machines 20,000 ║
Depreciation - factory ║
furniture 5,000 ║
1,594,000 ║
Add opening stock WIP 120,000 ║
1,714,000 ║
Less closing stock WIP 100,000 ║ -
1,614,000 ║ 1,614,000
========= ║ =========

447

SANTA TRADING ACCOUNT FOR THE YEAR ENDED JUNE 30, 2004
$ ║ $
Opening stock of finished ║ Sales 1,900,000
goods 200,000 ║
Cost of goods manufactured 1,614,000 ║
Purchases of finished goods 30,000 ║
1,844,000 ║
Less closing stock of ║
finished goods 250,000 ║
Cost of sales 1,594,000 ║
Gross profit 306,000 ║ -
1,900,000 ║ 1,900,000
========= ║ =========

SANTA PROFIT AND LOSS ACCOUNT FOR THE YEAR ENDED JUNE 30, 2004
$ ║ $
Discount allowed 2,000 ║ Gross profit 306,000
Office salaries & wages 320,000 ║ Discount received 6,000
Electricity ║ Loss (transferred
($200,000 x 0.20) 40,000 ║ to Capital A/C) 86,000
Water ($60,000 x 0.10) 6,000 ║
Depreciation - office ║
furniture 10,000 ║
General expenses 20,000 ║ -
398,000 ║ 398,000
======= ║ =======

448

SANTA BALANCE SHEET AS AT JUNE 30, 2004


---------------------------------------------------------------
$ $ $
Non-current Assets
Factory machines (net) 200,000
Factory furniture (net) 50,000
Office furniture (net) 100,000
350,000
Current Assets
Closing stock
Raw materials 40,000
Work in progress 100,000
Finished goods 250,000
390,000
Debtors 60,000
Cash and bank 70,000
520,000

Less Current Liabilities


Creditors 25,000
Accruals 30,000
55,000
Net current assets 465,000
815,000
=======

Financed By:
Capital 901,000
Less loss 86,000
815,000
=======
449

JOB COSTING
The production process is either continuous or discrete. A
continuous production process is suitable where the product is
homogeneous. In this case the product is being produced in
anticipation of demand, for example, the production of cement, soft
drinks and televisions. A discrete production process is suitable
where the product is produced to customer specification, in such
industries, as construction and printing. Job order costing is
usually used for a discrete production process, where cost is
accumulated for the product on a cost sheet. The cost sheet will
indicate the total cost for the product so that management can
decide on the final price for the product (if the quoted price can
be adjusted), or the future quoted price for similar products (if
the quoted price cannot be adjusted). The actual cost on the cost
sheet can be compared with the budgeted cost for the product and
material variances investigated.

From the information provided prepare a job cost sheet for


Quality Furniture Production Limited. The name of the customer
is Michael Williams, his cell phone number is 3719602 and his
home address is 10 Longwood Drive, Kingston.

Job Order #T1147


Quoted price for dining room table $16,000
11/8/2003 direct material issued to joinery department $3,000
12/8/2003 direct material issued to joinery department $2,000
13/8/2003 direct labour for joinery department $3,000
13/8/2003 direct material issued to upholstery department $800
13/8/2003 direct material issued to spraying department $500
14/8/2003 direct labour for spraying department $1,200
15/8/2003 direct labour for upholstery department $1,000
15/8/2003 allocation of indirect costs to joinery department $500
15/8/2003 allocation of indirect costs to upholstery department $200
15/8/2003 allocation of indirect costs to spraying department $500

The job was started on August 11, 2003 and completed on August
15, 2003.
450

Quality Furniture Production Limited


Job Cost Sheet
Job Order Number: Date Started:
Customer Name: Date Completed:
Telephone Number: (H)
(W)
(C)
Address:
E-mail Address: Quoted price:

Date Department A Department B Department C Total

Sub-Total

Sub-Total
Grand
Total
451

Quality Furniture Production Limited


Job Cost Sheet
Job Order Number: T1147 Date Started: 11/8/03
Customer Name: Michael Williams Date Completed: 15/8/03
Telephone Number: (H) (W) (C) 371-9602

Address: 10 Longwood Drive, Kingston


E-mail Address: Quoted price: $ 16,000
Date Joinery Upholstery Spraying Total
Department Department Department
Direct Material
11/8/03 $3,000 $3,000
12/8/03 $2,000 $2,000
13/8/03 $800 $800
13/8/03 $500 $500
Sub-Total $5,000 $800 $500 $6,300
Direct Labour
13/8/03 $3,000 $3,000
14/8/03 $1,200 $1,200
15/8/03 $1,000 $1,000
Sub-Total $3,000 $1,000 $1,200 $5,200
Overhead
15/8/03 $500 $500
15/8/03 $200 $200
15/8/03 $500 $500
Sub-Total $500 $200 $500 $1,200
Grand Total $8,500 $2,000 $2,200 $12,700
452

REVIEW QUESTIONS
(1) Distinguish between raw materials consumed and prime cost.

(2) Distinguish between manufacturing cost and transfer price.

(3)* Prime cost includes all _________________ costs.

(4) Explain the difference between a manufacturing cost center and


a manufacturing profit center.

EXERCISE 1 *

From the following information prepare a Manufacturing Account for


Turtle Ltd. for the year ended December 31, 2004.

$
Opening stock of raw materials 210,000
Closing stock of raw materials 160,000
Purchases of raw materials 620,000
Raw material returns 10,000
Manufacturing wages 590,000
Royalty 200,000
Electricity 470,000
Other factory utilities 240,000
Factory plant depreciation 40,000
Opening work in progress 65,000
Closing work in progress 45,000
453

EXERCISE 2 *

From the following information prepare the Manufacturing Account,


the Trading Account, and the Profit and Loss Account of Piglet
Enterprises for the year ended March 31, 2004.

$
Opening stock of raw materials 100,000
Closing stock of raw materials 80,000
Purchases of raw materials 440,000
Carriage inwards on raw materials 10,000
Direct manufacturing wages and salaries 800,000
Other direct costs 20,000
Electricity - factory 100,000
- office 20,000
Rent - factory 200,000
- office 40,000
Depreciation of office machines 10,000
Depreciation of office furniture 2,000
Depreciation of factory machines 50,000
Depreciation of factory furniture 1,000
Opening stock of finished goods 10,000
Closing stock of finished goods 30,000
Administrative expenses 280,000
Selling expenses 60,000
Sales 1,800,000
454

EXERCISE 3 *
$
Opening stock of raw materials 5,652
Closing stock of raw materials 12,152
Purchases of raw materials 460,000
Purchases of finished goods 50,000
Raw material returns 6,000
Direct wages 488,000
Opening stock - work in progress 37,000
Opening stock - finished goods 45,000
Closing stock - work in progress 39,000
Closing stock - finished goods 60,000
Administrative expenses 400,000
Sales commission expense 130,000
Rent 600,000
Electricity 120,000
Salaries 520,000
Insurance 25,500
Building depreciation 10,000
Plant depreciation 5,000
Motor vehicles depreciation 6,000
Carriage outwards 22,000
Bank charges 1,633
Discount received 8,000
Discount allowed 2,377
Sales 2,900,000
Repairs to factory plant 22,000
Furniture depreciation - factory 4,000
Furniture depreciation - office 7,000
Allowance for unrealized profit 7,500
Retained profit 100,000
Ordinary share capital 2,427,500
Debtors 260,000
Creditors 100,000
Prepayment 28,000
Accruals 130,162
Bank overdraft 240,000
Plant (net of depreciation) 800,000
Factory furniture (net of depreciation) 90,000
Office furniture (net of depreciation) 180,000
Motor vehicles (net of depreciation) 400,000
Building (net of depreciation) 1,200,000
455

ADDITIONAL INFORMATION:

(1) The goods manufactured are transferred from the manufacturing


department to the trading department at a profit margin of
one-sixth (1/6).

(2) The following apportionment should take place:

(i) Eighty per cent (80%) of the rent and ninety per cent
(90%) of the electricity should be charged to the factory
and the respective balance charged to the office.

(ii) $120,000 of the salaries relate to the factory.

(iii) Thirty per cent (30%) of insurance and building


depreciation should be charged to the office and the
balance charged to the factory.

(3) 50,000 units were produced during the financial year.

Prepare the following of Bear Limited for the year ended December
31, 2004:

(a) Manufacturing Statement


(b) Trading Statement
(c) Profit and Loss Statement
(d) Balance Sheet
(e) Manufacturing cost per unit
(f) Transfer price per unit
456

ANSWERS TO SELECTED REVIEW QUESTIONS


(3) Direct

EXERCISE 1

TURTLE LTD. MANUFACTURING ACCOUNT FOR THE YEAR ENDED


DECEMBER 31, 2004
-----------------------------------------------------------------
$ ║ $
Opening stock of raw ║ Trading A/C (manu-
materials 210,000 ║ facturing cost) 2,220,000
Purchases of raw materials 620,000 ║
830,000 ║
Less raw material returns 10,000 ║
820,000 ║
Less closing stock of ║
raw materials 160,000 ║
Cost of raw materials ║
consumed 660,000 ║
Manufacturing wages 590,000 ║
Royalty 200,000 ║
Prime cost 1,450,000 ║
Electricity 470,000 ║
Other factory utilities 240,000 ║
Factory plant dep. 40,000 ║
2,200,000 ║
Add opening stock WIP 65,000 ║
2,265,000 ║
Less closing stock WIP 45,000 ║ -
2,220,000 ║ 2,220,000
========= ║ =========

457

EXERCISE 2

PIGLET ENTERPRISES MANUFACTURING ACCOUNT FOR THE YEAR


MARCH 31, 2004
-----------------------------------------------------------------
$ ║ $
Opening stock of raw ║ Trading A/C (manu-
materials 100,000 ║ facturing cost) 1,641,000
Purchases of raw materials 440,000 ║
540,000 ║
Carriage inwards on raw ║
materials 10,000 ║
550,000 ║
Less closing stock of ║
raw materials 80,000 ║
Cost of raw materials ║
consumed 470,000 ║
Direct wages and salaries 800,000 ║
Other direct costs 20,000 ║
Prime cost 1,290,000 ║
Electricity 100,000 ║
Rent 200,000 ║
Depreciation - factory ║
machines 50,000 ║
Depreciation - factory ║
furniture 1,000 ║ -
1,641,000 ║ 1,641,000
========= ║ =========

458

PIGLET ENTERPRISES TRADING ACCOUNT FOR THE YEAR ENDED


MARCH 31, 2004
----------------------------------------------------------------
$ ║ $
Opening stock of finished ║ Sales 1,800,000
goods 10,000 ║
Manufacturing costs 1,641,000 ║
1,651,000 ║
Less closing stock of ║
finished goods 30,000 ║
Cost of sales 1,621,000 ║
Gross profit ║
(Profit & Loss A/C) 179,000 ║ -
1,800,000 ║ 1,800,000
========= ║ =========

PIGLET ENTERPRISES PROFIT AND LOSS ACCOUNT FOR THE YEAR ENDED
MARCH 31, 2004
-----------------------------------------------------------------
$ ║ $
Electricity 20,000 ║ Gross profit 179,000
Rent 40,000 ║ Loss 233,000
Depreciation - office ║
machines 10,000 ║
Depreciation - office ║
furniture 2,000 ║
Administrative expenses 280,000 ║
Selling expenses 60,000 ║
412,000 ║ 412,000
======= ║ =======

459

EXERCISE 3

BEAR LTD. MANUFACTURING STATEMENT FOR THE YEAR ENDED


DECEMBER 31, 2004
-----------------------------------------------------------------------
$
Opening stock of raw materials 5,652
Purchases of raw materials 460,000
465,652
Less raw material returns 6,000
459,652
Less closing stock of raw materials 12,152
Cost of raw materials consumed 447,500
Direct wages 488,000
Prime cost 935,500
Rent ($600,000 x 0.80) 480,000
Electricity ($120,000 x 0.90) 108,000
Salaries 120,000
Insurance ($25,500 x 0.70) 17,850
Building depreciation ($10,000 x 0.70) 7,000
Plant depreciation 5,000
Repairs to factory plant 22,000
Factory furniture depreciation 4,000
1,699,350
Opening stock of WIP 37,000
1,736,350
Less closing stock of WIP 39,000
Manufacturing cost 1,697,350
Manufacturing profit 339,470
Transfer price 2,036,820
=========
460

BEAR LTD. TRADING STATEMENT FOR THE YEAR ENDED DECEMBER 31, 1994
$
Opening stock of finished goods 45,000
Transfer price of goods manufactured 2,036,820
Purchases of finished goods 50,000
2,131,820
Less closing stock of finished goods 60,000
Cost of sales 2,071,820
Gross profit 828,180
Sales 2,900,000
=========

BEAR LTD. PROFIT AND LOSS STATEMENT FOR THE YEAR ENDED
DECEMBER 31, 2004
-----------------------------------------------------------------
$ $
Gross profit 828,180
Discount received 8,000
Manufacturing profit 339,470
1,175,650
Less operating costs
Administrative expenses 400,000
Sales commission 130,000
Rent ($600,000 x 0.20) 120,000
Electricity ($120,000 x 0.10) 12,000
Salaries ($520,000 - $120,000) 400,000
Insurance ($25,500 x 0.30) 7,650
Building depreciation ($10,000 x 0.30) 3,000
Motor vehicle depreciation 6,000
Carriage outwards 22,000
Bank charges 1,633
Discount allowed 2,377
Office furniture depreciation 7,000
Increase in allow. for unrealized profit 2,500
1,114,160
Profit 61,490
Retained profit brought forward 100,000
Retained profit carried forward 161,490
=======
461

BEAR LTD. BALANCE SHEET AS AT DECEMBER 31, 2004


$ $ $
NON-CURRENT ASSETS
Plant (net) 800,000
Factory furniture (net) 90,000
Office furniture (net) 180,000
Motor vehicles (net) 400,000
Building (net) 1,200,000
2,670,000
CURRENT ASSETS
Finished goods 60,000
Less allowance for
unrealized profit 10,000
50,000
Raw material stock 12,152
Work in progress stock 39,000
Debtors 260,000
Prepayment 28,000
389,152
LESS CURRENT LIABILITIES
Creditors 100,000
Accruals 130,162
Bank overdraft 240,000
470,162
(81,010)
2,588,990
=========

Financed By:
Ordinary share capital 2,427,500
Retained profit 161,490
2,588,990
=========
462

(e) Manufacturing Cost Per Unit = $1,697,350


50,000

= $33.947

 $33.95

(f) Transfer Price Per Unit = $2,036,820


50,000

= $40.7364

 $40.74

WORKINGS
Closing allowance for unrealized profit ($60,000 x 1/6) $10,000
Opening allowance for unrealized profit $ 7,500
Increase in allowance for unrealized profit $ 2,500
=======
463

Ch17
CHAPTER SEVENTEEN

FINANCIAL STATEMENT ANALYSIS

CHAPTER OBJECTIVES
After completing this chapter you should be able to:

 Distinguish between short-term solvency analysis and long-


term solvency analysis.

 Distinguish between vertical analysis and horizontal


analysis.

 Explain the meaning of trend analysis.

 Calculate short-term solvency ratios, long-term solvency


ratios, profitability ratios and share performance ratios.

 State clearly the different types of comparative analysis.


464

INTRODUCTION
Financial Accounting provides the users of the financial statements
with valuable information. The financial statements when analyzed
give an indication of the financial performance and financial
stability of the organization. Several different methods can be
utilized to analyze the financial statements. The most common method
is ratio analysis. A ratio is one figure expressed as a fraction or
percentage of another figure. A ratio is only useful when some form
of comparative analysis is done. For example, a return on investment
of 40% for 2004 might appear high, but if the return on investment
for 2003 was 80%, then the 2004 return on investment compared to
2003 is low. So some form of comparative analysis must be done if
the ratio is to be meaningful.

In analyzing financial statements you could analyze the audited or


un-audited financial statements. But it is better to use the audited
financial statements since these statements will indicate whether
the organization kept proper records and if the financial statements
reflect a true and fair view of the financial position of the
organization.

Several different methods can be utilized in analyzing financial


statements. The important methods are listed below:
Solvency Analysis
Short Term Solvency Analysis
Long Term Solvency Analysis
Profitability Analysis
Share Performance Indicators Analysis
Trend Analysis
Comparative Analysis

The analysis that is required will depend on the user of the


financial statement.
465

USER MAJOR ANALYSIS REQUIRED


Trade Union Profitability Analysis
Solvency Analysis
Shareholders Profitability Analysis
Solvency Analysis
Share Performance Indicators Analysis
Trend Analysis
Loan Creditors Profitability Analysis
Solvency Analysis
Trend Analysis
Comparative Analysis
Trade Creditors Short Term Solvency Analysis
Potential Investors Profitability Analysis
Solvency Analysis
Trend Analysis
Comparative Analysis
Management Solvency Analysis
Profitability Analysis
Share Performance Indicators Analysis
Trend Analysis
Comparative Analysis

TYPES OF FINANCIAL STATEMENT ANALYSIS

SOLVENCY ANALYSIS
Solvency analysis will give an indication of the ability of the
organization to "stay in business" in the short run and long run.
Solvency analysis can be subdivided into short-term solvency
analysis and long-term solvency analysis.
466

SHORT TERM SOLVENCY (LIQUIDITY) ANALYSIS

Short-term solvency analysis is usually referred to as liquidity


analysis. Short-term solvency analysis gives an indication of the
ability of the organization to "stay in business" in the short run.
The three (3) main short-term solvency ratios are the current ratio,
the acid test ratio or quick ratio, and working capital.

CURRENT RATIO

The formula for the calculation of the current ratio is as follows:

Current Ratio = Current Assets


Current Liabilities

This ratio gives an indication of the organization's ability to pay


debts due within a twelve months period (current liabilities). A
current ratio of 2 to 1 is widely accepted as the current ratio
benchmark (criterion or reference point). A current ratio of 2 to 1
means that current assets are two times greater than current
liabilities. This excess of current assets over current liabilities
indicates that the current assets are sufficient to pay the current
liabilities. The greater the excess of current assets over current
liabilities (high current ratio) suggests that the organization's
liquidity position is good. But this high current ratio might be
misleading if the majority of current assets are tied up in un-
collectible receivables or obsolete inventory.

QUICK RATIO OR ACID TEST RATIO

The formula for calculating the quick or acid test ratio is as


follows:

Quick or Acid Test Ratio = Current Assets – Non-liquid Assets


Current Liabilities
467

This ratio gives an indication of the organization's instant debt


paying ability for debts due within twelve months. A quick ratio of
1 to 1 is widely accepted as the quick ratio benchmark. The quick
ratio is a better indication of liquidity than the current ratio
since it excludes non-liquid assets such as, inventory and
prepayment. A quick ratio of 0.6 to 1 indicates that if all the
short-term creditors requested payment simultaneously, then the
organization would not be able to pay. So a quick ratio of less than
1 to 1 indicates that if all short-term creditors requested payment
simultaneously, then the organization would not be able to pay. A
quick ratio that is greater than 1 to 1 (for example, 1.1 to 1)
indicates that if all the short-term creditors requested payment
simultaneously, then the organization would be able to pay.

Working Capital

The formula for the calculation of working capital is as Follows:

Working Capital = Current Assets – Currently Liabilities

Working capital is the difference between current assets and


current liabilities. If current assets are greater than current
liabilities, then working capital is said to be positive. A
positive working capital is used to finance the daily operations of
the business. If current assets are less than current liabilities,
then working capital is said to be negative. A negative working
capital means that the business does not have enough funds to
finance its daily operations and will have difficulties in paying
its short-term debts (current liabilities).

Example 1
Current assets $250,000
Current liabilities $200,000

Working capital = current assets – current liabilities

Working capital = $250,000 - $200,000 = $50,000 positive


468

Example 2
Current assets $120,000
Current liabilities $150,000

Working capital = $120,000 - $150,000 = ($30,000) negative

The working capital of a business is constantly changing form over


time. For example, working capital in the form of cash $10,000 is
used to purchase goods. The working capital is now in the form of
goods (inventory). This inventory is sold on credit to a customer
for $12,000. Working capital is now in the form of accounts
receivable (trade debtor). When the trade debtor pays the $12,000
the working capital will change from accounts receivable to cash.
The difference of $2,000 in this case represents gross profit
(sales – cost of sales), that is, $12,000 - $10,000. The $2,000
profit represents an increase in working capital to the business.
When the goods were purchased for $10,000 cash, the form of working
capital changed but the amount of working capital $10,000 remained
the same. When the goods were sold on credit to a customer for
$12,000. The form of working capital changed and the amount of
working capital of the business increased by $2,000.

If $1,000 cash is used by the business to pay wages to an employee,


and the employee deposits this $1,000 in her bank account, in this
case the working capital is not changing form within the business,
but leaving the business. The working capital of the business will
decrease by $1,000. Over time the working capital of a business can
change form, it can increase, it can decrease, and it can also
remain the same.

Significance of working capital

Working capital is the ‘life blood’ of a business. Working capital


is used to finance the current operations of a business, that is,
to pay wages, purchase goods for resale, to pay trade creditors, to
buy office supplies and so on. It is important that the business
has an adequate amount of working capital.

Without an adequate amount of working capital the business will not


be able to pay trade creditors on time, the trade creditors may
decide not to extend any further credit, certain stock items may be
depleted and customers will go to competitors for these items.
469

Without an adequate amount of working capital the business will not


be able to pay expense creditors on time. The business will have a
bad reputation for paying bills late. This bad reputation will have
a negative impact on the level and amount of credit that is
extended to the business. An adequate amount of positive working
capital is vital to the survival of any business. An adequate amount
of working capital (positive working capital) does not necessarily
mean that the business has enough cash to pay its debts, since
current assets include non-cash items such as stock and debtors.

If the level of working capital is more than adequate, then the


cost of working capital will be too high. The more than adequate
working capital may be due to the level of inventory being too high
resulting in higher warehouse costs, and some of these overstocked
inventory items may become obsolete. The more than adequate amount
of working capital may be due to a high level of trade debtors,
some of which might become bad debts.

Creditors prefer a large amount of working capital for a business,


because this means that they have a greater chance of being paid.
Business owners prefer a small amount of working capital because of
the costs associated with working capital. A balance must be struck
to ensure the smooth operation of the business. The level of
working capital that is needed to ensure the smooth operation of
the business depends on the nature of the business, the expected
growth of the business, the seasonality of the product or service,
the competitive conditions within the industry, the production
cycle time period, and the size of the business.

A business involved in the construction of houses for sale will


need more working capital than a business that acts as an agent for
renting houses. Two businesses are in the same industry and are the
same size, and one is expecting a 10% growth and the other a 30%
growth for the financial year. The business expecting the 30%
growth will need more working capital than the one expecting a 10%
growth. A hotel operating in Negril, Jamaica will need more working
capital during the winter season. The winter season is usually the
busiest period for the hotel, with sustained high occupancy level
during this period.
470

Both company X and company Y are the same size with the same volume
of credit sales, and the same credit period of 15 days. Company Y
is in a different industry where the competition is fierce and the
average credit period has moved from 15 days to 30 days. If company
Y keeps it credit period of 15 days its credit customers will go to
its competitors who are offering a longer credit period. Because of
the fierce competition within this industry company Y will have no
choice but to increase its credit period to bring it in line with
the industry average or its credit sales will decline
significantly. Let us assume that company Y increases it credit
period from 15 days to 30 days and the volume of credit sales
remain the same. The longer credit period offered by company Y due
to the competitive conditions within the industry would require it
to obtain additional working capital to finance its accounts
receivable.

Both company AA and company BB are the same size producing


dissimilar products with the same cost. Company AA production cycle
time period for 1000 units is 4 days and company BB production
cycle time period for 1000 units is 7 days. Company BB will need
more working capital than company AA to finance its work in
progress.

Both company XX and company YY are in the same industry with the
same credit period. Company XX annual credit sales is $30 million
and its annual cash sales is $10 million. Company YY annual credit
sales is $10 million and its annual cash sales is $3 million.
Company XX with total sales of $40 million is larger than company
YY with total sales of $13 million. Company XX will have a larger
inventory level and a larger amount of accounts receivable than
company YY and will therefore require a larger amount of working
capital than company YY.

LONG TERM SOLVENCY ANALYSIS

Long-term solvency analysis gives an indication of the ability of


the organization to "stay in business" in the long run. The two (2)
main long-term solvency ratios are the gearing ratio and times
interest earned.
471

GEARING RATIO

The formula for the calculation of the gearing ratio for a company
is as follows:

Gearing Ratio = Long Term Loan Capital


Issued Share Capital + Reserves

The formula for the calculation of the gearing ratio for a sole
proprietorship is as follows:

Gearing Ratio = Long Term Loan Capital


Capital

The formula for the calculation of the gearing ratio for a


partnership is as follows:

Gearing Ratio = Long Term Loan Capital


Partners' Capital + Partners' Current Accounts

The gearing ratio gives an indication of the organization's


financial risk resulting from financing the business with long-term
loan(s). The gearing can be high geared or low geared. No benchmark
exists for this ratio. A gearing ratio of 200% indicates that long-
term loan creditors provided twice the amount of financing than the
owner(s) of the business. When the organization is high geared this
is an indication that long-term loan creditors have financed a
larger percentage of the business than the owner(s) of the business.
When the organization is low geared this is an indication that the
owner(s) of the business have financed a larger percentage of the
business than the long-term loan creditors. The higher the gearing
the greater the financial risk to the business, since the non-
payment of loan interest or loan principal could lead to bankruptcy.
472

TIMES INTEREST EARNED

The formula for the calculation of the times interest earned for a
company is as follows:

Times Interest Earned = Profit Before Interest Expense and Taxes


Interest Expense

Taxes relate to corporation taxes only.

The formula for the calculation of the times interest earned for a
sole proprietorship or partnership is as follows:

Times Interest Earned = Profit Before Interest Expense


Interest Expense

The interest expense relates to long-term loans and not to short-


term loans.

The times interest earned gives an indication of the organization's


financial risk resulting from financing the business from long term
loans. The higher the times interest earned the less likely it is
that the organization will fail as a result of declining profits.
Businesses with consistent and stable annual profits can afford to
obtain large sums of long term loans, when compared to seasonal
businesses (businesses in the tourist industry and businesses in the
fashion industry) with profits that may fluctuate widely from year
to year depending on how "good business was that season". A times
interest earned of 4 times indicates that the profits before
interest expense, and taxes in the case of a company, can decline by
4 times without the firm suffering any financial embarrassment from
its inability to pay interest expense.
473

PROFITABILITY ANALYSIS
Profitability analysis will give an indication of the excess of
income over expenses (profit) in relation to sales or investment.
Profitability analysis can also give an indication of the excess of
expenses over income (loss) in relation to sales or investment.
Profitability ratios are usually expressed in percentage terms. The
two most popular profitability ratios are the gross profit margin
and the net profit margin.

GROSS PROFIT MARGIN

The formula for the calculation of the gross profit margin is as


follows:

Gross Profit Margin = Gross Profit x 100


Net Sales 1

If the firm makes a gross loss then the gross profit margin will be
negative. The calculation will be as follows:

Gross Profit Margin = Gross Loss x 100


Net Sales 1

A positive gross profit margin of 20% means that for every $1 of


sales, the firm earns a gross profit of $0.20. A negative gross
profit margin of 6% means that for every $1 of sales, the firm makes
a gross loss of $0.06.
474

NET PROFIT MARGIN

The formula for the calculation of the net profit margin (profit
margin) for a sole proprietorship and a partnership is as follows:

Net Profit Margin = Net Profit x 100


Net Sales 1

The formula for the calculation of the net profit margin (profit
margin) for a company is as follows:

Net Profit Margin = Net Profit After Taxes x 100


Net Sales 1

A net profit margin of 15% for a sole trader or a partnership means


that for every $1 of sales the firm earns a net profit of $0.15.

A net profit margin of 15% for a company means that for every $1 of
sales the company earns an after tax profit of $0.15.
Just like in the calculation of the gross profit margin it is
possible to calculate a negative net profit margin. The calculation
for a negative net profit margin for a sole proprietorship,
partnership and company is as follows:

Net Profit Margin = Net Loss x 100


Net Sales 1

Net Sales = (Cash Sales + Credit Sales) - Sales Returns


475

SHARE PERFORMANCE ANALYSIS


Share performance indicators analysis gives an indication of the
profitability of the investment made by ordinary shareholders. The
two main share performance indicators are the earnings per share
(EPS) and the price earnings ratio.

EARNINGS PER SHARE

The formula for the calculation of the earnings per share is as


follows:

EPS = After Tax Profits Available to Ordinary Shareholders


Weighted average Number of Ordinary Shares Issued

The EPS gives an indication of the after tax profit earned by each
ordinary share. An EPS of $0.65 means that each ordinary share
earned an after tax profit of $0.65.

PRICE EARNINGS RATIO

The formula for the calculation of the price earnings ratio is as


follows:

Price Earnings Ratio = Market Price Per Share


Earnings Per Share

The price earnings ratio gives an indication of the future earnings


prospects of the ordinary share and the degree of investor
confidence in the ordinary share. The higher the price earnings
ratio the greater the degree of investor confidence in the ordinary
share. If Jack Ltd. price earnings ratio is 4 times and Jill Ltd.
price earnings ratio is 6 times, this means that investors have more
confidence in the future earnings prospect of Jill Ltd., the company
with the higher price earnings ratio.
476

HORIZONTAL ANALYSIS
Horizontal analysis occurs where each item in the Income Statement
(Trading and Profit and Loss Account) or Balance Sheet is compared
with the corresponding item for the previous period to determine the
percentage increase or decrease and the dollar value increase or
decrease. So, for example, the sales for February 2004 is compared
with the sales for January 2004, or the accounts receivable for the
financial year ended December 31, 2004 is compared with the accounts
receivable for the financial year ended December 31, 2003. An
example of horizontal analysis is shown below for two separate
items.

January 2004 February 2004 Increase/(Decrease)


$ $ % $
Sales 100,000 80,000 (20) (20,000)
Travelling 1,000 2,000 100 1,000

VERTICAL ANALYSIS
Vertical analysis occurs where each item in the Income Statement is
expressed as a percentage of net sales, and each item in the Balance
Sheet is expressed as a percentage of total assets. An example of
vertical analysis is shown below for a Trading Account.

$ %
Opening stock 20,000 10
Net purchases 160,000 80
180,000 90
Less closing stock 30,000 15
Cost of sales 150,000 75
Gross profit 50,000 25
Net sales 200,000 100
======= ===
477

TREND ANALYSIS
Trend analysis is done to determine whether a particular item is
improving or deteriorating over a particular period. The trend is
usually plotted on a graph. For example, we could examine the trend
of the gross profit margin for the five months period, January 2004
to May 2004.

COMPARATIVE ANALYSIS
INTRA-ORGANIZATION COMPARISON

Intra-organization comparison occurs where the financial statements


for one period are compared with the financial statements of another
period for the same firm.

INTER-ORGANIZATION COMPARISON

Inter-organization comparison occurs where the financial statements


of one firm for a particular period are compared with the financial
statements of another firm for the same period. Usually both firms
are in the same industry.

INDUSTRY COMPARISON

Industry comparison occurs where the financial statements of a firm


for a particular period are compared with the industry average for
that same period. The industry average of course relates to the
industry that the firm is in.
478

LIQUIDITY RATIOS

RATIO FORMULA FOR CALCULATION

Current Ratio Current Assets


Current Liabilities

Quick Ratio Current Assets – Non-Liquid Assets


Current Liabilities

Average Collection Period Trade Debtors x Days in the Year


Annual Net Credit Sales

Inventory Turnover Cost of Goods Sold


Average Inventory

Average Age of Trade Trade Creditors x Days in the Year


Creditors Annual Net Credit Purchases
479

ASSET MANAGEMENT RATIOS

RATIO FORMULA FOR CALCULATION

Tangible Non-current Asset Net Sales


Turnover Tangible Non-Current Assets

Total Asset Turnover Net Sales


Total Assets

Trade Debtors Turnover Annual Net Credit Sales


Average Trade Debtors

Inventory Turnover Cost of Sales


Average Inventory

Average Collection Period Trade Debtors x Days in the Year


Annual Net Credit Sales
480

DEBT MANAGEMENT RATIOS

RATIO FORMULA FOR CALCULATION

Total Debt to Total Total Debt


Assets Total Assets

Sole Proprietor- Long Term Loan Capital


ship Gearing Capital

Partnership Gearing Long Term Loan Capital


Partners' Capital + Partners' Current A/Cs

Company Gearing Long Term Loan Capital


Share Capital + Reserves

Company Times Profit Before Interest Expense and Taxes


Interest Earned Interest Expense

Sole Proprietor-
ship & Partnership Profit Before Interest Expense
Times Interest Interest Expense
Earned

Average Age of Trade Creditors x Days in the Year


Trade Creditors Annual Net Credit Purchases
481

PROFITABILITY RATIOS

RATIO FORMULA FOR CALCULATION

Gross Profit Margin Gross Profit x 100


Net Sales 1

Company Net Profit Net Profit After Taxes x 100


Margin Net Sales 1

Sole Proprietorship Net Profit x 100


and Partnership Net Net Sales 1
Profit Margin

Company Return on Net Profit After Taxes x 100


Total Assets Total Assets 1

Sole Proprietorship
and Partnership Return Net Profit x 100
on Total Assets Total Assets 1

Sole Proprietorship Net Profit x 100


Return on capital Closing Capital 1

Partnership Return on Net Profit x 100


capital Capital Accounts + Current Accounts 1

Return on Common Ordinary Shareholders' Profit x 100


Equity Capital Ordinary Share Capital + Reserves 1
482

SHARE PERFORMANCE RATIOS

RATIO FORMULA FOR CALCULATION

Return on Common Ordinary Shareholders' Profit x 100


Equity Capital Ordinary Share Capital + Reserves 1

Earnings Per Share Profit Available to Ordinary Shareholders


Weighted Average Number of Ordinary Shares Issued

Dividend Per Share Ordinary Dividends


Number of Ordinary Shares Issued

Dividend Payout Total Dividends x 100


Net Profit After Taxes 1

Price Earnings Ratio Market Price Per Share


Earnings Per Share

Dividend Yield Dividend Per Share x 100


Market Price Per Share 1

Book Value Per Share Ordinary Share Capital + Reserves


Number of Ordinary Shares Issued

NOTE: The profit figure in the earnings per share (EPS) calculation
refers to the after tax profit, net of profits attributable to
preference shareholders.
483

It is important to note that several ratios fall under more than one
category, such as the average collection period. This ratio is a
liquidity ratio as well as an asset management ratio.

We will now look briefly at those ratios in the various tables that
have not yet been explained.

AVERAGE COLLECTION PERIOD

This ratio indicates the average number of days it takes the


business to collect the amount outstanding from trade debtors.

INVENTORY TURNOVER

Turnover ratios are usually expressed in "times". For example, an


inventory turnover of 7 would be expressed as 7 times. This ratio
gives an indication of the efficiency of management in managing its
inventory.

The Average Inventory = Opening Stock + Closing Stock


2

AVERAGE AGE OF TRADE CREDITORS

This ratio indicates the average number of days it takes the


business to pay trade creditors.

TANGIBLE NON-CURRENT ASSET TURNOVER

This ratio gives an indication of the effectiveness of management in


the use of the business tangible non-current assets.
484

TOTAL ASSET TURNOVER

This ratio gives an indication of the effectiveness of management in


the use of the business tangible non-current assets and current
assets.

TRADE DEBTORS TURNOVER

This ratio gives an indication of the efficiency of management in


collecting from trade debtors and in the management of its credit
policy.

TOTAL DEBT TO TOTAL ASSETS

This ratio indicates the percentage of total funds provided by long-


term loan creditors and short-term creditors (current liabilities).

RETURN ON TOTAL ASSETS

This ratio gives an indication of the profit earned for every $1 of


investment in assets.

RETURN ON CAPITAL

This ratio gives an indication of the profit earned for every $1 of


investment in capital.

RETURN ON COMMON EQUITY CAPITAL

This ratio gives an indication of the profit earned for every $1 of


investment in equity capital. Equity capital equals ordinary share
capital plus (+) capital reserves and revenue reserves. Ordinary
shareholders' profit equals net profit after taxes minus (-)
preferred dividends.
485

DIVIDEND PER SHARE


This ratio indicates the dollar value of dividends earned by each
share.

DIVIDEND PAYOUT

This ratio indicates the percentage of after tax profits that are
distributed to shareholders.

DIVIDEND YIELD

This ratio indicates the percentage of dividends received by


shareholders in relation to the market price per share.

BOOK VALUE PER SHARE

This ratio indicates the dollar value of each ordinary share based
on the share capital and the reserves. Please note that the book
value per share is not the same as the market value per share. The
market value is the price that the share can be sold for by the
shareholder. The market value per share can be higher than or lower
than the book value per share.

CASH FLOW STATEMENT RATIOS


Cash flow statement analysis focuses on the cash generating
efficiency of the business. The four key ratios are the cash flow
yield, cash flow to sales, cash flow to assets, and free cash flow.
486
Time Limited
Cash Flow Statement for the year ended December 31, 2003
$’000
Cash Flows from Operating Activities
Profit before taxation 922
Adjustment for non-cash items
Depreciation 51
Profit on disposal of furniture ( 5)
(Increase)/Decrease in operating assets
Stock (108)
Insurance prepaid ( 2)
Trade debtors ( 17)
Increase/(Decrease) in operating liabilities
Trade creditors ( 13)
Accruals ( 8)
Cash generated from operations 820
Corporation tax paid (410)
Net cash provided by operating activities 410

Cash Flows from Investing Activities


Proceeds from disposal of furniture 16
Acquisition of furniture ( 40)
Net cash used by investing activities ( 24)

Cash Flows from Financing Activities


Dividends paid (300)
Proceeds from issue of shares 200
Net cash used by financing activities (100)

Net increase in cash and cash equivalents 286


Opening cash and cash equivalents 754
Closing cash and cash equivalents 1,040
=====

Additional information ($’000):


a. Net sales $10,195 b. Total assets $2,196

Calculate the following ratios:


1. Cash flow yield
2. Cash flow to sales
3. Cash flow to assets
4. Free cash flow
487

CASH FLOW YIELD

Cash Flow Yield = Net Cash Flows From Operating Activities


Profit After Tax

Cash Flow Yield = $410


$678

= 0.60

A cash flow yield of 0.60 times indicates that operating activities


are generating 60% less cash flow than net profit after tax. A cash
flow yield of 1.42 times for example indicates that operating
activities are generating 42% more cash flow than net profit after
tax.

CASH FLOW TO SALES

Cash Flow to Sales = Net Cash Flows From Operating Activities


Net Sales

Cash Flow to Sales = $410 X 100


$10,195 1

= 4.02%

A cash flow to sales ratio of 4.02% indicates that the business


generated a small positive net cash from sales.

CASH FLOW TO ASSETS

Cash Flow to Assets = Net Cash Flows From Operating Activities


Total Assets

Cash Flow to Assets = $410 X 100


$2,196 1

= 18.67%
488

A cash flow to assets ratio of 18.67% indicates that the cash flow
to assets is greater than the cash flow to sales of 4.02% because
the business has a good asset turnover ratio. If the cash flow to
assets ratio were lower than the cash flow to sales ratio of 4.02%,
this would indicate that the business has a poor asset turnover
ratio.

FREE CASH FLOW

Free cash flow equals net cash flow from operating activities minus
dividends paid plus proceeds from sale of property, plant and
equipment minus purchases of property, plant and equipment. A
positive free cash flow indicates that the business is able to meet
all of its cash commitments and has cash available to expand current
operations and, or reduce loan capital. A negative free cash flow
indicates that the business will have to borrow funds and, or issue
share capital and, or sell investments in order to maintain it
planned operating activity level.

Free cash flow ($’000) = $410 - $300 + $16 - $40


= $86

The free cash flow is a positive $86,000.


489

REVIEW QUESTIONS
(1) Distinguish between short-term solvency analysis and long term
solvency analysis.

(2) Distinguish between vertical analysis and horizontal analysis.

(3) What is meant by intra-organization comparison?

(4) State four liquidity ratios and the formula for calculating
these ratios.

(5) What is the meaning of a gross profit margin of 25%?

EXERCISE 1 *

From the information provided calculate the following ratios:

(a) Gross Profit Margin

(b) Net Profit Margin

(c) Current Ratio

(d) Quick Ratio

(e) Inventory Turnover

$
Net profit 10,000
Sales 160,000
Opening stock 14,000
Closing stock 10,000
Cost of sales 120,000
Current assets 50,000
Current liabilities 10,000
490

COLLEGE STUDENTS

EXERCISE 2 *

J. Golddigger Balance Sheet for 2004


February 29 January 1
$ $
Non-current Assets
Furniture and fixtures 22,000 20,000
Less allowance for depreciation 315 150
21,685 19,850

Current Assets
Closing stock 12,200 23,000
Accounts receivable 18,360 31,700
Investments in stocks 5,000 0
Deposit on call 10,000 0
Bank 2,200 26,500
Cash 2,030 9,640
49,790 90,840

Total assets 71,475 110,690


====== =======

Current Liabilities
Accounts payable 16,400 38,000
Accruals 3,100 3,000
Other creditor 0 20,000
19,500 61,000

Owner’s Equity
Capital 49,690 50,000
Add profit 3,285 3,690
52,975 53,690
Less drawings 1,000 4,000
Closing capital 51,975 49,690

Total liabilities 71,475 110,690


====== =======
491

J. Golddigger Income Statement for February 2004


$
Sales (net) 51,370

Less Cost of Sales


Opening stock 23,000
Purchases (net) 15,500
38,500
Less closing stock 12,200
26,300

Gross profit 25,070


Discount received 200
25,270

Less Operating Expenses


Wages and salaries 8,000
Rent expense 10,000
Travelling 100
Electricity 3,200
Discount allowed 400
Postage 20
Depreciation 165
Office supplies 100
21,985

Profit 3,285
======

Calculate the following ratios for February 2004:


a. Quick ratio
b. Gross profit margin
c. Net profit margin
d. Inventory turnover
e. Tangible non-current asset turnover
f. Total asset turnover
g. Return on total assets
h. Return on capital
492

EXERCISE 3 *

Hand and Foot


Income Statement for the year ended December 31, 2004
------------------------------------------------------------
$
Sales 647,000
Less Cost of Sales 280,000
Gross profit 367,000

Less Operating Expenses


Rent 72,000
Utilities 24,000
Office Supplies 2,000
Salaries 28,000
Depreciation 39,000
Interest on loan 8,500
173,500

Profit 193,500
=======

Hand and Foot Statement of Changes in Partners’ Equity


For the year ended December 31, 2004
------------------------------------------------------------------
Hand Foot Hand Foot Total
Capital Capital Current Current
Account Account Account Account
$ $ $ $ $
Balance 1/01/04 300,000 300,000 6,400 7,600 614,000
Interest on capital 0 0 30,000 30,000 60,000
Salary 0 0 30,000 36,000 66,000
Share of profits 0 0 33,750 33,750 67,500
Balance 31/12/04 300,000 300,000 100,150 107,350 807,500
======= ======= ======= ======= =====
493

Hand and Foot


Balance Sheet as at December 31, 2004
-----------------------------------------------------------
$
Non-current Assets
Furniture 780,000
Less Allowance for Depreciation 145,000
635,000

Current Assets
Stock 35,000
Trade Debtors 115,000
Bank 140,000
290,000

Less Current Liabilities


Current Portion of Loan 20,000
Trade creditors 24,000
Interest accrued 8,500
52,500

Net Current Assets 237,500


Total assets less current liabilities 872,500
Less Non-current Liabilities
Loan 65,000
807,500
=======

Financed By
Capital 600,000
Current Accounts 207,500
807,500
=======

Calculate the following ratios for 2004:


a. Partnership gearing
b. Partnership times interest earned
c. Partnership net profit margin
d. Partnership return on capital
e. Partnership gross profit margin
494

EXERCISE 4 *

Greek Limited
Income Statement for the year ended December 31, 2003
------------------------------------------------------------
$
Sales 1,305,000
Cost of Sales ( 750,000)
Gross Profit 555,000
Other Operating Income 20,000
Distribution Costs ( 145,800)
Administrative Costs ( 274,200)
Other Operating Costs ( 60,000)
Operating Profit 95,000
Finance Costs ( 20,000)
Profit Before Taxation 75,000
Taxation ( 12,800)
Profit After Taxation 62,200
======

Greek Limited Statement of Changes in Equity


For the year ended December 31, 2003
------------------------------------------------------------------
Ordinary Share General Retained Total
Share Premium Reserves Profits
Capital
$ $ $ $ $
Balance 1/01/03 200,000 37,500 20,000 85,000 342,500
After Tax Profits 0 0 0 62,200 62,200
Dividends 0 0 0 (30,000) (30,000)
Transfer 0 0 10,000 (10,000) 0
Balance 31/12/03 200,000 37,500 30,000 107,200 374,700
======= ======= ======= ======= =====

Number of ordinary shares issued on January 1, 2003 = 200,000


Number of ordinary shares issued on December 31, 2003 = 200,000
Market price per share on December 31, 2003 = $5.80
495

Greek Limited
Balance Sheet as at December 31
2003 2002
$ $
Non-current Assets
Furniture 512,500 500,000
Less Allowance for Depreciation 50,000 25,000
462,500 475,000

Current Assets
Stock 30,000 47,000
Trade Debtors 60,000 40,000
Bank 60,000 73,000
150,000 160,000

Less Current Liabilities


Trade Creditors 20,000 12,400
Rent Accrued 5,000 10,000
Corporation Tax Payable 12,800 20,100
Current Portion of Long-term Loan 50,000 50,000
87,800 92,500

Net Current Assets 62,200 67,500


Total Assets Less Current Liabilities 524,700 542,500

Less Non-current Liabilities


Long-term Loan 150,000 200,000
374,700 342,500
======= =======

Financed by
Ordinary Share Capital 200,000 200,000
Share Premium 37,500 37,500
General Reserves 30,000 20,000
Retained Profits 107,200 85,000
Common equity 374,700 342,500
======== ========
496
Greek Limited
Cash Flow Statement for the year ended December 31, 2003
$
Cash Flows from Operating Activities
Profit before taxation 75,000
Add interest expense 20,000
Operating profit 95,000
Adjustment for non-cash items
Depreciation 25,000
(Increase)/Decrease in operating assets
Stock 17,000
Trade debtors (20,000)
Increase/(Decrease) in operating liabilities
Trade creditors 7,600
Rent accrued ( 5,000)
Cash generated from operations 119,600
Corporation tax paid (20,100)
Net cash provided by operating activities 99,500

Cash Flows from Investing Activities


Acquisition of furniture (12,500)
Net cash used by investing activities (12,500)

Cash Flows from Financing Activities


Interest paid (finance costs) (20,000)
Dividends paid (30,000)
Loan payments (50,000)
Net cash used by financing activities (100,000)

Net decrease in cash and cash equivalents (13,000)


Opening cash and cash equivalents 73,000
Closing cash and cash equivalents 60,000
======
Calculate the following ratios for 2003:
a. Current ratio b. Acid test ratio
c. Inventory turnover d. Gearing
e. Times interest earned f. Total debt to total assets
g. Gross profit margin h. Net profit margin
i. Return on total assets j. Return on common equity
k. Dividend per share l. Dividend payout
m. Dividend yield n. Earnings per share
o. Book value per share p. Price earnings ratio
q. Total asset turnover r. Cash flow yield
s. Cash flow to sales t. Free cash flow
497

ANSWERS TO SELECTED REVIEW QUESTIONS

EXERCISE 1

WORKINGS
Gross Profit = Sales - Cost of Sales
= $160,000 - $120,000
= $40,000

Average Inventory = Opening Stock + Closing Stock


2

= $14,000 + $10,000
2

= $12,000

(a) Gross Profit Margin = $ 40,000 x 100


$160,000 1

= 25%

(b) Net Profit Margin = $ 10,000 x 100


$160,000 1

= 6.25%

(c) Current Ratio = $50,000


$10,000

= 5 to 1
498

(d) Quick Ratio = $50,000 - $10,000


$10,000

= 4 to 1

(e) Inventory Turnover = $120,000


$ 12,000

= 10 times

EXERCISE 2

Quick Ratio = $49,790 - $12,200


$19,500

= 1.9276 to 1

Gross profit margin = 48.8028%

Net profit margin = 6.3947%

Average Inventory = Opening Stock + Closing Stock


2

= $23,000 + $12,200
2
= $17,600

Inventory turnover = 1.4943 times

Tangible non-current asset turnover = 2.3689 times

Total asset turnover = 0.7187 times

Return on total assets = 4.5960%

Return on capital = 6.3203%


499

EXERCISE 3

Partnership gearing = 8.0495%

Partnership times interest earned = 23.7647 times

Partnership net profit margin = 29.9072%

Partnership return on capital = 23.9628%

Partnership gross profit margin = 56.7233%

EXERCISE 4

Interest = Financed Cost

Weighted average ordinary shares issued = $200,000 + $200,000


2

= $200,000

Current ratio = 1.7084 to 1

Acid test ratio = 1.3667 to 1

Inventory turnover = 19.4805 times

Total debt to total assets = 38.82448%

Gearing = 40.032%

Times interest earned = 4.11 times

Gross profit margin = 42.5287%

Net profit margin = 4.76628%

Return on total assets = 10.1551%


500

Return on common equity = 16.5999%

Dividend per share = $0.15

Dividend payout = 48.2315%

Dividend yield = 2.5862%

Earnings per share = $0.311

Book value per share = $1.8735

Price earnings per share = $18.6495 times

Total asset turnover = 2.1306 times

Cash flow yield = 1.2781 times

Cash flow to sales = 6.0919%

Free cash flow = $79,500 - $30,000 - $12,500


= $37,000
501

Ch18
CHAPTER EIGHTEEN

STOCKTAKING AND PAYROLL ACCOUNTING

CHAPTER OBJECTIVES
After completing this chapter you should be able to:

 Identify the different types of closing inventory.

 Explain the procedure for taking stock.

 Prepare a Stock Reconciliation.

 Prepare payroll accounting entries.

 Prepare a time sheet.

 Prepare a payroll summary.


502

INTRODUCTION
The financial statements should reflect a true and fair view of the
profitability and the financial position of the business. A material
overstatement or a material understatement in the closing inventory
value will result in the production of misleading financial
statements. A material overstatement of the closing inventory value
will result in a material overstatement of the net profit or an
understatement of a net loss in the Profit and Loss Statement, and a
material overstatement of current assets in the Balance Sheet. A
material understatement of the closing inventory value will result
in a material understatement of the net profit or an overstatement
of a net loss in the Profit and Loss Statement, and a material
understatement of current assets in the Balance Sheet.

The closing inventory for a financial year will become the opening
inventory in the next financial year. Therefore if the closing
inventory is overstated, then the opening inventory will also be
overstated. An overstated opening inventory will result in the net
profit being understated. So it is extremely important that the
stocktaking is accurate. An overstated ending inventory value will
lead to an overstated net profit for that financial year and an
understated net profit in the next financial year. So an error in
the closing inventory calculation can affect the business in
producing financial statements that reflect a true and fair view in
two financial years.

If the closing inventory is understated, then the opening inventory


will also be understated. An understated opening inventory will
result in the net profit being overstated.

SUMMARY

INVENTORY ERROR IN INVENTORY EFFECT ON NET PROFIT


VALUE
Closing Inventory Overstated Overstated
Closing Inventory Understated Understated
Opening Inventory Overstated Understated
Opening Inventory Understated Overstated
503

TYPES OF CLOSING INVENTORY


RAW MATERIAL INVENTORY

Raw material not yet used in the production process by a


manufacturing firm at the end of the accounting period, is referred
to as raw material inventory.

WORK IN PROGRESS INVENTORY

Work in progress inventory represents partly manufactured goods at


the end of the accounting period in a manufacturing firm. For partly
manufactured goods the degree of completion is taken into
consideration when estimating its closing inventory value.

FINISHED GOODS INVENTORY

Finished good not sold at the end of the accounting period by a


manufacturing firm or a trading firm is referred to as finished
goods inventory.

STATIONERY INVENTORY AND OFFICE SUPPLIES INVENTORY

Stationery and, or office supplies (for example, sugar and coffee)


represent goods purchased for use in the business. These goods are
used in all types of businesses, whether manufacturing, trading or
service. A material monetary amount of stationery or a material
monetary amount of office supplies not used by the business at the
end of the accounting period would be reflected as an inventory item
in the financial statements. If the stationery and, or office
supplies not used at the end of the accounting period does not
represent a material monetary amount, then this unused stationery
and, or office supplies would not be reflected as an inventory item
in the financial statements, but charged as an expense in the Profit
and Loss Account.
504

GOODS SENT ON A SALE OR RETURN BASIS

Goods or raw material sent to another business on a sale or return


basis which remains unsold by that business at the end of the
accounting period represents closing inventory for the firm that
sent the goods.

GOODS IN TRANSIT

Raw materials, finished goods, a material monetary value of


stationery or office supplies in transit at the end of the
accounting period which is owned by the firm, represents closing
inventory. In-transit means that the goods are being transported to
or from the business at the end of the accounting period.

PERPETUAL INVENTORY SYSTEM


In a perpetual inventory system the quantity for each stock item is
updated daily for any purchase or issue of that stock item.
Therefore, it is possible to obtain the quantity of each stock item
on a daily basis. Usually, in a computerized perpetual inventory
system when goods are purchased, the cost price of these goods is
debited to the Inventory Account and not the Purchases Account. (See
relevant General Ledger entries below for a business that maintains
a computerized perpetual inventory system, and Subsidiary Ledgers
for Debtors and Creditors).

PURCHASE OF GOODS ON CREDIT


DR. Inventory Account (with cost price)
CR. Creditors Account (with cost price)

PURCHASE OF GOODS WITH CASH OR CHEQUE


DR. Inventory Account (with cost price)
CR. Cash Account or Bank Account (with cost price)
505

SALE OF GOODS ON CREDIT


(1) DR. Cost of Sales Account (with cost price)
CR. Inventory Account (with cost price)

(2) DR. Debtors Account (with selling price)


CR. Credit Sales Account (with selling price)

SALE OF GOODS FOR CASH OR CHEQUE


(1) DR. Cost of Sales Account (with cost price)
CR. Inventory Account (with cost price)

(2) DR. Cash Account or Bank Account (with selling price)


CR. Cash Sales Account (with selling price)

PHYSICAL INVENTORY COUNT


At the end of the financial year most businesses involved in trading
goods usually close the business for a few days to take stock. Some
businesses do stocktaking on weekends or public holidays when the
business is closed. Stocktaking involves a physical count of each
stock item at the end of the financial year. Depending on the stock
item a physical count could involve weighing or measuring the item.
The total quantity of each stock item counted is then reflected on a
stock sheet. The cost price or net realizable value for the stock
item (whichever is lower) is then reflected on the stock sheet and
is multiplied by the total quantity of the stock item in order to
determine the total monetary value for that stock item. The monetary
value for each stock is then added in order to determine the total
value for the closing stock at the end of the financial year.

It is essential to have an accurate physical stock count. An


inaccurate physical stock count can lead to an understatement or an
overstatement of profits. In order to ensure an accurate physical
stock count the following points should be noted.

1. Set date for physical count at least two (2) months in advance
and inform the external auditors and all staff members.
506

2. Identify location of inventory items (for example, branches and


warehouses).

3. Identify counting groups and group leaders (do not include


employees responsible for the stock, such as warehouse
personnel).

4. Brief all employees involved in the physical count about the


process involved, their responsibilities, and stress the need
for accuracy.

5. Identify any goods received on a sale or return basis and not


yet sold at the year end date. A clear indication should be
made that these goods should not be counted.

6. Tag each inventory item that is counted, weighed or measured at


each location. The tags should be pre-numbered.

7. The physical count for each stock item should be verified


(checked) by a second employee. Any discrepancies between the
first physical count and the second physical count must be
rectified by a third and fourth count if necessary.

8. At the end of the stocktaking, the group leader should ensure


that each set of inventory items is tagged, within the location
that he or she is responsible for.

9. External auditors usually do an independent physical count of


a few stock items and compare their count with the count
completed by employees. If major discrepancies occur the
auditors will increase their sample size. Depending on the
nature and materiality of the discrepancies, the auditors may
request that the entire stocktaking be done over.
507

10. A special area should be identified for all damaged goods. Each
damaged item discovered during the physical count should be
transferred to this area and should not be included in the
physical count of "good" stock items. Damaged items should be
counted and tagged at the end of the stocktaking.

11. A special area should be identified for all obsolete goods.


Each obsolete item discovered during the physical count should
be transferred to this area and should not be included in the
physical count of "good" stock items. Obsolete items should be
counted and tagged at the end of the stocktaking.

12. All tags issued (both used and unused) should be retrieved and
accounted for at the end of the stocktaking.

13. Identify all goods in transit at the year end date that are
owned by the business. Determine the quantity, condition and
cost of these goods in transit.

14. Identify any goods sent on a sale or return basis and not sold
at the year-end date. Verify the quantity and condition of
these goods.

15. Compare the accounting records inventory value with the


inventory value based on the physical count for each location
(branch, head office, warehouse). Any material difference
should be investigated.
508

Let us now look at the stocktaking for Open Bookshop. The General
Ledger inventory balance at December 31, 2004 is $670,580. We will
first look at the Inventory Tag for each item and then prepare the
physical inventory stock list.

INVENTORY TAG 001

STOCK ITEM REFERENCE NUMBER : 25

NAME OF STOCK ITEM: Financial Accounting The ARE Way

DESCRIPTION OF STOCK ITEM: Hard Cover

1ST. PHYSICAL COUNT : 296

COUNTED BY : T. Baxter

2ND. PHYSICAL COUNT : 298

COUNTED BY : M. Mole

DISCREPANCY RECTIFIED BY : R. Rockstar

FINAL COUNT : 296

┌───┐ ┌───┐ ┌───┐


│ X │ GOOD │ │ DAMAGED │ │ OBSOLETE
└───┘ └───┘ └───┘
509

INVENTORY TAG 002

STOCK ITEM REFERENCE NUMBER : 27

NAME OF STOCK ITEM: Management Accounting

DESCRIPTION OF STOCK ITEM: Hard Cover

1ST. PHYSICAL COUNT : 120

COUNTED BY : T. Baxter

2ND. PHYSICAL COUNT : 120

COUNTED BY : M. Mole

DISCREPANCY RECTIFIED BY :

FINAL COUNT : 120

┌───┐ ┌───┐ ┌───┐


│ X │ GOOD │ │ DAMAGED │ │ OBSOLETE
└───┘ └───┘ └───┘
510

INVENTORY TAG 003

STOCK ITEM REFERENCE NUMBER : 30

NAME OF STOCK ITEM: Financial Analysis

DESCRIPTION OF STOCK ITEM: Soft Cover

1ST. PHYSICAL COUNT : 200

COUNTED BY : T. Baxter

2ND. PHYSICAL COUNT : 200

COUNTED BY : M. Mole

DISCREPANCY RECTIFIED BY :

FINAL COUNT : 200

┌───┐ ┌───┐ ┌───┐


│ X │ GOOD │ │ DAMAGED │ │ OBSOLETE
└───┘ └───┘ └───┘
511

OPEN BOOKSHOP STOCK LIST FOR THE FINANCIAL YEAR ENDED 31/12/04
STOCK DESCRIPTION PHYSICAL UNIT TOTAL
ITEM # COUNT PRICE VALUE
$ $
25 Fin. A/cting The ARE Way 296 1200.00 355,200
27 Management Accounting 120 1500.00 180,000
30 Financial Analysis 200 663.60 132,720
Physical Inventory Balance 667,920
=======

OPEN BOOKSHOP STOCK SUMMARY FOR THE FINANCIAL YEAR ENDED 31/12/04
STOCK DESCRIPTION QUANTITY UNIT TOTAL
ITEM # PRICE VALUE
$ $
25 Fin. A/cting The ARE Way 298 1200.00 357,600
27 Management Accounting 120 1500.00 180,000
30 Financial Analysis 200 663.60 132,720
41 Cooking Made Easy 1 260.00 260
General Ledger Inventory Balance 670,580
=======

At the end of the financial year the inventory value in the General
Ledger amounted to $670,580. The physical inventory count resulted
in a closing inventory value of $667,920. The following entry would
be recorded in the General Journal and then posted to the General
Ledger.

DR. Cost of Sales A/C $2,660


CR. Inventory A/C $2,660

So in effect we are writing off $2,660 of inventory. This inventory


loss may be the result of employee theft, employee error in
recording, customer pilferage and normal wastage.
512

Any material stock write off should be debited to the Profit and
Loss Account and not the Cost of Sales Account. Let us assume the
physical inventory count resulted in a closing inventory value of
$670,580 then no adjusting entry would be necessary since the
physical inventory value ($670,580) and the General Ledger inventory
value ($670,580) are equal. The closing inventory value based on the
physical count is usually lower than the General Ledger inventory
value because of two main factors, customer pilferage and employee
theft.

STOCK RECONCILIATION
Sometimes it is not possible to do the stocktaking at the year end
date due to a strike, hurricane or some other natural disaster. As a
result the stocktaking is done after the year end date. Under such
circumstances a Stock Reconciliation must be done in order to
determine the closing stock figure at the year end date. In some
cases stocktaking is done before the year end date and a Stock
Reconciliation is also required to determine the closing stock
figure at the year end date.

Inventory items may be destroyed during a fire at the business


premises. It will be necessary to determine the value of inventory
items that were destroyed in the fire. Under such circumstances a
Stock Reconciliation must be done in order to determine the value of
stock that was destroyed during the fire.

In the event of a burglary of stock at the business premises, it


will be necessary to determine the value of stock that was stolen.
Under such circumstances a Stock Reconciliation must be done in
order to determine the value of stock that was stolen during the
burglary.
513

EXAMPLE 1

Daffy Ltd. was unable to take stock for raw materials at the year
end date on June 30, 2004. The stocktaking for raw materials was
done on July 10, 2004. From the information provided calculate the
figure for raw material inventory on June 30, 2004.
$
Raw material inventory on July 10, 2004 160,000
Purchases of raw material July 1 to July 10, 2004 25,500
Raw material returns on July 1, 2004 800
Raw material used in production July 1 to July 10, 2004 40,000

RAW MATERIAL STOCK RECONCILIATION


$
Raw material inventory on July 10, 2004 160,000
Less raw material purchases July 1 to July 10, 2004 25,500
134,500
Add raw material returns on July 1, 2004 800
135,300
Add raw material used in production July 1 to
July 10, 2004 40,000
Raw material inventory on June 30, 2004 175,300
=======

In order to know whether you should add or subtract the item you
should ask yourself this question -

WAS THE ITEM IN STOCK AT THE YEAR END DATE?

The raw material purchased between the period July 1 to July 10,
2004 and included in the July 10, 2004 raw material stock figure was
clearly not in stock on June 30, 2004. Therefore these purchases
must be subtracted from the July 10, 2004 raw material stock figure
in order to determine the June 30, 2004 raw material stock figure.

The raw material return on July 1, 2004 was not included in the July
10, 2004 raw material stock figure. But this raw material must have
been in stock on June 30, 2004 for it to be returned on July 1,
2004. Therefore these returns must be added to the July 10, 2004 raw
material stock figure in order to determine the June 30, 2004 raw
material stock figure.
514

The raw material used in production during the period July 1 to July
10, 2004 was not included in the July 10, 2004 raw material stock
figure. It is logical to assume that the raw material that came in
first would be used first, that is, first in first out (FIFO). It is
not logical to assume that the raw material just purchased July 1 to
July 10, 2004 would be used first, that is, last in first out
(LIFO). Assuming the FIFO method was used then the raw material used
in production during the period July 1 to July 10, 2004 must have
been in stock on June 30, 2004. Therefore the raw material used in
production during the period July 1 to July 10, 2004 must be added
to the July 10, 2004 raw material stock figure in order to determine
the June 30, 2004 raw material stock figure.
515

EXAMPLE 2

Chance Ltd. was unable to take stock on December 31, 2004 because of
a strike by employees. The stocktaking was done on Saturday, January
8, 2005. From the information provided relating to finished goods
calculate the closing stock figure for December 31, 2004.

Closing stock on January 8, 2005 $280,000

NOTES:

(1) Purchases 2/1/05 to 7/1/05 $ 15,000


(2) Sales at selling price (mark-up 25%)
2/1/05 to 7/1/05 $ 40,000
(3) Purchases returns 5/1/05 to 7/1/05 $ 2,000
(4) Sales returns at selling price 2/1/05 to 7/1/05 $ 1,000
(5) Goods in transit on December 31, 2004 owned by
Chance Ltd. and received on January 7, 2005 $ 45,000
(6) Goods in transit on December 31, 2004 owned by
Chance Ltd. and received on January 10, 2005 $ 60,000
(7) Goods sent on a sale or return basis to R. Roll
at selling price of $20,000 which were not sold on
December 31, 2004 and not taken into consideration
on January 8, 2005. The margin on these goods is 15%.
(8) Goods received on a sale or return basis at a cost
of $6,000 and not sold on January 8, 2005. These
goods were included in the January 8, 2005 closing
stock figure.
(9) Damaged goods on hand on December 31, 2004 cost price
$10,000 and net realizable value $6,000. The damaged
goods were included in the closing stock on January 8,
2005 at cost price.
516

WORKINGS

Cost Price of Goods Sold = Selling Price - (Selling Price x Margin)


= $40,000 - ($40,000 x 20%)
= $32,000

Cost Price of Sales Returns = $1,000 - ($1,000 x 20%)


= $800

Cost Price of Goods Sent on a Sale or Return Basis


= Selling Price - (Selling Price x Margin)
= $20,000 - ($20,000 x 15%)
= $17,000

Goods purchased after the year end date are included in the closing
stock figure on January 8, 2005. These goods were not in stock at
the year end date and must be subtracted from the January 8, 2005
stock figure in order to determine the stock figure for December 31,
2004.

Goods sold after the year end date is not included in the closing
stock figure on January 8, 2005. These goods were in stock at the
year end date and must be added to the January 8, 2005 stock figure
in order to determine the stock figure for December 31, 2004. Sales
are reflected at selling price, but we need the cost price of these
goods. Please remember that closing inventory is valued at cost
price, unless the net realizable value is lower.

Purchases returns after the year end date are not included in the
closing stock figure on January 8, 2005. These goods were in stock
at the year end date and must be added to the January 8, 2005 stock
figure in order to determine the stock figure for December 31, 2004.

Sales returns after the year end date are included in the closing
stock figure on January 8, 2005. These goods were not in stock at
the year end date and must be subtracted from the January 8, 2005
stock figure in order to determine the stock figure for December 31,
2004. Sales returns are at selling price, but we need the cost price
of these goods.
517

Goods in transit on December 31, 2004 and received on January 7,


2005 would be on hand on January 8 and therefore included in the
stocktaking figure on January 8, 2005, so no adjustment is necessary
for this figure.

Goods in transit on December 31, 2004 and received on January 10,


2005 would not be on hand on January 8 and therefore would not be
included in the stocktaking figure on January 8, 2005, so the cost
of these goods must be added.

The goods sent to R. Roll on a sale or return basis and not sold at
the year end date are owned by Chance Ltd. These goods should be
included in the closing stock at cost price and therefore must be
added.

Goods received on a sale or return basis are not owned by Chance


Ltd. and should not be included in its closing stock figure. These
goods were incorrectly included in the stock figure and should
therefore be subtracted.

Damaged goods were included at cost price, but these goods should
have been included at net realizable value that is lower, so $4,000
($10,000 - $6,000) must be subtracted.
518

CHANCE LTD. STOCK RECONCILIATION


$ $
Closing stock on January 8, 2005 280,000
ADD
Sales at cost price 32,000
Purchases returns 2,000
Goods in transit received 10/1/05 60,000
Goods sent on a sale or return basis
at cost price 17,000
111,000
391,000
LESS
Purchases 15,000
Sales returns at cost 800
Goods received on a sale or return
basis at cost price 6,000
Mark down on damaged goods 4,000
25,800
Closing stock on December 31, 2004 365,200
=======
519

EXAMPLE 3

On the night of February 6, 2004 Hawk Enterprise business premises


was destroyed by fire. The firemen managed to salvage goods in
good condition amounting to $70,000 (cost price). The mark-up on
goods is 33⅓ per cent.
$
Sales 1/1/04 to 6/2/04 157,800
Opening stock 1/1/04 272,000
Sales returns 1/1/04 to 6/2/04 7,800
Purchases returns 1/1/04 to 6/2/04 2,000
Purchases 1/1/04 to 6/2/04 60,000

From the information provided calculate the cost price of goods


destroyed in the fire.

WORKINGS

Mark-up = 33⅓% = ⅓
Therefore Margin = ¼ = 25%

Net Sales = $157,800 - $7,800


= $150,000

Gross Profit = $150,000 x 0.25


= $37,500

Net Purchases = $60,000 - $2,000


= $58,000
520

HAWK ENTERPRISE STOCK RECONCILIATION


$ $
Opening stock 272,000
ADD
Net purchases 58,000
Gross profit 37,500
95,500
367,500
LESS
Net sales 150,000
Goods salvaged 70,000
220,000
Stock destroyed in fire on 6/2/04 147,500
=======

OR

The stock destroyed in the fire can be calculated using the


principles learned in preparing the Trading Account (accounting
format).

HAWK ENTERPRISE STOCK RECONCILIATION ACCOUNTING FORMAT


$ ║ $
Opening stock 272,000 ║ Net sales 150,000
Net purchases 58,000 ║ Goods salvaged 70,000
330,000 ║ Stock destroyed in the
Gross profit 37,500 ║ fire (missing figure) 147,500
367,500 ║ 367,500
======= ║ =======

521

PAYROLL ACCOUNTING ENTRIES


Depending on the organization employees can be paid weekly, or
fortnightly (every two weeks), or twice per month, or monthly.
Within the same organization some employees are paid weekly, some
fortnightly and others monthly. All employees are usually paid a
basic pay that is based on the number of hours worked or it can be a
fixed sum. The normal workweek is usually forty hours Monday to
Friday, that is, eight regular hours per day, but this sometimes
varies depending on the organization. The eight hours include one
hour for lunchtime but some organizations only allow employees
thirty minutes (30) for lunchtime.

Angella and Michael both work for Williams Ltd. at a basic rate of
$100 per hour. Angella is paid a basic pay, based on the number of
hours worked per week, whereas Michael is paid a fixed basic pay per
week. For the 40 hours week just ended, Angella worked 35 regular
hours and Michael 36 regular hours. (I said regular hours to
distinguish it from overtime hours that will be explained later).
Calculate the basic pay for Angella and Michael for the week just
ended. The normal workweek is Monday to Friday.

NAME OF EMPLOYEE REGULAR HOURS RATE PER HOUR BASIC PAY


Angella 35 $100 $3,500
Michael 40 $100 $4,000

Since Michael is paid a fixed basic pay per week, he is paid for
forty hours whether or not he works for forty hours.

Some organizations pay overtime for any work done in excess of the
regular daily working hours. If the normal workweek is Monday to
Friday the employees will be paid overtime for work done on
Saturdays, Sundays and Public Holidays.

DAYS OVERTIME RATE PAID


Monday to Friday one and a half time the basic rate per hour
Saturday one and a half time the basic rate per hour
Sunday two times the basic rate per hour
Public Holiday two times the basic rate per hour
522

Overtime is paid at time and a half or double time depending on the


day. It should be noted that some organizations do not pay overtime.

From the following information calculate the overtime pay for


Angella.

DAY NUMBER OF HOURS WORKED


Monday 10
Tuesday 9
Friday 10
Saturday 5
Public Holiday 4

WORKINGS

Time and a half rate = $100 X 1.5 = $150


Double time rate = $100 X 2 = $200

DAY OVERTIME HOURS RATE PER HOUR OVERTIME PAY


Monday 2 $150 $ 300
Tuesday 1 $150 $ 150
Friday 2 $150 $ 300
Saturday 5 $150 $ 750
Public Holiday 4 $200 $ 800
$2,300
======

Depending on the tax laws income tax is paid by employees and is


deducted at the source (P.A.Y.E.) pay as you earn. In other words,
income tax is deducted before employees are paid. Income tax
represents a statutory deduction. A statutory deduction is any
deduction that is required by law. Another statutory deduction is
N.I.S. (National Insurance Scheme). The N.I.S. is the social
security deduction. In Jamaica statutory deductions include Income
Tax, National Insurance Scheme (N.I.S.), Education Tax, and National
Housing Trust (N.H.T.). Deductions that are not required by law
include union dues, staff loan, other loans, credit union, pension,
life insurance, and health insurance.
523

Allowances such as rent, laundry, and travelling are sometimes paid


to employees in addition to their basic pay. Some organizations have
a productivity scheme whereby employees are paid an additional sum
if their productivity is in excess of the required normal
productivity level. In addition to allowances and productivity
incentives some organizations pay a Christmas bonus to employees at
Christmas.

From the following information prepare the relevant journal entries


and General Ledger Accounts.

WAGES EARNINGS PAYROLL SUMMARY FOR WEEK ENDED MAY 5, 2004

A. SAFE M. WILD B. BIKE T. FLIP TOTAL


$ $ $ $ $
BASIC 1,000 1,000 6,000 2,000 10,000
PAY
OVERTIME 100 - 200 - 300
PAY
OTHER - - - - -
PAY
GROSS 1,100 1,000 6,200 2,000 10,300
EARNINGS

WAGES DEDUCTIONS PAYROLL SUMMARY FOR WEEK ENDED MAY 5, 2004

A. SAFE M. WILD B. BIKE T. FLIP TOTAL


$ $ $ $ $
N.I.S. 30 30 - 75 135
INCOME TAX 80 50 530 200 860
N.H.T. 20 20 - 40 80
OTHER - - 30 - 30
TOTAL 130 100 560 315 1,105
DEDUCTIONS
524

WAGES NET PAY SUMMARY FOR WEEK ENDED MAY 5, 2004

A. SAFE M. WILD B. BIKE T. FLIP TOTAL


$ $ $ $ $
NET PAY 970 900 5,640 1,685 9,195

NET PAY = GROSS EARNINGS - TOTAL DEDUCTIONS

EMPLOYER'S STATUTORY DEDUCTION LIABILITY FOR WEEK ENDED MAY 5,2004.


N.I.S $120
N.H.T. $150

Each employee will receive a cheque for his or her net pay. So the
Bank Account will be credited with $9,195 and the Wages Account will
be debited with $9,195. B. Bike did not pay N.H.T. and N.I.S.
because he is a retired individual who was employed by the business.

GENERAL JOURNAL

DATE PARTICULARS FOLIO DR. CR.


$ $
5/5/04 Wages A/C 1,105
N.I.S. Payable A/C 135
Income Tax Payable A/C 860
N.H.T. Payable A/C 80
Other Payable A/C 30
Being wages deduction for
the week ended May 5, 2004.

5/5/04 N.I.S. Expense A/C 120


N.H.T. Expense A/C 150
N.I.S. Payable A/C 120
N.H.T. Payable A/C 150
Being statutory deductions
payable by the business.
525

Please NOTE that any statutory payment that the business must make
such as N.I.S. is an expense.

GENERAL LEDGER ACCOUNTS

DR. WAGES ACCOUNT CR.


$ ║ $
5/5/04 Bank A/C 9,195 ║
5/5/04 Sundries 1,105 ║

Please NOTE that the total debited in the Wages Account


= $9,195 + $1,105 = $10,300. This $10,300 represents the
total gross pay (earnings).

Since the $1,105 debited in the Wages Account is coming from several
different accounts (N.I.S. Payable A/C, Income Tax Payable A/C,
N.H.T. Payable A/C, and Other Payable A/C), the term `Sundries' is
used instead of writing the name of each account. Using the term
`Sundries' saves time and paper. The term `Sundries' is used
whenever the corresponding entries are in two or more accounts.

DR. N.I.S. PAYABLE ACCOUNT CR.


$ ║ $
║ 5/5/04 Wages A/C 135
║ 5/5/04 N.I.S. Expense A/C 120


526

DR. INCOME TAX PAYABLE ACCOUNT CR.


$ ║ $
║ 5/5/04 Wages A/C 860

DR. N.H.T. PAYABLE ACCOUNT CR.


$ ║ $
║ 5/5/04 Wages A/C 80
║ 5/5/04 N.H.T. Expense A/C 150

DR. OTHER PAYABLE ACCOUNT CR.


$ ║ $
║ 5/5/04 Wages A/C 30

DR. N.I.S. EXPENSE ACCOUNT CR.


$ ║ $
5/5/04 N.I.S. Payable A/C 120║

DR. N.H.T. EXPENSE ACCOUNT CR.


$ ║ $
5/5/04 N.H.T. Payable A/C 150║

527

TIME SHEET

From the following information prepare the time sheet, calculate the basic pay and
calculate the overtime pay, for the week ended December 27, 2003.

Pamela Foster, employee number 1002031 worked the following


hours at Flourish Limited, for the week ended 27/12/2003.

Day Date Time in Time Out


Saturday 20/12/2003 10:00 a.m. 12 noon
Monday 22/12/2003 9:00 a.m. 7:30 p.m.
Tuesday 23/12/2003 9:00 a.m. 5:00 p.m.
Wednesday 24/12/2003 9:00 a.m. 6:00 p.m.
Thursday 25/12/2003 9:00 a.m. 1:00 p.m.
Friday 26/12/2003 CLOSED CLOSED

Pamela’s basic rate per hour is $300. Pamela is only paid for the
hours that she works. It is the policy of the company to pay
overtime. Double time rate is paid for overtime done on a Sunday or
a public holiday. All other overtime is paid at time and a half.
Flourish Limited is open for business Monday to Friday, 9 a.m. to 5
p.m. Thursday, December 25, 2003 is Christmas day, a public
holiday.
528

Time Sheet

Name of Employee: Pamela Foster


Employee Number: 1002031
Date Time In Time Out Regular Overtime
Hours Hours
20/12/2003 10:00 a.m. 12 noon - 2
22/12/2003 9:00 a.m. 7:30 p.m. 8 2 ½
22/12/2003 9:00 a.m. 5:00 p.m. 8 -
24/12/2003 9:00 a.m. 6:00 p.m. 8 1
25/12/2003 9:00 a.m. 1:00 p.m. - 4
26/12/2003 - - - -

Workings

Time and a half rate = $300 X 1.5 = $450

Double time rate = $300 X 2 = $600

Basic Pay Calculation


Day Hours Rate Basic Pay
Monday 8 $300 $2,400
Tuesday 8 $300 $2,400
Wednesday 8 $300 $2,400
Thursday -
Friday -
24 $7,200
529

Overtime Pay Calculation


Day Hours Rate Overtime pay
Saturday 2 $450 $900
Monday 2 ½ $450 $1,125
Tuesday - - -
Wednesday 1 $450 $450

Thursday 4 $600 $2,400


Friday - - -
$4,875
530

PAYROLL SUMMARY
From the following information prepare a payroll summary for the month of
January 2004.

Matthew Mark Luke John Duck


Basic Salary $60,000 $60,000 $80,000 $80,000 $100,000

Overtime $10,000 $5,000 - - -


Monthly
Deductions

Life Insurance $1,000 - $500 - $2,000

Union Dues $500 $500 $500 $500 -

Credit Union $2,000 $3,000 - - -


Health
Insurance $3,000 $2,000 $3,000 $2,000 $3,000

Loan $4,000 - - - -

All employees pay social security of 1% of the basic salary. The


company operates a compulsory pension scheme to which all
employees contribute 5% of their basic salary. Duck makes an
additional 5% voluntary contribution to the pension scheme. All
employees should receive a Christmas bonus of 25% of basic
salary. Matthew, Mark and Duck are to receive a productivity
incentive of 50% of their basic salary. The company was unable
to pay the Christmas bonus and the productivity incentive in
December 2003 because of a cash flow problem. The income tax
rate is 20%.
531
Payroll Summary for January 2004
Matthew Mark Luke John Duck Total
$ $ $ $ $ $

Basic Pay 60,000 60,000 80,000 80,000 100,000 380,000

Overtime 10,000 5,000 - - - 15,000

Other 45,000 45,000 20,000 20,000 75,000 205,000


Gross Earnings 115,000 110,000 100,000 100,000 175,000 600,000

Deductions
Social
Security 600 600 800 800 1,000 3,800
Pension 3,000 3,000 4,000 4,000 10,000 24,000
Income Tax 22,280 21,280 19,040 19,040 32,800 114,440
Life
Insurance 1,000 - 500 - 2,000 3,500

Union Dues 500 500 500 500 - 2,000


Credit Union 2,000 3,000 - - - 5,000
Health
Insurance 3,000 2,000 3,000 2,000 3,000 13,000
Loan 4,000 - - - - 4,000

Other - - - - - -
Total 36,380 30,380 27,840 26,340 48,800 169,740

Net Pay 78,620 79,620 72,160 73,660 126,200 430,260


532

Year to Date $ $ $ $ $ $
Earnings 115,000 110,000 100,000 100,000 175,000 600,000
Social
Security 600 600 800 800 1,000 3,800
Pension
3,000 3,000 4,000 4,000 10,000 24,000
Taxable Pay 111,400 106,400 95,200 95,200 164,000 572,200
Income Tax 22,280 21,280 19,040 19,040 32,800 114,440

Taxable pay = gross earning minus (social security + pension)

Income tax = Taxable pay multiplied by income tax rate


533

REVIEW QUESTIONS
(1)* An understatement in the value of closing stock will result in
an __________________________ of net profit.

(2)* An overstatement in the value of closing stock will result in


an __________________________ of current assets.

(3) Briefly explain a perpetual inventory system.

(4) What is meant by stocktaking?

(5) What steps would you take to ensure an accurate physical stock
count during stocktaking?

(6) What is the purpose of a Stock Reconciliation?

(7) What are goods in transit, and what effect does the value of
these goods have on the closing stock?

EXERCISE 1 *

Genius Ltd. was unable to take stock at the year end date on June
30, 2004. The stocktaking was done on July 4, 2004. The margin on
goods sold is 20 per cent. From the following information calculate
the closing stock value for June 30, 2004.

$
(1) Closing stock on July 4, 2004 155,600
(2) Purchases for the period 1/7/04 to 3/7/04 32,500
(3) Returns outward on 2/7/04 600
(4) Sales at selling price for the period 1/7/04
to 3/7/04 35,000
(5) Returns inward at selling price for the period
1/7/04 to 3/7/04 4,000
(6) Goods in transit owned by Genius Ltd. on June 30,
2004 and received on July 2, 2004 51,825
534

EXERCISE 2 *

On November 27, 2004 Respect Ltd. had a fire at its business


premises that destroyed all its stock. From the information provided
calculate the cost price of the stock that was destroyed in the
fire. The margin on goods is 20 per cent.

$
Opening stock on January 1, 2004 100,000
Purchases 1/1/04 to 27/11/04 451,900
Sales 1/1/04 to 27/11/04 500,500
Purchases returns 1/1/04 to 27/11/04 1,900
Sales returns 1/1/04 to 27/11/04 500

EXERCISE 3 *

WAGES EARNINGS PAYROLL SUMMARY FOR WEEK ENDED JULY 7, 2004

M. TAXI B. CAKE T. BAKE Y. CUT TOTAL


$ $ $ $ $
BASIC 2,000 2,000 3,000 4,000 11,000
PAY
OVERTIME 500 600 400 200 1,700
PAY
OTHER - - - - -
PAY
GROSS 2,500 2,600 3,400 4,200 12,700
EARNINGS
535

WAGES DEDUCTIONS PAYROLL SUMMARY FOR WEEK ENDED JULY 7, 2004

M. TAXI B. CAKE T. BAKE Y. CUT TOTAL


$ $ $ $ $
N.I.S. 50 50 60 60 220
INCOME TAX 230 235 320 390 1,175
N.H.T. 40 40 60 80 220
EDUCATION 40 40 60 80 220
TAX
UNION DUES 30 30 30 30 120
INSURANCE 25 30 50 70 175
MORTGAGE - - - 500 500
OTHER 40 20 - - 60
TOTAL 455 445 580 1,210 2,690
DEDUCTIONS

WAGES NET PAY SUMMARY FOR WEEK ENDED JULY 7, 2004

M. TAXI B. CAKE T. BAKE Y. CUT TOTAL


$ $ $ $ $
NET PAY 2,045 2,155 2,820 2,990 10,010

REQUIRED:

(a) How much in total will be credited to the Bank Account for
wages?

(b) Prepare the relevant journal entry.


536

ANSWERS TO SELECTED REVIEW QUESTIONS

(1) Understatement

(2) Overstatement

EXERCISE 1

GENIUS LTD. STOCK RECONCILIATION


$ $
Closing stock on July 4, 2004 155,600
ADD
Sales at cost 28,000
Purchases returns 600
28,600
184,200
LESS
Purchases 32,500
Sales returns at cost 3,200
35,700
Closing stock on June 30, 2004 148,500
=======
537

EXERCISE 2

WORKINGS

Net Sales = $500,500 - $500


= $500,000

Gross Profit = $500,000 x 0.20


= $100,000

Net Purchases = $451,900 - $1,900


= $450,000

RESPECT LTD. STOCK RECONCILIATION


$ $
Opening stock 100,000
ADD
Net purchases 450,000
Gross profit 100,000
550,000
650,000
LESS
Net sales 500,000
Stock destroyed in fire 150,000
=======
538

EXERCISE 3

(a) $10,010

(b)
GENERAL JOURNAL

DATE PARTICULARS FOLIO DR. CR.


$ $
7/7/04 Wages A/C 2,690
N.I.S. Payable A/C 220
Income Tax Payable A/C 1,175
N.H.T. Payable A/C 220
Education Tax Payable A/C 220
Union Dues Payable A/C 120
Insurance Payable A/C 175
Mortgage Payable A/C 500
Other Payable A/C 60
Being wages deduction for
the week ended July 7, 2004.
539

addpr

ADDITIONAL PROBLEMS
540

PROBLEM 1 Recording Transactions in Books of Original Entry


(10 minutes)

Record the relevant transactions in the Sales Journal and Purchases


Journal.
$
April 1, 2004 L. Ackee started business with cash 80,000
April 1, 2004 Lodged cash to Current Account 60,000
April 2, 2004 Purchased goods by cheque 40,000
April 5, 2004 Sold goods on credit to C. Chin 1,000
April 6, 2004 Sold goods for cash 10,000
April 10, 2004 Purchased goods on credit from X. Tom 75,000
April 15, 2004 Sold goods on credit to B. Book 26,000
April 20, 2004 Sold goods on credit to C. Cook 38,000
April 25, 2004 Paid salary by cheque 2,000
April 27, 2004 Paid electricity bill by cheque 1,800
April 30, 2004 Water rate accrued 600
541

PROBLEM 2 Suspense Account (30 minutes)

After the preparation of the Trial Balance for Black Ltd. on March
31, 2004, an unexpected difference remains, and a Suspense Account
is opened for that amount. Subsequent investigations revealed all
the errors eliminating the Suspense Account credit balance of
$15,600. The errors discovered were:

(a) Office supplies purchased amounting to $6,800 was recorded in


the Purchases Account.

(b) The total discount allowed amounting to $2,000 was credited in


error to the Discount Allowed Account.

(c) A payment of $36,000 for cash purchases was recorded in the


Bank Account in the Cash Book, but was not posted to the Cash
Purchases Account.

(d) Interest income amounting to $27,800 was treated as interest


expense in the Interest Income Account.

(e) Repair to furniture amounting to $4,900 was debited to the


Furniture Account instead of the Repairs to Furniture Account.

(f) The purchase of office equipment amounting to $6,000 was


incorrectly posted to the Purchases Account.

REQUIRED: Prepare the relevant journal entries and the Suspense


Account.
542

PROBLEM 3 Trial Balance Preparation (30 minutes)

From the following information prepare the Trial Balance for


Flinstone Limited on May 31, 2004. The accounts should be listed in
numerical order.

ACCOUNT # NAME OF ACCOUNT AMOUNT


$
4209 Discount Allowed A/C 1,000
3202 Discount Received A/C 4,000
1103 Bank A/C (overdraft) 10,000
1202 Furniture A/C 70,000
2401 Ordinary Share Capital A/C 194,000
2402 10% Preference Share Capital A/C 20,000
2101 Creditors A/C 30,000
2102 Accruals A/C 10,000
1104 Debtors A/C 120,000
3102 Credit Sales A/C 400,000
3103 Returns Inward A/C 10,000
4102 Credit Purchases A/C 170,000
4103 Returns Outward A/C 8,000
4201 Salaries A/C 280,000
4212 Office Supplies A/C 25,000

PROBLEM 4 Disposal of Tangible Asset (10 minutes)

From the following information prepare the Disposal Account.

Cost of machine purchased on February 1, 1990 $400,000


Expected salvage value on January 31, 2000 $ 50,000
Method of depreciation - straight line
Machine sold on March 31, 1996 for cash $220,000
Depreciation is calculated on a pro rata basis.
Round all calculation to the nearest dollar.
The financial year end is December 31.
543

PROBLEM 5 Bank Reconciliation (20 minutes)

Bank balance as per Cash Book on June 30, 2004 $20,000


Unpresented cheques $50,000
Deposits not yet credited by the bank $30,000

Items on the Bank Statement not yet reflected


in the Cash Book
Standing order paid by the bank $ 2,000
Transfer from Frank Ltd. a debtor in settlement
of his debt $35,000
Transfer to Bay Ltd. a creditor in full settlement $10,000
Bank charges $ 150
Cheques deposited and dishonoured $ 200

What is the bank balance per the Bank Statement on June 30, 2004?
544

PROBLEM 6 Partnership (60 minutes)

TRIAL BALANCE OF SCOOBY, DOO AND DEXTER PARTNERSHIP ON


DECEMBER 31, 2004
NAME OF ACCOUNT DEBIT CREDIT
$ $
Property, plant and equipment(net) 390,000
Stock December 31, 2004 20,000
Trade Debtors 40,000
Bank 180,000
Capital Accounts
Scooby 100,000
Doo 100,000
Dexter 200,000
Current Accounts
Scooby 4,000
Doo 2,000
Dexter 8,000
Trade Creditors 26,000
Cost of Sales 280,000
Sales 576,000
Operating Expenses 136,000
Loan from Dexter (2008) 40,000
Drawings
Scooby 3,000
Doo 3,000
Dexter 4,000 -
1,056,000 1,056,000
========= =========

Interest of 10 per cent per annum is to be paid on each partner's


fixed capital, and the loan from Dexter. Scooby is to receive a
salary of $28,000 for working in the business. Interest of 20 per
cent is to be charged on the closing balance of each partner's
Drawings Account. The partners share profits and losses equally.

PREPARE THE FOLLOWING:


• Profit and Loss Statement
• Profit and Loss Appropriation Statement
• Current Accounts
• Balance Sheet
545

PROBLEM 7 Non-Profit Organization (35 minutes)

The following information relates to Staff Club for the financial


year ended December 31, 2004.

RECEIPTS PAYMENTS
$ ║ $
Cash Balance b/f 500 ║ Rent 7,500
Bank Balance b/f 2,000 ║ Fundraising expenses 6,250
Subscriptions for 2003 250 ║ Meeting expenses 1,250
Subscriptions for 2004 12,250 ║ Secretarial expenses 1,000
Subscriptions for 2005 1,250 ║ Prizes 2,000
Fundraising 15,000 ║ Cash balance c/f 3,250
║ Bank balance c/f 10,000
31,250 ║ 31,250
====== ║ ======

ADDITIONAL INFORMATION:

(1) Subscriptions in arrears at December 31, 2003 $500.


(2) The subscription for each member is $250. Fifty (50) members
were registered with the club in 2004.
(3) Meeting expenses outstanding at December 31, 2004 amounted to
$250.
(4) Rent paid in advance for January 2005 amounted to $500.
(5) Furniture and fittings on January 1, 2004 $11,250.
(6) Furniture and fittings on December 31, 2004 $10,750.

REQUIRED: Prepare the Income and Expenditure Account for the


financial year ended December 31, 2004 and a Balance
Sheet as at that date.
546

PROBLEM 8 Manufacturing Accounts (60 minutes)


$
Opening stock of raw materials 11,304
Closing stock of raw materials 24,304
Purchases of raw materials 960,000
Purchases of finished goods 100,000
Raw material returns 12,000
Direct wages 976,000
Opening stock - work in progress 74,000
Opening stock - finished goods 90,000
Closing stock - work in progress 78,000
Closing stock - finished goods 120,000
Administrative expenses 800,000
Sales commission expense 260,000
Rent 1,200,000
Electricity 240,000
Salaries 1,040,000
Insurance 51,000
Building depreciation 20,000
Plant depreciation 10,000
Motor vehicles depreciation 12,000
Carriage outwards 44,000
Bank charges 3,266
Discount received 16,000
Discount allowed 4,754
Sales 5,800,000
Repairs to factory plant 44,000
Furniture depreciation - factory 8,000
Furniture depreciation - office 14,000
Allowance for unrealized profit 15,000
Retained profit 200,000
Ordinary share capital 4,855,000
Debtors 520,000
Creditors 200,000
Prepayment 56,000
Accruals 260,324
Bank overdraft 480,000
Plant (net of depreciation) 1,600,000
Factory furniture (net of depreciation) 180,000
Office furniture (net of depreciation) 360,000
Motor vehicles (net of depreciation) 800,000
Building (net of depreciation) 2,400,000
547

ADDITIONAL INFORMATION:

(1) The goods manufactured are transferred from the manufacturing


department to the trading department at a mark-up of 20 per
cent.

(2) The following apportionment should take place:

(i) Twenty per cent (20%) of the rent and ten per cent
(10%) of the electricity should be charged to the office
and the respective balance charged to the factory.

(ii) $800,000 of the salaries relate to the office.

(iii) Seventy per cent (70%) of insurance and building


depreciation should be charged to the factory and the
balance charged to the office.

(3) 100,000 units were produced during the financial year.

Prepare the following statements for Bear Limited for the year ended
December 31, 2004:

• Manufacturing Statement
• Trading Statement
• Profit and Loss Statement
• Balance Sheet
548

PROBLEM 9 Accountant's Worksheet (45 minutes)

From the following information prepare the Accountant's Worksheet


for Rainy Day on December 31, 2004, showing the Unadjusted Trial
Balance, the Adjustments, the Adjusted Trial Balance, the Income
Statement and the Balance Sheet.

ACCOUNT # NAME OF ACCOUNT $


1201 Capital Account 177,000
1202 Drawings Account 15,000
3101 Purchases Account 126,000
3102 Purchases Returns Account 6,000
4101 Sales Account 303,000
3104 Opening Stock Account 69,000
3201 Salaries Account 60,000
3202 Rent Account 15,000
3203 Insurance Account 12,000
3210 Other Expenses Account 3,000
2203 Office Furniture Account 60,000
2204 Allowance for Depreciation Account 12,000
1004 Debtors Account 150,000
1002 Bank Account 63,000
1101 Creditors Account 75,000

Additional information :

(a) Provide for depreciation on the straight line basis at 10% per
annum on cost.
(b) Stationery expenses not paid at December 31, 2004 $3,000.
(c) Insurance prepaid $1,500.
(d) Stock at December 31, 2004 $66,000.

Other Accounts

ACCOUNT # NAME OF ACCOUNT


1006 Closing Stock Account
1008 Prepayment Account
1102 Accrual Account
3204 Stationery Expense Account
3209 Depreciation Account
4401 Trading Account
549

PROBLEM 10 Preparation of Financial Statements for a Sole Trader


(60 minutes)

Trial Balance of Flood Gate on June 30, 2004.

ACCOUNT # NAME OF ACCOUNT DEBIT CREDIT


$ $
1101 Petty Cash Imprest A/C 500
1102 Bank Account #1 19,500
1103 Bank Account #2 40,000
1104 Accounts Receivable A/C 90,000
1105 Allowance for Bad Debts A/C 750
1107 Bills Receivable A/C 5,000
1201 Machines A/C 100,000
1202 Allowance for Depreciation
on Machines A/C 56,667
1203 Furniture & Fixtures A/C 35,000
1204 Allowance for Depreciation
on Furniture & Fixtures A/C 5,000
2101 Creditors A/C 62,500
2201 Loan A/C 50,000
2301 Capital A/C 84,083
2302 Drawings A/C 15,000
3101 Cash Sales A/C 140,000
3102 Credit Sales A/C 215,000
3103 Sales Returns A/C 2,500
3201 Rental Income A/C 13,000
3202 Discount Received A/C 2,000
4102 Credit Purchases A/C 200,000
4104 Opening Stock A/C 40,000
4201 Wages and Salaries A/C 30,000
4202 Rent Expense A/C 24,000
4203 Insurance A/C 23,500
4204 Travelling A/C 3,500
4205 Stationery A/C 11,000
4206 Interest Expense A/C 10,000
4207 Water Rates A/C 6,000
4208 Electricity A/C 12,000
4209 Discount Allowed A/C 500 -
648,500 648,500
======= =======
550

The following transactions have not been recorded or adjusted for in


the books of Flood Gate.

(1) Debtors include $5,000 in respect of goods sent to Showgun on


a sale or return basis at a margin of 25%, and the goods
remained unsold at the year end date.

(2) The owner withdrew goods costing $2,500 for her own personal
use.

(3) A sales invoice for $15,000 was omitted from the Sales
Journal.

(4) A cheque for $500 received from Donman a debtor, and


lodged in Bank Account #1, was dishonoured by the bank.

(5) The allowance for bad debts should be 1% of debtors.

(6) Rental income for July 2004 amounting to $1,000 was received
in June 2004.

(7) A machine costing $25,000 that was purchased on March 10,


2001 was sold for $45,000 cash on June 26, 2004. The cash was
lodged in Bank Account #1.

(8) Travelling expense to be accrued $1,250.

(9) Insurance paid for January 2004 to December 2004 $20,000.

(10) Closing stock on June 30, 2004 $25,000.

(11) Provide for depreciation on machines and furniture and


fixtures.

(12) $10,000 was received in respect of a debt that was written


off 4 years ago. The amount was lodged in Bank Account #2.
551

Additional information:

(a) The machines are being depreciated on the straight line basis
at a rate of 20% per annum on cost.

(b) The furniture and fixtures were purchased July 1, 2001 and are
being depreciated on the straight line basis.

(c) Depreciation policy - a full month's depreciation is charged


on assets acquired or sold during the month.

(d) Loan repayment (principal) due within the next 12 months


$30,000.

(e) Round all calculations to the nearest dollar.

REQUIRED:

1. Prepare the Trading Statement for the financial year ended June
30, 2004 (vertical format). (14 marks)

2. Prepare the Profit and Loss Statement for the financial year
ended June 30, 2004 (vertical format). (26 marks)

3. Prepare the Balance Sheet as at June 30, 2004 (vertical


format). (40 marks)

4. Prepare the opening entries for July 1, 2004. (20 marks)


552

PROBLEM 11 Departmental Accounts (40 minutes)

From the following information prepare the Departmental Trading


and Profit and Loss Account for Tee Limited for the year ended
December 31, 2004.

Sales - Men Department 260,000


Women Department 1,340,000
Children Department 400,000
Cost of sales - Men Department 90,000
Women Department 480,000
Children Department 130,000
Wages and salaries - Men Department 24,000
Women Department 60,000
Children Department 44,000
Advertising 20,000
Insurance paid 5,200
Electricity 7,800
Rent 24,000
Administrative costs 104,000
Other costs 28,000
Insurance prepaid 200
Rent Accrued 2,000

MEN WOMEN CHILDREN


DEPARTMENT DEPARTMENT DEPARTMENT
ASSETS INSURED $100,000 $600,000 $300,000
FLOOR AREA 2,000 sq. ft. 6,000 sq. ft. 2,000 sq. ft.
# OF EMPLOYEES 4 10 6

COST BASIS OF ALLOCATION


Advertising Dollar sales
Insurance Value of assets insured
Electricity Floor area
Rent Floor area
Administrative costs Number of employees
Other costs Dollar sales
553

PROBLEM 12 Bank Reconciliation (25 minutes)

The following information relate to Skipper Limited.

Bank balance per Bank Statement on May 31, 2004 $24,000.

Unpresented cheques -
DATE CHEQUE NUMBER PAYEE AMOUNT
10/11/03 1035 R. Rat $5,000
08/05/04 1509 C. Cat $ 400
11/05/04 1517 B. Dog $1,000
30/05/04 1528 Frog Ltd. $ 600

Bank charges not yet reflected in the Cash Book $80.

A transfer of $6,000 from Native Jamrock a debtor in Cayman, to


Skipper Limited Bank Account in May 2004 was not reflected in the
Cash Book.

Cheque number 525 paid by Flipper Limited to S. Gilligan amounting


to $400 was deducted in error by the bank from Skipper Limited Bank
Account.

Fees for a bank loan to Skipper Ltd. debited by the bank on May 31,
2004 and not yet reflected in the Cash Book amounted to $1,200.

Cheque number 1516 paid to Moon Ltd. a creditor was correctly


recorded on the Bank Statement as $562 and incorrectly recorded in
the Cash Book as $526.

A deposit of $9,400 made on Wednesday, May 31, 2004 at 9:36 p.m. in


the night deposit at the bank was not reflected on the Bank
Statement.

Skipper Limited maintains a Debtor Control Account and a Creditor


Control Account in the General Ledger.

REQUIRED:
(a) Prepare a Bank Reconciliation Statement for May 31, 2004,
without adjusting the bank balance in the Cash Book.
(b) Prepare a Bank Reconciliation Statement for May 31, 2004, after
adjusting the bank balance in the Cash Book.
554

PROBLEM 13 Accountant’s Worksheet (40 minutes)

From the following information prepare the Accountant's


Worksheet for Doorbell on June 30, 2004, showing the
Unadjusted Trial Balance, the Adjustments, the Adjusted Trial
Balance, the Income Statement, and the Balance Sheet.

ACCOUNT NUMBER NAME OF ACCOUNT $


1002 Bank Account (overdraft) 2,000
1003 Opening Stock Account 8,000
1004 Debtors Account 24,000
1101 Land & Buildings Account 100,000
1102 Allowance for Depreciation on
Land & Building Account 10,000
1103 Furniture & Fixtures Account 20,000
1104 Allowance for Depreciation on
Furniture & Fixtures Account 8,000
2001 Creditors Account 7,000
2101 Capital Account 100,400
2102 Drawings Account 400
4201 Purchases Account 87,600
3001 Sales Account 140,600
3002 Sales Returns Account 2,600
3101 Discount Received Account 1,200
3102 Rental Income Account 24,000
4001 Salaries Account 46,800
4002 Insurance Account 1,640
4003 Electricity & Water Rates Account 2,160

Other Accounts

ACCOUNT NUMBER NAME OF ACCOUNT


1005 Prepayment Account
1006 Closing Stock Account
2002 Accruals Account
3201 Trading Account
4004 Buildings Depreciation Account
4005 Furniture & Fixtures Depreciation Account
555

Additional information:

(a) Provide for depreciation on land and buildings on the straight


line basis at 5 per cent per annum.
(b) Provide for depreciation on furniture and fixtures on the
straight line basis. The furniture and fixtures were purchased
on June 30, 1999.
(c) Insurance prepaid $240.
(d) Goods costing $2,000 were received on a sale or return basis
from Palace Ltd. on June 26, 2004. The goods remained unsold
at the year end date. The $2,000 was recorded in the Purchases
Journal in June 2004.
(e) Closing stock $16,000 (including the cost of goods received on
a sale or return basis).
556

PROBLEM 14 Correction of Errors (30 minutes)

B. Teddy discovered the following errors and omissions in her books


for the month of August 2004.

(a) Repair to machinery amounting to $400 was debited to the


Machinery Account instead of the Repairs to Machinery Account.

(b) Rent expense paid by cheque amounting to $6,000 was omitted


from the Bank Account and the Rent Expense Account.

(c) Cash sales amounting to $2,500 was debited in the Bank Account
only.

(d) A payment of $500 to T. Jewel a creditor was credited to the


Creditors Account. B. Teddy maintains a Subsidiary Ledger for
Creditors.

(e) Commission received amounting to $1,000 was debited in the


Commission Expense Account.

(f) The Credit Sales Account for August 2004 was undercast by
$7,000 and the Cash Purchases Account was overcast by $600.

(g) $4,000 paid for wages was credited in the Bank Account in the
Cash Book as $40,000 and debited in the Wages Account as
$40,000.

(h) B. Teddy withdrew goods from the business at selling price of


$8,000 for her personal use. No entry was made in the books
for this transaction.

(i) In August 2004, goods were sent on a sale or return basis to M.


Line. On August 31, 2004 these goods were in the warehouse of
M. Line, but $15,000 was recorded in the Sales Journal of B.
Teddy. The mark-up on cost for the goods is 20%. The closing
inventory included these goods at cost, so no adjustment is
necessary to the closing inventory. B. Teddy maintains a
Subsidiary Ledger for Debtors.
557

REQUIRED:

(a) Prepare the journal entries necessary to correct the above


errors and omissions (narration not required).

(b) Assuming that all errors and omissions were discovered what
was the amount of the opening balance in the Suspense
Account, if any?

Please state whether the opening balance is a debit balance or


a credit balance.
558

PROBLEM 15 Control Accounts (20 minutes)

From the following information prepare the Purchases Ledger Control


Account and the Sales Ledger Control Account for Y. Navigator for
the month of July 2004.

$
Opening balance 1/7/04
Sales Ledger Control A/C (Dr.) 25,000
Purchases Ledger Control A/C (Cr.) 18,000
Purchases Ledger Control A/C (Dr.) 1,000

Credit purchases 160,000


Credit sales 205,000
Returns outward 3,200
Returns inward 6,000

Cheque payment to creditors 138,000


Bills payable 200
Discount received 700
Set-off Purchases Ledger balance with
Sales Ledger balance 1,500

Receipts from debtors - cheques 162,000


Bills receivable 500
Discount allowed 3,800
Bad debts 1,000

Cheques received and lodged - dishonoured 500


Bills receivable dishonoured 200

The opening debit balance of $1,000 in the Purchases Ledger Control


Account was in respect of damaged goods returned to M. Kid. On
Monday, July 10, 2004 M. Kid sent goods costing $800 to replace some
of the damaged goods.
559

PROBLEM 16 Preparation of Financial Statements for a Sole Trader


(75 minutes)

From the information provided prepare the following :


• Adjusting Journal Entries (narration not required)
• An Adjusted Trial Balance
• Closing Journal Entries (narration not required)
• Trading Statement
• Profit and Loss Statement
• Balance Sheet
• Opening Journal Entries (narration not required)

Please note - an Accountant's Worksheet is not required.

A. Heat list of balances on December 31, 2004.

ACCOUNT # NAME OF ACCOUNT $


2301 Capital A/C 193,500
2302 Drawings A/C 12,000
4100 Cost of Sales A/C 159,000
3102 Sales A/C 254,000
1106 Closing Stock A/C 47,500
4201 Salaries A/C 60,000
4202 Rent A/C 16,500
4203 Insurance A/C 15,000
4204 Travelling A/C 5,000
4210 Other Expenses A/C 2,500
1203 Office Furniture A/C 60,000
1204 Allowance for Depreciation A/C 10,000
1104 Debtors A/C 80,000
1105 Allowance for Bad Debts A/C 5,000
1102 Bank A/C 22,500
2101 Creditors A/C 17,500

Adjustments not yet made:

(1) Depreciation is on the straight line basis at 5% per annum on


cost.
(2) Rent not paid at December 31, 2004 $1,500.
(3) Insurance prepaid $2,000.
(4) The allowance for bad debts should be 5% of debtors.
560

(5) B. Fall who owes the business $6,000 migrated. It is not


expected that B. Fall will pay the $6,000. Write off the $6,000
as a bad debt.
(6) Debtors include goods sent to Bay Ltd. on a sale or return
basis; the goods remain unsold at the year end date and were
included in the closing stock. The cost of the goods amounted
to $3,000, and was included twice in the closing stock figure.
The margin on the goods is 25 per cent.

Other Accounts

ACCOUNT NUMBER NAME OF ACCOUNT


1003 Opening Stock Account
1108 Prepayment Account
2102 Accrual Account
3105 Decrease in Allowance for Bad Debts Account
4205 Bad Debt Account
4209 Depreciation Account
4211 Increase in Allowance for Bad Debts Account
561

PROBLEM 17 Cash Flow Statement Preparation (45 minutes)

Blue Limited Balance Sheets at December 31


2003 2004
$ $
Non-current Assets
Furniture 704,000 956,000
Less accumulated depreciation 380,000 430,000
324,000 526,000

Current Assets
Stocks 108,000 314,000
Trade debtors 308,000 830,000
Prepayments 40,000 50,000
Cash 8,000 14,000
Bank 20,000 190,000
484,000 1,398,000

Less Current Liabilities


Accruals 104,000 200,000
Trade creditors 192,000 842,000
Taxation payable 128,000 208,000
424,000 1,250,000

384,000 674,000
======= =======

Owners’ Equity
Ordinary share capital 150,000 170,000
20% Preference share capital 40,000 40,000
General reserves 50,000 30,000
Retained profits 120,000 404,000
360,000 644,000

Non-current Liabilities
20% Debentures 24,000 30,000
384,000 674,000
======= =======
562

Blue Limited Profit and Loss Statements for the years December 31
2003 2004
$ $
Profit before taxation 380,000 816,000
Taxation (128,000) (208,000)
Profit after taxation 252,000 608,000
Preference dividends paid ( 8,000) ( 8,000)
Ordinary dividends paid (156,000) (316,000)
Retained profit for the year 88,000 284,000
Retained profit brought forward 32,000 120,000
Retained profit carried forward 120,000 404,000
======= =======

Additional information:

1. During the year to December 31, 2004, furniture originally


costing $20,000 was sold for $12,000. The accumulated
depreciation on the furniture disposed of was $15,000.

2. Debenture loans repaid during the financial year ended December


31, 2004 amounted to $10,000.

3. Bonus issue of ordinary shares out of general reserves amounted


to $20,000 on December 31, 2004.

Prepare the following for the year ended December 31, 2004:

a. Furniture Account.
b. Allowance for Depreciation on Furniture Account.
c. Furniture Disposal Account.
d. Debenture Account.
e. Cash Flow Statement (indirect method).
563

PROBLEM 18 Cash Flow Statement Preparation (45 minutes)

Click Limited Balance Sheets at June 30


2003 2004
$ $
Non-current Assets
Furniture 352,000 478,000
Less accumulated depreciation 190,000 215,000
162,000 263,000

Current Assets
Stocks 106,500 100,700
Trade debtors 169,500 219,300
Deposit on call 0 100,000
Bank 0 122,000
276,000 542,000

Less Current Liabilities


Accruals 6,600 68,000
Trade creditors 139,400 296,000
Bank overdraft 36,000 -
Taxation payable 64,000 104,000
246,000 468,000

192,000 337,000
======= =======

Owners’ Equity
Ordinary share capital 100,000 100,000
20% Preference share capital 20,000 20,000
Retained profits 60,000 202,000
180,000 322,000

Non-current Liabilities
20% Long term loan 12,000 15,000
192,000 337,000
======= =======
564

Click Limited Profit and Loss Statements for the years June 30
2003 2004
$ $
Profit before taxation 198,000 418,000
Taxation ( 72,000) (114,000)
Profit after taxation 126,000 304,000
Preference dividends paid ( 4,000) ( 4,000)
Ordinary dividends paid ( 78,000) (158,000)
Retained profit for the year 44,000 142,000
======= =======

Additional information:

1. During the year to June 30, 2004, furniture originally costing


$10,000 was sold for $2,000. The accumulated depreciation on
the furniture disposed of was $7,500.

2. Long term loan repaid during the financial year ended June 30,
2004 amounted to $12,000.

3. Sales for 2004 amounted to $6,061,000.

Required:

a. Prepare the Cash Flow Statement for the year ended


June 30, 2004 using the indirect method.
b. Calculate the Free Cash Flow.
c. Calculate the Cash Flow Yield.
d. Calculate the Cash Flow to Sales
e. Calculate the Cash Flow to Assets
565

PROBLEM 19 Statutory Body Financial Statements (35 minutes)

National Water Authority Trial Balance on March 31, 2004


Debit Credit
$’000 $’000
Inventories 370,304
Other receivables and prepayments 65,039
Due from Government 815,915
Trade accounts receivable (net) 959,946
Cash and bank 245,119
Deposits and retentions 349,020
Trade accounts payable 56,681
Other payables 22,383
Long term loan 19,500
Accumulated surplus April 1, 2003 8,433,737
Property, plant and equipment 4,952,100
Accumulated depreciation 772,608
Long term receivable 540,000
Operating revenue 5,665,584
Project management fees 11,544
Interest income 18,377
Salaries, wages and related costs 5,517,358
Repairs and maintenance 763,216
Utilities 885,586
Fuel and lubrication 38,936
Bank charges 9,639
Depreciation 150,575
Water purchases 31,673
Interest expense 4,028 -
15,349,434 15,349,434
========== ==========
Additional information:
2. Current portion of long term loan $4,500,000.
3. Current portion of long term receivable $200,000,000.

Required:
a. Prepare the Income and Expenditure Statement for
the year ended March 31, 2004.
b. Prepare the Statement of Changes in Equity for the
year ended March 31, 2004.
c. Prepare the Balance Sheet as at March 31, 2004.
566

PROBLEM 20 Statutory Body Financial Statements (30 minutes)

National Housing Authority list of balances on March 31, 2004


$’000
Operating expenses 1,516,271
Bonus on employees’ contribution 461,186
Bonus on employers’ contribution 1,622
Losses on projects 21,755
Subsidies for special projects 10,000
Interest income on loans to beneficiaries 2,766,218
Interest income on investments 1,811,715
Interest income on housing construction finance 36,722
Gain on disposal of housing units 14,315
Service charge on loans to beneficiaries 47,615
Penalty 115,533
Other income 38,845
Non-refundable employers’ contribution April 1, 2003 14,717,622
Non-refundable employers’ contribution for the year 2,918,545
Peril reserve April 1, 2003 400,000
Accumulated surplus April 1, 2003 12,615,714

Additional information:
1. Transfer $200,000,000 to the peril reserve.

2. Mortgage loss allowance should be increased by $107,333,000

Prepare the following:


a. Income and Expenditure Statement for the year ended March
31, 2004.
b. Statement of Changes in Equity for the year ended March
31, 2004.
567
PROBLEM 21 Financial Statement Analysis (30 minutes)

Blue Limited Balance Sheets at December 31


2003 2004
$ $
Non-current Assets
Furniture 704,000 956,000
Less accumulated depreciation 380,000 430,000
324,000 526,000

Current Assets
Stocks 108,000 314,000
Trade debtors 308,000 830,000
Prepayments 40,000 50,000
Cash 8,000 14,000
Bank 20,000 190,000
484,000 1,398,000

Less Current Liabilities


Accruals 104,000 200,000
Trade creditors 192,000 842,000
Current portion of debenture loan 10,000
Taxation payable 128,000 208,000
434,000 1,250,000

374,000 674,000
======= =======

Owners’ Equity
Ordinary share capital 150,000 170,000
General reserves 50,000 30,000
Retained profits 120,000 404,000
Ordinary shareholders’ equity 320,000 604,000
20% Preference share capital 40,000 40,000
360,000 644,000

Non-current Liabilities
20% Debentures 14,000 30,000
374,000 674,000
======= =======
568

Blue Limited Profit and Loss Statements for the years December 31
2003 2004
$ $
Profit before taxation 380,000 816,000
Taxation (128,000) (208,000)
Profit after taxation 252,000 608,000
======= =======

Blue Limited Statement of Changes in Equity


For the year ended December 31, 2004
------------------------------------------------------------------
Ordinary Preference General Retained Total
Share Share Reserves Profits
Capital Capital
$ $ $ $ $
Balance 1/01/04 150,000 40,000 50,000 120,000 360,000
After Tax Profits 0 0 0 608,000 608,000
Bonus Issue 20,000 0 (20,000) 0 0
Dividends 0 0 0 (324,000) (324,000)
Balance 31/12/04 170,000 40,000 30,000 404,000 644,000
======= ======= ======= ====== ======

Additional information:
a. The number of ordinary shares issued is 15,000.
b. The market price per ordinary share on December 31, 2004
is $55.65.
c. The bonus issue of 2,000 ordinary shares was on December
31, 2004.
d. Preference dividends of $8,000 were paid during 2004.

Calculate the following ratios for 2004:


1. Current ratio
2. Acid test ratio
3. Earnings per share
4. Price earnings ratio
5. Cash dividend per ordinary share
6. Cash dividend yield per ordinary share
7. Total cash dividend payout
8. Book value per ordinary share
9. Return on common equity
569

PROBLEM 22 Incomplete Records (80 minutes)

Carl Moore, a sole trader had not kept a full set of books. The
following is a summary of his bank transactions for the year
ended June 30, 2004.

Receipts Payments
$ $
Trade debtors 620,000 Trade creditors 545,000
Credit card (net) 80,000 Rent & rates 47,200
Electricity 9,000
Salaries 52,000
Overdraft interest 1,500
Accountant’s fee 15,000
Drawings 2,500
700,000 672,200
======= =======

1. Favourable bank balance per Cash Book on June 30, 2004, $8,400.

2. During the year discounts allowed by trade creditors amounted


to $2,700 and those allowed to debtors $3,680.

3. Goods stolen during the year amounted to $10,000 (at cost), of


which 90% is recoverable from the insurance company.

4. Closing debtors include $1,500 in respect of a bad debt that


should be written off.

5. For some of the sales, credit cards are accepted. Carl Moore
pays a credit card commission of 5%.

6. June 30, 2003 June 30, 2004


$ $
Stock 82,000 97,000
Trade debtors 58,600 71,500
Accountant’s fee payable 15,000 16,000
Rent paid in advance 1,000 1,200
Furniture (net) 40,000 38,000
Trade creditors 39,000 38,000
Credit card receivables (net) 2,650 1,500
570

Prepare the following:

a. A Statement of Affairs on June 30, 2003.

b. Trade Debtors Control Account for the year ended June 30, 2004.

c. Trade Creditors Control Account for the year ended June 30,
2004.

d. Credit Card Receivables Account (gross) for the year ended June
30, 2004.

e. The Trading and Profit and Loss Account for the year ended June
30, 2004.

f. The Balance Sheet as at June 30, 2004


571

PROBLEM 23 Incomplete Records (90 minutes)

Marie Hunter is a retailer, who had not kept a full set of


accounting records. The following is a summary of her bank
transactions in her Cash Book for the year ended December 31, 2004.

Receipts Payments
$ $
Trade debtors 461,500 Trade creditors 286,750
Cash sales 229,200 Rent 49,000
Electricity 12,725
Water 6,265
Salary 94,000
Accounting fee(2003) 10,000
Telephone 9,622
Overdraft interest 1,600
Drawings 1,700
Cash purchases 82,600
690,700 554,262
======= =======

You are given the following information:


1. Favourable bank balance per Cash Book on December 31, 2004
amounted to $92,600.
2. During the financial year discounts allowed by trade creditors
amounted to $860 and those allowed to trade debtors amounted
to $150.
3. Trade debt written off during the financial year amounted to
$750.
4. During the financial year goods returned by trade debtors
amounted to $2,600 and goods returned to trade creditors
amounted to $1,700.
5. During the financial year a trade debt of $2,000 owed by
Janice Davis was set off against the $2,900 owing to the same
Janice Davis a trade creditor.
6. Hanif Brown, a cash sales customer returned goods during the
financial year amounting to $580. Hanif Brown was repaid his
$580 from the daily cash sales on the day he returned the
goods.
572

7. Jada Aris, a cash sales customer returned goods during the


financial year amounting to $1,000. A friend of Miss Aris
returned the goods. Miss Aris did not collect her $1,000 until
January 16, 2005. She was repaid out of the cash sales for
January 16, 2005.
8. Marie Hunter keeps a cash float of $1,000 for change at her
retail outlet.
9. Marie Hunter agreed to pay her son Douglas, a commission of
10% of the net profit after charging such commission, for
working in the retail outlet on Saturday nights in 2004.
10. The rent is being paid under the terms of a shop rental lease
that commenced on January 1, 2002. Leasehold improvement
during January 2 to January 5, 2002 amounted to $80,000. The
leasehold improvement is being depreciated on the straight
line basis with a $0 residual value over the ten years period
of the lease. A full month’s depreciation was charged for
January 2002.
11. For the year ended December 31, 2003 the net profit was
$48,200 and drawings amounted to $700.
12. Stocks are valued at cost or net realizable value, whichever
is lower. The accounts are prepared under the historical cost
convention. All amounts are stated in Jamaican Dollars.
13. The accounting fee for preparing the 2004 financial statements
is estimated at $11,000.

14. December 31, 2003 December 31, 2004


$ $
Stock 57,680 59,820
Trade debtors 2,650 2,980
Trade creditors 780 1,400
Rent paid in advance 4,000 5,000
Accrued electricity 950 1,100

Required:
a. Prepare a Statement of Affairs on December 31, 2003.
b. Prepare the Trade Creditors Control Account for the year ended
December 31, 2004.
c. Prepare the Trade Debtors Control Account for the year ended
December 31, 2004.
d. Prepare the Income Statement for the year ended December 31,
2004.
e. Prepare the Balance Sheet as at December 31, 2004.
573

PROBLEM 24 Preparation of Financial statements for a Sole Trader


(40 minutes)

Campbell Super Store started business on December 1, 2003.


Campbell Super Store financial year end is December 31. List of
balances on December 31, 2003:

$
Sales 455,000
Returns inward 5,000
Returns outward 5,000
Carriage outward 7,500
Purchases 200,000
Opening stock 0
Closing stock 25,000
Advertising 10,000
Travelling 12,500
Insurance 4,000
Rent 11,000
Salaries 100,000
Telephone 2,500
Electricity 10,000
Water 500
Capital 100,000
Drawings 2,000
Insurance prepaid 44,000
Cash and bank 151,000

Required:
a. Prepare the closing entries in the book of original entry.
b. Prepare the Trading and Profit and Loss Account.
c. Prepare the Balance Sheet.
574

PROBLEM 25 Preparation of Financial statements for a Sole Trader


(40 minutes)

Gold medal Store list of balances on December 31, 2003

ACCOUNT # NAME OF ACCOUNT $


2301 Capital Account 165,200
2302 Drawings Account 14,000
4102 Purchases Account 117,600
4103 Purchases Return Account 5,600
3102 Sales Account 282,800
4104 Opening Stock Account 64,400
4201 Salaries Account 56,000
4202 Rent Account 14,000
4203 Insurance Account 11,200
4210 Advertising Expense Account 2,800
1203 Office Furniture Account 56,000
1204 Allowance for Depreciation Account 11,200
1104 Trade Debtors Account 140,000
1102 Bank Account 58,800
2101 Trade Creditors Account 70,000

Additional information:
1. Depreciation is to be provided on the reducing balance basis
at 10% per annum.
2. Travelling expenses not paid at December 31, 2003 amounted
to $2,800.
3. Insurance prepaid on December 31, 2003 amounted to $1,400.
4. Closing stock on December 31, 2003 amounted to $61,600.

Other Accounts

ACCOUNT # NAME OF ACCOUNT


1106 Closing Stock Account
1108 Prepayment Account
2102 Accrual Account
3301 Trading Account
4204 Travelling expense Account
4209 Depreciation Account
575

Required:

a. Prepare the Income Statement for the year ended December 31,
2003.

b. Prepare the Balance Sheet as at December 31, 2003.

c. Prepare the opening entries on January 1, 2004 in the book of


original entry.
576

PROBLEM 26 Preparation of Financial Statements for a Company


(25 minutes)

Flour Limited list of balances on December 31, 2003


$’000
15% Preference share capital 50,000
Ordinary share capital 250,000
General reserves 20,000
Retained profits January 1, 2003 35,000
Long term loan 100,000
Cost of sales 750,700
Sales 1,350,000
Other operating income 9,000
Distribution costs 120,100
Administrative costs 300,200
Other operating costs 51,600
Interest on loan 30,000
Closing stock 28,400
Trade debtors 417,500
Bank 99,000
Trade creditors 16,000
Preference dividends paid 7,500
Ordinary dividends paid 25,000

Additional information:

1. Transfer $2,500,000 to a general reserve account.

2. Corporation tax is estimated at $13,800,000 for the 2003


financial year.

3. Weighted average number of ordinary shares in issue during the


year amounted to 250,000.

Required:
a. Prepare the Income Statement for the year ended December
31, 2003.
b. Prepare the Statement of Changes in Equity for the year
ended December 31, 2003
c. Prepare a Balance Sheet as at December 31, 2003.
577
PROBLEM 27 Financial Statement Preparation - Company (30 minutes)

Cricket Limited list of balances year ended December 31, 2004


US$
15% Preference share capital 100,000
Ordinary share capital 100,000
General reserves 16,000
20% Debentures 100,000
Retained profit January 1, 2004 140,202
Sales 950,100
Cost of sales 636,560
Debenture interest paid for 2004 7,500
Salaries 136,250
Insurance 13,265
Utilities expense 21,853
Maintenance expense 222
Rental income 3,900
Rent expense 22,000
Depreciation 5,000
Equipment 50,000
Allowance for depreciation on equipment 20,000
Trade debtors 136,270
Cash and bank 206,480
Closing stock 177,002
Trade creditors 17,200
Preference dividends paid 15,000
Ordinary dividends paid 20,000

Additional information:
1. Rent expense unpaid on December 31, 2004 amounted to
US$2,000.
2. Rental income for January 2005 received in December 2004
amounted to US$300.
3. Insurance prepaid for 2005 amounted to US$2,400.
4. Provide for corporation tax of US$28,602 for 2004.
5. Transfer US$2,000 to general reserve.
6. Weighted average number of ordinary shares in issue during
the year amounted to 50,000.

Required:
a. Prepare the Income Statement.
b. Prepare the Statement of Changes in Equity
c. Prepare the Balance Sheet.
578

PROBLEM 28 Partnership Accounts (45 minutes)

Trial Balance of Bed and bath on December 31, 2004


Debit Credit
$ $
Bed Capital Account 200,000
Bath Capital Account 100,000
Bed Current Account 5,000
Bath Current Account 6,000
Furniture and Fittings 50,000
Motor Vehicle 250,000
Allowance for Depreciation
Furniture and Fittings 15,000
Motor Vehicle 120,000
Bed Drawings 6,000
Bath Drawings 4,000
Cash and Bank 90,400
Trade Debtors 176,200
Trade Creditors 36,200
Accruals 600
Closing Stock 80,600
Sales 930,125
Cost of Sales 480,120
Salaries 204,000
Utilities 20,605
Depreciation 45,000
Repairs 5,580
Bank charges 420 -
1,412,925 1,412,925
========= =========
Additional information:
1. Provide for interest on capital at 10% per annum.
2. Bath is to receive a salary of $96,000 for working in the
partnership business.
3. Profits and losses are shared equally.

Required:
a. The Income Statement for the year ended December 31, 2004
b. The Current Account for each partner.
c. The Balance Sheet as at December 31, 2004.
d. The Statement of Changes in Partners’ Equity for the year
ended December 31, 2004.
579

PROBLEM 29 Manufacturing Accounts (40 minutes)

Grass Limited list of Balances on December 31, 2004


$
Opening stock of raw materials 118,734
Closing stock of raw materials 94,400
Purchases of raw materials 514,800
Carriage inwards on raw materials 11,200
Direct manufacturing wages and salaries 902,500
Other direct costs 23,610
Electricity - Factory 117,800
- Office 23,600
Rent - Factory 236,000
- Office 54,000
Depreciation of office machines 14,000
Depreciation of office furniture 4,000
Depreciation of factory machines 59,000
Depreciation of factory furniture 1,500
Opening stock of finished goods 11,800
Closing stock of finished goods 35,100
Administrative expenses 330,720
Selling expenses 48,000
Sales 2,520,700

Prepare the following:


a. Manufacturing Statement
b. Trading Statement
c. Profit and Loss Statement
580

PROBLEM 30 Financial Statement Analysis (40 minutes)

From the information provided calculate the following ratios for


2003:

a. Current Ratio
b. Quick Ratio
c. Gross Profit Margin
d. Net Profit Margin
e. Return on Total Assets
f. Return on Equity Capital
g. Dividend Payout
h. Total Asset Turnover
i. Inventory Turnover

Time Limited Income Statement for the year ended December 31, 2003
$’000
Sales 10,195
Less cost of sales 6,120
Gross profit 4,075
Add other operating income 5
4,080
Less operating expenses 3,158
Net profit before taxation 922
Taxation 244
Net profit after taxation 678
Less ordinary dividends paid 300
Retained profit for the year 378
Retained profit brought forward 410
Retained profit carried forward 788
======
581

Time Limited Balance Sheet as at December 31


2003 2002
$’000 $’000

Non-current Assets
Furniture 520 500
Less allowance for depreciation 242 200
278 300

Current Assets
Closing stock 713 605
Insurance prepaid 18 16
Trade debtors 147 130
Bank 830 744
Cash 210 10
1,918 1,505

2,196 1,805
===== =====

Current Liabilities
Trade creditors 102 115
Corporation tax payable 94 260
Accruals 12 20
208 395

Shareholder’s Equity
Ordinary share capital 1,200 1,000
Retained profits 788 410
1,988 1,410

2,196 1,805
===== =====
582

PROBLEM 31 Hotel Accounting (30 minutes)

Blue Mountain Hotel Limited Rooms Department data for the financial
year ended September 30, 2004.
$
Revenue 7,203,020
Sales allowances 28,000
Expenses
Salaries and wages 840,000
Employee benefits (excluding free meals) 92,160
Cable and satellite television 45,000
Commissions 60,000
Complimentary guest services 8,630
Contract services 14,500
Guest relocation 5,000
Guest transportation 18,700
Laundry and dry cleaning 130,200
Linen 14,000
Operating supplies 90,600
Reservations 39,400
Telecommunications 4,100
Training 18,000
Uniforms 60,000
Miscellaneous expenses 850

Additional information:
1. The gross revenue includes a 10% tax.
2. Commission amounting to $1,250 payable to a travel agent should
be accrued.
3. Included in the $45,000 for cable and satellite television
expenses is an amount of $9,000 for October 1, 2004 to December
31, 2004.
4. Meals provided free of cost for the Rooms Department employees
amounted to $46,600.

Prepare the Rooms Department Income Statement for the year ended
September 30, 2004 in accordance with the Uniform Systems of
Accounts for the Lodging Industry.
583

PROBLEM 32 Hotel Accounting (30 minutes)


$
Revenue (including peanuts) 2,480,500
Sales allowances 2,000
Cost of beverage 893,760
Costs of peanuts 440
Beverage transfers to Food Department 25,400
Food transfers to Beverage Department 30,200
Service charge income 248,050
Expenses
Salaries and wages 690,000
Employee benefits (excluding free meals) 41,280
China, glassware, silver, and linen 46,400
Contract services 16,400
Laundry and dry cleaning 30,895
Licenses 5,000
Music and entertainment 37,500
Operating supplies 50,700
Telecommunications 2,560
Training 12,000
Uniforms 40,000
Miscellaneous expenses 150

Additional information:
1. Wages unpaid at the year end for the Beverage Department
amounted to $6,200. No adjustment was made to the accounts for
this amount.
2. Meals provided free of cost for the Beverage Department
employees amounted to $19,500.
3. Food transfers to the Beverage Department include $19,400 for
food used in the preparation of beverages and $10,800 for
appetizers, served as complimentary food during happy hour at
the bar.
4. Salaries prepaid at the year end for the Beverage Department
amounted to $6,000. No Adjustment was made to the accounts for
this amount.
5. One hundred and twenty non-hotel guests entering the nightclub
at the Hotel were charged a cover charge of $200 each.

Prepare Cedar Valley Hotel Limited Beverage Department Income


Statement for the year ended September 30, 2004 in accordance with
the Uniform Systems of Accounts for the Lodging Industry.
584

PROBLEM 33 Hotel Accounting (30 minutes)


US$ US$
Revenue (including sweets and cigarettes) 366,225
Sales allowances 1,020
Cost of food 83,750
Costs of sweets and cigarettes 4,050
Beverage transfers to Food Department 2,100
Food transfers to Beverage Department 2,555
Promotional meals 1,000
Employee meals (provided free of cost)
Food Department 5,925
Beverage Department 4,155
Rooms Department 4,060
Administrative Department 2,520
16,600
Meeting room rental income 40,660
Service charge income 15,912
Expenses
Salaries and wages 160,330
Employee benefits (excluding free meals) 39,750
China, glassware, silver, and linen 1,580
Contract services 2,050
Laundry and dry cleaning 4,250
Licenses 2,755
Music and entertainment 15,966
Operating supplies 23,980
Telecommunications 1,270
Training 4,800
Uniforms 15,425
Miscellaneous expenses 24

Additional information:
1. Training expenses prepaid at the year end amounted to US$270.
2. An invoice for contract services amounting to US$500 is not yet
reflected in the accounts.

Prepare the Food Department Income Statement for the year ended
December 31, 2004 for Casual Hotel Limited in accordance with the
Uniform Systems of Accounts for the Lodging Industry.
585

crpuzz

CROSSWORD PUZZLE

1 2 3 4 5 6 7 8 9 10 11
1 ||| |||
2 ||| ||| ||| ||| ||| ||| ||| ||| |||
3 ||| ||| ||| |||
4 ||| ||| ||| ||| ||| ||| ||| ||| |||
5 ||| ||| ||| |||
6 ||| ||| ||| ||| ||| |||
7 ||| ||| ||| ||| ||| ||| |||

8 ||| ||| |||


9 ||| ||| ||| ||| ||| ||| ||| |||
10 ||| ||| ||| ||| ||| ||| ||| ||| |||
11 |||

The first number indicates the column number and the second number
indicates the row number. Therefore 1,5 means column 1,row 5.

ACROSS

1,1 An accounting method developed by Michael Williams ___.


1,5 To increase an Asset Account you must _____ the account.
1,11 An amount owed by the business is called a _________.
2,3 The right hand side of an account is called the ______ side.
5,1 The profit expressed as a percentage of selling price.
586

DOWN

2,3 To increase a Liability Account you must ______ the account.


4,2 The left hand side of an account is called the _____ side.
4,8 The excess of income over expenses in the Profit and Loss
Account is referred to as net ______.
6,6 Each business transaction is recorded in a ____ of original
entry.
8,5 The loss made by a non-profit organization is referred to as
a _______.
10,1 The excess of expenses over income in the Profit and Loss
Account is referred to as the ___ loss.
11,5 An expense not paid at the end of the financial year.
587
588

crpuzz1
ANSWER TO CROSSWORD PUZZLE

1 2 3 4 5 6 7 8 9 10 11
1 A R E ||| M A R G I N |||
2 ||| ||| ||| D ||| ||| ||| ||| ||| E |||
3 ||| C R E D I T ||| ||| T |||
4 ||| R ||| B ||| ||| ||| ||| ||| ||| |||
5 D E B I T ||| ||| D ||| ||| A
6 ||| D ||| T ||| B ||| E ||| ||| C
7 ||| I ||| ||| ||| O ||| F ||| ||| C

8 ||| T ||| P R O F I T ||| R


9 ||| ||| ||| ||| ||| K ||| C ||| ||| U
10 ||| ||| ||| ||| ||| ||| ||| I ||| ||| A
11 L I A B I L I T Y ||| L
589

gry

GLOSSARY
The number following each definition gives the page(s) on which
additional information can be found.

A
Account - A record of financial transactions or financial entries
expressed in monetary terms, usually for a specific period of
time. An account contains a debit and a credit side. [16]

Accounting Equation - Assets equal liabilities plus owners' equity.


[227]

Accounting Standards - Provide guidelines for accountants in the


preparation of financial statements. [39]

Accrual Concept - Under this concept, costs (expenses) incurred in


one financial period should be matched against revenues that
are produced in that same financial period as a result of those
costs. [36]

Accrued Expense - An expense that is incurred during the accounting


period and is not paid for by the end of that same accounting
period. [144]

Accrued Income - Income that is earned during an accounting period


but is not paid to the firm by the end of that same accounting
period. [157]

Accumulated Fund - Capital within a non-profit oriented


organization. [408]
590

Adjusted Trial Balance - Trial Balance extracted from the General


Ledger at the end of the accounting period after the adjusting
entries have been posted; used to prepare the financial
statements. [206]

Annual General Meeting - An annual meeting of the shareholders of


a company. [344]

Assets - Economic resources that are owned or controlled by a firm


and are expected to benefit future operations. [12]

Authorized Share Capital - Indicates the maximum amount of each


class of share the company can legally issue. [345]

B
Bad Debt - An amount written off in the Profit and Loss Account as
an expense because the business knows with reasonable certainty
that it will not collect this amount from the debtor. [151]

Balance Sheet - A financial statement and not an account, and


therefore the assets can be on the left hand side and the
liabilities on the right hand side or the assets can be on the
right hand side and the liabilities on the left hand side. [221]

C
Capital Account - A Liability Account that reflects the amount
invested by the owner of the business (for example, Jack Sprat
Capital Account) or owners of the business (for example,
Ordinary Share Capital Account). [227,317]

Capital Expenditure - Refers to costs incurred, in the acquisition


of tangible non-current assets, and in material modification and
additions to these assets. [176]
591

Capital Reserve - Reserve that cannot be used to pay cash


dividends, that is, non-distributable reserve. [348]

Carriage Inward - Transportation cost on goods purchased. [218]

Carriage Outward - Transportation cost on goods sold. [219]

Cash Discount - Given for prompt payment. [70]

Cash Flow Statement - Indicates the increase or decrease in cash


for a particular accounting period. It also shows the sources of
cash and the application (use) of cash. [248,359]

Certificate of Incorporation – The birth certificate of a company.


[48,341]

Chart of Accounts - A list of ledger accounts of an accounting


entity giving the name of each account and the code for each
account (account number). [99]

Closing Entries - The process of closing all Nominal Accounts by


means of journal entries at the end of the financial year. [200]

Company - An artificial person, it comes into existence when the


Registrar of Companies issues a Certificate of Incorporation.
[48,340]

Consistency Concept - Under this concept, the accounting treatment


of like items should be consistent within each accounting period
and over successive accounting periods. [37]

Contingent Liability - A potential liability at the end of the


financial period that may or may not materialize. [168]

Credit - The right hand side of an account. [16]

Credit Note - Decreases the amount owed by customers. [50,56]

Current Asset - Asset that continually changes form within a 12


month period during the course of business. [13]
592

Current Liability - Debt payable by the firm within a 12 months


period. [13]

D
Debenture - Long term loan made to the firm. [349]

Debit - The left hand side of an account. [16]

Debit Note - Increases the amount owed by customers. [50,55]

Deficit - The excess of expenditure over income. [408]

Delivery Slip - Informs customers of goods being delivered. [50,57]

Depreciation - An allocation of the capitalised costs of a tangible


non-current asset (minus salvage value) over the estimated
useful life of the asset. [178]

Direct Cost - Cost that can be traced to a specific department.


[379]

Discount Allowed - A cash discount given for prompt payment


representing an expense to the business. [70]

Discount Received - A cash discount received for prompt payment


representing income to the business. [70]

Dishonoured Cheque - A cheque that is not paid by the bank on


which it is drawn. [292]

Double Entry Concept - Under this concept, every transaction in the


General Ledger is twofold. [16,36]

Drawee - The bank on which the cheque is drawn. [290]

Drawer - The person who signs the cheque. [290]


593

Drawings - The transaction whereby the owner of the business takes


cash or goods from the business for his or her personal use.
[167]

E
Entity Concept - Under this concept, it is assumed that the firm is
a separate entity from its owner(s). [34]

Expense - Cost incurred during the accounting period in order to


earn income for the organization. [15]

Extra-ordinary General Meeting - An emergency meeting of the


shareholders of a company. [344]

F
Financial Accounting - A system that involves the recording,
analysis, and communication of financial information of
organizations. [2]

Financial Accounting Cycle - A series of sequential steps leading


to the financial statements. [9]

Fixed Charge - A specific asset given as security for a loan. [343]

Floating Charge - No specific asset is given as security for the


loan. [343]

Funds Flow Statement -Indicates the increase or decrease in working


capital for a particular accounting period. It also shows the
sources of working capital and the application (use) of working
capital. [248]
594

G
GAAP - Generally Accepted Accounting Principles are concepts,
conventions, practices and principles that provide the basis
upon which the accounts are prepared. [34]

Going Concern Concept - Under this concept, it is assumed that the


firm will continue operations in the foreseeable future and
there is no intention to sell any significant part of the firm
or to liquidate the entire firm. [35]

Gross Loss - When cost of sales exceeds net sales. [217]

Gross Profit - When net sales exceed cost of sales. [216]

H
Historical Cost Concept - Under this concept, transactions are
recorded after they occur. [35]

I
Impersonal Account - Account relating to tangible items and account
relating to income, gains, profits, expenses, losses and costs.
[98]

Income - Revenue earned during the accounting period. [15]

Income and Expenditure Account - Indicates the surplus or deficit


for a particular accounting period. It also shows the expenses
and income for that period. [408]

Income Statement - Indicates the net profit or net loss for a


particular accounting period. It also shows the expenses and
income for that accounting period. [220]
595

Indirect Cost - Common costs incurred for the benefit of all


departments. [379]

Issued Share Capital - The number of shares issued to date


multiplied by the par value per share. [345]

L
Liability - Debt payable by the firm. [13]

Long Term Solvency - Indicates the ability of the organization to


"stay in business" in the long run. [470]

M
Margin - Profit expressed as a percentage of selling price. [162]

Mark-up - Profit expressed as a percentage of cost. [162]

Materiality Concept - Under this concept, the accounting treatment


of an item will depend on its significance. [37]

Money Measurement Concept - Under this concept, it is assumed that


all expenses, income, assets and liabilities are capable of
being measured in terms of a single monetary unit. [34]

N
Net Loss - When expenses exceed income. [218]

Net Profit - When income exceed expenses. [218]

Non-current Liability - Debt of the firm which will not become


payable within a 12 month period. [13]
596

Nominal Account - Account relating to expenses, losses, income,


profits, costs and gains. [98]

O
Objectivity Concept - Under this concept, the accounts are prepared
in such a way as to limit the scope for subjective judgement and
bias. [38]

Opening Entries - The process of opening the Asset Accounts and the
Liability Accounts at the beginning of the financial year. [228]

Ordinary Share - A company issues ordinary shares in order to raise


capital for the company. Ordinary shares are not entitled to any
fixed percentage in dividends. [345]

Owners' Equity - Capital invested in the business by its owners


plus reserves (undistributed profits). [14]

P
Partnership - A business owned by two or more persons. [46,316]

Payee - The person or firm that should receive the money from the
cheque. [290]

Personal Account - Account relating to transactions with persons.


[98]

Postdated Cheque - A cheque that is issued now but bears some


future date. [291]

Preference Share - A company issues preference shares in order to


raise capital for the company. Preference shares are entitled to
a fixed percentage in dividends. [346]
597

Prepayment - An expense that relates to a future accounting period


and is paid for before the beginning of this future accounting
period. [148]

Private Company - A company that restricts the right to transfer


its shares. [49,342]

Profit and Loss Account - Indicates the net profit or net loss for
a particular accounting period. It compares expenses with
income. [218]

Profit and Loss Appropriation Account - Distributes the profits or


losses of the partnership business or company. [316,349]

Proforma Invoice - Indicates to a prospective customer the cost of


goods if he or she decides to purchase the goods. [51,60]

Proxy - A legal document authorizing someone to vote on behalf of


a shareholder at a general meeting. [49,345]

Prudence Concept - Under this concept, income should not be


anticipated but recognized only when it is realized in the form
of cash or other asset that can be treated as cash. [36]

Public Company - A company that does not restrict the right to


transfer its shares. [49,342]

Purchase Order - Authorizes the purchase of goods or services.


[50,58]

Purchases Invoice - Received from suppliers for goods or services


purchased on credit. [50,52]

R
Real Account - Account relating to tangible items. [98]

Receipt - Given to customers to indicate the amount of money they


pay to the firm. [50,53]
598

Revenue Expenditure - Costs incurred in relation to cost of sales


and operating expenses. [176]

Revenue Reserve - Reserve that can be used to pay cash dividends


(distributable). [348]

S
Sales Invoice - Sent to customer for goods or services sold on
credit. [50,52]

Short Term Solvency - Indicates the ability of the organization to


"stay in business" in the short run. [466]

Sole Trader - A business owned by one person. [45]

Stable Monetary Unit Concept - Under this concept, it is assumed


that the value of the monetary unit used is constant over time.
[35]

Stale Dated Cheque - A cheque that is not lodged or cashed after


six months. [291]

Stock Requisition - Requests goods from stores or the warehouse.


[51,59]

Subscription - A fee paid to the club (non-profit organization) by


its members each year. [408]

Subscriptions in Advance - Subscriptions paid by members before


the due date. [408]

Subscriptions in Arrears - Subscriptions owed to the club by


members. [408]

Surplus - The excess of income over expenditure. [408]


599

T
Tangible Non-current Asset - Asset purchased by the firm to assist
it in earning its profit or to enhance its earning power. [12]

Time Period Concept - Under this concept, it is assumed that the


financial activities of a firm can be divided into time periods.
[36]

Trade Discount - Given by the firm to customers who buy goods in


bulk. [70]

Trading Account - Indicates the gross profit or the gross loss for
a particular accounting period. [216]

Trial Balance - A list of all accounts with debit and credit


balances in the General Ledger at the end of the accounting
period, usually in account number order. [121]

U
Unadjusted Trial Balance - The first Trial Balance that is
extracted from the General Ledger at the end of the accounting
period. [206]

Unearned Income - Payment received by the firm in the current


period in respect of future sales or service. [159]

Unrealized profit - Internal profit not yet realized by the


business. [443]
600
INDEX
xtx

A
A R E Method 19
Account 16
Account Number 99
Accountant's Worksheet 210
Accounting Equation 227
Accounting Standards 39
Accrual Concept 36
Accrued Expense 144
Accrued Income 157
Accumulated Fund 408
Acid Test Ratio 466
Adjusted Trial Balance 206
Adjustment 144
Allowance for Bad Debt 153
Allowance for Depreciation 178
Allowance for Discount on Debtors 154
Allowance for Unrealized Profit 156,444
Annual General Meeting 344
Articles of Association 48
Asset Liquidity 356
Assets 12
Asset Management Ratios 479
Auditing 6
Authorized Share Capital 345
Average Age of Trade Creditors 483
Average Collection Period 483

B
Bad Debt 151
Bad Debt Recovered 152
Balance Brought Down 120
Balance Brought Forward 120
Balance Carried Down 120
Balance Carried Forward 120
601

Balance Sheet 221


Balancing Accounts 120
Bank Reconciliation 293
Bank Statement 298
Beverage Department Income Statement 389
Bills Of Exchange 222
Bills Payable 222
Bills Receivable 222
Board of Directors 343
Bonds 349
Bookkeeping 2
Book Value Per Share 485
Books of Original Entry 71
Bounced Cheque 292
Business Organizations 44

C
Cancelled Cheque 291
Capital Account 227,317
Capital Expenditure 176
Capital Reserve 348
Capitalised 176
Carriage Inward 218
Carriage Outward 219
Cash Book 77
Cash Discount 70
Cash Equivalents 248
Cash Flow To Assets 487
Cash Flow To Sales 487
Cash Flow Statement 248,359
Cash Flow Statement Ratios 485
Cash Flow Yield 487
Certificate of Incorporation 48,341
Certified Cheque 292
Chart of Accounts 99
Cheque 290,291,292,293
Cheque Requisition Voucher 51,63
Closing Balance 120
Closing Inventory 503
602

Closing Entries 200


Company 48,340
Comparative Analysis 477
Concepts 34
Conservatism Concept 36
Consignment 167
Consistency Concept 37
Contingent Liability 168
Continuous Production Process 449
Control Account 133,272,273
Convertible Preference Share 346
Cost Center 442
Cost Of Production 434
Cost of Raw Materials Consumed 433
Credit 16
Credit Balance 120
Credit Card Statement 51,62
Credit Note 50,56
Crossed Cheque 290
Cumulative Preference Share 346
Current Account (Bank) 15
Current Account (Partner) 317
Current Assets 13
Current Liabilities 13
Current Ratio 466

D
Debentures 349
Debit 16
Debit Balance 120
Debit Note 50,55
Debt Management Ratios 480
Declaration of Compliance 341
Deficit 408
Delivery Slip 50,57
Departmental Accounts 378
Depletion Method of Depreciation 192
Depreciable Cost 178
Depreciation 178
Depreciation Policy 178
603

Direct Costs 379


Direct Method 249
Discount Allowed 70
Discount Received 70
Discrete Production Process 449
Dishonoured Cheque 292
Disposal of tangible non-current Assets 193
Dividend 347
Dividend Payout 485
Dividend Per Share 485
Dividend Yield 485
Double Entry Concept 16,36
Drawee 290
Drawer 290
Drawings 167

E
Earnings Per Share 475
E-Business 8
E-Mail 8
Entity Concept 34
Errors 123,124,125
Expenses 15
Extra-ordinary General Meeting 344
Extra-ordinary Resolution 344
External Auditing 6

F
Final Dividends 347
Financial Accounting 2
Financial Accounting Cycle 9,10,11
Financial Statement Analysis 463
Financing Activities 248
Finished Goods Inventory 503
Fixed Assets 176
Fixed Charge 343
604

Floating Charge 343


Food Department Income Statement 385
Free Cash Flow 488
Fundamental Principle of Debit and Credit 16
Funds Flow Statement 248

G
GAAP 34
Gearing Ratio 471
General Allowance For Bad Debts 153
General Journal 90
General Ledger 104
General Meeting 344
Going Concern Concept 35
Goods in Transit 504
Goods Receival Note 51,61
Goods sent on a Sale or Return Basis 504
Goodwill 12
Gratis Food Expense 390
Gross Loss 217
Gross Profit 216
Gross Profit Margin 473

H
Historical Cost Concept 35
Horizontal Analysis 476
605
I
Impersonal Accounts 98
Income 15
Income and Expenditure Account 408
Income Statement 220
Indirect Costs 379
Indirect Method 254,360
Industry Comparison 477
Intangible Assets 12
Interim Dividends 347
Internal Auditing 6
Internal Profit 437
Inter-organization Comparison 477
Intra-organization Comparison 477
Inventory 502
Inventory Tag 508
Inventory Turnover 483
Investing Activities 248
Issued Share Capital 345

J
Job Cost Sheet 450
Job Costing 449
Job Order Costing 449
Journal 71

L
Liabilities 13
Life Membership Fund 410
Limitation Of Income Statement 229
Limitation Of Balance Sheet 229
Liquidity Analysis 478
606

Long Term Investment 13


Long Term Receivable 12
Long Term Solvency 470

M
Management Accounting 5
Managerial Accounting 5
Managers Cheque 293
Manufacturing Account 435
Manufacturing Cost 434
Manufacturing Cost Per Unit 434
Margin 162
Mark-up 162
Matching Concept 36
Materiality Concept 37
Memorandum of Association 48
Money Measurement Concept 34

N
Net Book Value 187
Net Loss 218
Net Profit 218
Net Profit Margin 474
Nominal Accounts 98
Non-Current Assets 12
Non-Current Liabilities 13
Non-distributable Reserves 348
607
O
Objectivity Concept 38
Open Cheque 290
Opening Balance 120
Opening Entries 228
Operating Activities 248
Ordinary Resolution 344
Ordinary Shares 345
Overtime Rate 521
Owners' Equity 14

P
Par Value 348
Participating Preference Share 346
Partnership 46,316
Payee 290
Payment 15
Payment Voucher 51,63
Payroll Accounting Entries 521
Payroll Summary 530
Permanency 356
Perpetual Inventory System 504
Personal Accounts 98
Petty Cash Book 84
Petty Cash Voucher 51,64
Physical Inventory Count 505
Postdated Cheque 291
Preference Share 346
Prepayments 148
Price Earnings Ratio 475
Prime Cost 433
Principles Of Accounts 2
Private Company 49,342
Production Cost 434
Profitability Analysis 473
Profitability Ratios 481
608

Profit and Loss Account 218


Profit and Loss Appropriation Account 316,349
Profit Center 442
Proforma Invoice 51,60
Property, Plant And Equipment 12
Prospectus 342
Provision for Bad Debt 153
Provision for Depreciation 178
Provision for Discount on Debtors 154
Provision for Unrealized Profit 156,444
Proxy 49,345
Prudence Concept 36
Public Company 49,342
Purchase Order 50,58
Purchases Invoice 50,52
Purchases Journal 75

Q
Quick Ratio 466

R
Ratio Analysis 464
Real Accounts 98
Receipt 50,53
Receipts 15
Redeemable Preference Share 346
Reducing Balance Method of Depreciation 188
Register of Members 343
Registered Company 341
Residual Value 178
Resolution 344
Restricted Cross Cheque 291
Retained Earnings 348
Return on Common Equity 484
Return on Total Assets 484
Returns Inward Journal 76
609
Returns Outward Journal 76
Revaluation Method of Depreciation 191
Revenue Expenditure 176
Revenue Reserve 348
Rooms Department Income Statement 382

S
Sales Invoice 50,52
Sales Journal 74
Salvage Value 178
Share Performance Analysis 475
Share Performance Ratios 482
Share Premium 348
Short Term Solvency 466
Sole Trader 45
Solvency 465
Special Resolution 344
Specific Allowance For Bad Debt 153
Stable Monetary Unit Concept 35
Stale Dated Cheque 291
Standards 39
Statement Of Changes In Equity 324,352
Statutory Body 417
Statutory Deduction 522
Stock Reconciliation 512
Stock Requisition 51,59
Stocktaking 502
Straight Line Method of Depreciation 179
Subscriptions 408
Subscriptions in Advance 408
Subscriptions in Arrears 408
Subsidiary Ledgers 104
Sum of the Years' Digit Method of Depreciation 190
Sundries 525
Surplus 408
Suspense Account 125
610

T
Tangible Assets 12
Tangible Non-Current Asset Turnover 483
Time Period Concept 36
Time Sheet 527
Times Interest Earned 472
Total Asset Turnover 484
Total Debt to Total Assets 484
Trade Debtors Turnover 484
Trade Discount 70
Trademark 12
Trading Account 216
Transfer Price 443
Trend Analysis 477
Trial Balance 121

U
Unadjusted Trial Balance 206
Unearned Income 159
Unrealized Profit 443

V
Vertical Analysis 476

W
Working Capital 467
Work in Progress 440
Work In Progress Inventory 503

Das könnte Ihnen auch gefallen